0% found this document useful (0 votes)
1K views398 pages

W Harley Sobin, Kia Saeian, Patrick Sanvanson Managing Complex Cases

Copyright
© © All Rights Reserved
We take content rights seriously. If you suspect this is your content, claim it here.
Available Formats
Download as PDF, TXT or read online on Scribd
0% found this document useful (0 votes)
1K views398 pages

W Harley Sobin, Kia Saeian, Patrick Sanvanson Managing Complex Cases

Copyright
© © All Rights Reserved
We take content rights seriously. If you suspect this is your content, claim it here.
Available Formats
Download as PDF, TXT or read online on Scribd
You are on page 1/ 398

Managing

Complex Cases in
Gastroenterology

A Curbside Guide
W. Harley Sobin
Kia Saeian
Patrick Sanvanson
Editors

123
Managing Complex Cases in Gastroenterology
W. Harley Sobin • Kia Saeian
Patrick Sanvanson
Editors

Managing Complex Cases in


Gastroenterology
A Curbside Guide
Editors
W. Harley Sobin Kia Saeian
Division of Gastroenterology and Division of Gastroenterology and
Hepatology Hepatology
Medical College of Wisconsin Medical College of Wisconsin
Milwaukee, WI, USA Milwaukee, WI, USA

Patrick Sanvanson
Division of Gastroenterology and
Hepatology
Medical College of Wisconsin
Milwaukee, WI, USA

ISBN 978-3-031-48948-8    ISBN 978-3-031-48949-5 (eBook)


https://doi.org/10.1007/978-3-031-48949-5

© The Editor(s) (if applicable) and The Author(s), under exclusive license to Springer Nature
Switzerland AG 2023
This work is subject to copyright. All rights are solely and exclusively licensed by the Publisher, whether
the whole or part of the material is concerned, specifically the rights of translation, reprinting, reuse of
illustrations, recitation, broadcasting, reproduction on microfilms or in any other physical way, and
transmission or information storage and retrieval, electronic adaptation, computer software, or by similar
or dissimilar methodology now known or hereafter developed.
The use of general descriptive names, registered names, trademarks, service marks, etc. in this publication
does not imply, even in the absence of a specific statement, that such names are exempt from the relevant
protective laws and regulations and therefore free for general use.
The publisher, the authors, and the editors are safe to assume that the advice and information in this book
are believed to be true and accurate at the date of publication. Neither the publisher nor the authors or the
editors give a warranty, expressed or implied, with respect to the material contained herein or for any
errors or omissions that may have been made. The publisher remains neutral with regard to jurisdictional
claims in published maps and institutional affiliations.

This Springer imprint is published by the registered company Springer Nature Switzerland AG
The registered company address is: Gewerbestrasse 11, 6330 Cham, Switzerland

Paper in this product is recyclable


Preface

This book was designed to help gastroenterologists take care of their patients with
challenging problems. Every day in practice we face patients with issues that are not
simple to master. Oftentimes, we search out some guidance from “the experts,” and,
if they are available, we will curbside them, either by phone or in person.
This text is written from the standpoint of a community gastroenterologist curb-
siding his academic cohorts with his challenging cases. We present a patient with a
difficult GI problem and ask the academic expert how he, or she, would manage the
case. We have provided over a hundred examples of problem cases and asked the
experts at the Medical College of Wisconsin (MCW) for help in their management.
These cases run the gamut, including general GI, motility disorders, inflammatory
bowel disease, advanced therapeutics, hepatology, and disorders of gut–brain
interaction.
The main objective of this book is to compile the most clinically useful ques-
tions, present them to experts who frequently deal with these problems, and then
wrap up their knowledge within one clinically useful text. If a community gastroen-
terologist had endless access to an academic panel of GI experts, these are the ques-
tions they might ask. There are hundreds of useful clinical pearls between the pages
of this book which should help anyone taking care of patients with GI problems. We
want to thank the academic GI team at the Medical College of Wisconsin who, with
their hundreds of years of accumulated experience, share their invaluable counsel.
While the text may be targeted particularly toward practicing community gastro-
enterologists, it certainly can be of benefit to GI fellows in training, academic gas-
troenterologists, and general internists and surgeons with a strong interest in
gastroenterology as well as APPs.
Of course, these opinions shouldn’t be accepted as the only choice for your
patient. Indeed, in some cases, we share several opinions for managing the same
condition. But any decision-making in your individual patient should be done in the
context of the latest developments, and specific medical details of your particular
patient. But we hope this text will serve as a useful adjunct in managing your more
difficult patients.

v
vi Preface

Disclaimer—the cases presented in this text are hypothetical composites and


don’t represent specific individuals.
We would like to acknowledge the substantial help of Pilar Kuderer and Clarissa
Marsh in the preparation of this text. They were assisted by Emma Kiefer and
Kari Warren.

W. Harley Sobin
 Kia Saeian
Milwaukee, WI  Patrick Sanvanson
Contents

Part I General GI
1 
Introduction to the General GI Section ��������������������������������������������������   3
W. Harley Sobin
2 Eosinophilic Esophagitis����������������������������������������������������������������������������   7
Patrick Sanvanson
3 Oropharyngeal Dysphagia������������������������������������������������������������������������ 15
Patrick Sanvanson
4 Achalasia���������������������������������������������������������������������������������������������������� 17
Benson Massey
5 
Other Motility Disorders of the Esophagus and Achalasia�������������������� 25
Francis Edeani
6 GERD���������������������������������������������������������������������������������������������������������� 31
Patrick Sanvanson
7 Refractory GERD�������������������������������������������������������������������������������������� 37
W. Harley Sobin
8 Surgery for GERD ������������������������������������������������������������������������������������ 41
Jon Gould
9 Barrett’s Esophagus���������������������������������������������������������������������������������� 47
Kulwinder Dua and Wilfredo Pagani
10 PEG Tubes�������������������������������������������������������������������������������������������������� 53
William Berger
11 Autoimmune Metaplastic Atrophic Gastritis������������������������������������������ 65
Francis Edeani
12 
Subepithelial Lesions and NETs �������������������������������������������������������������� 69
Phillip Chisholm

vii
viii Contents

13 Viewpoints on Managing Common Clinical GI Disorders from


a Practitioner with Over 50 Years of “Real-World” Experience ���������� 73
Helmut Ammon
14 
Small Intestinal Bacterial Overgrowth���������������������������������������������������� 81
Benson Massey
15 Unusual Causes of Abdominal Pain and Controversies
in Diagnosis������������������������������������������������������������������������������������������������ 87
W. Harley Sobin and Patrick Sanvanson
16 
Vascular Disorders of the Intestine���������������������������������������������������������� 101
Jim Nelson
17 Polypectomy������������������������������������������������������������������������������������������������ 109
Zachary Smith and Matt Mohorek
18 Rectal Incontinence������������������������������������������������������������������������������������ 121
Ling Mei and Krupa Patel
19 Diverticulitis ���������������������������������������������������������������������������������������������� 129
William Berger, Keely Browning, and W. Harley Sobin
20 
Pancreatic Cysts and Recurrent Pancreatitis������������������������������������������ 139
Phillip Chisholm
21 Chronic Pancreatitis���������������������������������������������������������������������������������� 145
Srivats Madhavan
22 GI Oncology������������������������������������������������������������������������������������������������ 151
Ben George
23 Issues in Therapeutic Endoscopy�������������������������������������������������������������� 159
Kulwinder Dua
24 GI Pharmacology:�������������������������������������������������������������������������������������� 167
Elizabeth Pieper

Part II IBD Compendium


25 
Introduction to the Inflammatory Bowel Disease Compendium���������� 181
W. Harley Sobin
26 Crohn’s Ileitis �������������������������������������������������������������������������������������������� 183
Preetika Sinh
27 
Ulcerative Colitis Refractory to Mesalamine������������������������������������������ 189
Daniel Stein and Salina Faidhalla
28 
Managing Crohn’s in a Patient with Prior MI���������������������������������������� 193
Preetika Sinh
Contents ix

29 Managing Post-Op Crohn’s���������������������������������������������������������������������� 197


Amir Patel
30 Managing Pseudopolyps���������������������������������������������������������������������������� 199
Poonam Beniwal-Patel
31 
Ulcerative Colitis Refractory to Anti-TNF���������������������������������������������� 201
Daniel Stein and Salina Faidhalla
32 
Acute Severe Ulcerative Colitis���������������������������������������������������������������� 205
Amir Patel
33 Resistant/Refractory Proctitis������������������������������������������������������������������ 207
Poonam Beniwal-Patel
34 
Vaccinations in Newly Diagnosed IBD ���������������������������������������������������� 209
Preetika Sinh
35 
Ustekinumab vs Risankizumab in Crohn’s �������������������������������������������� 211
Daniel Stein and Salina Faidhalla
36 
Use of Vedolizumab in UC������������������������������������������������������������������������ 215
Daniel Stein and Salina Faidhalla
37 
Functional Diarrhea in IBD���������������������������������������������������������������������� 219
Amir Patel
38 
When to Postpone Infusions���������������������������������������������������������������������� 221
Poonam Beniwal-Patel
39 
Musculoskeletal Complaints in a Patient on Anti-TNFs������������������������ 223
Amir Patel
40 Use of Immune Modulators in Patients Being Started
on Anti-TNFs���������������������������������������������������������������������������������������������� 225
Daniel Stein and Salina Faidhalla
41 Pyoderma Gangrenosum�������������������������������������������������������������������������� 227
Poonam Beniwal-Patel
42  in a Patient Who Failed Mesalamine and Anti-TNF������������������������ 229
UC
Daniel Stein and Salina Faidhalla
43 Patients with IBD Who Are Squeamish About Rectal Meds
and Self-Injection �������������������������������������������������������������������������������������� 231
Amir Patel
44 
Colon Stricture in UC�������������������������������������������������������������������������������� 233
Poonam Beniwal-Patel
45 Nonspecific Ileal Ulcers ���������������������������������������������������������������������������� 235
Amir Patel
x Contents

46 Concern About Pneumocystis jirovecii������������������������������������������������������ 237


Poonam Beniwal-Patel
47 Bloating in Crohn’s������������������������������������������������������������������������������������ 239
Poonam Beniwal-Patel
48 
Severe Diarrhea in a Patient on Chemotherapy�������������������������������������� 241
Amir Patel
49 Microscopic Colitis������������������������������������������������������������������������������������ 243
Poonam Beniwal-Patel
50 Miscellaneous Questions About IBD�������������������������������������������������������� 245
Daniel Stein, Salina Faidhalla, and Amir Patel

Part III Disorders of Gut–Brain Interaction


51 
Introduction to Disorders of Gut-Brain Interaction������������������������������ 253
W. Harley Sobin
52 IBS-D���������������������������������������������������������������������������������������������������������� 255
W. Harley Sobin and Patrick Sanvanson
53 IBS-C���������������������������������������������������������������������������������������������������������� 261
W. Harley Sobin and Patrick Sanvanson
54 Functional Dyspepsia�������������������������������������������������������������������������������� 267
W. Harley Sobin and Patrick Sanvanson
55 Bloating ������������������������������������������������������������������������������������������������������ 271
W. Harley Sobin and Patrick Sanvanson

Part IV Hepatology Compendium


56 Introduction to the Hepatology Compendium���������������������������������������� 281
W. Harley Sobin
57 Liver Enzyme Elevation-negative Work-up�������������������������������������������� 285
Francisco Durazo
58 
Increased LFTs with Increased Iron�������������������������������������������������������� 289
Jose Franco
59 
Increased LFTs with Low Ceruloplasmin ���������������������������������������������� 291
Jose Franco
60 Normal Ceruloplasmin-Suspected Wilson’s�������������������������������������������� 293
Jose Franco
61 
Increased LFTs-Alpha 1 Antitrypsin Deficiency������������������������������������ 295
Jose Franco
Contents xi

62 Suspected Gilbert’s������������������������������������������������������������������������������������ 297


Jose Franco
63 
The Significance of AST > ALT���������������������������������������������������������������� 299
Francisco Durazo
64 Marked Hyperbilirubinemia�������������������������������������������������������������������� 301
Kia Saeian
65 Suspected NAFLD�������������������������������������������������������������������������������������� 303
James Esteban
66 Treatment of NAFLD�������������������������������������������������������������������������������� 307
Francisco Durazo
67 Treatment of MASLD�������������������������������������������������������������������������������� 309
Achutan Sourianarayanane
68 
Alcohol and Liver Disease in Women������������������������������������������������������ 319
Veronica Loy
69 Alcoholic Hepatitis ������������������������������������������������������������������������������������ 323
Francisco Durazo
70 
Fatty Liver on Ultrasound with Normal LFTs���������������������������������������� 325
Francisco Durazo
71 
Evaluation for Liver Damage from Chronic ETOH Use������������������������ 327
Francisco Durazo
72 
Ascites in a Patient Without Obvious Cirrhosis�������������������������������������� 329
Jose Franco
73 Ascites in Cirrhosis of Unclear Etiology with Increased
Mononuclear Cells ������������������������������������������������������������������������������������ 331
Jose Franco
74 Low SAAG Ascites ������������������������������������������������������������������������������������ 333
Jose Franco
75 Banding Esophageal Varices �������������������������������������������������������������������� 335
Kia Saeian
76 Gastric Varices�������������������������������������������������������������������������������������������� 337
Kia Saeian
77 
Increased Ascites in a Stable Cirrhotic���������������������������������������������������� 341
Kia Saeian
78 
Complications Related to Managing Ascites ������������������������������������������ 345
James Esteban
xii Contents

79 Portal Vein Thrombosis in Cirrhosis�������������������������������������������������������� 347


James Esteban
80 
PVT and Budd Chiari as a Complication of UC������������������������������������ 351
Kia Saeian
81 
HCC Management, Solitary Lesion in a Child’s A Cirrhosis���������������� 353
Francisco Durazo
82 
Management of Unresectable HCC in a Cirrhotic �������������������������������� 355
Francisco Durazo
83 
HCC in NAFLD Without Cirrhosis���������������������������������������������������������� 357
Francisco Durazo
84 
Managing Primary Biliary Cholangitis �������������������������������������������������� 359
Juan Trivella
85 Advanced PBC ������������������������������������������������������������������������������������������ 365
Juan Trivella
86 Autoimmune Hepatitis������������������������������������������������������������������������������ 369
Francisco Durazo
87 Autoimmune Hepatitis������������������������������������������������������������������������������ 371
James Esteban
88 Primary Sclerosing Cholangitis���������������������������������������������������������������� 375
Juan Trivella
89 PBC-AIH Overlap�������������������������������������������������������������������������������������� 379
Juan Trivella
90 Immune-Tolerant Chronic HBV�������������������������������������������������������������� 381
James Esteban
91 Immune-Active Chronic HBV������������������������������������������������������������������ 385
James Esteban
92 HBV Treatment Discontinuation�������������������������������������������������������������� 391
James Esteban
93 Differentiating Acute HBV from an Acute Exacerbation
of Chronic HBV������������������������������������������������������������������������������������������ 395
James Esteban
94 
Acute HBV and Risk of HBV Reactivation �������������������������������������������� 397
James Esteban
95 Complicated HCV�������������������������������������������������������������������������������������� 401
Kia Saeian
Contents xiii

96 Acute Liver Failure������������������������������������������������������������������������������������ 403


Kia Saeian
97 
Delayed Presentation of Tylenol Overdose���������������������������������������������� 405
Kia Saeian
98 Candidates for Liver Transplant?������������������������������������������������������������ 407
Veronica Loy
99 
Frustrations with Waiting for a Transplant�������������������������������������������� 411
Veronica Loy
100 Complications Posttransplant������������������������������������������������������������������ 415
Veronica Loy

Index�������������������������������������������������������������������������������������������������������������������� 419
Contributors

Helmut Ammon Division of GI and Hepatology/Department of Medicine—GI,


Medical College of Wisconsin, Milwaukee, WI, USA
Poonam Beniwal-Patel Medicine, Gastroenterology/Hepatology Division,
Medical College of Wisconsin, Milwaukee, WI, USA
GI and Hepatology Division, Medical College of Wisconsin, Milwaukee, WI, USA
William Berger Division of Gastroenterology and Hepatology, Medical College
of Wisconsin, Milwaukee, WI, USA
Keely Browning Madison Medical Affiliates, Milwaukee, WI, USA
Department of Gastroenterology, Medical College of Wisconsin, Milwaukee,
WI, USA
Phillip Chisholm Department of Medicine, GI/Hep Division, Medical College of
Wisconsin, Milwaukee, WI, USA
Kulwinder Dua Division of Gastroenterology and Hepatology, Department of
Medicine, Medical College of Wisconsin, Milwaukee, WI, USA
GI and Hepatology Division, Dept. of Medicine, Medical College of Wisconsin,
Milwaukee, WI, USA
Francisco Durazo Division of Gastroenterology and Hepatology, Department of
Medicine, Medical College of Wisconsin, Milwaukee, WI, USA
Francis Edeani Division of Gastroenterology and Hepatology, Department of
Internal Medicine, Medical College of Wisconsin, Milwaukee, WI, USA
James Esteban Division of Gastroenterology and Hepatology, Department of
Medicine, Medical College of Wisconsin, Milwaukee, WI, USA
Salina Faidhalla Department of Medicine Division of Gastroenterology and
Hepatology, Medical College of Wisconsin, Milwaukee, WI, USA

xv
xvi Contributors

Jose Franco Department of Medicine-Division of Gastroenterology and


Hepatology, Medical College of Wisconsin, Milwaukee, WI, USA
Ben George Division of Hematology and Oncology, Department of Medicine,
Medical College of Wisconsin, Milwaukee, WI, USA
Jon Gould Department of Surgery, Medical College of Wisconsin,
Milwaukee, WI, USA
Veronica Loy Division Gastroenterology and Hepatology, Department of
Medicine, Medical College of Wisconsin, Milwaukee, WI, USA
Srivats Madhavan Department of Medicine, Division of Gastroenterology and
Hepatology, Medical College of Wisconsin, Milwaukee, WI, USA
Benson Massey Division of Gastroenterology and Hepatology, Department of
Medicine, Medical College of Wisconsin, Milwaukee, WI, USA
Ling Mei Department of Medicine, GI/Hepatology Division, Medical College of
Wisconsin, Milwaukee, WI, USA
Matt Mohorek Department of Gastroenterology, GI Associates,
Wauwatosa, WI, USA
Jim Nelson Department of Medicine, Division of Gastroenterology and
Hepatology, Medical College of Wisconsin, Milwaukee, WI, USA
Wilfredo Pagani Division of Gastroenterology and Hepatology, Medical College
of Wisconsin, Milwaukee, WI, USA
Amir Patel Dept. of Medicine, Division of Gastroenterology and Hepatology,
Medical College of Wisconsin, Milwaukee, WI, USA
Division of Gastroenterology and Hepatology, Department of Internal Medicine,
Medical College of Wisconsin, Milwaukee, Milwaukee, WI, USA
Krupa Patel Methodist Medical Group, Dallas, Texas, USA
Elizabeth Pieper Department of Gastroenterology and Hepatology, Froedtert
Hospital, Milwaukee, WI, USA
Kia Saeian Division of Gastroenterology & Hepatology, Medical College of
Wisconsin, Milwaukee, WI, USA
Patrick Sanvanson Division of Gastroenterology and Hepatology, Department of
Medicine, Medical College of Wisconsin, Milwaukee, WI, USA
Preetika Sinh Division of Gastroenterology and Hepatology Medical College of
Wisconsin, Milwaukee, WI, USA
Zachary Smith Division of GI and Hepatology, Department of Medicine, Medical
College of Wisconsin, Milwaukee, WI, USA
Contributors xvii

W. Harley Sobin Division of Gastroenterology and Hepatology, Medical College


of Wisconsin, Milwaukee, WI, USA
Achutan Sourianarayanane Division of Gastroenterology and Hepatology,
Department of Medicine, Medical College of Wisconsin, Milwaukee, WI, USA
Daniel Stein Division of Gastroenterology and Hepatology, Department of Internal
Medicine, Medical College of Wisconsin, Milwaukee, WI, USA
Juan Trivella Division of Gastroenterology and Hepatology, Department of
Medicine, Medical College of Wisconsin, Milwaukee, WI, USA
Part I
General GI
Chapter 1
Introduction to the General GI Section

W. Harley Sobin

In the General GI section, we start with the case-based management of complicated


eosinophilic esophagitis. The use of PPIs, budesonide, fluticasone, and dupilumab
are outlined. Dosage and prolonged duration of therapy are discussed. Management
of severe strictures and fibrostenotic disease is outlined.
Next is a case discussing the diagnosis and management of oropharyngeal dys-
phagia, including how to direct the correct questions in taking a history and how to
work with your speech therapist.
In reviewing achalasia, the author discusses the tendency to misdiagnose achala-
sia as GERD. He differentiates the management of types 1, 2, and 3 achalasia and
discusses the advantages of Heller myotomy and POEM. Diagnosis and manage-
ment of achalasia in the elderly are discussed, as are the diagnosis of pseudoachala-
sia and difficulties in diagnosing achalasia in someone on opioids.
Multiple examples of motility disorders of the esophagus including non-­achalasia
cases are provided. Some of the issues discussed include the evaluation of a patient
with chest pain and dysphagia; use of esophageal manometry in diagnosing motility
disorders; diagnosis and treatment of jackhammer esophagus and ineffective esoph-
ageal motility disorders; opioid-induced esophageal motility disorders; and the
diagnosis and treatment of corkscrew esophagus.
There are case examples of some of the intricacies in the treatment of GERD and
pH testing in the diagnosis and management of GERD. There is a review of various
options to treat refractory GERD, as well as the management of patients with func-
tional heartburn and reflux hypersensitivity.

W. H. Sobin (*)
Division of Gastroenterology and Hepatology, Medical College of Wisconsin,
Milwaukee, WI, USA
e-mail: [email protected]

© The Author(s), under exclusive license to Springer Nature 3


Switzerland AG 2023
W. H. Sobin et al. (eds.), Managing Complex Cases in Gastroenterology,
https://doi.org/10.1007/978-3-031-48949-5_1
4 W. H. Sobin

An esophageal surgeon, using case examples, discusses the complex decisions


about whether to operate in GERD. He reviews the benefits of the LINX and Toupet
procedures over the Nissen fundoplication; who is a candidate for magnetic sphinc-
ter augmentation (MSA), performing an MRI in someone with an MSA; surgical
management of GERD in the obese patient; and issues with transoral incisionless
fundoplication (TIF).
There are case examples of Barrett’s esophagus with and without dysplasia.
Surveillance guidelines in patients with Barrett’s with and without dysplasia are
discussed. Endoscopic modalities for the treatment of dysplasia are reviewed, as
well as the pros and cons of performing ablation, techniques of performing ablation,
and EMR of esophageal nodules. Newer techniques for screening and diagnosing
Barrett’s are outlined.
Case examples of problems in the management of PEG tubes are discussed
including: dealing with a PEG tube that won’t stop draining; PEG tube tracts that
won’t close; dealing with clogged G tubes and J tubes; techniques for placing a G-J
tube; controversies in placing a PEG in someone who has ENT cancer; avoiding
complications inserting PEG tubes; and how to manage those complications should
they occur. Issues with replacing a PEG tube are discussed, as are the management
of a prematurely pulled-out PEG tube and managing a buried bumper.
Case examples of the diagnosis of pernicious anemia (PA) are provided.
Pathophysiology of autoimmune metaplastic atrophic gastritis (AMAG), surveil-
lance for gastric cancer in AMAG, HP, and the development of environmental meta-
plastic atrophic gastritis (EMAG) are discussed, as are the distinctions between
EMAG and AMAG and techniques of gastric mapping and its use in PA, AMAG,
and EMAG.
In the section on subepithelial lesions and NETs, we provide case examples of
gastric submucosal lesions and how to determine their etiology. The management of
gastric GISTs, neuroendocrine tumors of the gut, and treatment of a small rectal
carcinoid are reviewed. There are three types of gastric carcinoids, and their evalu-
ation and treatment are outlined.
There is a chapter on viewpoints on managing common clinical GI disorders
from a practitioner with over 50 years of “real-world” experience. He discusses case
examples of difficulties managing: chronic constipation, IBS-D, gastroparesis,
GERD, microscopic colitis, fecal incontinence, and diverticulitis.
A case of severe SIBO and its management is discussed. Clinical findings, diag-
nosis, pathophysiology, predisposing mechanisms, and antibiotic choice are
reviewed. How should you manage a patient with SIBO where antibiotics are no
longer working?
A case of a patient with a functional GI disorder who has an abnormal breath test
is discussed. Problems with interpretation of breath tests, a comparison of glucose
and lactulose breath tests, and the benefit of adding scintigraphy to breath testing
are highlighted. The problem of frequent false positive breath tests is delved into
and explained. A strong case for limiting breath testing to patients who have a pre-
disposing reason to have SIBO is made.
1 Introduction to the General GI Section 5

Unusual causes of abdominal pain and controversies in the diagnosis of chronic


abdominal pain are the subject of another chapter. Case examples of difficult-
to-­make diagnoses in the determination of an etiology of chronic abdominal pain
are reviewed. These include acalculous cholecystitis, median arcuate ligament com-
pression syndrome, disorders of gut-brain interaction, narcotic bowel syndrome,
microscopic colitis, chronic mesenteric ischemia, sphincter of Oddi dysfunction,
endometriosis, chronic abdominal wall pain, epiploic appendagitis, porphyria, scle-
rosing mesenteritis, and abdominal migraine. Conflicts about ordering CT scans for
chronic abdominal pain are discussed. The peculiar circumstance of peptic ulcer
disease and cholecystitis first presenting as complicated disease in previously
asymptomatic elderly and RA patients is highlighted.
Case examples of patients with-acute mesenteric insufficiency, mesenteric vein
thrombosis, chronic mesenteric ischemia, non-occlusive mesenteric ischemia, colon
ischemia, and median arcuate ligament syndrome are reviewed. Difficulties in diag-
nosis and management are discussed.
There are case examples that describe the pros and cons of cold snare and hot
snare polypectomy. Other topics discussed in the chapter on polypectomy include
the use of cold snares for removing large laterally spreading granular tumors; choice
of snare in doing a polypectomy in a patient receiving anticoagulation; use of clips
in cold snare vs hot snare polypectomies; and whether or not to hold anticoagulation
or antiplatelet agents prior to elective colonoscopy. There is also a discussion of
injection technique and other recommended maneuvers in EMR, including a review
of obstacles encountered. The authors also discuss which polyps to send to the sur-
geon; endoscopic features that suggest deep submucosal invasion; management of
pedunculated polyps, including polyps with a thick stalk; and how to deal with post-­
polypectomy bleeding.
In a chapter on rectal incontinence, the authors provide case examples describing
the management of passive, urge, and seepage incontinence. They describe the best
techniques for rectal examination in the incontinent patient. They extensively dis-
cuss the evaluation of fecal incontinence and examples of etiologic causes. This
includes diagnosing fecal incontinence due to overflow and its management.
Managing diarrhea in fecal incontinence is extensively reviewed including the use
of biofeedback therapy, surgery, and sacral nerve stimulation in various patients
with incontinence. Incontinence in the post-stroke patient is discussed, as is manag-
ing the patient with intermittent involuntary seepage.
In the chapter on diverticulitis, we provide case examples discussing controver-
sies in the management of diverticulitis including whether all patients need to
receive antibiotics; how to manage smoldering diverticulitis; when to recommend
elective surgery; and how to avoid recurrent diverticulitis. The entities of symptom-
atic uncomplicated diverticular disease (SUDD) and segmental colitis associated
with diverticulosis (SCAD) and their management are reviewed.
There are case studies that demonstrate when (and how) to evaluate a pancreatic
cyst, including the use of EUS and FNA, the analysis of amylase and CEA in cyst
fluid, and the evaluation of pancreatic IPMNs. Other subjects reviewed include how
6 W. H. Sobin

to evaluate the patient with idiopathic recurrent pancreatitis, how to diagnose and
manage pancreas divisum, sphincter of Oddi dysfunction, and autoimmune
pancreatitis.
Case studies diagnosing and evaluating etiologies for chronic pancreatitis in
alcohol abusers and those without a history of alcohol use are reviewed. The utility
of the secretin-stimulated MRCP is discussed as are the management of chronic
pain in chronic pancreatitis and the management of pancreatic stones.
In a section on GI oncology, the author provides case studies demonstrating the
use of chemotherapy in colon cancer, Lynch syndrome, EGJ adenocarcinoma, and
pancreatic cancer. Different management for right-sided vs. left-sided colon cancer
is discussed. The different approaches to chemotherapy, immunotherapy, and surgi-
cal therapy in Lynch syndrome are reviewed. There is a wide-ranging discussion of
different responses to chemotherapy and immunotherapy in different GI cancers.
There are case studies reviewing the management of refractory benign esopha-
geal strictures, the use of stents in benign and malignant esophageal strictures, and
the role of stents in extrinsic malignant esophageal obstruction. Issues concerning
the diagnosis and staging of pancreatic cancer and the diagnosis of pancreatic can-
cer when preliminary tests are negative are discussed.
A PharmD specializing in GI pharmacology provides case studies analyzing the
use of GI drugs in treating EOE, HP, HBV, HCV, IBS, IBD, pancreatic insuffi-
ciency, and autoimmune hepatitis. Issues concerning the use of budesonide and
fluticasone are discussed. Obstacles with patients using quadruple therapy for HP
are outlined. Many more challenges are reviewed. Important drug-drug interactions,
drug-food interactions, drug toxicity, and insurance coverage are highlighted.
Chapter 2
Eosinophilic Esophagitis

Patrick Sanvanson

Case 1 A 28-year-old man presents with a food impaction after eating a turkey
sandwich. He has a history of food sticking over the last couple of years but has
always gotten it down by swallowing water. He tends to cut his food into small
pieces, but this time he swallowed a too large piece of meat. On endoscopy, he
appears to have a mildly narrowed esophagus; it has multiple rings and some lon-
gitudinal furrows. The piece of chicken is removed with a Roth net. Biopsies show
80 eos/hpf in the upper and lower esophagus.
He is placed on pantoprazole but feels like food continues to stick
intermittently.
Would you agree with that initial treatment and how would you pro-
ceed now?
Yes, it sounds like he has a classic presentation of eosinophilic esophagitis with
greater than 15 eosinophils per high-power field. Obviously, you have the other
components with the rings and the furrows, but those won’t necessarily make the
diagnosis of eosinophilic esophagitis (EOE).
The histologic diagnosis seems pretty clear for EOE, and the history of food
impaction is one of the more common presentations of eosinophilic esophagitis.
They placed him on a PPI right away, which I agree with. There is usually a delay
in diagnosis of EOE, and in this case, it sounds like he’s had symptoms going on for
the last couple of years.
After starting pantoprazole, his symptoms are not completely relieved. The ques-
tion I ask in patients with incomplete relief is as follows: Is he compliant with the
pantoprazole, and is he taking it correctly?

P. Sanvanson (*)
Division of Gastroenterology and Hepatology, Department of Medicine, Medical College of
Wisconsin, Milwaukee, WI, USA
e-mail: [email protected]

© The Author(s), under exclusive license to Springer Nature 7


Switzerland AG 2023
W. H. Sobin et al. (eds.), Managing Complex Cases in Gastroenterology,
https://doi.org/10.1007/978-3-031-48949-5_2
8 P. Sanvanson

I always like to bring these patients back into the clinic, to explain the disease
EOE, and to inform them that they have one of the more common complications,
which is a food impaction. I let him know that this is a medical condition that we
can manage but can’t necessarily cure and then to also give him insight into the
therapy itself.
So, he’s been placed on proton pump inhibitor therapy, and the question is as
follows: Why do EOE patients respond to this? Is it because you’re managing acid
reflux and that’s stimulating the disease, or is it because you’re blocking eotaxin-3
expression, which subsequently has an effect on eosinophil recruitment?
Obviously, PPIs have been used extensively to treat EOE, but this patient is hav-
ing inadequate relief. So, we question what dose he’s on, how long he’s been on the
medication, whether he is compliant with the medication dosing, and how is he tak-
ing the pantoprazole, before we conclude that he hasn’t necessarily responded to
PPI therapy.
Do you normally start with pantoprazole or a different PPI and what dose
do you like to start with?
It varies and is generally dependent on insurance coverage. Numerous PPIs have
been tried with EOE. If I’m using pantoprazole, I will usually start with 40 mg twice
a day. I usually err on the side of using high-dose PPI. This may be supra-­therapeutic,
but I just don’t want to underdose these patients. If I’m using omeprazole, I would
also use 40 mg twice a day. So, the key is that whatever PPI you choose, always
make sure to err on the side of the higher dose, just so you don’t underdose them
prior to their repeat endoscopy.
There have been studies looking at the potency of different PPIs, but these were
based on GERD data, not EOE data, so, we can’t be sure this extrapolates to
EOE. However, in GERD, pantoprazole is considered one of the least potent acid-­
suppressing PPIs, while rabeprazole and dexlansoprazole are the most potent.
Omeprazole and esomeprazole are in the middle.
.
The patient is taking pantoprazole 40 mg bid and has been taking it on an empty
stomach a half hour before meals for the past 8 weeks. While there might be mild
improvement, he is still complaining of dysphagia.
What would you do, would you repeat endoscopy, would you dilate the
esophagus, and would you start him on steroids?
In terms of symptoms, it’s always hard with EOE patients, because I feel like
their symptoms vary so much from person to person, and a lot of these patients no
longer know what normal is.
In making therapeutic decisions, symptoms can be confusing, and so I usually
like to guide my decision-making on endoscopic evaluation with biopsies. So, I like
to treat the patient for 8–10 weeks, and then, at that point I would go in and do
another EGD and get repeat biopsies of the esophagus.
I don’t typically dilate on that follow-up endoscopy. However, if there is an iso-
lated ring in the distal esophagus at the GE junction level, I will often dilate those
patients on the subsequent EGD. Also, if I am dealing with a narrow caliber
2 Eosinophilic Esophagitis 9

esophagus where the only scope I can advance is an ultrathin endoscope, I feel those
patients are at very high risk of food impaction, so, I will frequently do a bougie
dilation on the follow-up EGD.
However, if there is no severe narrowing, only a mild diffuse narrowing, where
the esophageal diameter is well above 12 mm, and they’re able to chew their food
and get it down, I will not dilate at the 8-week follow-up. In those patients, I will
give them more chances to respond to additional medical therapy. The reason I try
to defer dilation as long as possible is that we’ve seen many patients that have been
on medical therapy, and once you get their inflammation under control, the esopha-
gus starts to remodel and a lot of their stricturing disease just goes away. I’m not
necessarily hesitant to dilate their esophagus, but if it’s not absolutely necessary,
I’d rather avoid it. With any type of procedure, there are potential complications,
and in terms of dilating patients with EOE, the initial reports were concerning due
to an increased perforation risk. However, those initial concerns have been
mitigated.
However, it is clear that a lot of EOE patients may experience significant chest
discomfort and chest pain post-dilatation. So, if I’m going to dilate them, I warn
them ahead of time that if we do dilate you, you might experience some degree of
chest discomfort post-procedure.
However, not everyone is comfortable dilating EOE. You mustn’t be too aggres-
sive with dilation, start with lower caliber dilators, and then slowly move your
way up. Using these precautions, the risk is fairly low. You have to realize, in treat-
ing EOE that this is not a condition that is going to lead to cancer or shorten
someone’s life, so it is well worthwhile to be conservative, to go out of your way
to avoid a perforation. This is one of the reasons why I’m not too aggressive with
dilation.
How will the results of endoscopy and biopsy on that second EGD affect
your management plans?
Obviously, the main thing is measuring the response to therapy. So, you are look-
ing for histologic improvement, and you are evaluating endoscopic improvement
with regard to stricturing and narrowing. These are important, along with symptom
improvement as well.
With our biopsies, the goal, from a histologic standpoint, is to have essentially
zero eosinophils per high-power field, but typically, we accept less than 15 eosino-
phils per high-power field as an adequate goal for response. However, there are
some patients where I’ll start up PPI therapy after the initial endoscopy showed an
eosinophil count of 100/hpf, and they may go down to 14 eos/hpf with treatment,
and I say, okay, that’s an adequate response.
However, then you have subsets of patients that present with a low-grade eosino-
philia like 25 eos/hpf, and now they’ve down to 13 or 14. So you’ll wonder, if that
is really a significant response, or not, but usually we are also taking into account
any improvement in symptoms or endoscopic appearance.
If their response to therapy is suboptimal and the patient doesn’t have a decrease
in their eosinophil count to less than 15 eos/hpf, what do we do next? Then, you
10 P. Sanvanson

started talking about alternate therapies. Our other therapies involve making a deci-
sion between using topical steroids versus doing an elimination diet. As of May
2022, there is another therapy, the newly approved dupilumab.
So, I always give my patients the option of choosing between elimination diet
versus topical steroids. Unfortunately, none of the topical steroids are FDA-approved
therapies. The two most commonly used are budesonide and fluticasone which have
been found to be relatively equivalent in terms of clinical response.
Unfortunately, a lot of our guidance on steroid choice depends on cost. I don’t
necessarily have a favorite, budesonide, or fluticasone. I base it more on the cost to
the patient and the convenience of taking the particular regimen. From the stand-
point of topical steroids, the goal is to decrease the inflammation within the
esophagus.
If I’m using budesonide, I start at 1 mg twice a day. If we can get their insurance
to cover a compounded version of budesonide, it is much simpler for the patient to
use this, but, unfortunately, a lot of insurance companies don’t cover the com-
pounded version, which makes the use of budesonide much more difficult. Those
patients have to compound their own budesonide. They take respules of budesonide,
which are designed for use with nebulizers in asthma, mix the liquid medicine in
these respules with a high viscosity agent, usually sucralose or honey, and then
swallow the admixture. You can see how that’s pretty cumbersome for a lot of
patients to take and requires a patient who is very motivated and compliant.
The other option is fluticasone, which is administered by inhaler, but you have to
educate the patient to spray it in their mouth and not inhale it into their lungs. There
have been studies that show a number of patients make that mistake. So, it’s key to
educate patients on how to take fluticasone properly. From a dosing standpoint, I
recommend two puffs at 220 μg per puff, i.e., 440 μg twice a day. Occasionally, I
will go up to 880 μg twice a day in patients who are initial non-responders. Some
patients who are initial non-responders will respond to 880 μg bid. Once I start ste-
roids, I’ll usually wait 8–10 weeks then re-biopsy the esophagus and assess the
response.
The response rate to elimination diets is thought to be above 50%. Obviously,
this requires a very dedicated patient that they have to be extremely compliant with
the diet. It requires working closely with our GI dietician as well. When we start an
elimination diet, we also repeat endoscopy after each manipulation to gauge
response to the withdrawal of a particular food group. When we decide to reintro-
duce a particular food, we also repeat endoscopy to gauge response. l will explain
the elimination diet strategy to see if this is something the patient wants to pursue.
In addition to being on a strict diet, the other challenging factors are the cost and the
time required for multiple endoscopies to assess response.
In addition, our Wisconsin patients are not thrilled about the idea of restricting
wheat and dairy, the two major food groups that are frequently eliminated. I do have
a number of patients that are on elimination diets, but unfortunately these are the
select few who are very compliant people that are willing to put in the time and the
work to do the elimination diet. However, if somebody has some hesitancy, they feel
like they can’t do it, we try our other options.
2 Eosinophilic Esophagitis 11

Pantoprazole is continued at 40 mg BID. The patient opts to be placed on ste-


roids, and fluticasone is started at 440 mg bid. Repeat endoscopy with biopsy is
done 8 weeks later and shows histologic response. Symptomatically, the patient is
doing very well with much-improved swallowing.
How long will you maintain PPIs and steroids?
We have learned that these patients will need some type of long-term mainte-
nance of whatever therapy they’re on, as EOE is a chronic illness. We know that
when they stop the therapies most people will have a recurrence of the esophageal
eosinophilia and, then, subsequently, the scarring and other classic findings of EOE.
From a PPI standpoint, if they’re doing well on twice a day dosage, I’ll decrease
it to once a day and see how they do from a symptom standpoint. If they’re doing
fine, I won’t repeat the endoscopy, but if they’re not doing well, I may consider
repeating the biopsies.
Topical steroids are a little trickier. I usually keep them on some type of long-­
term therapy. However, we know that if you decrease the maintenance dose of
budesonide or fluticasone from their initial induction dose, then probably about
25–50% of people will have a recurrence of their esophageal eosinophilia. So, we
have some shared decision-making to decide whether to lower the dose. If patients
are happy, and they’re doing well on their current regimen, I give them the option of
maintaining their current dose of topical steroids without taper.
We do see some problems with steroid compliance since it’s a twice-daily regi-
men. If it’s very cumbersome for the patient, I may decrease it from twice a day to
once a day and see how they do, but we do know from randomized, placebo-­
controlled studies that a lot of these patients, essentially 50% or more, may have a
recurrence of their esophageal eosinophilia within 6-month time.
It sounds like you are keeping most people on these medicines indefinitely
Exactly, assuming that they’re getting some benefit from them, I always compare
it to blood pressure issues. Typically, when people have hypertension, unless you
get rid of some other factor, it becomes a lifelong management issue. With EOE,
there may be some things that we can modify, like diet, that may help maintain
remission. Unfortunately, most of the studies suggest a recurrence of eosinophilia
and recurrence of symptoms once you stop their maintenance therapy.
In which cases will you use dupilumab therapy?
Dupilumab is basically the first FDA-approved medication for EOE. It’s obvi-
ously a systemic therapy that should target some of these remodeling pathways and
inflammatory pathways and hopefully prevent fibrostenotic disease. There are a
number of other agents in the pipeline that are coming out as well.
Dupilumab is thought to work by inhibiting signaling of interleukin 4 and 13. It’s
a monoclonal antibody against IL-4 receptor. Historically, it’s been approved for
other disease processes, including atopic dermatitis, asthma, chronic rhinosinusitis,
and now EOE. The data are quite strong in terms of response of esophageal eosino-
philia, the classic endoscopic findings of EOE, and symptom improvement as well.
The population, for sure, that may benefit from this treatment are those who are
refractory to PPIs and steroids. Patients who find budesonide or fluticasone too
cumbersome to take may also be good candidates. Right now, we have to see where
12 P. Sanvanson

dupilumab lands in terms of insurance coverage. Dupilumab is given as one injec-


tion once a week. We do have a number of patients that are refractory to PPI therapy
and topical steroid therapy who will be considered for dupilumab therapy in the
near future.
What would be very interesting is to use dupilumab in our EOE patients who also
have eosinophilic asthma or atopic dermatitis. I think these patients should be per-
fect candidates, where you try to reduce the medications they’re on, save them
money, and hopefully give them back some more quality of life. In the long term, if
you can treat multiple illnesses with the same medication, it would be a significant
advantage.
So, I think that if patients fail to respond to combination therapy, then you should
at least introduce the idea of using dupilumab therapy.
Besides for dysphagia, what other symptoms do you tend to see in your EOE
patients?
I think there are variable presentations, but the other common complaint besides
dysphagia is heartburn. The hard part in that scenario is deciding whether the heart-
burn is due to the EOE or is it reflux-related. Sometimes, that requires additional
investigations to figure that out.
Another symptom we occasionally see is nausea, which is probably more com-
mon in the pediatric population. Our pediatric colleagues are more likely to see
EOE presenting with food intolerance and weight loss in their patients.
We occasionally see bloating, although not that commonly. Regurgitation is
more often seen with reflux disease than with eosinophilic esophagitis. However, if
someone has an EOE-related stricture, then they’re more likely to have esophageal
reflux and other related issues depending on the severity of the stricture.
However, once again, these EOE patients have varied symptoms that don’t nec-
essarily correlate with the severity of their endoscopic findings. Some patients have
a wide-open esophagus yet have severe symptoms, while other patients may have
close to a pinhole esophagus and will have essentially no symptoms. So, evaluating
symptoms is important but can be misleading.
Case 2 Another patient, a 32 year-old man, presents with a food impaction. He
has a known history of EOE but got lost to follow-up 10 years ago, shortly after
diagnosis. He has had progressive dysphagia which he has managed by cutting his
food into small pieces, avoiding tough meats, like steak, no salads. In spite of this,
food is frequently getting stuck, and now, he presents to the ER because a piece of
sausage got stuck and he was unable to vomit it out. On endoscopy, the esophagus
is quite narrow. The food stuck in the mid-esophagus is able to be removed. The
endoscope is unable to be passed all the way down the esophagus because it is
too narrow.
How do you manage a patient who has a long duration of untreated EOE
and has developed significant narrowing, significant fibrosis?
I’ve seen a number of patients that have had more fibrostenotic disease. It’s
presumed that there is more fibrosis because the disease has been persisting
2 Eosinophilic Esophagitis 13

untreated for a longer period of time. EOE starts as an inflammatory process, and
then over time, the inflammation progresses and develops into more and more
fibrotic disease.
I think the pathophysiology is probably fairly similar to patients with more acute
presentations. EOE starts as an immune response to some type of antigen stimulus,
whether it’s a food or whether it’s environmental, with probably some degree of a
genetic abnormality involved as well.
We do know that in a long term, some patients will develop a narrow caliber
esophagus, as a result of tissue remodeling and fibrosis. Subsequently, there is a
change in the mechanical properties of the esophagus, where fibrosis leads to issues
with decreased distensibility and dysmotility.
The medical treatment of these patients is similar to those with a normal caliber
esophagus. However, because of the severe narrowing of the esophagus, I may mod-
ify the particular medicine regimens that they’re on. I might give lansoprazole,
which is available as an oral dissolvable formulation. I may also prescribe a capsule
version of omeprazole, rather than the pill form, where you open up the capsule and
put the medicine in applesauce and swallow that just to make certain that they’re
actually able to get their pills down and get an adequate trial of the medication.
In addition, if there is severe narrowing, with an extremely narrow caliber esoph-
agus, I’ll frequently have to go down with an ultrathin endoscope and begin gradual
dilation over a guidewire. When I dilate, I am not too aggressive, starting off. I’ve
had patients that come in with a 5-mm esophagus. I may do a 21 French bougie
dilation, which is about 7 mm. Initially, we might tell them to stay on a liquid diet,
or more of a pureed type of diet, as opposed to swallowing solid foods. One of the
more important parts of education is to make sure that the patient takes his time with
eating, chewing his food extremely well, and drinking a lot of liquid in-between
bites as well.
If it is the very narrow esophagus, how often would you repeat dilation?
A stricture from EOE acts differently than a GERD-related stricture. In a tight
GERD-related stricture, we might bring them back weekly, but in an EOE-related
stricture I want to give their medications time to work. I feel that, with a lot of these
EOE patients, once you treat the inflammatory component of the disease with your
medication, the dilations are more effective, compared to long-standing peptic,
GERD-related strictures.
So, if they’re able to tolerate something liquid and perhaps other foods as well,
usually I’ll try to go 8 weeks between dilations. If, however, the patients contacting
me say they’re not doing well, they’re miserable, and everything is getting stuck,
then I’ll bring them back earlier, at 4-week intervals. However, I’ve never really had
to dilate them more frequently than that.
A part of the reason is that I just want to see what their response to therapy is. A
lot of times, patients are able to tolerate some type of diet once they’re on medical
therapy, they’re maintaining their weight, and they’re able to maintain their hydra-
tion. However, it’s also important with EOE patients that every time you see them
you try to confirm compliance with medication. I had a patient that came in with an
14 P. Sanvanson

extremely narrow caliber esophagus. I did six or seven dilations on him, and he just
wasn’t improving. He wasn’t responding to PPI therapy, and he wasn’t responding
to topical steroid therapy. We even tried switching his steroid formulations, and
there was still no improvement. Then, after multiple ED visits, he finally confessed
that he was not complying with taking his meds the way he was instructed. Once he
started complying with the medication instructions, there was rapid remodeling of
the esophagus and I was quickly able to dilate him sufficiently.
Chapter 3
Oropharyngeal Dysphagia

Patrick Sanvanson

A 78-year-old man with a history of Parkinson’s disease is complaining of dyspha-


gia. A swallowing evaluation suggests that he has oropharyngeal dysphagia.
Despite maneuvers by the speech therapist, he continues to have problems.
How do you manage patients with oropharyngeal dysphagia?
You always need to know the underlying pathophysiology of the dysphagia.
Is it stroke-related? Is it related to a neuromuscular issue, like ALS, or Parkinson’s
disease? We also see some patients that have head and neck cancer, and they’ve had
surgical therapy or radiation therapy that has resulted in dysphagia. So, you always
want to consider whether the underlying condition can be rehabilitated and whether
improvement is possible? If you’re dealing with ALS, you’re very limited from a
therapy standpoint because the disease progresses, which is often the case in
Parkinson’s disease as well. On the other hand, you have some stroke patients that
may have a significant degree of rehabilitative potential. When there are no obvious
explanations for the dysphagia, we may be witnessing idiopathic oropharyngeal
dysphagia, and these patients may present in a variety of ways with various rehab
potential.
However, whenever I have a patient who says they have difficulty swallowing, I
always ask them, “Do you feel like you have coughing when you swallow? Do you
ever feel like things go down the wrong pipe? Have you ever felt that you needed to
have the Heimlich maneuver, because you were unable to breathe when these epi-
sodes were occurring?”
There are other patients who may be completely asymptomatic but present with
aspiration pneumonia. Obviously, the most important thing that we want to avoid

P. Sanvanson (*)
Division of Gastroenterology and Hepatology, Department of Medicine,
Medical College of Wisconsin, Milwaukee, WI, USA
e-mail: [email protected]

© The Author(s), under exclusive license to Springer Nature 15


Switzerland AG 2023
W. H. Sobin et al. (eds.), Managing Complex Cases in Gastroenterology,
https://doi.org/10.1007/978-3-031-48949-5_3
16 P. Sanvanson

with these patients is aspiration, because that can lead to significant morbidity and
mortality. Oropharyngeal dysphagia is a very common disease, with significant
costs to the healthcare system. In evaluating these patients, I think the thing that's
key is to have a close relationship with your speech therapist and to understand what
the patient’s video swallow study shows.
So, if you’re worried that a patient has oropharyngeal dysphagia, the typical first
approach will be to do a video swallow study to see if they have any penetration
above the vocal cords or if they have any aspiration. Then, if they do, what is the
consistency of the food that is aspirated? Do they have liquid aspiration? Do they
have aspiration of pudding? Do they aspirate thicker liquids? Do they have aspira-
tion for solid foods as well? And sometimes, from that standpoint, initial manage-
ment is to try to determine if there are certain foods that can be tolerated without
aspirating. If so, the speech therapist can work with the patient to see if they can
modify the consistency of some of the other foods. If the patient aspirates liquids,
then they can be taught to thicken up the liquid that they’re taking in. If they cannot
swallow any foods without aspirating, then you have to talk about whether or not
alternate delivery of nutrition is necessary, including enteral feeding or G-tube
placement to get you through the initial phases of the oropharyngeal dysphagia.
The speech therapists are good at teaching certain maneuvers that patients can do
to limit their aspiration risks. One of these is the Shaker exercise, which requires a
head lift, to try to strengthen some of the pharyngeal swallowing musculature. This
maneuver can also help with upper esophageal sphincter opening. Obviously, the
limitation of many of these exercises is that you have to have a patient who is capa-
ble of following instructions. If the patient is neurologically devastated, that's less
of an option.
Also, if you have a patient with restricted neck mobilization, because they have
had a big spinal surgery or other type of neurosurgery, this also limits your ability to
do certain swallowing exercises, including the Shaker exercises. Then, there are
things down the pipeline that hopefully will address some of these issues. We are
doing research protocols in our laboratory, working with a swallow resistance exer-
cise device, which is a pressure collar that people wear. Patients basically practice
swallowing against resistance. In some of the initial data, we're able to improve
aspiration and penetration in a subset of individuals who wear this collar, including
some with Parkinson’s disease. These are all areas of study where hopefully we can
modify the swallowing mechanism, find a way to strengthen their musculature to
delay the disease progression, and prevent or delay aspiration.
However, in patients who have persistent aspiration, we consider PEG tubes.
Although PEG tubes are not necessarily the best thing in demented patients that
have oropharyngeal dysphagia, they may be options for patients that have ALS,
Parkinson’s disease, and some stroke patients to hopefully maintain their nutri-
tional status.
Chapter 4
Achalasia

Benson Massey

Case 1 A patient is referred to your surgeon by a community gastroenterologist


requesting a fundoplication for chronic gastroesophageal reflux. The gastroenter-
ologist thinks the surgeon should consider doing a Toupet procedure because his
patient, in addition to having chronic reflux esophagitis, has ineffective esophageal
motility. The surgeon, concerned about the presentation, is asking you to review the
case, wondering if he has been misdiagnosed.
This is a 40-year-old man with a long history of heartburn and regurgitation.
This has been going on for a couple of years and is getting worse. He has lost about
ten pounds over the past couple of months. The patient frequently has regurgitation
after eating, and over time, the patient has transitioned to a softer and more liquid
diet. He avoids eating for several hours before bedtime. In fact, he occasionally
induces vomiting before going to bed because, on occasion, he has woken up in the
middle of the night regurgitating liquid, occasionally solid material.
His gastroenterologist did an EGD that showed some retained liquid and super-
ficial ulceration in the distal esophagus, all of which he interpreted as reflux esoph-
agitis. The GE junction was somewhat tight, which he interpreted as a reflux
stricture. He dilated the stricture using a TTS balloon (to 18 mm), with minimal
symptomatic improvement. The patient has been taking PPIs long term, and yet his
symptoms keep getting worse.
The gastroenterologist has previously sent patients to this surgeon for anti-reflux
surgery with great results. He knows that the surgeon likes to have manometry done
beforehand. His GI practice still uses older water-infusion manometry catheters,
and his experience doing manometry is somewhat limited. He goes ahead and

B. Massey (*)
Division of Gastroenterology and Hepatology, Department of Medicine, Medical College of
Wisconsin, Milwaukee, WI, USA
e-mail: [email protected]

© The Author(s), under exclusive license to Springer Nature 17


Switzerland AG 2023
W. H. Sobin et al. (eds.), Managing Complex Cases in Gastroenterology,
https://doi.org/10.1007/978-3-031-48949-5_4
18 B. Massey

performs the procedure, after placing the manometry catheter without using fluo-
roscopy. His interpretation of the study is that there is clearly decreased pressure in
the esophagus with limited, if any peristalsis. He is unable to detect a lower esopha-
geal sphincter zone. He interpreted this to mean that the LES was nonfunctional,
blown-out.
When the patient comes to see you, he brings the manometry tracing which you
review. Looking at the tracing, you suspect that the probe never passed the LES and
never entered the stomach. Without being able to evaluate the LES, you are not sure
if you are dealing with GERD in a patient with ineffective motility or if the patient
may have Type 1 achalasia.
How often do you see patients being diagnosed with GERD, who actually
have achalasia?
First, it can be very difficult to distinguish GERD from achalasia by history, and
there is a lot of overlap, particularly in early achalasia. We use dysphagia for solids
and liquids as a marker of achalasia, but early on in achalasia there may just be
dysphagia for solids. This may be very hard to distinguish from the symptoms of
someone with a peptic stricture or even something like eosinophilic esophagitis. So,
the symptom overlap may confuse the diagnosis early on.
Second, because, achalasia is several orders of magnitude less frequent than
GERD, anyone who’s coming in with esophageal symptoms, even if they seem a
little weird, like dysphagia for solids and liquids, they're still more likely to have
GERD than they are achalasia, just a priori without doing any investigation at all.
Interestingly, the reliability of the clinical history for diagnosing GERD is poor.
If you look at the Diamond study [1], done some years ago, which looked at primary
care doctors trying to determine whether a patient had GERD based on symptoms,
their decision-making was not much better than flipping a coin (sensitivity and
specificity of 63%, based on the history when you’re using actual acid exposure as
the gold standard). Gastroenterologists don’t do much better. The sensitivity and
specificity of their history are about 67% and 70% which still mean that about a
third of the time they got it wrong [1].
So, with GERD and achalasia both being tricky diagnoses, unless someone pres-
ents with advanced achalasia where they’re having really obvious features, such as
an esophagus that is massively filled with liquid in a manner that really gets your
attention, I think it’s really a bit of a challenge.
There are a few clinical clues that may help distinguish the two. First, the patient
described here is regurgitating. When you have achalasia and you regurgitate, the
stuff tastes the same coming up as when it goes down. When GERD patients regur-
gitate, it will taste sourer, unless the patient is on vigorous PPI therapy for reflux, in
which case the regurgitation may not taste acidic.
An endoscopic technique to help determine whether achalasia is present is to
inflate the esophagus with a lot of air, watch if the GEJ opens up, and then inflate
the stomach with a lot of air and retroflex and see if the GE junction opens up. In
achalasia, the GEJ stays shut; in GERD, it tends to open wide. In fact, if you look
from below, in retroflex, it’s hard to keep the stomach insufflated in GERD
because so much air is leaking back up through the opening (that’s only in very
4 Achalasia 19

end-stage conditions like systemic sclerosis, where there is a very hypoten-


sive LES).
So, it’s a pretty accurate sign if you see the GEJ opening up, and the patient is
belching air, that the patient doesn’t have achalasia (if they’ve not already had
surgery).
Another thing you can do, if you’re unsure whether you’re dealing with GERD
or achalasia, is to combine your endoscopy with a BRAVO study. If your EGD is
unrevealing and the Bravo returns negative, then you’re not dealing with GERD,
and you have to really start thinking about another diagnosis like achalasia.
You decide to repeat the manometry yourself. You pass the probe into the stomach
with fluoroscopic guidance. On the tracing, you note absent peristalsis, but the IRP
is only borderline elevated, with some decreased relaxation. You still think this is
likely achalasia, but you want more convincing evidence.
What other tools can help define whether the patient has achalasia?
When you’re dealing with someone with absent contractility, the question is as
follows: Is this achalasia or is this a scleroderma-like dead esophagus? Is that pos-
sibly borderline high IRP in supine swallows really due to high deglutitive residual
tone in the LES? To help determine whether an elevated IRP may be clinically
important, you should also do the manometry sitting up as well as supine. If the IRP
is normal on swallows while sitting, the high IRP during supine swallows should be
disregarded.
When the IRP is also elevated during sitting swallows, there can still be a con-
cern that this reflects muscle activity of the diaphragmatic crura (particularly in
patients with obesity or lung disease) or could reflect a structural problem (e.g., a
paraesophageal hernia or tight fundoplication wrap). This is where provocative
pharmacologic testing during manometry using amyl nitrite can help with the diag-
nosis. Amyl nitrite completely abolishes smooth muscle tone in the esophageal
body and LES for around 30 s. If the nadir pressure at the esophagogastric junction
after amyl nitrite falls over 10 mmHg lower than the IRP on swallows, then you
have impaired LES relaxation. If there is really no change with amyl nitrite, then the
cause of the pressure is not smooth muscle tone in the LES.
Do you rely on the timed barium esophagram to help make a diagnosis of
achalasia?
No, I don’t think that the timed barium esophagram is as helpful in making a
diagnosis of achalasia as it is in measuring the response to therapy. The timed bar-
ium esophagram can show you a delay in esophageal emptying, but it can’t differ-
entiate whether the delay is due to decreased esophageal contractions or to EGJOO
or a combination of the two. We feel we get much more information by doing an
impedance study at the time of manometry. Using impedance, if liquid is not empty-
ing normally, I can determine whether it relates to decreased squeeze of the esopha-
geal body, or increased resistance at the GEJ, or elevation of gastric pressure (in an
obese patient there is elevation of intragastric pressure that may delay esophageal
emptying). I can also assess the height of the fluid column remaining in the esopha-
gus at different times, when the patient is in a seated position.
20 B. Massey

After you evaluate your manometry and impedance studies, you are con-
vinced the patient has achalasia. How do you approach the management of the
different types of achalasia?
For patients with spastic achalasia, the type 3 achalasia, we often try pharmaco-
logic therapy first, a calcium channel blocker, like nifedipine or sublingual hyoscya-
mine, and many of them will get relief from pharmacology alone. In a number of
these patients, the dysphagia is caused more by spastic contractions than outflow
obstruction. We find that a number of these patients will respond to low-dose sub-
lingual hyoscyamine. Even though it doesn't normalize the motility, they get symp-
tomatic relief without needing to do an intervention on the GEJ and spastic
contractions in the esophageal body.
In type 2 achalasia, I would not try pharmacologic therapy because it will
decrease pan-pressurization and you need that pressure to help empty the esophagus.
I would also avoid doing a POEM in type 2 because you still have intrinsic motil-
ity that can help empty the esophagus in a non-peristaltic fashion, and you destroy
that with a POEM. When you’re doing a POEM, you are essentially creating a
scleroderma-like esophagus, and you’re getting rid of all motility. What you need in
a type 2 is to get rid of the EGJOO either with a Heller myotomy or pneumatic dila-
tion. There is usually enough remaining intrinsic esophageal motility that the reduc-
tion in LES tone produced by pneumatic dilation should be sufficient to relieve
symptoms in type 2. However, the reduced esophageal body contractility in type 1
(end-stage achalasia) usually requires a more complete ablation of the LES tone,
which necessitates a myotomy.
In type 1, I am worried less about a POEM ablating esophageal body contractil-
ity, because it is already essentially absent. However, the concern for increased
reflux after POEM remains.
People talk about pan-pressurization (in type 2 achalasia) as if it is a bad thing. I
think it is a good thing, a driving force that can help to empty the esophagus. So, if
the esophagus is pan-pressurized to 30 mm of mercury, and you can reduce the pres-
sure at the LES to 10 mm of mercury, now you’ve got a 20-mm mercury driving
pressure to empty out the esophagus. The difficulty comes when there’s not suffi-
cient esophageal squeeze pressure and then you have to really obliterate all the tone
at the LES, allowing things to empty by gravity.
However, when you obliterate all tone at the GEJ, you also knock out your anti-­
reflux barrier. With a Heller, you can restore some of that by also performing a Dor
or Toupet anti-reflux procedure, but with a POEM you can’t. I have some concerns
about the POEM long term. I think there’s the potential for a lot of reflux damage
that can accumulate over time, including Barrett’s esophagus and possibly associ-
ated neoplasia.
One of the things to watch out for post-achalasia treatment is that patients will
feel great when you see them 1 or 2 months after their initial therapy. Some gastro-
enterologists mistakenly tell their patients not to come back unless they’re having
problems. That’s not a good idea. Many patients, especially after a POEM, will have
significant reflux. Many patients will put on a lot of weight since they’re able to eat
a lot better now that their obstruction is relieved. As a result, they develop even more
4 Achalasia 21

reflux related to weight gain, and many will develop reflux esophagitis and even
reflux strictures. Also, pills are more likely to lodge unawares in the esophagus of
these patients, so they are more prone to pill injury from NSAIDS, etc. Therefore, I
think you have to monitor these patients long-term and continue to re-educate them
regarding potential problems.
I also think that with the surgical myotomy in type 2 achalasia, the surgeons should
try to disrupt as little of the muscle in the esophageal body above the LES as possible.
There is an understandable tendency to err on the side of cutting higher into the
esophageal body, to avoid the disaster of an incomplete myotomy of the LES. We’re
starting to see an increased number of patients who have a blown-out esophagus,
years after their surgery because too much esophageal muscle has been destroyed.
Case 2 An 82-year-old woman is referred to you for dysphagia. She has had dys-
phagia for 6 months that has been getting progressively worse. It was initially for
solids, but now she has dysphagia for liquids as well. Her community gastroenter-
ologist did an EGD that showed a corkscrew esophagus. The LES was somewhat
tight and seemed to have a birds-beak appearance. The scope easily passed the zone
of resistance at the LES and popped into the stomach. He did a TTS dilation of the
LES, and the swallowing improved for about a week, and then, swallowing got worse.
He suspects the patient may have a variant of achalasia and wants your opinion.
He thinks she probably is a candidate for Botox. While he thinks a pneumatic dila-
tion would help her, he is reluctant because of her age and asks your opinion.
You examine the patient. She is somewhat frail and fairly immobile. She is reluc-
tant to have esophageal manometry.
Would you ever inject Botox without a manometry documenting achalasia?
I don’t want to empirically treat someone for achalasia without manometry.
There are different presentations and different types of achalasia. Some have spastic
contractions, and some have a dead and scleroderma-type esophageal body. You
manage these differently.
For the patient who complains that they don’t want to go through an esophageal
manometry because they are afraid of the catheter placement being painful, we will
place the catheter at the time of endoscopy while they are sedated, so it should not
be a problem.
Do you ever see 80-year-old patients presenting with a new diagnosis of
achalasia?
I do see achalasia being diagnosed in patients who are in their 70s and 80s. Of
course, many patients have had their disease for years before the diagnosis is made.
I’ve seen some patients where I have looked back in their old X-ray jacket and saw
a dilated esophagus on a CAT scan done years before symptom onset that was not
appreciated at the time.
Of course, when you diagnose achalasia in elderly, frail patients there is a reluc-
tance to send them to surgery because of the higher risk of postoperative complica-
tions. They’re also reluctant to have the patient get a pneumatic dilation for the same
reason, because there’s a 2% risk of potential perforation. So, you inject Botox and
the patient does great.
22 B. Massey

However, when you decide to treat the older patient with Botox because it is less
invasive are you just delaying the inevitable? They do great for, at best, a year, and
then you inject them again. After a while, you’ve injected a lot of Botox down there,
the patient develops antibodies, and the Botox stops working. In addition, all that
Botox may or may not cause fibrosis which surgeons aren't too happy about.
So, if the patient survives long enough to outlast the Botox what are you going to
do? You’ve got to do something, but it’s going to be at higher risk. When you decide
whether you should start Botox on an elderly patient, that is a challenging decision
that your present self has to make, that your future self is going to have to live with.
So, they’re now five years older, and the Botox is not working. You have to do some-
thing more substantial, but now they’re only a poorer surgical candidate.
In this case the patient received a TTS dilation that helped very briefly.
What do you think about doing an empiric TTS dilation in a patient with
achalasia?
I actually think it is a good idea. A number of patients with achalasia also have a
stricture at the GEJ. Empirically, dilating the stricture can improve their dysphagia,
and some of them may not need further treatment for their achalasia for a long time.
It may be hard to tell that they’ve got a stricture down there, and you can’t see the
stricture because the LES isn’t relaxing. However, I do have any number of achala-
sia patients who, in addition to their achalasia, have a stricture at the GE junction.
You won’t know it unless you pass a 60 French bougie across there or inflate a TTS
20-mm balloon.
I’ve had some patients with achalasia that probably had the condition for 10 or
20 years without bothersome symptoms, and then, they get a stricture, and that’s
the thing that tips them over. If I dilate the stricture, they can live with the achala-
sia. I wouldn’t do that in a 20-year-old, but a 70-, 80-year-old, if you can find a
stricture and dilate it, and they get symptomatic relief I wouldn’t treat their acha-
lasia any further, although they still need to be followed and their symptoms
monitored.
We don’t really know how Botox works to relieve symptoms in achalasia. We
assume that it works on the motility of the sphincter, but is it possible that it works
more on the sensory component? I had a guy this last year who was in his nineties
with well-documented achalasia. We treated him with Botox, and his symptoms of
dysphagia and regurgitation got dramatically better, but then we noticed that he also
had iron deficiency anemia. I’m not too excited about working up iron deficiency
anemia in a 90-year-old, but then, 2 or 3 weeks later he started having black stool.
He was on an antiplatelet agent for heart disease, and his blood count was dropping
more. So, we felt we had to do a colonoscopy, but we didn’t find anything. So, what
was causing this dark stool? Did he have something concerning down in his
small bowel?
At that point, he’s saying his swallowing is doing great, because of the Botox,
and so we decided to do a capsule endoscopy, which he swallowed without diffi-
culty. It was a 12-h capsule, and then, we went to read the images and the capsule
sat in the esophagus for 11 h. Then, it finally came out, but he felt no dysphagia at
all. Clearly, the esophagus wasn’t emptying correctly, and he just didn’t feel it.
4 Achalasia 23

Case 3 A 64-year-old woman with a history of small cell lung cancer that has been
in remission is referred with dysphagia. Dysphagia started about a month earlier
and rapidly worsened. She is now having marked difficulty swallowing solids and
liquids.
A CT scan shows a mildly dilated esophagus but no suggestion of metastatic
disease in the region of the esophagus or stomach. An EGD is performed that shows
a birds-beak-type LES with increased resistance, but eventually the scope success-
fully pops through. No lesions are seen in the esophagus and cardia, etc. A TTS
dilation is performed without prolonged clinical benefit. You suspect the patient has
pseudoachalasia.
What are some of the markers that make you suspect pseudoachalasia?
Historical markers for pseudoachalasia include older age, more rapid onset of
symptoms, and more substantial weight loss. A clue during endoscopy is that it may
be more difficult to get the scope through the GEJ. In idiopathic achalasia, the GEJ
may not open with air insufflation alone, but you can easily advance the scope
through the sphincter into the stomach. One of the markers of pseudoachalasia is
significant difficulty advancing the scope into the stomach. There may be tumor
infiltrating the GEJ which may be difficult to visualize.
Are there manometric distinctions that can help differentiate between idio-
pathic achalasia and pseudoachalasia?
Idiopathic achalasia is very uncommon, pseudoachalasia is extremely uncom-
mon, and pseudoachalasia as a paraneoplastic phenomenon is extremely rare. To tell
you whether there are manometric findings that can help distinguish pseudoachala-
sia from achalasia is difficult because there are so few cases to evaluate. It might be
possible there will be different responses to pharmacologic provocative maneuvers,
but at this point we really have insufficient data to tell, and there are so few cases
out there.
Case 4 A 43 year-old man presents to his community gastroenterologist with dys-
phagia of one-month duration. He has dysphagia for solids and liquids. He has
been taking high doses of Percocet for months after a back injury. The gastroenter-
ologist does an EGD that suggests achalasia. He does a manometry that is consis-
tent with achalasia. He sends the patient to you because he refers all his pneumatic
dilations to you.
Do you approach the diagnosis of achalasia differently in patients on
opiates?
If you look at patients with type 3 achalasia and patients with EGJOO, there is an
increased number of people in these cohorts who take high doses of narcotics. If you
look at their responses to pharmacologic provocative testing during manometry, the
responses to amyl nitrite and CCK are different in patients who have opiate-­
associated achalasia from those patients with idiopathic achalasia.
We only see this clinical picture in patients who are taking high doses of narcot-
ics, at least 30 milliequivalents of morphine daily, not just one or two Percocet a
day. Even before we knew about the connection of achalasia with opiates, we
24 B. Massey

realized that patients on opiates did not get as good a result with achalasia therapy
as patients who were not on opiates. The symptomatic outcomes were especially
poor in people who appeared to have type 3 achalasia who were treated with abla-
tive therapies.
On the other hand, we have seen that if you can get these patients completely off
opiates, many of them will get total relief from their symptoms. So, that becomes
the goal.
For whatever reason, we are seeing fewer of these patients than before, perhaps
because community gastroenterologists have been made aware of this connection
and they’re explaining to their patients that the opiates are the problem. Anyway,
when we see patients like this, we will not refer them for surgery until they’ve gone
off narcotics. If they do that, symptoms will resolve in most patients.
One thing about my approach to treating opiate-induced esophageal dysmotility
is that I don’t want to be overly aggressive in patients who have symptoms that are
simply a nuisance and not a threat to life (e.g., not having substantial weight loss or
recurrent pneumonias). I am loath to recommend a POEM to someone whose condi-
tion is potentially reversible upon stopping opiates or dropping to a lower dosage,
because the POEM creates a scleroderma-like esophagus, something that can’t be
reversed.

Reference

1. Dent J, Vakil N, Jones R, Bytzer P, Schöning U, Halling K, Junghard O, Lind T. Accuracy of


the diagnosis of GORD by questionnaire, physicians and a trial of proton pump inhibitor treat-
ment: the Diamond Study. Gut. 2010;59(6):714–21.
Chapter 5
Other Motility Disorders of the Esophagus
and Achalasia

Francis Edeani

Case 1 The patient is a 35-year-old woman who presents with a history of


­occasional chest pain which is clearly related to meals, definitely worse when
­eating. The pain is described as an ache or a cramp. She also has the sensation of
food sticking intermittently, both solids and liquids. Her symptoms are exacerbated
when she is stressed. Her physical exam is normal. An EKG is normal. She is treated
with PPIs for one month without relief.
How do you evaluate a patient with chest pain and dysphagia during
swallowing?
Dysphagia to solids and liquids suggests a motility disorder of the esophagus.
We generally start with the simplest test—an esophagram. This is noninvasive, pro-
vides a “roadmap” and can give a general idea about esophageal motility, shows the
presence of strictures or rings, and may provide some information about the muco-
sal lining. Challenge with a barium tablet or marshmallow can sometimes reproduce
the patient’s symptoms and can demonstrate stasis of the food bolus.
The gold standard test for esophageal motility disorders is esophageal manome-
try. EGD is typically performed as a complementary test to assess for lesions due to
reflux esophagitis, eosinophilic esophagitis, infectious esophagitis (such as CMV,
HSV, candida in immunocompromised patients), esophageal strictures, rings or
webs, and malignancy. A tight esophagogastric junction during EGD may suggest
the presence of an esophageal outflow obstruction motility disorder. If the EGD
with biopsies is normal, esophageal manometry is performed. In patients with high
probability for esophageal motility disorder as determined by findings on

F. Edeani (*)
Division of Gastroenterology and Hepatology, Department of Internal Medicine,
Medical College of Wisconsin, Milwaukee, WI, USA
e-mail: [email protected]

© The Author(s), under exclusive license to Springer Nature 25


Switzerland AG 2023
W. H. Sobin et al. (eds.), Managing Complex Cases in Gastroenterology,
https://doi.org/10.1007/978-3-031-48949-5_5
26 F. Edeani

esophagram (bird-beak appearance, esophageal spasms, dysmotility, etc.), a


manometry catheter can be placed during EGD. Esophageal dilation should be per-
formed if a ring or stricture is present.
Aren’t the manometry results altered by the sedation?
Earlier studies raised concerns about the potential impact of sedation on esopha-
geal motility. However, recent studies have failed to show any significant differ-
ences in esophageal motility between post-sedated and unsedated studies.
We avoid the use of topical lidocaine spray to numb the oropharynx because this
may affect safe swallowing during manometry and may potentially affect esopha-
geal motility.
In this case, the EGD was normal, and biopsies returned negative for EOE. The
manometry revealed a normal pressure LES that relaxed normally, with an IRP of
8. Esophageal peristalsis was coordinated, but contractions were of increased
amplitude. Out of ten swallows, the distal contractile integral (DCI) was over 8000
on four swallows with the highest reaching 10,000.
The DCI represents the vigor of esophageal contractions based on pressure mea-
surements from multiple sensors across the body of the esophagus. The IRP is a
measure of LES relaxation pressure computed by measuring the pressures of any 4 s
with the lowest pressures over a 10-s window.
Findings in this case are consistent with Jackhammer esophagus. The Chicago
Classification v 4.0 states that if at least 20% of swallows have a DCI > 8000, this
is consistent with Jackhammer esophagus. In this case, 40% were higher than 8000.
In Jackhammer, the forceful contractions produce luminal occlusion which can
cause dysphagia and chest pain.
How does Jackhammer differ from nutcracker esophagus and esopha-
geal spasm?
Nutcracker is a terminology that isn’t used anymore. In the past, nutcracker
esophagus was used to refer to patients who had multiple swallows with a
DCI > 5000 but <8000. However, subsequently it was noted that many asymptom-
atic individuals have DCIs > 5000, so this term has been removed from the Chicago
classification.
Esophageal spasms, which are non-peristaltic contractions, are characterized by
short distal latency, defined by the time from initiation of the swallow to the bolus
reaching the distal esophagus in less than 4.5 s.
In these cases, the lower esophageal sphincter may not have enough time to relax
before the bolus arrives and the food bolus gets stuck, which can cause chest pain
and/or the sensation of dysphagia. This rapid emptying is also seen in patients with
type 3 achalasia where most contractions are spastic.
How do you treat patients with Jackhammer esophagus?
Jackhammer esophagus represents hypercontractile peristalsis, and so, we use
medications that cause smooth muscle relaxation. We typically start with pepper-
mint oil and, if ineffective, calcium channel blockers. Patients on calcium channel
blockers should be monitored for side effects such as hypotension, lightheadedness,
and headache. For peppermint, we recommend two Altoids three times a day, before
meals. If this is ineffective, we can add hyoscyamine 2.5 mg (two tablets) SL before
5 Other Motility Disorders of the Esophagus and Achalasia 27

each meal. If that doesn’t work, we may go to diltiazem 30–90 mg 4 times daily.
Nifedipine has been used in the past at a dose of 10 mg three times daily 20 min
before meals, but sublingual formulations should be avoided as this can cause an
exaggerated drop in blood pressure. If symptoms improve or resolve with calcium
channel blockers, we decrease the dose to once daily or take only as needed.
For patients who do not improve with the above, a trial of neuromodulators is
recommended. We recommend a low-dose tricyclic antidepressant such as amitrip-
tyline or nortriptyline, taken at bedtime. Symptoms are reassessed after 3 months.
Are Botox injections useful?
Botox may be useful for hypercontractile and impaired relaxation disorders of
the lower esophageal sphincter but is not used for hypercontractile disorders of the
esophageal body. We don’t use Botox injections along the length of the body of the
esophagus like we do with the LES in achalasia. In achalasia, we are simply knock-
ing out the LES. In Jackhammer, we can’t afford to knock out all peristalsis by
injecting along the length of the esophagus; we might go from having too strong
contractions to no contractions and create a more serious problem. Patients with
Jackhammer esophagus may improve over time allowing for discontinuation of
smooth muscle-relaxing medications. For the same reason, POEM is not being used
in Jackhammer esophagus.
Case 2 A 45-year-old woman is referred for difficulty with swallowing that has
been present for 6 months. She is aware of the sensation of solid food sticking and
liquids going down slowly. The symptoms are slowly getting worse. She has no
smoking or drinking history. She has been getting increased heartburn. An EGD is
done which appears normal, and biopsies return negative for EOE. A timed esopha-
gram shows mild-to-moderate esophageal dysmotility. Manometry reveals a normal
LESP with normal relaxation, with an IRP of 9. However, esophageal contractions
show low amplitude.
How do you approach a patient with dysphagia and low-amplitude
contractions?
This is a very common finding when we do manometry to investigate dysphagia.
In patients with a LES that relaxes appropriately, we divide swallows of low
amplitude into weak contractions, where the DCI is <450, and failed contractions
where the DCI is <100. The diagnosis of ineffective esophageal motility (IEM) is
used if 50% or more swallows have a DCI < 100, or 70% have a DCI < 450. The
most recognized finding with IEM is gastroesophageal reflux. The lack of effective
esophageal contractions interferes with the clearance of gastric refluxate, as well as
the ingested bolus.
While ineffective or absent contractility is seen in scleroderma, this is not
really part of the differential for our patient, who has a normal LES. In sclero-
derma, the LES is hypotensive at baseline, predisposing to severe reflux. Ineffective
esophageal motility can occasionally be seen in other mixed connective tissue
diseases.
Occasionally, we are asked whether GERD produces decreased esophageal
motility. There is no evidence to support this. GERD can be associated with
28 F. Edeani

dysphagia, related to the inflamed mucosa. PPIs can help relieve dysphagia in this
instance, but there is no evidence that esophageal motility is altered by GERD or
administration of PPIs.
IEM is poorly understood, but it is very common in patients investigated with
manometry for dysphagia. Many asymptomatic controls can have a degree of inef-
fective motility.
Case 3 A 38-year-old man has a history of difficulty swallowing. He is on chronic
opiates for a bad back, taking three Percocet a day. He smokes 1 PPD and has 2–3
drinks a day. He denies heartburn, nausea, and vomiting. His dysphagia started
several months earlier. He has problems swallowing solids and liquids that have
gradually worsened. His physical exam is unremarkable. An EGD is negative for
any stricture, and biopsies are negative for EOE. Manometry shows simultaneous
contractions, with no true peristalsis. Overall pressure measurements are low dur-
ing swallowing, but there is an elevated LESP that does not relax appropriately on
swallowing, with an IRP of 20.
How would you evaluate and manage this patient?
This patient appears to have opioid-induced esophageal motility disorder. The
absence of coordinated esophageal contractions along with a hypertonic LES that
does not relax adequately with swallowing is suggestive of achalasia. However, the
diagnosis of achalasia should not be made in patients on opioids because these
drugs can cause a similar motility disorder.
The mechanism of opioid-induced esophageal motility disorders is not com-
pletely understood. Stimulation of μ opioid receptors has been associated with
symptoms of dysphagia and heartburn. Opioids decrease esophageal peristalsis and
affect function of the LES through impairment of nitric oxide pathway. We don’t
want to send the patient for unnecessary achalasia surgery when the syndrome may
be caused by opiates.
The management in this case requires getting the patient off opioids and repeat-
ing the manometry if symptoms persist. If the pattern persists then the patient has
true achalasia, but in most cases the manometry will normalize off opioids. We
don’t know the true incidence of opioid-induced dysphagia, but it would appear to
be relatively uncommon considering the number of patients on chronic opioids and
the small proportion of these patients that present with dysphagia. However, in eval-
uating patients with swallowing disorders, getting them off opioids may alleviate
their symptoms.
Case 4 A 30-year-old man presents with dysphagia for solids and liquids going on
for three months. He has lost 12 pounds over that time. No history of opiate use, no
smoking, and drinks only occasional alcohol. On EGD, he has a tight LES, but the
scope passes through with a gentle pop as it is gently advanced into the stomach.
Manometry is performed and demonstrates incomplete relaxation of the LES, with
an IRP of 17, absent esophageal peristalsis, and no pan-pressurization.
5 Other Motility Disorders of the Esophagus and Achalasia 29

How would you manage this patient?


This appears to be a classic type 1 achalasia. The patient is in the expected age
range for achalasia, 25–60 years old. The history of dysphagia with solids and liq-
uids is suggestive of esophageal motility disorder. This patient is also not on opi-
oids. The high IRP and failed esophageal body peristalsis all meet the criteria for
type 1 achalasia.
We recommend a timed barium esophagram in our patients with achalasia, which
can serve as a baseline for comparison with follow-up studies in the future.
Treatment of type 1 achalasia is directed at disrupting the LES as there is no
peristalsis. The least invasive treatment is a trial of smooth muscle-relaxing medica-
tions such as peppermint and calcium channel blockers. Botox may relieve dyspha-
gia with efficacy lasting from a few weeks to several months. Repeat Botox
injections can be performed, but patients develop antibodies, and the efficacy of
repeated injections decreases.
Other treatments for achalasia depend on the age, comorbidities, and expected
life expectancy of the patient. Pneumatic dilation can relieve dysphagia for up to 5
or more years, but the potential complication is esophageal perforation. Therefore,
pneumatic dilation should only be considered in a patient who is a candidate for
cardiothoracic surgery in the unfortunate event of esophageal perforation.
Heller myotomy and POEM are other therapies for patients who are surgical
candidates. POEM is less invasive and recovery is quicker, but there is a small risk
of esophageal perforation, and all patients will have esophageal reflux post-op.
Because of the universal problem with reflux after a POEM, most centers will place
all their patients on PPIs post-op and attempt to de-escalate over time.
There is much more experience with the Heller myotomy, which has been per-
formed for decades. Because the Heller myotomy is typically combined with a fun-
doplication, reflux is not as prevalent a problem compared with POEM
postoperatively.
The choice between pneumatic dilation, Heller myotomy, and POEM in patients
who are surgical candidates depends on local expertise. In our center, most patients
have had Heller myotomy, but the number getting a POEM is increasing.
In this patient, we mentioned that pan-pressurization was absent. What
would it mean if pan-pressurization was present?
This would be consistent with type 2 achalasia, where the LES does not relax,
and contractions occur in all segments of the esophageal body simultaneously, lead-
ing to pressurization of the esophageal lumen. An accurate diagnosis of the achala-
sia subtype has implications for the treatment options. For instance, patients with
type 3 achalasia should have POEM or long Heller myotomy with proximal exten-
sion of myotomy.
Case 5 An 85-year-old woman, who is chronically ill, presents with food impac-
tion. EGD is performed, and there is soft food sitting in a corkscrew-shaped esopha-
gus. After the food is removed, the LES zone appears mildly tight, but the scope
30 F. Edeani

could pop into the stomach. Dilation with a TTS balloon is performed, but there is
not much symptomatic improvement. Manometry is performed (somewhat limited),
and the LES pressure is elevated and does not relax, with an IRP of 16. There is no
normal esophageal peristalsis.
How would you manage the case of a patient with corkscrew esophagus
where the LES does not relax appropriately?
This patient sounds like she may have type 1 achalasia. However, in patients
presenting with findings of achalasia in this age range, we need to rule out pseudo-
achalasia. It’s important to make sure that there is no GEJ cancer causing obstruc-
tion and would also perform a chest radiograph to rule out any mediastinal mass.
Once we’ve ruled out pseudoachalasia, we would generally treat a patient like this
with Botox. This may provide some dysphagia relief for a few to several months,
before needing to repeat injection. As we mentioned earlier, pneumatic dilation
should be avoided unless the patient were a candidate for cardiothoracic surgery
should a complication occur. This patient also would not be a great candidate for a
POEM or Heller myotomy because of her age and frailty. Medical therapy can be
considered although this is the least effective treatment option. Long-acting nitrates
such as isosorbide dinitrate 5 mg may be administered sublingually 10–15 min
before meals. Sublingual nitroglycerine 0.4 mg is an alternative. PDE5 inhibitors
(e.g., sildenafil), anticholinergics (e.g., atropine), and beta-adrenergic agonists (e.g.,
terbutaline) have been used for treatment of achalasia, but there are limited data on
their efficacy. If the patient is already on an anti-hypertensive that is not a calcium
channel blocker, you could consult with her primary care physician about switching
her medication to a calcium channel blocker.
We encounter a number of elderly patients with corkscrew esophagus some
of whom present with food impactions. Is this simply an aging process of the
esophagus?
Corkscrew esophagus is a radiographic finding on barium esophagram character-
ized by non-peristaltic contractions and may reflect esophageal spasm. However,
most patients with esophageal spasm do not have corkscrew morphology on esoph-
agram. Age-related esophageal dysmotility is a nonspecific finding on esophagram
and is termed presbyesophagus. With advancing age, more people develop changes
in esophageal motility including an increase in the number of simultaneous contrac-
tions, lower amplitudes of contraction, and diminished LES relaxation. Patients
may remain asymptomatic early on, but later some will develop dysphagia and even
food impactions.
Chapter 6
GERD

Patrick Sanvanson

Case 1 A 35-year-old man with a BMI of 28 is referred to you with a history of


chronic heartburn. He doesn't smoke. He has only one to two drinks on weekends.
He stops eating 2 h before bedtime and elevates the head of his bed. Despite this, he
has ongoing heartburn. He started taking over-the-counter omeprazole at bedtime
and initially had limited response. He spoke to his primary care doctor who cor-
rected the timing of the PPI, and taking it before breakfast worked much better. As
soon as he stopped the omeprazole, his heartburn came right back again. Rather
than simply continuing the omeprazole, his primary care doctor referred him to you.
Would you accept the diagnosis of GERD in this case? Or are there other
conditions you would consider? How would you manage this?
He has a pretty classic presentation of GERD, with chronic heartburn. He takes
the medication and responds pretty well to it. Initially, he was taking it incorrectly,
at nighttime. We usually tell our patients to try to take PPIs 30 min before breakfast
when the proton pumps are the most active to be able to expose more of the proton
pumps to the drug.
In terms of risk factors for Barrett’s esophagus, he is overweight. On the other
hand, he is young, under 50 years old, and has no alarm symptoms like dysphagia
or weight loss. We usually don’t recommend endoscopy looking to rule out Barrett’s
esophagus in someone who is 35. All things considered, he’s probably at less of a
risk for Barrett's esophagus right now.
I think the other thing that is always important in taking care of these patients is
to instruct them early on about lifestyle factors, including trying not to eat late at
night, elevating the head of your bed, and weight loss. The medications we give

P. Sanvanson (*)
Division of Gastroenterology and Hepatology, Department of Medicine, Medical College of
Wisconsin, Milwaukee, WI, USA
e-mail: [email protected]

© The Author(s), under exclusive license to Springer Nature 31


Switzerland AG 2023
W. H. Sobin et al. (eds.), Managing Complex Cases in Gastroenterology,
https://doi.org/10.1007/978-3-031-48949-5_6
32 P. Sanvanson

decrease gastric acid, but they do nothing for esophageal clearance. They do nothing
to correct a hiatal hernia. The lifestyle measures will help decrease the volume of
refluxed material.
In this case, when the patient stops the drug, his symptoms come right back. At that
point, I have a discussion with him about potential next steps. For some patients I will
say, “It’s okay, it sounds like you have reflux disease. You respond well to the therapy,
so we can just keep you on the 20-mg dose of PPI.” Unfortunately, at this time, he will
need to stay on the PPI indefinitely unless some other risk factors are modified.
Hopefully, decreasing his weight may help with some of the reflux that he’s having.
Some patients don’t like to be on medications long term, and that’s understand-
able. So, in those patients, I might give them the option of potentially doing a scope,
looking for erosive esophagitis. Seeing that they have erosive esophagitis or a hiatal
hernia may be more of a reason to keep them on medication long term, since they
have structural findings that will continue to promote reflux and evidence of esopha-
geal damage already present.
Endoscopy can also be helpful in excluding alternate diagnoses, like eosinophilic
esophagitis (EOE). Do we need to do an endoscopy simply to rule out Barrett’s
esophagus in this gentleman? No. Usually, we don’t recommend screening for
Barrett’s esophagus at the age of 35. American College of Gastroenterology guide-
lines for screening for Barrett’s esophagus recommend it be limited to men with
chronic reflux symptoms and two or more risk factors including age > 50, Caucasian
race, presence of central obesity, a history of smoking, and a family history of
Barrett’s esophagus or esophageal adenocarcinoma. Unfortunately, I’ve had patients
who were in their twenties that I’ve diagnosed with Barrett’s esophagus.
In considering whether to endoscope this patient, it is unlikely that he has
Barrett’s esophagus, but I might give him the option, particularly if he feels like he
doesn’t want to be in therapy long term.
If you decide to do an endoscopy, do you want the patient off of PPIs?
Yes, I try to stop PPIs at least two months beforehand, because that way I feel like
you’re not masking anything, you’re not masking erosive esophagitis, and you’re not
masking EOE. Then, if there is no evidence of Barrett’s esophagus, and there is no
erosive esophagitis, then we are managing a non-erosive reflux disease patient. They
may have reflux that isn’t causing esophagitis, or they may have functional heart-
burn or esophageal hypersensitivity. Regardless, when dealing with non-­erosive dis-
ease, it’s less essential to be on medication, except from a quality-of-life standpoint.
If you do the endoscopy and it’s grossly normal, are you prepared to do pH
testing at that time?
It depends on how severe their symptoms are. If they’re tolerating their symp-
toms pretty well, and the endoscopy is completely normal, then I wouldn’t necessar-
ily do pH testing. If their symptoms are mild, we might just suggest taking an H2
blocker, which is quite safe.
But if the patient wants to see if they truly have reflux disease, or you want to
determine if this is functional heartburn, or esophageal hypersensitivity, at that point
I do offer pH testing. Unfortunately, pH testing is not the most sensitive testing in
the world.
6 GERD 33

We do know that probably around 70% of patients with heartburn don’t have evi-
dence of erosive changes on endoscopy. So, from that standpoint, all we can tell him
is he doesn’t have reflux diseases causing damage or ulceration of their esophagus.
If they do want further evaluation to see if this is truly reflux disease, the next
option would be doing pH testing, whether it’s catheter-based pH testing or it’s a
wireless Bravo pH testing.
Which do you prefer in terms of pH testing?
It all depends on the patient. I think they both have their benefits. When I do pH
testing to determine if reflux is occurring, then I stop their PPI therapy for at least 7
days and test off therapy.
If somebody is unable to tolerate any catheter within their nose for a 24-h time
period, they make the decision for you, and you do a Bravo study. However, if you
have previously done an endoscopy and you want to save the patient the cost of
another endoscopy, and if they’re able to tolerate a catheter in their nose, then choos-
ing a catheter-based test is prudent.
The Bravo device might be advantageous if you have a patient with nonspecific
symptoms like chest pain that occurs intermittently, not daily. A Bravo study usually
gives you 96 h worth of data. The pH catheter just studies 24 h of data. So, in that
scenario, a Bravo study may be more likely to capture the event in question.
On the other hand, occasionally we’ll have patients that come in with some vague
symptoms like a globus sensation or some kind of cough-related issue, and we want
to answer the question whether these symptoms are reflux-related. There are two
advantages to the pH catheter device compared to the Bravo. The pH catheter has
two sensors, so it can measure acid levels proximally and distally. The Bravo just
measures at one location, usually 6 cm above the LES. In addition, the pH catheter
measures impedance, so it can detect reflux of liquid that is non-acid, as well as acid.
So, using the pH catheter with its proximal sensor and impedance monitoring,
we can determine whether there are proximal reflux events that correspond with
coughing or globus.
Then once again, if the patient just can’t tolerate a catheter within their nose for
24 h, we have to use the Bravo and recognize that the data collection has its limita-
tions. So, it depends on the patient and the clinical scenario, and then, we give them
our recommendation of which test we think will give us the higher yield for what
we’re looking for.
Now, we’ve discussed doing pH studies off PPIs. Occasionally, we will do the
studies on PPI to assess the patient’s response to therapy. If the patient is still symp-
tomatic, in spite of taking PPIs, doing the pH study while on therapy can be benefi-
cial to determine whether the patient is having residual acid reflux, and if you’re
using the pH catheter, you can also see if they are having non-acid reflux.
How do you approach patients who are complaining of heartburn waking
them at night, who are taking a PPI once a day in the morning?
We know that PPIs work very well at healing erosive esophagitis, and when
patients complain of ongoing heartburn in spite of PPIs, there are two things I ques-
tion. Is the diagnosis of GERD correct? Is the patient on a PPI regimen with suffi-
cient potency?
34 P. Sanvanson

However, if somebody’s complaining more about nocturnal symptoms, for


example, they feel like they wake up in the middle of the night choking on acid or
coughing. In these patients, it’s very important to stress lifestyle factors. So, elevat-
ing the head of the bed and not eating late at night, usually at least 2 h before lying
down, are essential.
Then, if that is not working, I will often add an H2 blocker like famotidine at
bedtime. The main reason I do that is because a lot of nocturnal reflux events are
thought to be histamine-driven. So, frequently, H2 blockers are a little bit more
effective for nighttime reflux. Unfortunately, there may be tachyphylaxis after
extended H2 blocker use, where patients over time become tolerant to the medica-
tion, and it has less efficacy. So that’s always a concern.
Then, another option, although it’s not currently commercially available (until a new
manufacturer starts delivering it), is a device for patients who have supra-­esophageal
reflux symptoms, where you think they’re coughing at night because of acid going
above the level of the upper esophageal sphincter (UES). This is an upper esophageal
sphincter assist device that was developed at the Medical College of Wisconsin. It was
initially known as the Reza band, but is now known as the reflux band. This band
applies some pressure at the level of the upper esophageal sphincter to prevent acid
going above the UES. This may be an option for a subset of patients, although certain
patients can’t tolerate anything around their neck while they sleep. The nice thing about
the reflux band is that it’s not a medication, and it’s quite safe. When you stop using it,
you just go back to your baseline. So, there’s no dependency that occurs.
Have you ever used nighttime omeprazole-bicarbonate (Zegerid)?
It’s an option that I’ve only used in a small set of patients. More often, what I’ll
do is try to switch them to a more potent PPI to see if that’s more effective. In addi-
tion to that, I may give them a double dose where they take the second dose 30 min
before dinner time. Or, if they have predominantly nighttime symptoms, stop the
morning dose and simply prescribe their PPI 30 min before dinner. I will also occa-
sionally use dexlansoprazole (Dexilant), which is a PPI with a longer half-life.
Have you ever done genetic testing for the CYP2C19 polymorphism to see if
they were rapid metabolizers?
No, usually I just increase the dose or give one of the more potent PPIs, including
rabeprazole or dexlansoprazole. Then, if somebody is not responding to double-­
dose higher potency PPI, I will want to perform pH testing on medication to deter-
mine whether they have residual acid reflux.
While there may be some patients who are more likely to be nonresponders
because of genetic polymorphisms, more commonly you wonder if that is just one
contributing factor. Other factors that might exacerbate reflux include structural
ones like a hiatal hernia, or perhaps, there’s some dietary noncompliance issues, or
perhaps, there are other modifiable factors like weight. Then, there is also a subset
of our patients that have some degree of esophageal hypersensitivity or functional
heartburn that might cause them to be nonresponders.
So, before I go down that line of considering genetic testing, I think the key is
always to see (1) whether or not you’re treating the right condition, (2) whether
you're suppressing acid sufficiently, with pH testing, and (3) seeing if there’s a
symptom correlation with events, which might suggest hypersensitivity.
6 GERD 35

However, at this point, I’m not doing the genetic polymorphism tests. I think that
as genetic testing becomes more available and, hopefully, cost-efficient, I may want
to add in these tests. Right now, from my experience, I just don't know how it's
going to impact clinical practice. I feel that even if they are hyper-metabolizers, at
the end of the day, you’re probably just going to increase the dose of PPI or change
to the more potent agent, regardless.
Case 2 The patient is a 40-year-old white woman who has been taking OTC PPIs
for months without relief from heartburn. An EGD is performed, and the results are
normal and biopsies negative. You decide to do a Bravo procedure to evaluate
whether the symptoms are reflux-related. The Bravo procedure is done off PPI and
reveals the time with pH < 4 is only 2%, a negative study. However, a symptom log
was reviewed, and the patient did note episodes of symptoms that did correlate with
isolated reflux episodes.
How would you approach this situation?
Yes, this is a very difficult group of patients. The most important thing is that
these are patients who you definitely want to avoid sending for anti-reflux surgery.
When dealing with patients who have esophageal hypersensitivity or functional
heartburn, there are not a lot of well-defined therapies to use. From the standpoint
of understanding their symptoms, I think it is always important to tell these patients
that although their symptoms may feel like severe reflux disease, their esophagus is
NOT severely damaged, and the pH testing demonstrates that they are not refluxing
large amounts of acid into their esophagus.
There are some patients that have reflux hypersensitivity who may respond to a
larger dose of acid-reducing medication, PPI therapy. Further suppressing their acid
levels below the normal level may actually help with some of their reflux symptoms,
whether it’s heartburn or regurgitation, or whatever.
However, a lot of the time, with many of these patients, we consider what other
options are available. A lot of the strategies aim to work on the esophageal sensitiv-
ity levels. We turn to medications that have been used for other neuropathic condi-
tions or visceral hypersensitivity issues as well. The ones that I have the most
experience within this population are low-dose tricyclic antidepressants (TCAs),
whether it’s nortriptyline, amitriptyline, or imipramine.
We’ve had some success using these as adjunctive agents in terms of altering the
sensory threshold to esophageal hypersensitivity. We also try dietary therapies try-
ing to limit trigger foods in patients whose symptoms are worsened with particu-
lar foods.
Sometimes, we’ve tried diaphragmatic breathing techniques in this patient popu-
lation as well. Hopefully, in the future, we will develop some targeted therapies,
because a lot of the therapies we’re using now are neuromodulating agents that are
used for other purposes and extrapolated to the esophagus.
It is known that TCAs and SSRIs can help with symptom improvement by
increasing the sensory threshold for pain perception. People have done some esoph-
ageal balloon distension studies. They’ve done esophageal acid perfusion studies,
and some of these neuromodulating agents have been shown to be beneficial in
patients who have esophageal hypersensitivity or functional heartburn.
36 P. Sanvanson

Comments on Case 2: Dr. Sobin


This patient clearly does not have increased acid reflux but appears to have reflux
hypersensitivity. I give patients the “princess and the pea” analogy to explain their
heightened sensitivity. These patients are distinguished from those who have normal
24-h pH testing and no correlation between symptoms and pH reflux episodes. This
is so-called functional heartburn. In both these instances, we think there is a func-
tional component to their symptoms.
In reflux hypersensitivity, I would continue a PPI even though 24-h acid levels
are not increased, seeing how the patient’s symptoms are exacerbated when there
are episodes of reflux. If the symptoms are severe, I might turn to a central neuro-
modulator. Trazodone and SSRIs have been used and found effective in treating
functional esophageal syndromes, although large population studies are lacking.
SSRIs have been found more helpful in treating functional esophageal disorders
than other functional disorders of gut-brain interaction. Tricyclics may also be tried,
as noted by Dr. Sanvanson.
Chapter 7
Refractory GERD

W. Harley Sobin

Case 1 A 41-year-old white man with a BMI of 26 presents with a history of chronic
acid reflux symptoms for the past year. He has frequent heartburn and intermittent
regurgitation. He has taken occasional OTC Prilosec. You put him on esomeprazole,
40 mg a day, and see him back in 1 month. Despite therapy, he has ongoing
heartburn.
Is it surprising to you that the symptoms of heartburn aren’t gone after a
month of therapy?
Many people assume that PPIs universally clear up GERD symptoms, but only
60% of patients with GERD, at best, will become asymptomatic after a month of
PPI therapy.
When patients aren’t responding to PPIs what is the checklist you go
through?
I always review whether they are actually taking the medicine and timing it
appropriately. Many people take their PPIs with meals or HS which leads to poor
results. I consider alternative diagnoses: Could the patient have pill esophagitis—
are they taking minocycline, doxycycline, potassium pills, or alendronate? Might
there be an underlying motility disorder—either gastroparesis exacerbating GERD
or achalasia masquerading as GERD? Could it be eosinophilic esophagitis—is there
a history of food impactions or multiple allergies?
He had mistakenly been taking his PPI HS, so, instead, he starts to take his
esomeprazole 30 min before breakfast and presents a month later with almost the
exact same complaints.

W. H. Sobin (*)
Division of Gastroenterology and Hepatology, Medical College of Wisconsin,
Milwaukee, WI, USA
e-mail: [email protected]

© The Author(s), under exclusive license to Springer Nature 37


Switzerland AG 2023
W. H. Sobin et al. (eds.), Managing Complex Cases in Gastroenterology,
https://doi.org/10.1007/978-3-031-48949-5_7
38 W. H. Sobin

So now it has been 2 months since starting daily PPI and he is still symptom-
atic. How would you proceed?
At that time, I would investigate further with an EGD. However, I would want to
do the study off PPIs; if I do it on PPIs, I may get misleading information. Although
I will be looking for endoscopic evidence of reflux esophagitis, I realize that the
majority of patients with true reflux do not have erosive esophagitis on their index
endoscopy, only about 30% do. If I see erosive esophagitis, I have my diagnosis, but
since the majority of reflux patients have NERD (non-erosive reflux disease), I
would be prepared to do a pH study if the endoscopy is normal. I would have the
BRAVO probe ready to go. I would also do biopsies to rule out eosinophilic esopha-
gitis, etc.
You instruct him to taper his PPI off over a week to avoid acid rebound, and then,
he stays off it for a month. His symptoms go back to baseline. An EGD is performed,
and on EGD, the esophageal mucosa appears normal. Esophageal biopsies are
negative for EOE, Barrett’s esophagus, etc. The BRAVO probe is introduced and
shows time with pH < 4 of 10%, a positive result.
How would you manage this?
At this time, I would go to bid PPI 30 min before breakfast and before dinner. In
one study of pH results in patients with refractory heartburn, the pH results were
abnormal on one PPI daily in 31% of patients, but only 7% in patients on two PPI
daily [1].
If patients don’t respond sufficiently to this, I will add an H2 blocker at bedtime.
This works well in many patients, but eventually a lot of them develop
tachyphylaxis.
The other thing I might do is switch PPIs. There is a gradation of potency of dif-
ferent products which goes-pantoprazole is weakest, omeprazole and esomeprazole,
somewhat stronger, rabeprazole and dexlansoprazole-the strongest.
In addition, there are patients who may have a genetic polymorphism in
CYP2C19. They may be hyper-metabolizers of PPIs who will do better on rabepra-
zole than most of the other PPIs. Esomeprazole should do better as well.
Another trick, in patients who suffer from nocturnal heartburn is giving Zegerid,
which is immediate release, not enteric coated, omeprazole-bicarbonate. This does
not have to be taken before meals; it may be taken at bedtime.
A common problem is that patients may have improvement in their heartburn but
still suffer from regurgitation. These patients generally have a significant anatomic
defect, hiatal hernia, etc. I will reemphasize to these people the importance of ele-
vating the head of their beds and other anti-reflux measures.
Are there other medical regimens that may improve regurgitation?
Taking baclofen before meals may be efficacious. Baclofen is a GABA-B ago-
nist, and GABA-B inhibits TLESRs. However, most patients don’t tolerate baclofen
well, certainly long term.
The patient does not respond to protonix 40 mg bid and nighttime famotidine. He
is switched to rabeprazole and finds dramatic improvement. He is able to discon-
tinue his ranitidine and go to once daily rabeprazole before breakfast.
7 Refractory GERD 39

Patient 2 A 38-year-old woman with chronic heartburn and regurgitation. Three


months earlier, she had an EGD because of melena at which time she was found to
have a hiatal hernia, reflux esophagitis (LA grade A), and a small duodenal ulcer.
Biopsies were negative for Barrett’s esophagus and HP. She was started on PPIs.
After 2 months, the heartburn has improved, but she complains of severe regurgita-
tion. A repeat EGD is performed that demonstrates healing of the esophagitis and
duodenal ulcer. However, her LES appears quite lax, and she has a 3-cm sliding
hiatal hernia.
How would you approach this patient?
I think she is probably going to require anti-reflux surgery to fix the hernia and
correct the reflux. Our surgeons do Nissen, Toupet, and LINX procedures to correct
these hernias. Their philosophy is outlined in the next chapter.

Reference

1. Charbel S, Khandwala F, Vaezi MF. The role of esophageal pH monitoring in symptomatic


patients on PPI therapy. Off J Am Coll Gastroenterol ACG. 2005;100(2):283–9.
Chapter 8
Surgery for GERD

Jon Gould

Case 1 A 40-year-old man, with 10 years of medically refractory reflux, remains on


omeprazole. Despite omeprazole, he has severe regurgitation. When he stops
omeprazole, he has severe heartburn as well.
Workup reveals a 4-cm hiatal hernia, normal esophageal motility, and a positive
pH study with good symptom correlation. He has avoided having GERD surgery
because of horror stories on the Internet about difficulties swallowing, not being
able to vomit after surgery, or having severe gas bloat.
Most patients with GERD are being treated medically. In a case like this, the
patient is really suffering despite medical therapy. Overall, is surgery
underused?
Tons of patients suffer from this condition; GERD is a really common disease.
Medical therapy with PPIs is frontline treatment, it’s used frequently. It’s accessible
to patients. They can prescribe it for themselves. Primary care doctors hand it out
frequently. We all prescribe it.
However, there are data that suggest that if you look at people who truly have a
well-established GERD diagnosis, who are on a PPI, as many as one-third of those
people are not completely satisfied with their treatment. We end up operating on a
very small subset of those patients, less than 1% of people who would meet surgical
criteria, which means symptoms of GERD refractory to medical therapy. We’ve got
a small subset of people getting surgical therapy, and we’ve got a lot of people that
are treading water, feeling like there’s got to be something better, but they’re not
ready to commit to surgery.

J. Gould (*)
Department of Surgery, Medical College of Wisconsin, Milwaukee, WI, USA
e-mail: [email protected]

© The Author(s), under exclusive license to Springer Nature 41


Switzerland AG 2023
W. H. Sobin et al. (eds.), Managing Complex Cases in Gastroenterology,
https://doi.org/10.1007/978-3-031-48949-5_8
42 J. Gould

Fundoplication outcomes can be excellent when done properly in the appropriate


patient. However, a lot of patients who have GERD interventions end up back on
PPIs. There are doctors who tell their patients to avoid fundoplication because they
only last a few years, and there are surgeons who have an opinion that these things
never fall apart. The truth is somewhere in the middle. These procedures are intended
to be lifelong interventions, but in the end, the bottom line is that you fail on
some people.
We can do an intervention, but the patient may be left with side effects. In the
end, are they better off? In some cases, no. There are some surgical failures. These
repairs can work for a while and then fall apart. There are some bad outcomes. The
wrong procedure might be done on the wrong patient.
The operations are not always as effective as we want them to be. So, we’ve got
this issue. We’ve got a prevalent disease with a lot of people who are suffering, who
aren’t completely happy with their current treatment. However, they’re not ready to
make the next leap because they don’t feel like there’s an ideal option for them.
The good news is that newer techniques and learning from past mistakes allow
us to have better surgical results.
You mention surgical repairs coming apart. Do the sutures loosen? Is that
what is happening here?
We used to think that the sutures that we placed in the hiatus were tearing and
the repair was disrupting. Now, we think that the repair doesn’t disrupt, and it
dilates over time. The hiatus stretches out, and then, the stomach herniates
through there.
So now, what people are trying to do is close it tight enough that when it stretches,
it doesn’t stretch too much. You make it tight enough to prevent recurrence but not
so tight that you get symptoms.
So, I tighten it to where I think it’s about right. My way of doing it is a little dif-
ferent from the next guy. It’s very much art, and it’s judgment. For what it's worth,
good judgment comes from having bad judgment a few times.
The 40-year-old patient described above seems to be a good candidate for
surgery. He has a 10-year history of symptoms, severe regurgitation in spite of
PPIs, and a disordered anatomy. I have always thought that the recommended
surgical therapy for these patients was a Nissen. If the patient had disordered
esophageal motility, you would do a Toupet instead. In this case the motility
looks normal so would you do a Nissen?
No, we had a multi-society GERD Consensus Conference [1] with GI and sur-
gery societies participating, and we reviewed a lot of data comparing Nissen with
Toupet. We looked at a lot of different outcomes that matter, including hiatal hernia
recurrence over time, dysphagia requiring an intervention, gas-bloat symptoms,
inability to vomit, and recurrence of reflux symptoms at 1 year. What the data
showed is that patients did better with a partial wrap (Toupet) than a complete wrap
(Nissen). Our panel recommended that adult patients with GERD would benefit
more from a partial wrap than a complete wrap. We shouldn’t be doing as many
Nissens as we do.
These studies also show that esophageal motility does not make much of a differ-
ence. If your motility is normal, a partial wrap has fewer side effects, with equal
8 Surgery for GERD 43

GERD outcomes to a full wrap. If your motility’s abnormal, same statement, it


probably doesn’t matter as much as we once thought. The only exception is those
cases with extreme dysmotility and real aperistalsis.
The bottom line is that the Toupet is better than the Nissen. The problem is that
surgeons in the USA haven’t been trained to do Toupets, and they only learn how to
do Nissens. We’re the only country in the world that does Nissens. Last month, I did
my first Nissen in the last 5 years, and I’ve completely switched to partial wrap
fundoplication.
You mention esophageal manometry. Do you think everyone should get an
esophageal manometry prior to GERD surgery?
The guidelines say you should, and it’s nice to have a baseline, in case things
don’t go well. However, as I just pointed out, it doesn’t make as big a difference as
we once thought, unless there is severe aperistalsis. That being said, I do them in
about 75% of cases.
What do you think about the LINX anti-reflux system, in comparison with
fundoplication?
I’m a fan. It’s an early technology, but so far, the data suggest that GERD may be
better treated with magnetic sphincter augmentation (MSA) than fundoplication.
So far, in terms of symptom recurrence, and perioperative complications, the
data suggest better outcomes with MSA. This includes dysphagia, which is counter-
intuitive, because the biggest problem with the LINX is concern about dysphagia.
When we look out over a longer time interval, at more than 2 years, quality of life
measures favor MSA.
The great thing about LINX is that it’s very easy to put in. The dissection is very
minimal compared to what you have to do for a wrap. You can put it in, and if there
are problems, you can take it out very easily. There are way fewer decision points,
and the technique is very standardized. There’s an established technique that’s
taught to every surgeon.
I think the physics at the diaphragm is different for a LINX than for a fundoplica-
tion. You don’t have this big bulky wrap pushing on the diaphragm. You’ve got a
magnet that’s sitting there without any tension. It’s not pushing as hard, and it even-
tually gets encapsulated in some scar tissue that helps hold it into place. So, you
don’t see the same rate of anatomic failures in hiatal hernias after a LINX that you
do after a fundoplication. What I say is “let’s do the least invasive option that leaves
us as many options as we can possibly still have on the table.”
What if you do a LINX and it’s not working well, is it permanently embed-
ded? And what about erosions with the LINX?
Most of the failures we see are people who develop dysphagia that doesn’t
improve. There is also a very small incidence of erosions about 0.2%. Most of the
erosions occurred on devices that were overly tight. The philosophy over time has
changed from making it tight to the point where you can see the things squeezing
the esophagus to actually having it loose enough that it doesn’t compress at all.
In terms of managing dysphagia, if you go in to remove the device a year later,
you find that every bead has a perfectly formed scar on top of it. It’s like a little
capsule that you touch with a little cutter, and the magnet pops out and you grab the
magnet and you just ride around each side and you cut the wire.
44 J. Gould

However, what we’ve found is that even though you’ve removed the device,
there’s scar tissue remaining that changes the compliance there. When you take it
out and don’t do anything, the dysphagia is gone and about 75% of people don’t get
the reflux back.
Do you take all patients with post-op dysphagia from the LINX back to
surgery?
No, there’s a reversal time frame, and at first, you need to try and ride it out.
Some people will react to the implant with scarring, and you try to dilate it. You
might also prescribe a brief course of oral steroids to soften up the scar tissue.
You would try to manage it with dilation and steroids for a few months. If people
are struggling, asking to get them out, I would try to wait at least 2 or 3 months,
unless there’s a major issue. Most of these people will improve sufficiently to avoid
surgery.
The patient under discussion has a 4-cm hiatal hernia. I thought that the
LINX was approved only for patients with a hiatal hernia under 3 cm. Does
that mean a LINX is contraindicated in this patient?
The original MSA studies only looked at patients with small hiatal hernias. Large
hernias were a relative contraindication because they were not studied in the pivotal
and premarket trials conducted to attain original FDA approval. Now, appropriate
studies have been done with larger hernias, and currently, the FDA indications for
LINX include larger hernias.
If that’s the case, would you prefer to do an MSA for most of your GERD
operations?
To be a candidate, the patient can't have any MSA contraindications, which
include a BMI > 35, ineffective motility (IEM) on manometry, baseline dysphagia
that is moderate or worse, or an allergy to nickel. Insurance is a frequent obstacle,
Medicare and Medicaid don’t cover it; private insurance is about 50/50.
All else being equal, I would say that the MSA is simpler, more likely to be per-
formed without an overnight hospital stay, easily reversible, and may have a lower
failure rate. When MSA fails, Toupet is an option—not the other way around in my
opinion, so MSA saves options in the event of a failure. In the short term, however,
MSA is more likely than Toupet to cause dysphagia.
The advantages of the Toupet include that it is universally covered; it’s okay in
patients with IEM and dysphagia, and there’s no foreign body/implant, no worries
about MRI.
In my opinion, both are good options. All else being equal, I think a patient who
meets criteria and has access to either may be better off with the MSA due to the
long-term thoughts and preservation of options as described.
What are the concerns regarding MSA and MRIs?
The current generation of LINX is compatible with a 1.5 TESLA MRI. About
90% of the MRIs done in this country utilize an MRI 1.5 T or weaker. Most of the
bone and joint and spine MRIs are all 1.5. However, some of the breast indications
use a 3 Tesla MRI, and those functional brain MRIs are 7.
If you use an MRI that’s stronger than the device is compatible with, it depolar-
izes the magnets. So instead of attracting each other, they repel, and people get their
reflux back. That’s usually the worst thing that happens.
8 Surgery for GERD 45

Patients are educated and given a card. You would be amazed that someone could
have an operation and have a magnet implanted in their body, and then when they
go to get an MRI, they check the form saying “no implants.” I’ve had it happen to
two of my patients who didn’t follow our warnings.
Case 2 A 45-year-old man with a BMI of 22 has a long history of GERD well con-
trolled on PPI. His only symptom is heartburn, and it is totally relieved with PPI. He
had an EGD that showed a small hiatal hernia with a mildly lax lower esophageal
sphincter and Grade A esophagitis.
He is concerned about continuing PPIs long term because he read on the inter-
net that PPI use can cause cancer and dementia and wonders whether he should
just proceed with a hiatal hernia repair?
How do you feel about operating on patients whose symptoms are well con-
trolled on PPIs?
When I see people that are well controlled on PPIs but say that they want to get
off them, I’ll have a debate with them about whether that’s the right thing to do.
Because if their symptoms are completely under control, there’s a chance they’ll do
worse with surgery. I may give them a side effect from an operation that might ulti-
mately fail.
Of course, there’s potential downsides to PPIs, but things have been blown out of
proportion. For the patient who uses the C word (cancer), who doesn’t have Barrett’s
or dysplasia on endoscopy, I tell them that’s simply not a big concern. There’s
downsides to an invasive intervention. So, I’ll actually talk those people back.
I think that a patient who’s well managed medically on a PPI, who has symptoms
under control, whose quality of life as it relates to their GI symptoms is acceptable
to them is best left alone. Now, I might meet them in the future to discuss surgery if
things change, but for now I would counsel against surgery.
On the other hand, if I’m dealing with a patient whose main complaint is regur-
gitation, a PPI is not going to fix that. Most people with bad regurgitation have a
significant hiatal hernia. They have an anatomic issue.
Case 3 A 38-year-old woman has a long history of severe reflux. She has a BMI of
40. On EGD, she has a 1-cm hiatal hernia. PPIs are not controlling her symptoms.
How do you manage GERD in the obese patient?
If a patient with a BMI of 40 has medically refractory reflux, the best GERD
operation that I can do for them is a gastric bypass. We call it an esophageal discon-
nect instead, which is because many insurance companies will deny it if we call it a
bypass, claiming that they don’t cover cosmetic operations for obesity. So, I have to
get a medical doctor on the phone and tell him or her what I’m trying to accomplish.
We can usually get it covered if we call it a disconnect and have a peer-to-­
peer review.
The reason it works so well is that we make a small pouch, about a 20-cc gastric
pouch, and there are very few parietal cells in there. Obviously, there’s no bilio-­
pancreatic secretions that can get in there. There’s no pepsin that can get in there. So
that’s how it works to prevent reflux. Of course, this patient is obese and will lose
weight, which is a factor for all these other things.
46 J. Gould

So, a gastric bypass or a disconnect is good in the high BMI patients. We know
that BMI is a risk factor for hiatal dilation leading to fundoplication failure. We also
prefer a gastric bypass for GERD in some patients with scleroderma and others with
profound motility problems. A bypass doesn’t rely on a valve that’s tight enough to
prevent reflux, but not so tight that they’ll have a swallowing problem that gets
worse. It just diverts all that caustic stuff.
Case 4 A 63-year-old female, is 5 years status post an endoscopic TIF (transoral
incisionless fundoplication). Her symptoms prior to surgery were heartburn and
regurgitation refractory to medical therapy. She had a positive pH test, with no
hiatal hernia. Following the TIF, her symptoms resolved for 3 months, but after that
her symptoms came back, and they pretty quickly returned to baseline.
What is your impression of the new surgical-endoscopic hybrid procedure,
the transoral incisionless fundoplication (TIF)?
The word on the street is that because you’re doing it endoscopically, you don’t
burn any bridges whatsoever and that you leave everything on the table. You haven't
done anything to harm the patient, and you have every option that you would want
to have in your back pocket if that should fail.
However, the issue with TIF has always been that if a patient has a hiatal hernia,
it’s hard to do a wrap, have it in the chest, and have it work at all. So now people are
doing the cTIF, where they go in laparoscopically or robotically, fix the hernia, and
then close all the incisions and step back and do a blind endoscopic TIF.
However, our Consensus Conference reviewed the data and found that as time
goes by the failure rate of a TIF is going to exceed the failure rate of a fundoplica-
tion. Therefore, we recommended that adult patients with GERD would get greater
benefit from fundoplication than TIF.
We need to come up with an endoscopic treatment for medically refractory
reflux; we have to do it. Of all the things that are out there, TIF is really the closest
thing I think, to what we do in surgery. We’re not there yet.
In terms of the ease of a redo, if the TIF fails, I find that these are really hard
things to undo. The people that claim that it’s easy are cutting corners and putting
wraps on top of TIFs. However, this stuff is bound to fail. The only technique that
works is to do a minimally invasive hernia repair with takedown of the TIF and
conversion to a Toupet. It’s really hard to remove all those TIF anchors. I would
rather redo a fundoplication any day of the week than someone who had a TIF.

Reference

1. Slater BJ, Collings A, Dirks R, Gould JC, Qureshi AP, Juza R, Rodríguez-Luna MR, Wunker
C, Kohn GP, Kothari S, Carslon E. Multi-society consensus conference and guideline on the
treatment of gastroesophageal reflux disease (GERD). Surg Endosc. 2023;37(2):781–806.
Chapter 9
Barrett’s Esophagus

Kulwinder Dua and Wilfredo Pagani

Case 1 A 60-year-old Caucasian man is referred to your office for Barrett’s esoph-
agus. He was diagnosed with Barrett’s esophagus when he had an EGD for
hematemesis during a vacation in Florida 1 year ago. The EGD revealed a small
duodenal ulcer, which was the source of the bleeding. Biopsies were negative for
helicobacter pylori, but he was also found to have C4M5 Barrett’s esophagus. There
was intestinal metaplasia without dysplasia on biopsy. He has been on PPIs, feels
fine, and is without heartburn. He was told that he should follow up with a local
gastroenterologist in 1 year.
Would you repeat the endoscopy at this time?
When deciding on surveillance frequency, it is important to consider the length
of the segment of BE and whether dysplasia is present or not. In those patients with
non-dysplastic Barrett’s esophagus (NDBE), the most recent guidelines recommend
assigning a surveillance interval of 3 years for those who have a long segment of
NDBE (≥3 cm) and a 5-year interval for those with a short segment of NDBE
(<3 cm) [1]. So, I would repeat the endoscopy in two more years, not now.

K. Dua (*)
Division of Gastroenterology and Hepatology, Department of Medicine, Medical College of
Wisconsin, Milwaukee, WI, USA
e-mail: [email protected]
W. Pagani
Division of Gastroenterology and Hepatology, Medical College of Wisconsin,
Milwaukee, WI, USA
e-mail: [email protected]

© The Author(s), under exclusive license to Springer Nature 47


Switzerland AG 2023
W. H. Sobin et al. (eds.), Managing Complex Cases in Gastroenterology,
https://doi.org/10.1007/978-3-031-48949-5_9
48 K. Dua and W. Pagani

What is your recommendation regarding surveillance vs. treatment if a


patient is found to have dysplasia, and what would be the timing if you decided
on surveillance alone?
There are three pathological determinations of dysplasia—low grade, high grade,
and indeterminate. If dysplasia is diagnosed, a second pathologist with expertise in
BE should review the diagnosis. In patients with confirmed dysplasia by a second
expert pathologist [1, 2], the surveillance frequency is guided by the grade of dys-
plasia. Patients with high-grade dysplasia (HGD) should not be offered surveil-
lance; they need to undergo endoscopic eradication therapy or surgery. In patients
with low-­grade dysplasia (LGD), there is a lot of debate about whether surveillance
or ablation should be performed.
Therefore, the current guidelines for LGD recommend shared decision-making
with the patient regarding the risks and benefits of endoscopic eradication therapy
vs surveillance. In a patient who chooses to pursue surveillance, an endoscopy in
6 months should be performed and then annually if LGD remains present.
In cases where the pathologist is unable to discern if there is dysplasia or if the
histologic findings are due to inflammatory changes, a diagnosis of indeterminate
for dysplasia is given. Again, a second expert pathologist should review the diagno-
sis. In patients with a confirmed diagnosis of indeterminate for dysplasia, current
guidelines recommend performing a surveillance endoscopy 6 months after diagno-
sis and then annually if indeterminate for dysplasia remains present.
What endoscopic approaches are available for treatment?
Endoscopic eradication therapy (EET) is recommended by all GI societies for
patients with HGD or intramucosal cancer (IMC), and it can also be used in LGD to
prevent progression [1, 2]. EET includes endoscopic mucosal resection (EMR) for
visible lesions and ablation therapies, which include thermal ablation and cryo-
therapy ablation [2].
Photochemical ablation which was used in the past has fallen out of favor, mainly
because it was expensive, caused increased discomfort in patients, and was not very
effective. The most studied and established ablative treatment is radiofrequency
ablation (RFA) with studies showing rates of complete eradication of intestinal
metaplasia of 54–100% and rates of complete eradication of dysplasia of 80–100%.
There are multiple RFA catheters with different sizes which can provide circumfer-
ential RFA vs focal RFA therapy.
Another thermal ablation therapy used in the treatment of BE is argon plasma
coagulation (APC) [4]. APC is a contact-free technique that uses ignited ionized
argon gas to achieve tissue ablation. Hybrid APC is an alternative technique that
involves a submucosal fluid injection prior to performing APC to reduce the depth
of thermal injury and minimize post-ablation stricture formation [5].
Finally, cryotherapy ablation can also be performed with liquid nitrogen-based
cryotherapy or balloon-based nitrous oxide therapy which also appears to be effec-
tive [6]. RFA continues to be the most used ablation technique [1] given its exten-
sive data confirming its safety and effectiveness, with the use of cryotherapy ablation
techniques for those patients who fail RFA or experience increased discomfort
with RFA.
9 Barrett’s Esophagus 49

Case 2 A 62-year-old man has an EGD performed because of chronic heartburn.


He is found to have Barrett’s C3 M3, and biopsies show low-grade dysplasia. He is
treated with PPIs for 6 months, and a repeat endoscopy still shows low-grade dys-
plasia confirmed by a second pathologist.
Will you invariably recommend ablation?
I think there is no question but that ablation should be performed for high-grade
dysplasia. I think the management of low-grade dysplasia is less clear-cut. There are
certainly studies that show that low-grade dysplasia will occasionally turn into high-­
grade dysplasia in some patients.
However, here’s the problem. I talk to the patient, and I tell them, “You have got
low-grade dysplasia. Because of that you are not a candidate to have repeat surveil-
lance endoscopies every 3 years. No, now I need to scope you again in 3 or 6 months
to begin with, and then, we need to do more extensive, quad biopsies every 1 cm. If
your low-grade dysplasia remains low grade on repeat, then I will repeat again in 6
months, and if biopsies are stable, maybe spread it out to 1 year. As long as you have
low-grade dysplasia, we will be doing this every year. If it turns into high grade, we
must treat it.”
“Alternatively, you have another option. I could ablate it, which, in many centers,
is the standard of care. After ablating, even if I eradicate the dysplasia, you are still
going to need follow-up endoscopes every 6 months, and then every year, the same
way we would if you had not had ablation.”
So, many of my patients say, “Then what’s the point?”. They hear about potential
chest pain and potential stricture formation, with ablation, and so they may opt-out.
That’s okay, unless, or until, they develop high-grade dysplasia at some time in the
future, which may never happen. So, I have that discussion with the patient.
Then, there are other patients who I push for ablation. I had a patient whose
brother died at a young age with esophageal cancer. I recommended that since he
had low-grade dysplasia, he needed to be ablated. I had another patient who had
low-grade dysplasia who was getting chemotherapy for myeloma, and the concern
was that the chemotherapy could cause some accelerated high-grade dysplasia
development or carcinoma development, and the patient opted for ablation. Then,
there are some other patients who are absolutely terrified, and they want ablation.
Even though we are going to keep scoping them as though they never had ablation,
they want it because it’ll give them some peace of mind.
If I’m dealing with a patient who previously had high-grade dysplasia which we
eradicated but now they return on follow-up surveillance with some low-grade dys-
plasia on one or two biopsies, we recommend ablation. Assuming that your patholo-
gist’s findings are confirmed, once they have had an element of high-grade dysplasia,
I will definitely treat the patient who returns with low-grade dysplasia.
The majority of my patients who are found to have low-grade dysplasia de novo
decide that since they’re going to be endoscoped every 6 months for at least a year,
and then every year after that, despite ablating or no ablating, they decide that they
might as well just hold off on ablation.
50 K. Dua and W. Pagani

Eventually, we may see some national guidelines come out with firm recommen-
dations that once you eradicate the Barrett’s esophagus in low-grade dysplasia, you
can now get a little bit laid back and push the endoscopies out to once every 3 years.
If that recommendation comes out, we may want to do more ablations.
The patient chooses to have an ablation.
How do you perform your ablations at MCW?
We no longer use photodynamic therapy to ablate Barrett’s esophagus. Currently,
the most common technique used for eradication is radiofrequency ablation and that
is what we use. The probes come in various shapes and sizes. We have a probe that
ablates 360° covering large areas, another one with 90-degree ablation, a longer
90-degree ablator, and 90-degree Ultra. The 360-degree probe has to be passed over
a guide wire outside the endoscope, whereas the 90-degree probes are mounted on
the endoscope like a cap. Then, there is a probe that can be introduced through the
accessory channel of the endoscope. We also have cryoablation catheters that we
generally reserve for patients who fail RFA ablation, although this approach can
also be used upfront.
Is this generally a pretty benign procedure?
Patients can feel significant chest pain and odynophagia following ablative ther-
apy, especially after using the 360-degree probes and ablating a long segment of
Barrett’s esophagus. Patients can also develop ulcers and strictures, especially if
ablation is applied over overlapping areas that otherwise should be avoided. Rare
perforations have occurred.
Do all patients respond to ablation or are some resistant? If so, what do you
do in cases that are resistant to ablation?
A few patients are resistant, which seems to occur in patients who have reflux
that is not adequately controlled, generally associated with a large hiatal hernia. No
matter how much PPI you give them, you cannot control their reflux. So, I will
encourage those patients to get a laparoscopic repair of their hernia. Of course, we
first try a very strict anti-reflux regimen including weight loss, smoking cessation,
and exercise. In others who fail RFA, some practitioners start using cryoablation.
Case 3 A 57-year-old man with a BMI of 32 is referred with a finding of a nodule
in the background of Barrett’s esophagus. The patient was first diagnosed with
Barrett’s esophagus 3 years earlier, and at that time, there was no dysplasia pres-
ent. He has been receiving pantoprazole since then. Now, on his first follow-up
EGD, he is found to have C4M6 Barrett’s esophagus with a 3-mm nodule present.
He is sent to you for a second opinion.
When you see a small nodule will you biopsy it, or do an ablation, or EMR?
If somewhere in the Barrett’s esophagus segment, there is a nodule, then one
should do an EMR. There are two reasons to avoid simply doing a biopsy. First,
there could be something deeper that is more sinister, where your biopsy may come
back showing high-grade dysplasia, but deeper down there is carcinoma. Second,
your biopsy may cause some submucosal fibrosis and that may create difficulties
with the endoscopic mucosal resection. If there is a bump or a nodule, we always do
9 Barrett’s Esophagus 51

EMR rather than ablation. As compared to EUS, histopathology of the resected tis-
sue will also give better depth of invasion and margin involvement if it is
adenocarcinoma.
Will you ablate the surrounding Barrett’s esophagus at the same time?
This can be done, but we generally avoid it for fear of perforation, especially if
using balloon ablation and cryogas. We usually wait for the histopathology results
and plan accordingly. Generally, waiting for 8 weeks after EMR/ESD is
recommended.
How do you evaluate the remainder of the field of Barrett’s esophagus?
Once I see a nodule, I will do 4-quadrant biopsies every 1 cm. I want to see if
there is any multifocal dysplasia or even carcinoma. You can have carcinoma that
may be completely flat and miss it if you don’t do biopsies. Always use NBI (or
other such imaging on other company endoscopes) and magnification in all patients
with Barrett’s at each examination. There are several other enhanced imaging
modalities available such as confocal microscopy [7] and OCT (see WATS-3D below).
What would you do if the biopsies of the remaining Barrett’s esophagus
show no dysplasia, would you do ablation?
If the nodule shows high-grade dysplasia or carcinoma in situ, I would want to
eradicate the remaining Barrett’s esophagus, even if the remaining Barrett’s esopha-
gus has no dysplasia on biopsy. If there is one site that declared itself, I expect that
the entire field of Barrett’s esophagus is probably unstable and hence would favor
eradication.
Are there new techniques available for screening and diagnosing Barrett’s
esophagus in the community?
There are new techniques being studied, but not widely available, they are not
being done here at MCW. Endoscopy continues to be the mainstay for the diagnosis
of Barrett’s esophagus (BE). However, this strategy is not cost-effective and carries
a potential risk of harm from the procedure. Because of this, there is much interest
in the development of tools and testing modalities which are less invasive and more
cost-effective for screening and diagnosing BE with substantial progress over the
last decade. Of these, the modality that has the most data available is the use of teth-
ered cell collection devices in combination with biomarker testing. Devices such as
the Cytosponge™ with trefoil factor family protein 3 (TFF3) immunohistochemical
staining, EsophaCap™ with a methylated DNA marker panel, and EsoCheck™ also
with a methylated DNA marker panel have shown good accuracy in diagnosing BE
[7]. Of these, the most studied has been the Cytosponge™ with TFF3 immunohisto-
chemical staining, with one study showing up to a 90% sensitivity and 94% specific-
ity when compared to endoscopy in patients with a BE segment of 2 cm or greater [8].
Given the progress in the development of these devices, the most recent guide-
lines have included using a swallowable, non-endoscopic capsule sponge device
combined with a biomarker as an acceptable alternative to endoscopy for BE screen-
ing [1]. However, these devices have not become widely adopted in the USA as of
this moment.
52 K. Dua and W. Pagani

Another promising technology is the use of a wide-area transepithelial sampling


with computer-assisted three-dimensional (WATS-3D) analyses [3, 7]. This tech-
nology provides advanced sampling by using an abrasive brush to sample deeper
layers and larger surface areas of the esophagus. These specimens are then analyzed
by a computer software which identifies abnormal cells. Studies have shown that
WATS3D may increase the yield of dysplasia detection with a recent meta-analysis
[9] showing that WATS3D increased the absolute yield of dysplastic BE by 7.2%
(95% Cl 3.9–11.5) over conventional forceps biopsies. Currently, one of our society
guidelines (AGA) supports its use as an adjunctive technique to sample the sus-
pected or established BE [7].

References

1. Shaheen NJ, Falk GW, Iyer PG, Souza RF, Yadlapati RH, Sauer BG, Wani S. Diagnosis
and management of Barrett’s esophagus: an updated ACG guideline. Am J Gastroenterol.
2022;117(4):559.
2. Sharma P, Shaheen NJ, Katzka D, Bergman JJ. AGA clinical practice update on endo-
scopic treatment of Barrett’s esophagus with dysplasia and/or early cancer: expert review.
Gastroenterology. 2020;158(3):760–9.
3. Qumseya B, Sultan S, Bain P, Jamil L, Jacobson B, Anandasabapathy S, Agrawal D, Buxbaum
JL, Fishman DS, Gurudu SR, Jue TL. ASGE guideline on screening and surveillance of
Barrett’s esophagus. Gastrointest Endosc. 2019;90(3):335–59.
4. Manner H, Rabenstein T, Pech O, Braun K, May A, Pohl J, Behrens A, Vieth M, Ell
C. Ablation of residual Barrett’s epithelium after endoscopic resection: a randomized long-­
term follow-up study of argon plasma coagulation vs. surveillance (APE study). Endoscopy.
2014;46(01):6–12.
5. Kolb JM, Shah S, Chahine A, Chang K, Samarasena JB. Hybrid argon plasma coagulation for
Barrett’s esophagus. VideoGIE. 2021;6(8):339–41.
6. Canto MI, Shaheen NJ, Almario JA, Voltaggio L, Montgomery E, Lightdale CJ. Multifocal
nitrous oxide cryoballoon ablation with or without EMR for treatment of neoplastic Barrett’s
esophagus (with video). Gastrointest Endosc. 2018;88(3):438–46.
7. Muthusamy VR, Wani S, Gyawali CP, Komanduri S, Conference CB. AGA clinical practice
update on new technology and innovation for surveillance and screening in Barrett’s esopha-
gus: expert review. Clin Gastroenterol Hepatol. 2022;20(12):2696–706.
8. Kadri SR, Lao-Sirieix P, O’Donovan M, Debiram I, Das M, Blazeby JM, Emery J, Boussioutas
A, Morris H, Walter FM, Pharoah P. Acceptability and accuracy of a non-endoscopic screening
test for Barrett’s oesophagus in primary care: cohort study. BMJ. 2010;341:c4372.
9. Codipilly DC, Chandar AK, Wang KK, Katzka DA, Goldblum JR, Thota PN, Falk GW, Chak
A, Iyer PG. Wide-area transepithelial sampling for dysplasia detection in Barrett’s esophagus:
a systematic review and meta-analysis. Gastrointest Endosc. 2022;95(1):51–9.
Chapter 10
PEG Tubes

William Berger

Case 1 We have a 75-year-old man who had a PEG placed following a stroke a
month earlier. He is sent back to the office because he has continuous drainage from
the PEG site.
Many gastroenterologists find this a common complication. How do you
deal with it?
First of all, what sort of drainage are you dealing with? In some patients, it's only
clear liquid which may be gastric fluid, which can be very irritating. Other patients
have feedings come out around the tube, and still others have a lot of mucus or even
purulent discharge.
The most common sort of drainage is tube feedings. To have tube feedings drain-
ing out the stoma, there must be something causing an abnormal pressure differen-
tial. Because if you remember back to the work of William Beaumont, he studied a
French trapper named Alexis St. Martin, who had been shot in the abdomen and had
a large unhealed fistula between his skin and his stomach. Beaumont was able to
look into the trapper’s stomach and observe what goes on in the stomach during
digestion. The hole was so big that Beaumont could actually peer into it, and he
could place various things into the stomach and watch how the stomach reacted. The
point of the story is that the trapper could eat regular food, and it didn't come pour-
ing out onto his shirt because the pressure inside the stomach is typically negative
relative to the outside. So, if the pressure inside the stomach is negative, we should

W. Berger (*)
Division of Gastroenterology & Hepatology, Medical College of Wisconsin,
Milwaukee, WI, USA
e-mail: [email protected]

© The Author(s), under exclusive license to Springer Nature 53


Switzerland AG 2023
W. H. Sobin et al. (eds.), Managing Complex Cases in Gastroenterology,
https://doi.org/10.1007/978-3-031-48949-5_10
54 W. Berger

not see any leak through the G tube site. So then, why do we have any leak? Why
would we have a higher pressure in the stomach than the atmospheric pressure out-
side, which is what causes liquid to leak out?
One possibility is that you just have dramatic overfeeding. Another could be that
the patient has gastroparesis, and the stomach fills up but doesn't empty properly. In
fact, any kind of gastric motility disorder, outlet obstruction, or anything else caus-
ing increased pressure in the stomach will cause more leakage.
A third, which I see not infrequently, is a PEG tube placed in the antrum. During
gastric peristalsis, antral contractions start on the incisura and push toward the pylo-
rus generating high pressures. When the tube is in the antrum, these high pressures
can force fluid out around the tube because the tract becomes the path of least resis-
tance. The proximal stomach is for capacitance—that’s where the adaptive relax-
ation and accommodation occur. The antrum—beyond the incisura—is functionally
a different organ as far as acid secretion, peristalsis, and the like.
Now, why do some patients have increased gastric acid leakage? The acid leak-
age usually is because they're hyper-secretors. For these patients, you place them on
a PPI, and frequently, the problem goes away. Not only does it alleviate the caustic-
ity of the discharge, but also the volume of it.
If you have purulent drainage, it usually means you have an infection. Sometimes,
it just looks like mucus, but quite often it's an infection in the abdominal wall.
Sometimes it’s obvious just looking at it, but other times you get an ultrasound or
CT, and you may even find a small abscess there. With the infection, you occasion-
ally have to take the G tube out, leave it for a few days, and then put another G tube
in at a different location.
What do you think leads to these infections?
By taking a PEG tube and dragging it down the patient’s throat, you're taking a
foreign body, coating it in saliva, and sticking it in a fresh wound. So, it’s not sur-
prising that we see some immediate infections.
Delayed infections, months or years later? I don't know what causes them, but
usually there's some kind of nidus, and it's not just a local cellulitis. Usually, there's
an abscess. Usually, even if you give them antibiotics, it may clear up for a while,
but it comes right back because the foreign body is still there. So, I usually take the
G tube out, let it heal, and settle down at least a few days, with or without antibiot-
ics, and then, put another G tube in at another site.
Sometimes, there’s no infection, but you’ll see marked irritation on the skin
surrounding the G tube probably related to the caustic damage from gastric acid. I
treat this like a bad diaper rash with absorbent pads and zinc oxide. PPIs should
help as well.
What if we have a G tube that’s in the body of the stomach, not the antrum,
there's no gastroparesis, and you have ongoing drainage that’s either clear or
slightly mucoid. There’s some local irritation around the surrounding skin
from leakage. What are some strategies to deal with this?
In all these cases of G tube dysfunction, you want to identify any physiological
conditions that are correctable. If you find them and correct them, there is a pretty
high success rate. In this case, there's no obvious underlying condition, and then, you
10 PEG Tubes 55

have to do a systomatic approach. By aspirating gastric contents through the PEG


tube we can quantify gastric residuals. If that is increased, solve that problem first.
Next is usually the PPIs, which at least make it somewhat less caustic and
sometimes cut down the volume of drainage. In the worst cases we will place an
ostomy bag around the stoma if the leakage is severe. These unfortunate patients
have to hook themselves up for feedings and then drain the excess fluid that
comes out.
We always want to make sure that we are not dealing with a buried bumper. In
those cases, they're pumping in the food, and it comes right back out around the
stoma. So, one of the things I almost always do is scope the patient to see what the
site looks like, make sure the bumper is intact and not buried, and make sure the
tube is not located in the antrum. If you go down and the tube is in a normal posi-
tion, there's not a big gastric pool, the pylorus is wide open, yet they still seem to
leak, and you have two options. Either try replacing, repositioning the G tube in a
new location, or simply apply symptomatic control of the leakage.
Have you ever dealt with this problem by leaving the tube out for a number
of hours and let the hole close up by itself so the opening is smaller?
That presumes that resistance to flow around the tube is the problem. Following
that logic, we see people placing bigger and bigger tubes in an attempt to plug the
hole. So, if you start out with a 20 Fr tube, the hole may loosen, so it is now 22 Fr
diameter. Then, if you place a 22 Fr tube in to plug it up, it’ll become a 24 Fr hole,
and it just gets progressively wider. It loosens itself up as you go. Wanting to just
plug it up to take care of the problem is destined to fail. It may give you a temporary
sense of control, but the underlying problem is the pressure differential, and trying
to increase the resistance so that it won’t leak while you still have the underlying
pressure differential is at best a temporizing measure.
It makes you feel better, so you can send them back to the nursing home, but the
only way they don’t come back is if they get sick of dealing with you and send them
to someone else.
On a similar note, we occasionally get consulted about patients who have had a
surgical J tube placed with a lot of bile leakage. The feedings don’t come back, but
bile leakage can be massive and caustic to the skin. It turns out that it is almost
always because somebody put a balloon tube in the jejunum and it’s causing a par-
tial downstream obstruction for fluid coming from upstream. Peristalsis is generat-
ing internal pressure, while clamping down on the balloon blocking the lumen just
as the bile bolus is being pushed from upstream. This just makes the J tube stoma
the path of least resistance. So, what you do is just get rid of the balloon and secure
the jejunostomy tube externally by suturing it in place.
Do you ever consult wound care nurses?
Often, one of their tricks is to place the ostomy bag over the tube, as I alluded to
previously. It’s messy and it’s inconvenient, but it will work. That’s a worst-case,
last-resort scenario. The wound care nurses are basically going to deal with what
comes out, and they’re going to assume it’s going to continue to come out because
they are ostomies. That’s what these nurses work with. Otherwise, like I said, you
just need to try to figure out what’s coming out and why it’s coming out.
56 W. Berger

Do you ever use antibiotics or antibiotic ointments?


If it is at all purulent, the ointments aren’t really going to help, it has to be sys-
temic, and you can usually get that with pills, occasionally you have to resort to IV
antibiotics. Quite often because the foreign body is still there, and that’s what’s
really acting as the nidus of infection, I usually end up taking the tube out. Of
course, I have a pretty low threshold for doing that. I will put down an NJ tube or
something else to take care of them for 3, 4, and 5 days until that stoma cools down
and closes up and then replace the tube somewhere else.
In some cases, where PEG tubes have been in place for a long time and then
are removed because the patient no longer needs them, the tract fails to close.
How do you deal with this?
Again, consider the underlying cause. Typically, it is because the tube has been
in so long that the tract—that was originally lined with granulation tissue, which
seals up pretty quickly—has become epithelialized. Usually for those, I try using a
silver nitrate stick inside the tract to try to ablate the squamous tissue and try to cre-
ate some granulation tissue. Then, you take a four by four and fold it corner to cor-
ner until you have this little pyramid, stick that in there, and cover it over tightly
with tape. That’ll keep it from leaking. I won’t send them to surgery to close the
fistula until they’ve tried a few sessions of that over a couple of weeks. Denuding
the epithelium in the tract can be painful, so if you are going to be aggressive, first
injecting lidocaine around the stoma can be kind.
Clogged NG, NJ, and PEG tubes are a common problem. Do you have strat-
egies to deal with these?
One thing to be aware of is that crushed pills can clog a tube. Sometimes, liquid
medicines are used for this reason. However, you need to know that a lot of the
medicines that come in liquid form are sweetened, because it is assumed that they’re
for children. They’ll often use a sugar-free sweetener, usually sorbitol. By the time
you get three or four medicine doses of sorbitol a day, you have a real diarrhea prob-
lem. What people don’t appreciate is that there are other ways to avoid crushed
meds other than using liquid meds. You can just crush them very finely, or you can
disolve them with water.
Some of the more common causes for a clogged J tube include administering
meds like Carafate (which ironically is meant to be delivered to the stomach) or
potassium pills (also ironic, as this used to only come in liquid form). These will
both clog up a J tube quickly.
We will occasionally use Viokase (pancreatic enzyme) and bicarbonate to unclog
tubes. G tubes are fairly short and wide enough that they usually don’t clog. There
are devices you can twirl down a G tube to unclog it. So, it turns out that it's the J
tubes that are the biggest problem. I usually just start out with coke because it’s
cheap, it’s easy, and it’s immediately available. The carbonic acid in coke will clean
your car battery terminals and also clean out G tubes and J tubes reasonably well.
The biggest reason for clogging a lot of the J tubes out there is both NJ tubes and
G-J tubes are side holes. What happens is they’ll have a little tiny hole at the end.
That gets clogged up immediately, but feeds and meds can still come out the side
holes. Then, more stuff packs up and the most distal side hole gets plugged. Then,
one by one, all the other side holes clog up.
10 PEG Tubes 57

So, I use a J tube of polyvinyl chloride (PVC) with a blunt cut end and no side
holes. The blunt end may be more difficult to get past the pylorus. If you can maneu-
ver the blunt end into the jejunum, you can just use a 3- to 5-ml syringe, shoot some
water through, and the clog will shoot right out. Some folks, who don’t understand
basic hydraulics will put on a big ol’ 60cc syringe, but you get your most hydraulic
force using a small barrel syringe.
Case 2 A 56-year-old man had a PEG placed for bulbar palsy 2 years earlier. The
original tube had been replaced with a MIC PEG replacement a year ago, which
itself has been replaced several times for occlusions. Now, he is found to have vom-
iting after feeding and is found to have evidence for gastroparesis.
How would you manage this?
I would exchange the PEG tube for a G-J tube. I find them very easy to do,
although for some reason, most people seem to think it’s too complex or difficult,
but yes, it sounds like the patient needs it. I tracked 179 J tubes and found that about
half of those were for aspiration and about a quarter were for gastroparesis. PEG-J
seemed to work pretty well for both.
Now, many patients with gastroparesis have poorly controlled diabetes. Poor
control causes the gastroparesis, and the gastroparesis leads to retention or dump-
ing, resulting in poor control. I recall a study I think out of Iowa from the late nine-
ties which studied patients with a glycohemoglobin of at least 14, who had been
hospitalized twice in the last year for inability to tolerate feeds and been on TPN at
least once. They put those folks on PEG-J tube feedings and then used a continuous
pump, 24 h a day, and administered ultralente insulin to bring their glucose under
control. Within 2–3 months, those people were able to come off J tube feedings and
eat normally. After 2 years of follow-up, none of them ended up back in the hospital
and their glycohemoglobin stayed under 11. Okay, so in many cases, we can make
a huge difference in gastroparesis by controlling the underlying problem.
Can you describe the method(s) you like to use to place a G-J tube?
I like to start out with a large bore G tube and that’s one of the reasons that we
have the 24 French PEG set. We scope through the G tube using a 4.2-mm ultra-thin
gastroscope that will fit through a 24 Fr tube but not the routine 20 Fr PEG tube. Go
through the G tube with the thin endoscope, past the pylorus, past the ligament of
Treitz, and pass a wire down the endoscope well into the jejunum. Then, we with-
draw the scope over the wire and use the wire to place the J tube into the jejunum.
Although it may seem that this requires special equipment, most GI laboratories
have an ultra-thin “neonatal” scope (also useful for trans-nasal endoscopy). The
larger PEG tube can easily be ordered, even up to 28 Fr. Other benefits to this tech-
nique are that no medication is needed, airway issues are avoided, and it is fast. This
can also be done at the time of initial PEG placement, if you already know that a G
tube alone will be inadequate.
Case 3 A 65-year-old man is found to have laryngeal cancer. He is started on
radiation treatment, and his oral intake decreases substantially. A PEG is desired.
58 W. Berger

Would you do the routine sort of PEG in someone with laryngeal cancer?
What do you think about concerns of potentially seeding tumor doing this, do
you think they are overblown?
The literature has been a little all over the place with that. The frequency of seed-
ing is fairly low, and estimates have ranged from 0.6% to 5% of cases. It’s important
to differentiate between seeding and metastasis. Seeding means that you took laryn-
geal cancer cells and implanted the cells at the PEG site, while metastasis means
that the tumor has developed the ability to spread throughout the body on its own.
These have very different prognostic significance.
The thing is that if seeding is going to occur, it usually doesn’t show up for 6–12
months. Many of the people who have seeding are those who are going to die within
a year or two anyway. However, if you have a more viable patient, seeding can usu-
ally be taken care of with focal radiation therapy or resection, so it’s usually
manageable.
In reality, this is an uncommon complication, but I try to teach the fellows the
Russell (introducer) PEG insertion technique, which does not involve pulling the
PEG tube down the throat. This technique can also be beneficial in patients with
narrowing of the esophagus, where you are not sure if you can get your bumper to
go through or if you have a patient with narrowing high up in the larynx and you
worry that you might obstruct the airway if you drag a bumper through. In cases like
this, the Russell technique can also come in handy. This is the technique the radiolo-
gists use. In fact, we use the same kit with T-fasteners and dilators with a peel-away
sheath that IR uses.
This said, I certainly don’t criticize gastroenterologists who place PEGs through
the routine Ponsky technique even in patients with active ENT or esophageal
cancers.
Case 4 A patient is malnourished and has poor oral intake, and a swallowing exam
shows that he aspirates. As part of his workup for weight loss, an abdominal CAT
scan is done and is essentially negative. The decision is made to place a
PEG. Reviewing the CT scan, however, it appears there is not a good window for
you to do PEG placement.
Do you trust the CT interpretation, or do you find that when you attempt
PEG placement in someone like this you are often successful in spite of the CT
interpretation?
I really don’t place too much reliance on the CT because you’re going to inflate
the stomach with air, and who knows what you’ll find. The CT is showing a tiny
little collapsed balloon of stomach that’s up underneath the ribs, where it should be
when it’s empty, but when you inflate it, either with the endoscope or when a radi-
ologist pumps air in with an NG tube, then it becomes a football, and you may have
easy access.
So, it’s never bad to have a CAT scan. I’ve had cases where a CAT has been done,
and we find that the patient had an umbilical hernia repair with a big chunk of mesh
in the way. It’s good to know about the mesh, because you can’t put your PEG tube
through the mesh. If you ever hear the word mesh in a surgical repair, you definitely
10 PEG Tubes 59

want to get a CT or an ultrasound before doing a PEG so that you can map it out and
mark where the mesh is on the abdominal wall. And then you can try to be at least
2 cm away from it when you put your PEG in.
The other thing that’s interesting from a radiologic perspective is that if you have
a patient with an upper abdominal midline incision, and you’re afraid that the colon
may be adhered by scar tissue, there is a technique to make sure the colon is not in
the way of your planned PEG placement. The patient drinks about a half cup of
barium mixed with a half cup of water the day before and then does the PEG under
fluoroscopy. By that time, barium will be distributed through the colon pretty evenly,
so imaging can help me find a window, massage the colon out of the way, or confirm
that there is no window.
Case 5 A patient with advanced type 1 diabetes has refractory gastroparesis. The
desire is to be able to provide enteral feeds, while at the same time performing a
“venting gastrostomy.”
How would you do this?
I would place a 28F PEG tube and put a Y connector on before putting the J tube
in. It's a little bulky, but it works pretty well. Then, I usually use a bile collection
bag. On the bottom of that bag is a valve that you can easily open for venting and
drainage.
Also, on occasion, we’ve had patients with apparent duodenal obstruction sent
for venting gastrostomies where we have been able to place a venting G tube and a
feeding G-J tube because we were able to get our endoscope past the obstruction
and feed a wire and allow for a feeding jejunostomy. One caveat is if there is a lot
of drainage of gastric fluid and/or bile, this needs to be reduced as much as possible.
A PPI can help with gastric secretions, but bile may need to be re-infused through
the J tube. If not, significant potassium and bicarbonate loss can occur.
Case 6 A 65-year-old woman is sent to have PEG placement. At the time of the
procedure, you do not see good trans-illumination within the stomach. There is a
good finger indentation, however.
Would you proceed with this case?
I have aborted in many of these cases, but there are others where you don’t see a
good light, but there might be a good explanation for this, like they weigh 400
pounds. Sometimes I’ll go ahead and try it using the safe-track technique (see below).
Case 7 A PEG is placed in an 82-year-old woman. Placement was somewhat chal-
lenging but was eventually successful. She goes for a CT scan 2 weeks later to help
evaluate some abdominal pain, and the CT scan shows the PEG tube traversing the
liver and going into the stomach. It is not causing any bleeding.
How would you manage this? Have you had, or seen other practitioners
have, complications of PEG placement where another organ was entered? Do
you have any advice on how to avoid these complications?
This problem should be avoidable using the safe-track technique. That’s where
you get some fluid in your syringe, and you place a nice long needle on it. Then, you
60 W. Berger

advance the needle along your proposed tract aspirating as you go in. Keep an eye
on the inside of the stomach with your scope to see where the needle’s going to
come in.
Just as the needle becomes visible in the stomach, the person who is looking at
the screen will say “Needle” and the person putting the needle in and watching the
syringe should say “Air,” when bubbles suddenly aspirate into the syringe. (Note:
The person watching the syringe should not be trying to watch the screen and the
person who’s watching the screen should not be trying to glance at the syringe.)
Then, if you get the audible from both people simultaneously—in stereo—you
know that you’re in the stomach and you didn’t hit the colon or anything else. If you
get back air but don’t see the needle in the stomach, it may be passing through the
colon. If you’re not seeing the needle in the stomach but you get a big flashback of
red blood, you’re probably going through the liver.
However, in this case they’ve already gone through the liver, unaware they had
done so, and had no major clinical problems. Going through the liver is actually not
that uncommon. Jeffrey Ponsky actually reported a case where they intentionally
went through the liver to place a PEG in a kid with a huge liver that covered the
whole belly, and there was no problem.
There have been two times that I’ve encountered this scenario, and in neither
case, was there a problem. Just remember that with PEGs through the liver the prob-
lems are when it comes time to replace the tube.
The tract needs to be really mature—at least a few months. Then, the tube must
be removed atraumatically. This means endoscopically snared and pulled back up
the esophagus. If you were to try “traction removal” as the tube was designed for,
you could stimulate significant bleeding. This is if you are lucky enough to know
your tract is through the liver.
However, if you try to replace a tube in a tract without knowing it is through the
liver, you could end up with the tip of the tube in the liver parenchyma. Tube feed-
ings making their way into the hepatic vein is uncool. There is even a case report of
someone testing their tube placement with an air flush, which immediately became
a fatal air embolism.
When you replace a PEG tube do you do it endoscopically, or blindly? And
do you always do a gastrografin study to make sure it's in the right place?
The original tube is designed to be removed by traction, without endoscopic
guidance, and we have not seen significant complications when we simply pulled
them out.
We usually don’t get a gastrografin study to confirm the location inside the stom-
ach. What I do is just go ahead and feed that replacement tube in about 9 cm through
the tract, with an uninflated balloon and put suction on it. If I get back some gastric
fluid, bile, or mucus, then I know I’m in the correct place. Then, blow up the bal-
loon, cinch it up a bit, and send them home.
If I can aspirate, that’s my preferred method, I think that using gastrografin in
every case is a bit of overkill, but you want to be really careful with your replace-
ment tubes to make sure that you aspirate some good gastric fluid. I had one where
the tract was not solid, although 14 months old. It had somehow been placed through
the posterior wall of the stomach, interposing the omentum in the tract. The
10 PEG Tubes 61

replacement tube initially landed in the peritoneal cavity between the abdominal
wall and the stomach. However, when we tried to aspirate, we weren’t getting good
gastric juice back, and we knew not to start feedings.
The other thing you should do if you’re trying to aspirate fluid and you get noth-
ing back is to inject some water into the tube and then see if you can suck that back.
If you’re in the peritoneal cavity and you put 60 ML of water in, you won’t get that
return because it’s not going to sit there. If that’s the case, then we’ll definitely get
an X-ray to check on the position. So, if you can put fluid in and suck fluid out, then
you’re in the stomach.
The first thing to remember is that the first replacement is the most dangerous.
It’s not pulling out the original PEG tube that is the tricky part, it’s putting that first
replacement tube in. It depends on how long the original tube has been in. Your task
may be more daunting if the original tube has only been in for a month or two. In
that case, you could potentially rupture the tract with the pullout. If it’s less than 2
weeks, obviously this requires an emergency room visit, because you won’t have a
mature tract. On the other hand, if it’s been a year and a half, 2 years, that tract is as
solid as it’s going to be.
It turns out that in Europe it is mandated to change the primary PEG tube once a
year, but also to change any balloon tubes every 3 months. To replace, PEG tubes
involves taking a patient from the nursing home, putting them in an ambulance,
transporting them to the emergency room, having somebody take out the old tube,
and put the new tube in. If mandated to do that four times a year, it would be easier
and cheaper to just replace the primary tube endoscopically.
That’s what they usually do in Europe, just replace the primary tube. So, you
have a Ponsky tube in there with a bumper, no balloon. We can usually get at least 2
years out of a Ponsky primary tube.
When you place a new PEG do you put gauze under the bumper?
Initially, I do, on day 1. That way, any discharge or bleeding will collect in the
gauze, but when we loosen the tube the next day, they take the gauze out and leave
it out. Usually, the stoma is dry and there’s no need to have gauze there.
We’re concerned that if we leave the gauze there and the patient goes back to the
nursing home, it won’t get changed and you increase the risk of infection long term.
How tight do you make the PEG bumper, and how much do you loosen it the
next day?
We used to make it very tight to prevent pneumoperitoneum, but since we started
using CO2 instead of air during our PEG placements, we have much less problem
with pneumoperitoneum. So, we usually leave 0.5 cm of play at placement. The
trauma surgeons seem to clinch their bumpers in real tight and they are more likely
to get a buried bumper or an ulcer underneath the internal bolster, just from pressure
necrosis which can happen within a few days.
When you place a PEG, the thing is that you’re going through perpendicularly
oriented layers of muscle in the gastric wall and the abdominal wall. So, when your
needle goes through those layers, it just spreads them apart and you get a pretty
good seal. There was a paper by Ponsky, again, back in the eighties, where he inves-
tigated how tight you needed to cinch up the bumper. He put 10 PEG tubes in 10
dogs. The first one he put in and cinched it up nice and tight. The second one, he put
62 W. Berger

a centimeter loose, and the next dog was 2 cm loose and then 3 cm and so forth. He
got all the way up pushing 10 cm of tube into the abdomen. All 10 dogs did fine.
There were no leaks anywhere.
Now, I’m not leaving it that loose. I want it so that several days later there is at
least 0.5–1.0 cm of play. Now, right after the procedure I might leave 0.5 cm of play
but when we examine the patient the next day the bumper may be tight, no play,
because the abdominal wall had increased in thickness due to either edema from the
injury or hematoma.
The other thing that’s interesting is that you do have a difference in your abdomi-
nal wall thickness between when you’re laying down and sitting up. So, somebody
lying down may have 0.5–1.0 cm play, but when they sit up, there may be no play at
all. So, if it’s really cinched down, then the amount of time they spend sitting up
may be the amount of time that they’re getting pressure necrosis.
It’s one of those things to be aware of. You can’t make it too loose; you can defi-
nitely make it too tight.
Will you place a PEG in someone on DOACs or Plavix?
I try to get the patient to hold those, simply because you’re likely to get bleeding
from either the mucosa or the skin edge. If they can’t and they need the tube, I go
ahead anyway. If you get mucosal bleeding, usually that stops. That’s one of the
times that I might tighten up the internal bolster because that’ll hold pressure on it
and then loosen it the next day. Bleeding at the incision site, worst-case scenario,
you go in there with some silver nitrate and you burn it to make it stop, but I like to
see the bleeding stop before I send them back upstairs.
I will hold the DOACs for a couple of days, I will hold warfarin until the INR is
1.5. I don’t hold the antiplatelet meds, but if the patient can’t hold the anticoagula-
tion because of an acute post-stroke or some other reason, I just do what needs to
be done.
Case 8 You are attempting to place a PEG but the light shows up in an undesirable
location—either too high—at the xiphoid, or too low, or too much to the left or right.
Will you proceed in these situations? What is the risk involved? Is there
some way of minimizing the risk?
If I have to, although usually you’ll find that if you get the stomach well inflated
then you will have multiple potential sites. Inside the stomach, I would like to be
2 cm proximal to the incisura on the anterior wall of the stomach. On the outside, I
try to stay off of the midline because the Linea alba doesn’t make for a good tract,
going through the tendon. I also try to stay 2 cm off the rib margin because that can
be really uncomfortable. For those patients, it hurts every time they take a breath
because the tube is rubbing against their lower rib.
As I mentioned before, the safe-track technique can help reassure you, but I also
make sure to use tiny finger circles at candidate sites, not just finger pokes, which
can be misleading.
Case 9 A 37-year-old man who suffered severe brain anoxia following a self-­
hanging attempt has a PEG tube placed. Two weeks later, he involuntarily yanks the
tube out.
10 PEG Tubes 63

How would you manage this situation?


You generally don’t see gastric perforations in people who’ve yanked out their
PEG tubes too early. It seals up, as long as you don’t put a lot of stuff in the stomach.
I’d keep him NPO on NG suction for 24 h and then feed him via the NG tube on
days 2 and 3 and then go ahead and place another PEG.
You generally end up placing a totally new PEG rather than using the old tract.
When the tube comes out, the muscle fibers typically fall back and seal pretty
well. It may be that the tract is already established, within even a few days, but if
you go back in and you try to put a needle through where the old tract was, it rarely
comes out where the inside tract was in the stomach. So, you generally end up plac-
ing a new PEG in a totally new location.
A case reported in the Milwaukee Journal-Sentinel told of a local nursing home
patient who returned from the hospital with a fresh PEG tube, which he pulled out
on transit to the nursing home. The charge nurse, having replaced PEG tubes before,
dutifully put in a new tube and fed through it. Unsurprisingly, feeds went into the
peritoneal cavity, the patient got septic, and he died. Our post-procedure orders
clearly state that if a PEG tube comes out under any circumstances in the first 2
weeks, bring the patient to the emergency room.
How do you manage the buried bumper?
Usually, if there’s still a tract that communicates with the inside of the stomach,
I can put a wire down the tube and assuming the wire makes its way into the stom-
ach, and I grab the wire and pull the tube out over it. Then, I just place another PEG
tube over the same wire by a regular Ponsky technique and make sure the tube
length and play are appropriate and that the external bolster is not over-tight.
When you talk about using a wire to place your J tubes do you always try to
use fluoroscopy? What techniques do you use when fluoro is not available?
I use fluoro when I can, but it’s not always available. When you don’t have access
to fluoro, the way you control the wire is critical, but working with the fellows I
always hold the wire. You have to keep the wire from coiling in the stomach. The
person holding the wire has to keep his hand completely steady, because the wire
moving forward, even a few inches as you’re feeding the tube in will cause the tube
to coil up in the stomach. Then, once it’s coiled, even if you’re holding the wire firm
after that, then it just pulls the wire back and continues coiling up in the stomach.
Because I’m tall, I can just hold it up, arm length, really high, and then try to
keep the wire as straight as possible between my hand and the patient’s nose (NJ) or
G tube (PEG-J) and don’t let the wire move. One thing that can be very helpful for
the PEG-J is to loosen the external GT bumper to allow the GT to be fed into the
stomach ~10 cm toward the pylorus before feeding the PEG-J tube in. Then, there’s
only a short distance between the tube tip and the pylorus, so the tube is less likely
to coil in the stomach.
Finally, try to avoid any big loop in the wire in the stomach during scope with-
drawal, and decompress the stomach of as much gas as possible as you withdraw
your scope.
Chapter 11
Autoimmune Metaplastic Atrophic
Gastritis

Francis Edeani

Case 1 A 48-year-old diabetic male patient is referred to you by his primary care
physician for chronic osmotic diarrhea, anemia, and a sore tongue. Laboratory
evaluation reveals macrocytic anemia, and a vitamin B12 level returns low. The
diagnosis of pernicious anemia is entertained.
How would you confirm this diagnosis?
I would order intrinsic factor antibodies and anti-parietal cell antibodies. If
intrinsic factor antibodies return positive, these are highly specific for PA. However,
they are not that sensitive. Antibodies to parietal cells are more sensitive (80%) but
not that specific. In addition, I order a gastrin level. In advanced PA, there is hypo-
chlorhydria or achlorhydria with a resultant hypergastrinemia.
The patient’s intrinsic factor and anti-parietal cell antibodies both return posi-
tive, and gastrin level is elevated at 540.
How would you further evaluate this patient?
In patients with pernicious anemia, I would do endoscopy with gastric mapping.
I suspect the patient has autoimmune metaplastic atrophic gastritis (AMAG). I want
to confirm the presence of gastric atrophy, evaluate the extent of inflammation, and
look for any changes of metaplasia or dysplasia.
Which portions of the stomach tend to be involved in AMAG?
AMAG tends to involve the fundus and body of the stomach, generally sparing
the antrum. Autoimmune destruction of oxyntic mucosa leads to a decrease in acid,
pepsin, and intrinsic factor. The antrum is not involved and so antral G cells

F. Edeani (*)
Division of Gastroenterology and Hepatology, Department of Internal Medicine, GI, Medical
College of Wisconsin, Milwaukee, WI, USA
e-mail: [email protected]

© The Author(s), under exclusive license to Springer Nature 65


Switzerland AG 2023
W. H. Sobin et al. (eds.), Managing Complex Cases in Gastroenterology,
https://doi.org/10.1007/978-3-031-48949-5_11
66 F. Edeani

proliferate, stimulated by hypochlorhydria and resulting in hypergastrinemia. The


hypergastrinemia also leads to ECL proliferation, which may stimulate growth of
carcinoids.
Endoscopically what do you tend to see in AMAG?
You tend to see loss of gastric rugae in the gastric body and fundus, and because
of thinning of gastric mucosa, submucosal vessels may stand out. Uneven destruc-
tion of oxyntic mucosa may lead to pseudopolyps—representing residual oxyntic
mucosa that has not been destroyed.
Are patients with AMAG at a higher risk of developing gastric cancer?
Yes, they are at a heightened risk for gastric carcinoids and gastric adenocarci-
noma. The gastric carcinoids result from the ECL proliferation that results from
hypergastrinemia. The gastric adenocarcinoma is a result of inflammation leading to
metaplasia eventually resulting in dysplasia that over time may evolve into adeno-
carcinoma. There are various types of metaplasia that may occur—pseudopyloric
metaplasia, incomplete intestinal metaplasia, or complete intestinal metaplasia.
Because of the increased risk of gastric malignancy how often would you
repeat surveillance endoscopy?
Every 3 years and more often if there is a family history of gastric malignancy.
This patient is diabetic. Is there an increased incidence of other “autoim-
mune diseases in AMAG?”
Yes, about 1/3 of AMAG patients have autoimmune thyroid disease and 10%
have diabetes.
Case 2 A 55-year-old Caucasian man is evaluated and referred to gastroenterol-
ogy for endoscopic evaluation of occult blood loss anemia. An EGD is performed,
which shows erythema and edema in the gastric mucosa of the antrum. Biopsies
revealed atrophic gastritis, and Warthin silver stain reveals H. pylori organisms.
What is the association between HP and atrophic gastritis?
The most common association is environmental metaplastic atrophic gastritis
(EMAG). EMAG differs from AMAG in that antibodies for IF and parietal cells are
generally not present, and gastrin levels are normal. HP is the most common cause
of EMAG, but HP has also been linked as a possible trigger in some cases of AMAG.
EMAG differs from AMAG in other ways as well. Atrophic gastritis can be pres-
ent anywhere in the stomach in EMAG and actually is most common in the antrum
(which is usually spared in AMAG). Normal gastrin levels mean that ECL hyperpla-
sia and carcinoid tumors are uncommon.
EMAG may progress to dysplasia and there is an increased incidence of gastric
adenocarcinoma just as with AMAG. Indeed, HP is the most common etiologic fac-
tor associated with gastric adenocarcinoma.
Can you prevent the development of gastric adenocarcinoma by eradicating
HP in cases of EMAG?
It is clear that treating HP will not prevent all cases of adenocarcinoma. Once
dysplasia has occurred, the disease has usually progressed too far. Indeed, once
11 Autoimmune Metaplastic Atrophic Gastritis 67

extensive intestinal metaplasia has occurred many cases may already be too far
gone. However, it is felt that if you treat EMAG prior to development of extensive
metaplasia most cancers may be prevented.
Do all polypoid lesions have to be removed in AMAG and EMAG?
No, many polyp-like lesions are actually pseudopolyps, normal viable oxyntic
mucosa that remains adjacent to areas of oxyntic destruction, but all polypoid
lesions should be biopsied to make sure there is no dysplasia or cancer.
You mention gastric mapping. What technique do you recommend?
I would get at least two biopsies from both the antrum and the body, biopsying
both lesser and greater curvature, and one from the incisura. In addition, any suspi-
cious or polypoid lesions should be biopsied. If polypoid lesions are biopsied, you
should also biopsy flat areas adjacent to the polyps.
It is crucial to label the biopsies from the antrum and body specifically. There is
a form of metaplasia where mucosa in the body of the stomach can be replaced by
antral-type mucosa, termed pseudopyloric metaplasia. The only way to make this
distinction histologically is to label the biopsies from the corpus separately.
Chapter 12
Subepithelial Lesions and NETs

Phillip Chisholm

Case 1 A 68-year-old man has an EGD performed for chronic heartburn. He is


found to have mild grade A esophagitis accounting for his symptoms, but inciden-
tally noted is a lesion indenting the gastric fundus, about 1.5 cm in size. The overly-
ing mucosa appears normal.
How would you determine the location of this lesion and its malignant
potential?
Lesions that are present under the mucosal layer of the stomach may be intramu-
ral or extramural. These are usually found incidentally at the time of endoscopy-like
in this case.
Extramural lesions could include surrounding organs, or blood vessels, pancre-
atic pseudocysts, or liver cysts. Intramural lesions could be tumors or intramural
vessels.
Determining the etiology of the lesions can be done with tunneled biopsies—
where using a jumbo biopsy forceps multiple biopsies are done, one on top of the
next. More often we evaluate them with EUS. EUS allows a determination of the
location of the lesion, what layer it’s located in, and allows for FNA to arrive at
histology.
Many of these lesions are submucosal, and one of the most common gastric sub-
mucosal lesions is the GIST. On EUS, these lesions are hypoechoic, usually in the
fourth layer, rarely in the second. GISTs are most commonly found in the stomach
(40–60%), next most frequently in the small intestine (20–30%), and less common
in the colon and rectum (15%), and <1% are found in the esophagus. On histology,
GISTs are spindle cell tumors that stain positive for C-kit+ (CD117) and CD34.

P. Chisholm (*)
Hep Division/Department of Medicine—GI, Medical College of Wisconsin,
Milwaukee, WI, USA
e-mail: [email protected]

© The Author(s), under exclusive license to Springer Nature 69


Switzerland AG 2023
W. H. Sobin et al. (eds.), Managing Complex Cases in Gastroenterology,
https://doi.org/10.1007/978-3-031-48949-5_12
70 P. Chisholm

The differential of submucosal gastric lesions includes leiomyomas. These are


also usually located in the fourth layer on EUS, occasionally the second. These are
differentiated from GISTs by histology, which reveals smooth muscle actin and
stains are negative for spindle cells and C-kit.
A third submucosal lesion is the lipoma. These are most common in the colon,
but can be present anywhere in the GI tract. The overlying mucosa appears normal
but is occasionally yellow. When using biopsy forceps to push on the lesion, there
is the characteristic pillow or tent sign.
The patient has an EUS with FNA, and histology is consistent with a GIST.
How would you manage this lesion?
For gastric GISTs <1 cm in size, observation alone is recommended. For lesions
>2 cm, it is felt that surgical resection is necessary. This lesion is 1.5 cm in size, and
careful follow-up observation is warranted unless the patient wants to proceed with
surgery.
Case 2 A 59-year-old man was found, on screening colonoscopy, to have a small
subepithelial lesion in the rectum removed with snare cautery that revealed a well-­
differentiated neuroendocrine tumor. It was completely excised, and there was no
lymphovascular invasion. The quantity of Ki67 was <1%. It was felt to be a rectal
carcinoid.
What is the significance of finding a neuroendocrine tumor in the GI tract?
Neuroendocrine tumors of the gut are generally termed carcinoid tumors.
Neuroendocrine tumors in the pancreas are pancreatic islet cell tumors or PNETs.
Some neuroendocrine tumors may be functioning, secreting hormones, while others
are non-functioning. Functioning tumors include VIPomas, gastrinomas, insulino-
mas, and glucagonomas. These are usually PNETs. Small bowel carcinoids may be
functional, releasing serotonin and other vasoactive amines.
The patient described above has a small rectal carcinoid. Not expecting this to be
functional we wouldn’t bother with hormonal testing. Endoscopic resection is gen-
erally curative.
Case 3 A 63-year-old man without a significant past medical history is admitted to
the hospital with melena and acute anemia. An EGD is performed, which reveals a
2.5 cm ulcerated lesion in the gastric body. The remainder of the examination is
unremarkable. Biopsies obtained from the lesion reveal a gastric neuroendocrine
tumor, a carcinoid. A gastrin level returned 25. It was felt that the patient probably
had a type 3 carcinoid.
What is the implication of this finding? Is this usually as innocent as the
rectal carcinoid described earlier?
Gastric carcinoids have been divided into three types. Type 1 is generally innocu-
ous and represents a growth of enterochromaffin-like cells (ECL) as a reaction to
hypergastrinemia. These patients have atrophic gastritis, often with associated per-
nicious anemia. The elevated gastrin is an appropriate reaction to hypochlorhydria
or achlorhydria. This has also rarely been described in patients on chronic PPI ther-
apy. Type 1 gastric carcinoids have a very low malignant potential.
12 Subepithelial Lesions and NETs 71

Type 2 gastric carcinoids are also a reaction to hypergastrinemia and also grow
from ECL cells. They are often multifocal. In type 2 carcinoids, the tumors arise
from hypergastrinemia from ZE syndrome. These carcinoids are usually benign,
but there needs to be further searching to locate another biologically active NET
of the pancreas or chest, which is secreting gastrin. These tumors may be
malignant.
Type 3 gastric carcinoids also develop from ECL cells. However, the growth of
these cells is not stimulated by hypergastrinemia. These carcinoids tend to have a
high malignancy risk with poor prognosis. These cases require surgical resection
and lymphadenectomy.
Case 4 A 51-year-old woman with a history of hypothyroidism presents with symp-
toms of epigastric discomfort and fatigue. She has no heartburn and denies nausea,
vomiting, or weight loss. Her sister died from gastric adenocarcinoma at age 48.
Laboratory studies show a Hbg of 11, ferritin of 6, iron of 18, and vitamin B12 of
100. Upper endoscopy reveals atrophic mucosa with a 3-mm polyp in the fundus
that is removed. Histology reveals a monomorphic collection of neuroendo-
crine cells.
How would you interpret this case?
This patient almost certainly has pernicious anemia with associated gastric atro-
phy with hypochlorhydria or achlorhydria and intrinsic factor deficiency. These
patients tend to have a rebound hypergastrinemia that stimulates growth of the ECL
cells. In addition, the achlorhydria is associated with decreased iron absorption and
a mixed anemia, both IDA and megaloblastic type (the latter due to low vitamin
B12). This fits the criteria for a type 1 gastric carcinoid, which tends to run a very
benign course.
What further testing should be done in a patient with suspected gastric
carcinoid?
Further testing might include checking a gastric pH, as well as biopsies for HP. A
fasting gastrin level should be checked off of PPI. In patients with elevated gastrin
suspected of having Zollinger-Ellison syndrome, a secretin test should be per-
formed. In normal controls, secretin should cause a decrease in serum gastrin. In
Z-E syndrome, there is an inappropriate rise in serum gastrin. Some of these patients
will also benefit from a CT chest and abdomen along with somatostatin scintigra-
phy—either dotate or octreotide PET scan.
What about small bowel carcinoids?
Small bowel carcinoids account for a large number, about 40% of small bowel
malignancies. They are most common in the ileum. Small bowel carcinoids tend to
be slow-growing tumors, but they cause an intense desmoplastic reaction that might
lead to small bowel obstruction. In addition, some of these lesions are functioning,
releasing serotonin and other biologically active amines. These can cause a carci-
noid syndrome that includes diarrhea, flushing, and hypotension. Usually, carcinoid
syndrome from small bowel carcinoids occurs in patients who have liver metasta-
ses. It is diagnosed by an elevation of 5 HIAA.
Chapter 13
Viewpoints on Managing Common Clinical
GI Disorders from a Practitioner
with Over 50 Years of “Real-World”
Experience

Helmut Ammon

Constipation

Case 1 A 35-year-old woman is having problems with chronic constipation. She


would prefer to use a natural product because she doesn’t like the idea of tak-
ing drugs.
Our first choice is Metamucil, which works in over 30%, maybe 50% of our
patients. We start out telling the patient to take one packet of Metamucil in a glass
of water, with a meal, preferably breakfast. If taken before a meal, patients tend to
fill up too quickly. Usually, one packet a day is sufficient; rarely, a patient needs a
second packet with lunch or dinner. We tell the patient that their constipation is a
result of not having enough water in their stool and that Metamucil mixed in water
takes care of that problem along with providing bulk.
The patient tries the Metamucil but stops it because she gets too gassy, even if
she cuts back to half a packet a day.
She agrees to try some drug. What do you like to use here?
Everyone these days seems to prescribe Miralax, which works well, but another
agent to keep in mind, which has been used over the decades, is milk of magnesia
(MOM). It’s not the most fashionable drug these days, but it works quite predict-
ably, is quite cheap, and can be used on a chronic basis, as long as the patient
doesn’t have renal insufficiency. We start with one tablespoon a day, but if needed
you can go to one tablespoon two or three times a day, with meals, with plenty of
fluid [1].

H. Ammon (*)
Division of GI and Hepatology/Department of Medicine, Medical College of Wisconsin,
Milwaukee, WI, USA
e-mail: [email protected]

© The Author(s), under exclusive license to Springer Nature 73


Switzerland AG 2023
W. H. Sobin et al. (eds.), Managing Complex Cases in Gastroenterology,
https://doi.org/10.1007/978-3-031-48949-5_13
74 H. Ammon

The important thing to remember in treating constipation is not to overdo things


and create severe diarrhea, where the patient gets tempted to take Lomotil or
Imodium. You want to be consistent, and not get into a pattern of blasting them out
and then going in the opposite direction, and then having to go back and forth. You
want to get them on a regular regimen they can adhere to where they take the same
dose of medicine every day and have an adequate bowel movement on an ongo-
ing basis.
When you start patients on treatment for constipation, they shouldn’t expect to
experience a bowel movement the very next day, it takes time to prime the pump.
They needn’t worry if it takes a couple of days for them to start going, there is noth-
ing wrong with that.
How do you feel about the use of Surfak or Colace?
These meds are surfactants, they are detergents, and they all inhibit electrolyte
and water absorption and stimulate secretion. Generally, they are quite effective, but
you can get the same effect by using olive oil. These triglycerides are broken down,
and their fatty acids act as detergents. Some people use linseed oil, and in the old
days, they used castor oil. Ricinoleic acid, the fatty acid in castor oil, is poorly
absorbable, which makes it effective [2]. We tell patients to take one or two table-
spoons of olive oil and put it on their salad, or cook with it, and get the same laxa-
tive effect.
How about the use of senna or bisacodyl (Dulcolax)?
Senna is considered safe [3]. Its long-term use can cause melanosis coli.
Bisacodyl can cause significant mucosal injury [4, 5]. For this reason, we don’t like
to use it on a chronic basis. Dulcolax suppositories have been known to cause rectal
ulcers due to their local effects.
How do you feel about the use of anorectal manometry?
We usually reserve this for patients who don’t get relief from their constipation
with our medications. Our patients appreciate the fact that we are trying to come up
with a scientific explanation for their symptoms, using anorectal manometry.
Unfortunately, when you diagnose dyssynergic defecation, it turns out that many
patients are not good candidates for biofeedback. In the end, there are relatively few
patients who will benefit from anorectal manometry.

Acute Diarrhea

Case 2 A 38-year-old man who you have seen previously calls with the complaint
of new-onset diarrhea of 5-day duration without any clear-cut precipitating factors.
In a case like this is it necessary to do any testing? How would you manage
the patient? Is Pepto-Bismol useful?
Most patients with this problem have self-limited viral, or less often, bacterial
diarrhea that will resolve with no treatment, so there is no indication for stool cul-
tures. We simply tell patients to stay well hydrated, occasionally recommending
13 Viewpoints on Managing Common Clinical GI Disorders from a Practitioner… 75

Gatorade. We do not prescribe Pepto-Bismol. It would be hard to appreciate whether


Pepto was of any benefit, since the diarrhea will generally resolve in a few days,
with or without treatment.

IBS-D

Case 3 A 28-year-old woman is having diarrhea and cramps over the past 6
months. Her primary care doctor orders laboratories, including a celiac panel and
stools, including stool for calprotectin, and all tests return negative. He prescribes
a low FODMAP diet but she gets no relief. She is referred to you for a second
opinion.
How would you proceed?
I assume you are trying to describe a patient with functional complaints. In real-
ity, you end up with a colonoscopy to exclude organic disease first. The main tool
here is the history. Why in the world did this 28-year-old woman develop these
symptoms at this time? What happened in her life to precipitate this change? We ask
the patient to tell us her story because oftentimes the answer is in the telling. When
there is a strong psychological underpinning, we may refer her to a psychologist or
psychiatrist. Most gastroenterologists are not trained to take a detailed psychologi-
cal history.
In terms of management, we often try Metamucil first, and it absorbs excess
water and provides bulk, which may be beneficial. Imodium can certainly be used.
An antispasmodic occasionally works. Peppermint oil is an over-the-counter anti-
spasmodic that is inexpensive. Some patients will respond to dicyclomine or hyo-
scyamine. It is important to reassure the patient of the benign nature of her symptoms,
which can produce anxiety. Oftentimes medication is not necessary, and we adhere
to what Marvin Schuster once said, “Don’t medicalize IBS, don’t make it into a
disease, it’s part of the human condition.”
What about using cholestyramine in IBS-D or post-cholecystectomy
diarrhea?
For cholestyramine to work, there must be some bile acid malabsorption.
According to Camilleri, 25–50% of patients with IBS-D have bile acid malabsorp-
tion, and it is not so clear whether they all will respond to cholestyramine, but it may
be worth a trial [6, 7]. However, it’s a different story if the patient has had a chole-
cystectomy, in which case there may be bile acid malabsorption, which can cause
diarrhea and cramps, simulating IBS-D. These patients will frequently respond to
cholestyramine.
Cholestyramine works best when given with meals. The bile acid pool is largest
in the morning, so giving it with breakfast has the largest benefit. Cholestyramine
binds bile acids as they get dumped into the small intestine in response to a meal.
Patients with post-cholecystectomy diarrhea respond well to cholestyramine, but
oftentimes this is a self-limited disease that corrects over time. Before embarking on
76 H. Ammon

long-term treatment, a therapeutic trial with cholestyramine 4 g with each meal and
at bedtime for 72 h will tell. If the diarrhea is not controlled, it is not caused by bile
acids. Most patients get away with taking it with breakfast or with breakfast and
lunch. Since cholestyramine can bind medications as well, the patient needs to be
instructed to take all medications at least 1 h apart from cholestyramine. Patients
taking cholestyramine on a chronic basis are at risk of developing deficiencies in
fat-soluble vitamins and need to be monitored accordingly.

 astroparesis (We Are Presenting Several Scenarios Here


G
for Comparison)

Cases 4–6 The first patient is a 35-year-old woman who complains of chronic full-
ness and nausea but has no history of diabetes. She experiences significant weight
loss of about 15 pounds over 2 months. She claims to avoid eating because it exac-
erbates her nausea. An EGD is performed and is negative except for mild gastric
retention of semi-solid food. A gastric emptying scan at 4 h showed 30% retention
(normal < 10%).
The second patient is a 50-year-old obese woman with type 2 diabetes of 5-year
duration who gives a history of persistent nausea and vomiting. Her hemoglobin
A1C is 7. EGD and a gastric emptying scan are consistent with gastroparesis.
The third patient is a 55-year-old diabetic patient with poorly controlled diabe-
tes and a HbA1C of 11. She suffers from peripheral neuropathy in her toes and
retinopathy.
In the first case, history is very important. What happened to this 35-year-old
woman to create these symptoms at this point in her life? Was there some precipitat-
ing cause, any depression or other factor? When patients are nauseated, they usually
have delayed gastric emptying but what came first, the nausea or the
gastroparesis?
The most worrisome thing in this history is the fifteen-pound weight loss. Has
she had a full work-up for weight loss? Might she have an eating disorder? Take a
calorie count to document whether there has been decreased caloric intake.
In the second case, we question the correlation between diabetes and gastropare-
sis seeing how the diabetes is only of 5-year duration and there is no peripheral or
autonomic neuropathy.
In the third case, it is clear that diabetes is an important factor. In dealing with
patients with gastroparesis and diabetes that is poorly controlled the first thing you
need to do is to get the diabetes under better control. However, an interesting pearl
about diabetic gastroparesis is that the symptoms may fluctuate. There are times
when symptoms are worse and times when they are better. Sometimes the symp-
toms correlate with diabetes control, and sometimes they don’t.
In treating gastroparesis, we generally try a prokinetic agent first, and it is usu-
ally metoclopramide. If the metoclopramide is helping, we will continue it for 4–6
13 Viewpoints on Managing Common Clinical GI Disorders from a Practitioner… 77

weeks, we avoid using it any longer. The longer the patient takes metoclopramide
the risk of tardive dyskinesia gets much higher and the efficacy of the drug dimin-
ishes, because of tachyphylaxis. So, we usually use a 4 to 6-week cutoff. You don’t
want to get complications from your therapy.

GERD

Case 7 A 35-year-old man with chronic heartburn takes PPIs and his symptoms
keep recurring whenever he tries to stop the medicine. Occasionally, he gets break-
through symptoms despite once daily PPI. An EGD is performed off of PPIs, which
returns negative.
Do you have a favorite PPI? Is there one that is superior? How long will you
maintain a patient on PPIs? What do you make of the fact that the PPI is work-
ing inconsistently?
Our favorite PPI is the PPI that works for that patient (unfortunately we’re often
stuck with the one the insurance company will pay for). Not all PPIs work the same
in all patients, some will respond better to one PPI than another. We generally try
omeprazole first and go from there.
If a patient is getting a suboptimal result from his PPI, the first thing to do is to
make sure he is taking it properly, a half hour before the meal. If they are not
responding to several different PPIs, it is possible they are a fast PPI metabolizer, in
which case they may do better with rabeprazole [8].
We believe in maintaining PPIs as long as they are needed, we are not that con-
cerned about long-term side effects from PPIs, which seem questionable. We feel
it’s more important to keep the patient out of the hands of the surgeons for as long
as possible. If patients need to remain on PPIs indefinitely, we are comfortable
with that.

Microscopic Colitis

Case 8 A 67-year-old woman presents with chronic diarrhea. A screening colonos-


copy 2 years ago was negative. The patient has been on statins for 20 years.
The patient had a negative colonoscopy. How would you evaluate her now?
In this age range, microscopic colitis is a distinct possibility. A negative colonos-
copy (without biopsy) 2 years ago does not rule that out. So, performing a colonos-
copy with biopsy is indicated.
The colonoscopy is grossly normal but biopsies show microscopic colitis.
How would you proceed?
Our first choice is usually budesonide.
How long will you continue budesonide?
78 H. Ammon

If budesonide works, the condition often improves, and you can taper off the
medicine. If you stop the medicine and symptoms return, they usually don’t recur
immediately, so the patient usually experiences a hiatus in drug therapy. If there is a
clear relapse, we restart budesonide and try to taper the dose to 6 mg or 3 mg. In
most cases, long-term therapy is not needed, but if it is necessary, we are generally
comfortable with long-term treatment with 3 mg and to some extent, 6 mg.

Fecal Incontinence

Case 9 An 83-year-old woman presents with the complaint of occasional fecal


incontinence. She is aware of the urge to defecate, but when these “incidents” occur
she can’t “hold it in” long enough. This happens about once a week, and during
these episodes, she actually defecates into her pants. There is nothing to suggest
that she has constipation with overflow incontinence. On your rectal examination,
she has some anal tone, but it is diminished. Her last colonoscopy was 5 years ago
and was negative except for diverticulosis. Her only meds are anti-hypertensives
and statins.
What are your considerations in managing this patient?
In a woman with anal incontinence, an obstetric history is important. Were there
vaginal deliveries, and if so, how many? Were there any lacerations at the time of the
deliveries? We want to know if there is a rectocele or cystocele. So, if there was a
significant obstetric history, we definitely want the patient to get a good pelvic
examination.
What is the stool pattern? If the stool is soft or watery or even of normal consis-
tency, we might try Imodium first. If the patient has hard balls of stool we probably
would not. Metamucil can also be helpful in some of these patients.
We also want to know when the incidents occur in relation to the time of day, the
time of meals, and the type of food eaten. If a connection can be established, a
behavior may be able to be changed.
The question of whether to proceed with defecography in an 83 years old, if a
rectocele is highly likely, is debatable. If the patient is not a good surgical candidate,
it is not worth putting her through the examination, and in elderly patients, you
don’t want to be overly aggressive.

Diverticulitis

Case 10 A 35-year-old man comes in with a history of two episodes of diverticulitis


in the last 9 months.CAT scan documented sigmoid diverticulitis in both cases. Both
episodes were treated with outpatient antibiotics.
13 Viewpoints on Managing Common Clinical GI Disorders from a Practitioner… 79

How would you manage this patient?


The unusual thing in this case is the patient’s young age. We haven’t been told
whether he has diffuse diverticulosis, or diverticulosis that is limited to the sigmoid,
which may alter our management.
In this case it was limited to the sigmoid.
The fact that the patient has had two episodes of diverticulitis in 9 months at such
a young age makes it very likely that he will have more episodes in the future. We
would put him on a high-fiber diet, and Metamucil. We would have a low threshold
to refer him to a surgeon if there was another episode of diverticulitis in the future.
At this age, the likelihood of recurrence is high, and he is a good surgical risk, but
you need to know your surgeon, and a colon resection for diverticulitis is not always
a walk in the park.

References

1. Loening-Baucke V, Pashankar DS. A randomized, prospective, comparison study of polyethyl-


ene glycol 3350 without electrolytes and milk of magnesia for children with constipation and
fecal incontinence. Pediatrics. 2006;118(2):528–35.
2. Ammon HV, Thomas PJ, Phillips SF. Effects of oleic and ricinoleic acids on net jejunal water
and electrolyte movement. Perfusion studies in man. J Clin Invest. 1974;53(2):374–9.
3. Godding E. Laxatives and the special role of Senna. Pharmacology. 1988;36:230–6.
4. Saunders DR, Sillery J, Rachmilewitz D, Rubin CE, Tytgat GN. Effect of bisacodyl on the struc-
ture and function of rodent and human intestine. Gastroenterology. 1977;72(5 Pt 1):849–56.
5. Saunders DR, Haggitt RC, Kimmey MB, Silverstein FE. Morphological consequences of
bisacodyl on normal human rectal mucosa: effect of a prostaglandin E1 analog on mucosal
injury. Gastrointest Endosc. 1990;36(2):101–4.
6. Camilleri M. Bile acid diarrhea: prevalence, pathogenesis, and therapy. Gut Liver. 2015;9:332–9.
7. Camilleri M, Vijayvargiya P. The role of bile acids in chronic diarrhea. Am J Gastroenterol.
2020;115(10):1596–603.
8. Sachs G, Shin JM, Howden CW. Review article: the clinical pharmacology of proton pump
inhibitors. Aliment Pharmacol Ther. 2006;23(Suppl. 2):2–8.
Chapter 14
Small Intestinal Bacterial Overgrowth

Benson Massey

Case 1 A 64-year-old woman presents with abdominal distension, flatus, diarrhea,


weight loss, and anemia. She has malodorous, fatty, foul-smelling stools several
times a day. She has systemic sclerosis with significant GI tract involvement. In
addition, she has had recurrent bouts of small bowel obstruction, which have led to
repeated explorations and resultant adhesive disease. In addition, she’s had explor-
atory laparotomies for profound pneumoperitoneum and pneumatosis in the bowel,
but no perforation or ischemia has been identified. On presentation, she has anemia
with low vitamin B12, hypoalbuminemia, low vitamin D, and elevated INR.
Do you think this patient has SIBO?
Yes, I think it’s very likely that the patient has many manifestations of SIBO
syndrome [1] and has many factors that would predispose to having SIBO. In SIBO,
you have an abnormal accumulation of primarily anaerobic bacteria in the small
bowel, in sufficient quantities to disrupt normal intestinal digestion, absorption, and
transit, leading to various clinical manifestations.
What are the consequences of this anaerobic proliferation? One is consumption
of vitamin B12 by the bacteria within the gut. They use this as part of their metabo-
lism. Interestingly, however, while bacterial metabolism markedly reduces vitamin
B12 it often generates folate. So, patients with SIBO will often have a very high
serum folate, along with a low vitamin B12. In addition, not only does bacterial
metabolism in SIBO lead to B12 deficiency, but it also causes breakdown of carbo-
hydrates and proteins so that they’re no longer available for human nutrition. In
addition, bacterial action will cause bile acid hydrolysis. As a result, we don’t have

B. Massey (*)
Division of Gastroenterology and Hepatology, Department of Medicine, Medical College of
Wisconsin, Milwaukee, WI, USA
e-mail: [email protected]

© The Author(s), under exclusive license to Springer Nature 81


Switzerland AG 2023
W. H. Sobin et al. (eds.), Managing Complex Cases in Gastroenterology,
https://doi.org/10.1007/978-3-031-48949-5_14
82 B. Massey

bile acids to help micellize fats. Therefore, fat, protein, and carbohydrate deficien-
cies are encountered. Bacterial overgrowth also leads to damage to the intestinal
lining, resulting in reduced disaccharide levels. The divalent sugars are not absorbed
and empty into the colon to be fermented by bacteria, resulting in an increase in the
luminal osmolality, giving us an osmotic, laxative-type diarrhea.
Furthermore, because of bacterial enterocyte damage, particularly in the distal
small bowel, we can have a reduction of the enterohepatic circulation of bile acids.
Therefore, more bile acids will leak into the colon, which stimulates further fluid
secretion, yet another reason to have diarrhea.
As a consequence of this anaerobic proliferation, many patients have diarrhea,
some have steatorrhea and weight loss, and in severe cases, we can see severe
protein-­calorie malnutrition, with low serum albumin and edema. As a result of
vitamin B12 losses, we can see manifestations of pernicious anemia and peripheral
neuropathies.
After being seen in the ER, the patient gets a KUB that demonstrates massively
dilated loops of bowel, but she also has air under the diaphragm and there is pneu-
matosis intestinalis. CAT scan shows no other signs of perforation, ischemia, or
inflammation.
Now, in this case, the patient has pneumoperitoneum and pneumatosis with-
out obvious ischemia, perforation, or obstruction. Can this be due to bacterial
overgrowth?
Yes, with all the tremendous fermentation that’s going on, these patients often
complain of bloating and excessive flatus. In extreme cases, the increased gas from
bacterial fermentation can lead to pneumatosis intestinalis and, rarely, even pro-
found pneumoperitoneum.
What are the mechanisms that predispose a patient to develop SIBO?
The most common is impaired clearance through the gut. We see this in patients
with intestinal strictures, patients who’ve developed adhesions, chronic partial small
bowel obstruction, and patients with multiple small bowel diverticula. These can all
act as areas of stasis that become a nidus for bacterial proliferation. In addition,
some patients have coloenteric fistulas that lead to a direct backflow of bacteria into
the small intestine.
The next most common etiology for delayed clearance would be impaired motil-
ity. This woman has systemic sclerosis, which is a classic disorder associated with
delayed motility. This can also be seen in chronic intestinal pseudo-obstruction and
certain neurodegenerative disorders, such as multiple sclerosis. Certainly, we always
have to remember drugs, particularly narcotics.
Another etiology for SIBO is impaired bacterial killing. One example of this is
the patient with profound achlorhydria, as is seen in autoimmune gastritis. This can
also happen in certain bypass situations where the stomach has been mostly
excluded.
Yet another mechanism is when there’s excess substrate for bacterial growth,
because, normally, nutrients are being cleared (absorbed) from the small bowel, but
if you have chronic pancreatitis, or if you have a reduction of lipase and amylase for
any reason, you may have nutrients sitting around longer in the small intestine,
encouraging the development of bacterial overgrowth.
14 Small Intestinal Bacterial Overgrowth 83

We can also see bacterial overgrowth with immune deficiencies such as common
variable immunodeficiency.
The patient underwent a glucose breath test where glucose was administered and
breath samples were collected for hydrogen and methane over a course of 3 h.
Normally, the fasting values for hydrogen after glucose administration are less than
10. This patient started out with an extraordinarily high baseline of 238 and subse-
quently had whopping elevations after administration of glucose. The diagnosis of
SIBO was confidently confirmed.
How do you treat your patients with SIBO?
There are no controlled studies on this, but I will start with the cheapest antibi-
otic that the patient hasn’t already been exposed to. I keep them on this until they
develop resistance, which will eventually happen. Hopefully, you can get 6 months,
maybe a year, out of the antibiotic, but at some point, it’s going to fail, and then, you
have to switch to another agent, and then, after that one fails, switch to yet
another one.
In the past, I’d use agents such as metronidazole or fluoroquinolones, but it’s
been clear for years now that there are potential toxicities with long-term use of
these, such as peripheral neuropathy and tendon rupture, and so, I avoid their long-­
term use.
I will say that in my experience, rifaximin, usually in the dose of around 1200
milligrams a day, is actually quite helpful, but the problem is that it does not have
an FDA-approved indication for treating SIBO, so we’re using it off-label, and it’s
very expensive. It’s very hard to get it covered by insurance even after appeal.
The patient receives three courses of different antibiotics and responds for
months at a time. Now, she is no longer responding after trials of various
antibiotics.
What do you do for the patient who initially responded to several rounds of
antibiotics but now nothing is working?
First, I would confirm that the SIBO condition is still active. I would do an
updated breath test on the current therapy, and if it’s abnormal, I would get a duode-
nal aspirate for quantitative culture and sensitivity. Using this, we can try to find out
what the dominant organism or organisms are and what antibiotics they are suscep-
tible to. We can also make sure the patient hasn’t developed yeast overgrowth from
all these antibiotics, but in severe end-stage cases, you may need to resort to an
elemental diet or TPN.
Case 2 This is a 24-year-old law student who’s been having intermittent abdominal
cramps, occasional rectal urgency, and some diarrhea. She has a frequent sense of
bloating and passing gas after meals.
She’s working all day to get her law degree, neglecting a lot of healthy lifestyles,
eating a highly processed takeout diet, and has gained 10 pounds since the start of
the semester.
Because of her GI symptoms, she started searching online and read about
SIBO. She took a home lactulose breath test where she collected breath samples
after consuming some lactulose and sent these off to the commercial laboratory.
Her early hydrogen readings were all less than 10 until 60–75 min out where they
84 B. Massey

went up to the 30–45 range. She did feel slightly bloated at that time. This was
reported out as a positive breath test for SIBO, and she was instructed to contact
her physician for antibiotic treatment.
She has no other past medical history and is not taking any medications. When
you examine her, she has an elevated BMI; otherwise, her examination is normal
and her laboratory studies are normal.
Do you agree that this patient has SIBO?
I think it is dubious. If you’re considering the diagnosis of SIBO in a patient, you
have to consider what predisposing conditions might be leading up to it. In the
absence of those predisposing conditions, the diagnosis is unlikely [1], and clearly,
many of the symptoms that may be attributed to SIBO, including diarrhea and fla-
tus, are much more commonly seen in various functional disorders such as func-
tional bloating, IBS-D, and IBS-C.
Then, we have to realize that the testing for SIBO really is imperfect. There is no
gold standard that’s agreed on. So, with imperfect testing, if you’re not careful about
who you choose to investigate for SIBO, you will have a lot of false positives.
However, let’s step back a minute and talk about breath testing, which is the main
way people try to diagnose SIBO in this day and age. There are some generally
agreed-upon principles about breath testing. One is that human metabolism does not
produce hydrogen or methane. They are a product of bacterial metabolism, and
when hydrogen or methane are produced, they diffuse into the circulation at what-
ever site they’re being produced, be it mouth, small intestine, large intestine, or
bladder, and then, they’re transported to the lungs, and these gasses are expelled
during exhalation. These can be detected with very sensitive gas analyzers. So, if
you see hydrogen and methane in someone’s breath, it tells you that bacterial fer-
mentation is going on somewhere in their body. It does not tell you where this is
occurring or just what is being fermented.
By itself, the production of hydrogen or methane does not confirm that a disorder
like SIBO is present. Why is that? First of all, we have to realize that fermentation
in the gut is a normal process that depends on what you eat and where the bacteria
are located. Levels of bacteria are low in the stomach and proximal small bowel and
orders of magnitude higher in the colon than in the small bowel, even in someone
who has small bowel bacterial overgrowth. Therefore, when you give a patient a
carbohydrate test substance and detect a rise in hydrogen or methane production in
the breath, you have to be certain that the production is not occurring in the colon
before you diagnose the patient as having SIBO.
The problem with using lactulose for breath testing is that once this undigestable
(by humans) carbohydrate reaches the colon, bacteria there will metabolize this and
generate gas that can be detected in the breath. Most people using this test make the
assumption that SIBO must be present if hydrogen or methane concentrations in
breath specimens rise significantly (10–20 ppm) before 90 min. This is based on an
assumption that the orocecal transit time is 90 min or longer. However, in a land-
mark study of diarrhea-predominant IBS patients when the lactulose was given with
a radioactive tracer, so that the location of the lactulose bolus could be determined
when the breath hydrogen levels increased, it was found that in 79% of the
14 Small Intestinal Bacterial Overgrowth 85

“abnormal (hydrogen rose before 90 min)” tests, the rise in hydrogen actually
occurred after the bolus had already reached the colon. Indeed, the mean orocecal
transit time in these cases was 41 min, well under 90 min. This is consistent with
prior work showing that functional diarrhea patients have a more rapid orocecal
transit time. Therefore, using lactulose breath testing without concurrently measur-
ing the orocecal transit time leads to a tremendous rate of false-positive tests [1].
The only reasonably reliable test result with lactulose is a negative one.
How about glucose breath testing, which was the gold standard for breath test-
ing? The working assumption for a glucose breath test is that all glucose is com-
pletely absorbed within the small intestine. Therefore, if you see a rise in hydrogen
or methane, after giving glucose, this has to be due to bacteria in the small bowel
metabolizing glucose. This is a reasonable assumption so long as you can be certain
that the patient does not have rapid gastric emptying, rapid small bowel transit, or
small bowel mucosal disease. Any of these three factors could lead to incomplete
small bowel absorption of glucose, resulting in colonic fermentation of that glucose,
and hence a false-positive glucose breath test.
At MCW, we have the ability to perform glucose breath testing with concurrent
scintigraphy, and so, we can monitor where the head of the bolus is when we start
detecting bacterial fermentation. We only make the diagnosis of SIBO if the gas
levels rise before the bolus reaches the cecum.
It turns out that about half the time when we see abnormal glucose breath tests,
they’re actually falsely positive from rapid transit. In our studies, the overall average
orocecal transit time was 75 min. So, the studies that argue that any time you see a
rise of hydrogen before 90 min SIBO must be present are erroneous.
If you look at people who’ve had major upper GI tract surgery, such as a gastric
bypass, the vast majority of these have an abnormal breath test, but in most cases,
it’s abnormal due to rapid transit. However, even without such surgery, about one in
nine abnormal breath tests were falsely positive, which is better than with lactulose,
but still points out a high rate of false-positive testing if one were to use glucose
alone without a concurrent measure of orocecal transit time.
Clearly, defining SIBO based on breath tests is problematic. What about per-
forming quantitative bacterial cultures from small bowel aspirates obtained during
endoscopy? This approach is very expensive and invasive. It’s going to cost several
thousand dollars by the time you count in the endoscopy and the culture processing
in the microbiology laboratory. Furthermore, there is a concern that obtaining sam-
ples from only the duodenum will not detect bacterial overgrowth located more
distally. Finally, there is uncertainty about what concentration of bacteria represents
a true problem with overgrowth. While previous work has suggested that patients
with symptomatic SIBO have concentrations of >105/ml, a recent study in healthy
subjects on a high-fiber diet can have bacteria concentrations this high. Furthermore,
treating patients for SIBO based on a finding of this relatively low concentration in
duodenal aspirates has not been shown to result in better clinical outcomes.
Ultimately, the problem we are facing today is one of indiscriminate testing in
patients with nonspecific symptoms who have no predisposing conditions or find-
ings that would lead one to suspect the presence of SIBO (i.e., low pretest
86 B. Massey

probability). In this setting, I would expect the pretest probability of having SIBO is
at best 1%, but for the point of argument let’s say it is as high as 10%. If we perform
in this patient group a glucose breath test, which has a sensitivity and specificity of
about 80%, then among those with a positive result about 4% (a third, at the higher
pretest prevalence) will actually have SIBO.
So, if you start doing more indiscriminate testing, you’re going to diagnose a lot
more people with SIBO who don’t really have SIBO. Therefore, I think we have to
come up with a more rational approach to testing here. First of all, I think we have
to increase the pretest probability, and to do that, we have to start limiting testing to
patients who have abnormalities that are associated with SIBO. Patients with ane-
mia, particularly vitamin B12 deficiency-related anemia, and those with steatorrhea,
or low albumin. Those people who have conditions that are known to predispose to
SIBO, such as chronic small bowel pseudo-obstruction, common variable immuno-
deficiency, or systemic sclerosis, are reasonable candidates (if actually symptom-
atic). We can’t just keep testing people with nonspecific symptoms, and we also
need to have a reliable testing protocol. We can’t use lactulose breath testing unless
it’s combined with scintigraphy to measure orocecal transit. You also need to be sure
that patients are on the appropriate pretest (no FODMAPs) diet before breath test-
ing. Because constipated patients can release preformed colonic gas during breath
testing, this needs to be treated before performing breath testing. Breath test find-
ings cannot be interpreted accurately in patients with major foregut surgery without
a concurrent measure of transit time. If a patient with no good a priori indication
nevertheless tests positive on glucose breath testing, consider repeating the test with
a concurrent measure of orocecal transit time. Otherwise, if one truly thinks the test
result is accurate, then one is obligated to investigate further to try to find the under-
lying reason the patient has developed SIBO, to see if that can’t be corrected, rather
than just treating the test result with antibiotics.

Reference

1. Massey BT, Wald A. Small intestinal bacterial overgrowth syndrome: a guide for the appropri-
ate use of breath testing. Digest Dis Sci. 2021;66(2):338–47.
Chapter 15
Unusual Causes of Abdominal Pain
and Controversies in Diagnosis

W. Harley Sobin and Patrick Sanvanson

In the selections below, we briefly examine a number of controversies in the diag-


nosis of abdominal pain, along with a review of some of the rarer etiologies of
abdominal pain. They are not meant to be an exhaustive examination of the topics,
but, rather, a brief review of diagnoses to consider in the patient with an elusive
presentation, a checklist of zebras to keep in mind.
Case 1 A 35-year-old woman presents with chronic abdominal pain. The pain has
been present off and on over the past 2 years. The pain is located in the mid-­
abdomen. It is worse when she is stressed. It does not radiate. Bowel movements are
normal, and stool for H pylori was negative. A trial of PPIs for a month was not
helpful. Her CBC, CMP, and lipase were all normal. An abdominal ultrasound was
normal. She had an esophagogastroduodenoscopy (EGD) by a different gastroen-
terologist that was negative. You tell the patient that there is no sign of serious
pathology and you explain the functional nature of her symptoms. She does not buy
into your explanation; instead, she is upset that nothing has been found and requests
a CT scan.
How beneficial is a CT scan in evaluating chronic abdominal pain?
In a young patient like this, with negative laboratory results, a CT scan is unlikely
to add much useful information. CT scans are vastly over-utilized in the diagnosis
of chronic abdominal pain. They are costly, expose the patient to radiation, and have

W. H. Sobin (*)
Division of Gastroenterology and Hepatology, Medical College of Wisconsin,
Milwaukee, WI, USA
e-mail: [email protected]
P. Sanvanson
Division of Gastroenterology and Hepatology, Department of Medicine, Medical College of
Wisconsin, Milwaukee, WI, USA
e-mail: [email protected]

© The Author(s), under exclusive license to Springer Nature 87


Switzerland AG 2023
W. H. Sobin et al. (eds.), Managing Complex Cases in Gastroenterology,
https://doi.org/10.1007/978-3-031-48949-5_15
88 W. H. Sobin and P. Sanvanson

a high incidence of insignificant incidentalomas. Unless there are findings on physi-


cal or basic laboratory data to suggest organic disease, CT scans have a low likeli-
hood of providing data that will alter management.
Incidental findings increase healthcare costs for limited benefit for the patient.
There is a certain allure to CT scans that drive patients to ask for them and doctors
to order them, when there are no findings of organic disease on initial examinations
to explain symptomatology [1–3]. However, if there is nothing to suggest organic
disease, we should try to resist that temptation.
After explaining your reasons to decline ordering the test, the patient becomes
argumentative. She talks about the stress in her life and says that it will relieve her
stress to have the CT scan.
It is very problematic to order a CT scan simply to allay a patient’s fear if you
fully believe the CT scan will be negative and will more likely cause harm than
benefit. After explaining your arguments, after explaining the nature of functional
symptoms, the majority of patients will be happy to learn that they do not have seri-
ous organic pathology. They will be happy to drop the issue and go on with their
lives, but we have all experienced a few challenging patients who make demands
and try to coerce us [4].
This patient alludes to all the stress in her life, and patients with increased stress
are more likely to have functional GI pain syndromes and disorders of gut-brain
interaction [5]. Many of these patients may benefit from psychotherapy. There is a
high frequency of anxiety and somatization in those patients who have disabling
IBS symptoms.
In all cases, it is essential to have a strong doctor-patient relationship and spend
the time necessary to explain the nature of DGBI. Some patients will definitely
benefit from the use of central neuromodulators that can work on abdominal pain
and anxiety and depression. In a case like this, communication and strategies to
address the stress are much more likely to help the patient than ordering a CT
scan [6].
The patient, rather than going to see a mental health specialist, makes an
appointment with her OB-GYN doctor, who reluctantly orders the CT scan. The CT
scan fails to show anything in the upper or mid-abdomen to explain her symptoms
but does show some thickening in the area of the cecum. The reading reports that
this could be due to under-distension and presence of stool, but cannot rule out a
cecal mass.
How likely is it that the patient has colon cancer?
Unfortunately, this finding, a thickening of the colon wall, is notoriously nonspe-
cific. A routine contrast CT scan (as opposed to CT colonography or PET scan
showing intrinsic colon disease) has a very high number of false-positive results,
due to stool, lack of adequate distension, or other artifacts. In one study [7], only
13% of patients with cecal thickening on CT scan had pathology on follow-up colo-
noscopy. This is an example of one of the hazards of ordering a CT, the inciden-
taloma, which probably occurs in about 5% of all cases.
15 Unusual Causes of Abdominal Pain and Controversies in Diagnosis 89

Case 2 A 45-year-old slightly obese woman presents with intermittent RUQ pain.
On the examination, she has very minor RUQ tenderness. Laboratories, including
LFTs, are normal. An US (ultrasound) is normal. She wants to have her gallbladder
removed.
Would you send her to a surgeon to take out her gallbladder?
While the ultrasound is good for ruling out cholelithiasis, it is not as good at rul-
ing out microlithiasis and does not evaluate biliary dyskinesia. On the one hand,
there are patients who have a normal ultrasound whose pain is due to gallbladder
disease, but, on the other, we do not like to send a patient to surgery without some
objective abnormality on imaging or laboratory.
Therefore, if the patient’s symptoms sound typical of cholecystitis, but the US is
normal, we like to order scintigraphy with a CCK-HIDA scan. If there is non-filling
of the gallbladder, or if the ejection fraction after injection of CCK is significantly
<38%, this suggests biliary dyskinesia and an increased incidence of microlithiasis
and/or chronic cholecystitis [8]. In these cases, if there are no other findings to
explain their symptoms, we will send patients for surgical evaluation, knowing that
we can’t guarantee a cholecystectomy will resolve the problem in all of them. Many
patients will get symptomatic improvement after surgery, but a few will not, and
therefore, an abnormal CCK-HIDA scan has to be evaluated with some healthy
skepticism [9].
Case 3 A 35-year-old woman complains of mid-abdominal pain. The pain is fairly
chronic but worsened by meals. She has chronic constipation, only moving her bow-
els every 2–3 days. You examine her and hear a bruit over her abdomen. Because of
this, you send her for a CT scan that reveals median arcuate ligament narrowing of
the celiac axis.
How likely is it that this is causing the pain?
Both superior mesenteric artery compression (SMA syndrome) and median arcu-
ate ligament compression (MALS) on CT scan have been objects of controversy.
There are patients who have these radiographic findings who are asymptomatic
[10], and a number of patients have had surgical repairs that did not result in
improvement [11]. There are certainly a number of patients with SMA syndrome
and MALS who have improved with surgery, but in other cases the X-ray findings
end up being a red herring [12].
It is important to rule out other etiologies for abdominal pain. Only then, if the
history is consistent with a vascular cause of pain, should angiographic studies
be performed to measure the flow velocity across the vessel and document
whether there is significant stenosis. Not everyone with the radiologic finding
has significant clinical disease. It is important to inform the patient that although
these may be the correct diagnoses, frequently patients will go for surgery/revas-
cularization depending on the diagnosis, and the symptoms may not completely
resolve.
90 W. H. Sobin and P. Sanvanson

Some factors suggesting a good result with celiac decompression for MALS
include postprandial pain, age between 40 and 60, and weight loss of more than 20
lbs. [13]. Factors that suggest a poorer outcome include alcohol, and drug use and a
history of significant psychiatric disorder [13, 14].
Case 4 A 35-year-old woman comes in for a second opinion. She complains of
persistent abdominal pain located on the right side of her abdomen. The pain has
been there for a year but has been getting more intense recently. When you push on
her abdomen, she complains of extreme pain, but the abdomen is very soft and there
is no guarding. Her vitals are normal, and her heart rate is 70, even when she com-
plains of severe pain. Her basic laboratory panel is normal. She has had an US and
EGD elsewhere, which were normal. Your impression is that her pain is probably
functional.
It is very difficult to manage patients with chronic abdominal pain who have
no apparent organic abnormalities. Do you have any special insight into treat-
ing these patients?
Many of the patients we see with complaints of chronic abdominal pain will not
have an organic reason. Generally, when they do, it can be elicited with a basic his-
tory, physical examination, and simple laboratory panel. Not every patient com-
plaining of pain warrants an EGD, colonoscopy, and CT scan.
How do we approach these patients who lack any organic findings? Some of
them fit established criteria for well-defined disorders of gut-brain interaction such
as irritable bowel syndrome (IBS), functional dyspepsia (FD), and centrally medi-
ated abdominal pain syndrome (CAPS), which, although not associated with organic
findings, are accepted as diagnostic entities with a well-defined treatment algorithm.
Not all of these patients come to medical attention, but the ones who do, and the
ones who are more resistant to treatment, tend to have higher levels of psychological
distress [15].
Many of these patients have complaints that are vastly exaggerated relative to
their physical findings. These patients tend to perseverate over symptoms, catastro-
phize, and, in many ways, end up inflating the severity of a symptom that would
normally seem innocuous. It is a form of visceral hypersensitivity [16].
Some patients get secondary gain from their role as a patient, getting attention
from doctors, family, and friends that reinforces the behavior. If we recognize that a
patient has symptoms that are way out of proportion to the physical and laboratory
findings, we can often anticipate that these factors are at play.
The patient described above most likely has a disorder of gut-brain interaction
(DGBI, used to be called functional GI disorder-FGID). In DGBI patients, painful
stimuli are exaggerated rather than inhibited. Many patients who do not respond to
primary treatment regimens have this sort of misalignment. It is essential to explain
to patients that this is a positive diagnosis, so that they don’t go around thinking that
no one can come up with an answer for their symptoms. There are specific treat-
ments for these disorders, and it is important to communicate this insight to the
patient [5]. There are more insights into management of DGBI in other chapters,
later in this text.
15 Unusual Causes of Abdominal Pain and Controversies in Diagnosis 91

Case 5 A 46-year-old man comes to you with the complaint of abdominal pain. The
patient had an episode of pancreatitis 1 year ago and was placed on narcotic pain
medications as an inpatient, which were continued as an outpatient. He has not seen
a gastroenterologist since discharge from the hospital. He has reportedly quit drink-
ing, although he keeps smoking, but still complains of abdominal pain and keeps
asking his primary care doctor to renew his narcotics. His doctor recently told him
that he would not refill the narcotics unless he saw a gastroenterologist who recom-
mended they be continued. The patient is complaining of severe, constant pain. His
vitals are stable. On examination, the only thing of note is that his abdomen feels
full, probably from retained stool, but his abdomen is soft and lacks involuntary
guarding, although he complains of diffuse tenderness.
Will you continue narcotic pain medications in a patient like this?
While narcotic pain medications may benefit patients with acute pancreatitis in
the short term, their efficacy wears off quickly and these drugs need to be stopped
as early as possible. One concern is addiction, the second is tolerance, with patients
requesting ever higher doses, the third is side effects such as opioid-induced consti-
pation, and the fourth is the most remarkable, that after a while, continued use of
narcotics actually increases the level of pain, causing hyperalgesia [17].
You need to explain to the patient that you think the drugs are actually contribut-
ing to the pain. They need to start tapering the dose of narcotics while getting onto
alternative analgesics such as gabapentin or duloxetine. The gastroenterologist may
need to enlist help from a pain specialist. Unfortunately, patients tend to resist this
explanation, but it is a necessary first step.
Case 6 A 64-year-old woman complains of intermittent diarrhea and abdominal
cramps that have been ongoing for 3 months. She avoids eating to avoid diarrhea
and has lost 10 lbs. as a result. She had her last screening colonoscopy at age 60,
and it was negative. Her 35-year-old daughter was recently diagnosed with IBS,
and she questions whether that is her diagnosis as well. Her CBC, iron studies, and
chemistries were all normal.
How would you manage a patient like this?
In a 35-year-old with a history of chronic diarrhea and pain, IBS-D is often the
explanation, but in a patient who is over the age of 50, with no history of functional
GI illness, IBS is infrequent. In this case, there is a good chance that what sounds
like IBS-D is actually microscopic colitis [18], and a repeat colonoscopy with ran-
dom biopsies is indicated.
What if a patient in this age range presented with upper abdominal pain
and ulcer-like symptoms, but had a negative EGD and US. Would you think
this is likely functional dyspepsia (FD)?
No, once again it is much less common for a disorder of gut-brain interaction
(DGBI) like FD to first present at 64 years, and, in this age range, organic pathology
needs to be ruled out [19]. An EGD would be recommended, and, if that is negative,
a triple-phase CT scan to rule out pancreatic malignancy or other organic pathology
may be warranted in the appropriate patient.
92 W. H. Sobin and P. Sanvanson

Case 7 A 78-year-old man comes into the ER with a history of moderately severe
pain in his mid-abdomen. He had several similar episodes of pain over the prior
4 months, but this one was more severe and lasted about 3 h before remitting. His
CBC, lactate, and chemistries were normal. A CT scan showed diffuse atheroscle-
rosis involving mesenteric vessels. The small bowel and colon appeared normal.
Is this likely acute bowel ischemia?
The finding of diffuse atherosclerosis will be present in a large number of indi-
viduals in this age range who are totally asymptomatic. The absence of acute bowel
wall edema makes it less likely; this is acute ischemic colitis, or acute mesenteric
insufficiency. Making a diagnosis of chronic mesenteric ischemia is more difficult.
It is generally necessary to rule out other causes of pain, which might require upper
endoscopy, ultrasound, or dedicated cross-sectional imaging.
When would you proceed to revascularization?
If this patient, with documented mesenteric atherosclerosis, has recurrent epi-
sodes of pain that are suggestive of ischemia, and the work-up shows no other etiol-
ogy for the pain, we would refer the patient to a vascular surgeon for consultation.
Case 8 A 37-year-old woman has intermittent RUQ pain after eating fatty foods.
The pain is very similar to the symptoms she had prior to cholecystectomy 1 year
earlier. She had cholelithiasis, but her symptoms weren’t much better after chole-
cystectomy and still occur intermittently. She is referred to you for the evaluation for
sphincter of Oddi dysfunction (SOD).
What is your approach to patients with potential SOD disorder?
In this setting, it is essential to know what her liver enzymes are during the epi-
sodes of pain. We would also get a magnetic resonance cholangiopancreatography
(MRCP) to rule out a retained common duct stone. Assuming there is no stone, we
would consider the diagnosis of an SOD disorder only if there is an appropriate
bump of liver function tests (LFTs) during the episode that might suggest intermit-
tent spasm or obstruction. The finding of elevated LFTs and dilation of the bile duct
must be present to consider this diagnosis. In the past, we used to consider endo-
scopic retrograde cholangiopancreatography (ERCP) and manometry based on the
symptoms alone. This approach was associated with a high frequency of complica-
tions [20]. Now, if we have the requisite elevation of laboratories and bile duct dila-
tion, we would go directly to ERCP and sphincterotomy without manometry.
Case 9 A 46-year-old woman is seen with the complaint of chronic suprapubic
pain. She has a history of constipation for which she started taking laxatives.
Although her constipation improved, her suprapubic pain did not. In addition, she
is troubled with low back pain. She is pre-menopausal, and the suprapubic pain is
worse during her menstrual periods. Her gynecologist ordered a pelvic ultrasound
that was negative. She had a negative screening colonoscopy 1 year ago. The pos-
sibility of endometriosis is entertained.
How do you proceed with investigating possible endometriosis?
Diagnosis of endometriosis is challenging, and basic tests may return negative. A
history of dyspareunia, dysmenorrhea, low back pain, abnormal uterine bleeding,
15 Unusual Causes of Abdominal Pain and Controversies in Diagnosis 93

and bowel or bladder abnormalities are common symptoms [21]. Pelvic ultrasound
and examination by a gynecologist with special interest/training in diagnosing and
managing patients with endometriosis are essential since this is frequently a diffi-
cult diagnosis to make. Gynecologists will occasionally obtain pelvic MRIs for
additional information. A GI work-up to exclude other gastrointestinal pathology
may be warranted. The ultimate diagnosis of endometriosis is made by
laparoscopy.
Is a diagnostic and therapeutic medical trial ever appropriate before per-
forming laparoscopy?
In patients with typical but less severe symptoms, a trial of oral contraceptive
pills (OCPs) may be given. Pain relief with OCPs supports the diagnosis. But if
symptoms are more severe, or patients do not respond to OCPs, laparoscopy may be
performed prior to starting GnRH agonists or antagonists.
Case 10 A 27-year-old woman comes in complaining of chronic abdominal pain.
He says the pain is steady, unrelated to meals, occasionally worse when he defe-
cates, particularly if he is straining. His appetite is good, and he denies nausea or
weight loss. On examination, his abdomen is soft, and bowel sounds are normal, but
he has localized abdominal wall tenderness. When you ask him to tense his abdomi-
nal muscles and raise his head (while supine), the tenderness seems to worsen.
Basic laboratories are all normal.
Is it possible to get chronic abdominal pain from the abdominal wall?
Yes, some patients develop chronic abdominal wall pain [22]. The Carnett
maneuver, described above, is important in making the diagnosis [23]. In some
cases, patients may have an anterior cutaneous nerve entrapment syndrome. We
have tried lidocaine patches in a few, generally with limited success, but, in the
patient with more severe pain, we will contact the anesthesiologists in the pain
clinic to consider a local injection [24, 25]. For patients who don’t respond to injec-
tion, abdominal wall neurectomies have been successful [26].
Another cause of abdominal wall pain is a Spigelian hernia, an uncommon her-
nia that may present with pain alone in some individuals. If we are considering this
diagnosis, a good ultrasound can confirm it, and we will generally get an experi-
enced surgeon to weigh in.
Case 11 A 57-year-old man presents with LLQ pain of 4-h duration. After evalua-
tion in the ER, he is found to be afebrile but has mild leukocytosis of 14,000.
Diverticulitis is suspected, but the CT scan reveals a round fat-attenuating structure
with associated inflammation on the lateral wall of the rectosigmoid consistent with
epiploic appendagitis.
Epiploic appendagitis is a self-limited entity that presents just like diverticulitis,
but does not require antibiotics. In uncomplicated cases, it can be managed with
NSAIDs. Rather than a mini-perforation of a diverticulum as we see in diverticuli-
tis, this involves inflammation/ischemia of the appendages of the bowel mesentery.
The presentation oftentimes is suggestive of diverticulitis or appendicitis but
requires a CT scan for diagnosis. We emphasize the self-limiting nature and
94 W. H. Sobin and P. Sanvanson

symptoms that average about 10 days. However, if conservative treatments fail and
symptoms persist/worsen, rarely patients may need surgery [27, 28].
Case 12 A 27-year-old woman presents to the ER with the complaint of abdominal
pain, nausea, and vomiting and also pain and tingling in her hands and legs. She has
had three other ER visits for abdominal pain and has had an EGD, CT scan x 2, and
US, all of which were negative. She is having her menstrual period. On the examina-
tion, she seems agitated and mildly confused. Pulse is 115, and BP is 150/110. Her
abdomen is moderately tender and distended with increased tympany and high-­
pitched bowel sounds. A KUB showed an ileus with no free air. Because of her
altered mental status, a drug screen was sent off and returned negative for cocaine
or other drugs. Because of her multiple ER visits with negative findings in the face
of abdominal pain, now accompanied by symptoms suggesting CNS abnormality, a
urine porphobilinogen (PBG) was sent off. Result is 12 mg/24 h
(normal < 4 mg/24 h.).
How would you manage this patient?
The finding of elevated urine PBG is specific for porphyria, and this patient’s
clinical course is consistent with porphyria [29]. Since she is symptomatic and her
urine PBG is significantly elevated, treatment should be administered with IV hemin
(Panhematin). While waiting for hemin to be made available, IV glucose loading
should be initiated [30]. Hemin should be administered once daily for 4 days
or longer.
For patients with repeated episodes of acute intermittent porphyria (AIP), there
is a drug, givosiran, which interferes with the gene that is induced during an AIP
attack, thereby preventing attacks or decreasing their intensity. This drug needs to
be administered subcutaneously on a monthly basis.
Case 13 A 67-year-old man presents with a complaint of chronic abdominal pain
gradually worsening over the past month. He has a past history of cholecystectomy
when he was 30 years old and also required surgery for small bowel obstruction
when he was 35 years old. On examination, there is mid-abdominal fullness and a
possible mass. You order a CT scan that reveals a 5-cm mass in the mesentery and
a fat ring sign. There is calcification within the mass and surrounding lymph nodes.
The differential diagnosis is most likely sclerosing mesenteritis (or panniculitis), or
less likely neoplasm.
How would you proceed?
Laparoscopy and tissue biopsy are required.
The surgical findings and biopsy are consistent with sclerosing mesenteritis.
How would you manage the patient?
Treatment for sclerosing mesenteritis involves a course of steroids and long-term
therapy with tamoxifen. Tamoxifen acts as an anti-fibrotic agent. Prednisone is
administered for 3 months and then, if there is symptomatic improvement, tapered
over 3 months. Tamoxifen is continued indefinitely [31, 32].
15 Unusual Causes of Abdominal Pain and Controversies in Diagnosis 95

Case 14 A 36-year-old man presents for an office consultation with a history of


recurrent episodes of abdominal pain and nausea. His past history includes
migraine headaches. When you ask him whether the abdominal pain coincides with
the migraine headaches, he says, “I think it does. I hadn’t thought of that. I don’t
get abdominal pain with every migraine but, yes, when I get the pain, it is often
shortly after the start of a migraine.”
He is feeling well when seen. His examination is unremarkable, and baseline
laboratories are normal.
How do you make a diagnosis of abdominal migraine?
Patients usually have a history of migraine headaches. The abdominal pain may
or may not coincide with the migraines. Episodes of pain are described as “parox-
ysmal,” coming on suddenly and then often lasting for 15–20 min. Patients often
have nausea and vomiting along with headaches that may accompany the pain. The
abdominal pain is usually relieved with migraine medications. Meds used for pro-
phylaxis of migraines may prevent episodes of abdominal pain [33]. There is no
specific test to diagnose abdominal migraine. As with many of the other uncommon
etiologies for abdominal pain, the diagnosis requires ruling out other organic causes
for the GI symptoms. If abdominal migraine is suspected, a clinical trial of sumat-
riptan should be given. If the patient responds, but has repeated episodes of abdomi-
nal pain, medications used to prevent migraine headaches can be offered. Beta
blockers, calcium channel blockers, valproic acid, and topiramate have all been
used prophylactically.
Case 15 A 55-year-old woman with rheumatoid arthritis (RA) presents to the ER
with a 2-h history of excruciating abdominal pain. On examination, there is marked
tenderness and rebound. Non-contrast CT scan reveals free air and inflammatory
change around the stomach suggestive of perforated ulcer. The patient was on meth-
otrexate for her RA and was taking naproxen on a regular basis over the past
3 months for increased joint pains. When questioned, she denied having chronic
abdominal pain, although she experienced some mild nausea or “indigestion” for
the past week. The patient is sent to surgery, and a perforated gastric ulcer is identi-
fied and oversewn.
Isn’t it extraordinary for a peptic ulcer to lead to perforation without warn-
ing signs of abdominal pain leading up to that?
There is a high frequency of peptic ulcers in patients with RA taking NSAIDs. In
one study [34], 26% of patients taking naproxen for 3 months developed peptic
ulcers. Only 7% had abdominal pain.
The finding of asymptomatic peptic ulcers is not limited to patients with rheuma-
tologic disorders. In one Chinese study, largely asymptomatic patients were having
EGDs to screen for gastric cancer [35]. In that study, 11% had peptic ulcer disease
(PUD) and 2/3 of ulcer patients were asymptomatic. Current smoking and obesity
were associated with increased incidence of asymptomatic ulcers. Smaller ulcers
and healing ulcers were less likely to cause symptoms.
96 W. H. Sobin and P. Sanvanson

It is not unusual for peptic ulcers to first present with a complication. In about
25% of PUD patients, the first manifestation is perforation or bleeding. Of
patients with bleeding ulcers, somewhere between 43% and 87% are
asymptomatic.
It has been argued that NSAID use may increase the likelihood of asymptomatic
PUD due to its analgesic effects. However, more recent studies show that most
NSAID-associated gastric ulcers are symptomatic [36].
Age also factors in when determining the incidence of asymptomatic ulcers.
Only 7% of patients under 50 years old were without abdominal pain, while 29% of
patients over 60 were without pain [37].
Case 16 A 70-year-old woman is brought into the ER by her daughter with a temp
of 100 and new-onset confusion. She is on prednisone 5 mg daily for temporal arte-
ritis. There was no prior history of abdominal pain. On examination, she has mini-
mal tenderness in the epigastric area and RUQ. She has a WBC of 10,000 with 5%
bands. An US suggests cholecystitis. A CT scan is ordered that suggests phlegmon-
ous cholecystitis but is otherwise negative.
Isn’t it unusual to see a patient with cholecystitis presenting with altered
mental status and minimal abdominal pain?
Older patients who present to the ER with abdominal pathology are generally
more likely to present with advanced-stage disease. Elderly patients are more likely
to have vague symptoms, nonspecific physical findings, and may lack laboratory
abnormalities. Elderly patients with cholecystitis lacked the classic pain patterns in
84% and had no pain at all in 5%. They were more likely to have an absence of
leukocytosis (41%) and fever (56%) and are much more likely to require surgery on
presentation [38].
The most common finding in elderly patients presenting to the ER with acute
abdominal pain was acute cholecystitis. In younger patients, the most common eti-
ologies were nonspecific abdominal pain followed by appendicitis.
When elderly patients do present with appendicitis, they can have unusual pre-
sentations. Only 23% had fever, and many more elderly patients were found to have
diffuse tenderness, distension, and rigidity because they waited longer before pre-
senting to the ER [39, 40].
Another cause of intra-abdominal emergency in the elderly is an abdominal aor-
tic aneurysm (AAA). Many significant AAAs may be asymptomatic and may not be
diagnosed until there is rupture [41, 42]. Of course, in elderly patients presenting
with abdominal pain, vascular insufficiency always has to be considered. The clas-
sic story is the elderly patient with severe pain that seems out of proportion with the
physical findings [43].
Besides ischemic bowel, AAA, and cholecystitis, other causes of abdominal pain
that are more common in the elderly include acute pancreatitis and diverticulitis.
They are also more likely to have intra-abdominal cancer.
Another factor that may lead to decreased symptoms and physical findings in the
elderly is the increased use of medications such as beta blockers and steroids that
may mask normal alterations in vital signs.
15 Unusual Causes of Abdominal Pain and Controversies in Diagnosis 97

References

1. Mwinyogle AA, Bhatt A, Ogbuagu OU, Dhillon N, Sill A, Kowdley GC. Use of CT scans for
abdominal pain in the ED: factors in choice. Am Surg. 2020;86(4):324–33.
2. Kanzaria HK, Hoffman JR, Probst MA, Caloyeras JP, Berry SH, Brook RH. Emergency physi-
cian perceptions of medically unnecessary advanced diagnostic imaging. Acad Emerg Med.
2015;22(4):390–8.
3. de Burlet KJ, MacKay M, Larsen P, Dennett ER. Appropriateness of CT scans for patients with
non-traumatic acute abdominal pain. Br J Radiol. 2018;91(1088):20180158.
4. Roberts LW, Dyer AR. Caring for “difficult” patients. Focus. 2003;1(4):453–8.
5. Drossman DA. David sun lecture: helping your patient by helping yourself—how to improve
the patient–physician relationship by optimizing communication skills. Off J Am Coll
Gastroenterol|ACG. 2013;108(4):521–8.
6. Drossman DA. Do psychosocial factors define symptom severity and patient status in irritable
bowel syndrome? Am J Med. 1999;107(5):41–50.
7. Cai Q, Baumgarten DA, Affronti JP, Waring JP. Incidental findings of thickening luminal gas-
trointestinal organs on computed tomography: an absolute indication for endoscopy. Am J
Gastroenterol. 2003;98(8):1734–7.
8. Sharma BC, Agarwal DK, Dhiman RK, Baijal SS, Choudhuri G, Saraswat VA. Bile litho-
genicity and gallbladder emptying in patients with microlithiasis: effect of bile acid therapy.
Gastroenterology. 1998;115(1):124–8.
9. Gudsoorkar VS, Oglat A, Jain A, Raza A, Quigley EM. Systematic review with meta-analysis:
cholecystectomy for biliary dyskinesia—what can the gallbladder ejection fraction tell us?
Aliment Pharmacol Ther. 2019;49(6):654–63.
10. Park CM, Chung JW, Kim HB, Shin SJ, Park JH. Celiac axis stenosis: incidence and etiologies
in asymptomatic individuals. Korean J Radiol. 2001;2(1):8–13.
11. Ylinen P, Kinnunen J, Höckerstedt K. Superior mesenteric artery syndrome. A follow-up study
of 16 operated patients. J Clin Gastroenterol. 1989;11(4):386–91.
12. Cohen LB, Field SP, Sachar DB. The superior mesenteric artery syndrome. The disease that
isn't, or is it? J Clin Gastroenterol. 1985;7(2):113–6.
13. Reilly LM, Ammar AD, Stoney RJ, Ehrenfeld WK. Late results following operative repair for
celiac artery compression syndrome. J Vasc Surg. 1985;2(1):79–91.
14. Skelly CL, Stiles-Shields C, Mak GZ, Speaker CR, Lorenz J, Anitescu M, Dickerson DM,
Boyd H, O'Brien S, Drossos T. The impact of psychiatric comorbidities on patient-reported
surgical outcomes in adults treated for the median arcuate ligament syndrome. J Vasc Surg.
2018;68(5):1414–21.
15. Keefer L, Drossman DA, Guthrie E, Simrén M, Tillisch K, Olden K, Whorwell PJ. Centrally
mediated disorders of gastrointestinal pain. Gastroenterology. 2016;150(6):1408–19.
16. Mayer EA, Gebhart GF. Basic and clinical aspects of visceral hyperalgesia. Gastroenterology.
1994;107(1):271–93.
17. Drossman D, Szigethy E. The narcotic bowel syndrome: a recent update. Am J Gastroenterol
Suppl. 2014;2(1):22–30.
18. Nguyen GC, Smalley WE, Vege SS, Carrasco-Labra A, Flamm SL, Gerson L, Hirano I,
Rubenstein JH, Singh S, Stollman N, Sultan S. American Gastroenterological Association
Institute guideline on the medical management of microscopic colitis. Gastroenterology.
2016;150(1):242–6.
19. Moayyedi PM, Lacy BE, Andrews CN, Enns RA, Howden CW, Vakil N. ACG and CAG clin-
ical guideline: management of dyspepsia. Off J Am Coll Gastroenterol|ACG. 2017;112(7):
988–1013.
20. Cotton PB, Pauls Q, Keith J, Thornhill A, Drossman D, Williams A, Durkalski-Mauldin V. The
EPISOD study: long-term outcomes. Gastrointest Endosc. 2018;87(1):205–10.
21. Ballard KD, Seaman HE, De Vries CS, Wright JT. Can symptomatology help in the diagnosis
of endometriosis? Findings from a national case–control study—part 1. BJOG Int J Obstet
Gynaecol. 2008;115(11):1382–91.
98 W. H. Sobin and P. Sanvanson

22. Srinivasan R, Greenbaum DS. Chronic abdominal wall pain: a frequently overlooked problem:
practical approach to diagnosis and management. Am J Gastroenterol. 2002;97(4):824–30.
23. Takada T, Ikusaka M, Ohira Y, Noda K, Tsukamoto T. Diagnostic usefulness of Carnett's test
in psychogenic abdominal pain. Intern Med. 2011;50(3):213–7.
24. Alnahhas MF, Oxentenko SC, Locke GR, Hansel S, Schleck CD, Zinsmeister AR, Farrugia
G, Grover M. Outcomes of ultrasound-guided trigger point injection for abdominal wall pain.
Dig Dis Sci. 2016;61:572–7.
25. Boelens OB, Scheltinga MR, Houterman S, Roumen RM. Management of anterior cutaneous
nerve entrapment syndrome in a cohort of 139 patients. Ann Surg. 2011;254(6):1054–8.
26. van Assen T, Brouns JA, Scheltinga MR, Roumen RM. Incidence of abdominal pain due to the
anterior cutaneous nerve entrapment syndrome in an emergency department. Scand J Trauma
Resusc Emerg Med. 2015;23(1):1–6.
27. Sand M, Gelos M, Bechara FG, Sand D, Wiese TH, Steinstraesser L, Mann B. Epiploic
appendagitis–clinical characteristics of an uncommon surgical diagnosis. BMC Surg.
2007;7(1):1–7.
28. Akubudike JT, Egigba OF, Kobalava B. Epiploic appendagitis: a commonly overlooked dif-
ferential of acute abdominal pain. Cureus. 2021;13(1):e12807.
29. Herrick AL, McColl KE. Acute intermittent porphyria. Best Pract Res Clin Gastroenterol.
2005;19(2):235–49.
30. Ortega AJ, Cherukuri S, Kalas MA, Lee B, Guzman J, Robles A, Zuckerman MJ, Al-Bayati
I. A perfect storm: abdominal pain and ileus explained by acute intermittent porphyria caused
by prehospitalization and intrahospitalization factors. J Investig Med High Impact Case Rep.
2022;10(2):3247096221109206.
31. Danford CJ, Lin SC, Wolf JL. Sclerosing mesenteritis. Off J Am Coll
Gastroenterol|ACG. 2019;114(6):867–73.
32. Anderson KE, Bloomer JR, Bonkovsky HL, Kushner JP, Pierach CA, Pimstone NR, Desnick
RJ. Recommendations for the diagnosis and treatment of the acute porphyrias. Ann Intern
Med. 2005;142(6):439–50.
33. Roberts JE, deShazo RD. Abdominal migraine, another cause of abdominal pain in adults. Am
J Med. 2012;125(11):1135–9.
34. Hawkey CJ, Laine L, Simon T, Quan H, Shingo S, Evans J. Incidence of gastroduodenal ulcers
in patients with rheumatoid arthritis after 12 weeks of rofecoxib, naproxen, or placebo: a mul-
ticentre, randomised, double blind study. Gut. 2003;52(6):820–6.
35. Lu CL, Chang SS, Wang SS, Chang FY, Lee SD. Silent peptic ulcer disease: frequency, fac-
tors leading to “silence,” and implications regarding the pathogenesis of visceral symptoms.
Gastrointest Endosc. 2004;60(1):34–8.
36. Kim HM, Cho JH, Choi JY, Chun SW, Kim YJ, Cho HG, Song SY, Han KJ. NSAID is inversely
associated with asymptomatic gastric ulcer: local health examination data from the Korean
National Health Insurance Corporation. Scand J Gastroenterol. 2013;48(12):1371–6.
37. Hilton D, Iman N, Burke GJ, Moore A, O’Mara G, Signorini D, Lyons D, Banerjee AK, Clinch
D. Absence of abdominal pain in older persons with endoscopic ulcers: a prospective study.
Am J Gastroenterol. 2001;96(2):380–4.
38. Parker LJ, Vukov LF, Wollan PC. Emergency department evaluation of geriatric patients with
acute cholecystitis. Acad Emerg Med. 1997;4(1):51–5.
39. Elangovan S. Clinical and laboratory findings in acute appendicitis in the elderly. J Am Board
Fam Pract. 1996;9(2):75–8.
40. Kraemer M, Franke C, Ohmann C, Yang Q, Acute Abdominal Pain Study Group. Acute appen-
dicitis in late adulthood: incidence, presentation, and outcome. Results of a prospective mul-
ticenter acute abdominal pain study and a review of the literature. Langenbeck's. Arch Surg.
2000;385:470–81.
41. Fielding JW, Black J, Ashton F, Slaney G, Campbell DJ. Diagnosis and management of 528
abdominal aortic aneurysms. Br Med J (Clin Res Ed). 1981;283(6287):355–9.
15 Unusual Causes of Abdominal Pain and Controversies in Diagnosis 99

42. Marston WA, Ahlquist R, Johnson G Jr, Meyer AA. Misdiagnosis of ruptured abdominal aortic
aneurysms. J Vasc Surg. 1992;16(1):17–22.
43. Bala M, Kashuk J, Moore EE, Kluger Y, Biffl W, Gomes CA, Ben-Ishay O, Rubinstein C,
Balogh ZJ, Civil I, Coccolini F. Acute mesenteric ischemia: guidelines of the world Society of
Emergency Surgery. World J Emerg Surg. 2017;12(1):1–1.
Chapter 16
Vascular Disorders of the Intestine

Jim Nelson

Case 1: Acute Mesenteric Insufficiency (AMI) A 64-year-old male smoker is


recovering from a recent MI in a rehab center. One evening, he develops acute
abdominal pain and is brought into the ER. There is moderate abdominal tender-
ness on the examination. WBC 15,000. Elevated lactate and D-dimer. Creatinine is
1.6. KUB reveals thumbprinting.
There is a suspicion of intestinal ischemia here. There is mild renal insuffi-
ciency. Is a contrast CT required for diagnosis? Can an MRA be used instead?
That’s one of the gray areas and really depends on how significant the renal
insufficiency is. Generally speaking, a mild degree of chronic kidney disease is not
a major contraindication to using CT in these circumstances. There are many rea-
sons why a CTA is the preferred study. The sensitivity and specificity of a CTA are
much better compared to an MRA [1]. In addition, CTAs don’t take nearly as long
to complete. So, if the patient has difficulty lying down for a long time, is in a sig-
nificant amount of pain, or can’t hold their breath for any period of time, it makes
MRA much more difficult and less useful. In addition, the MRA often lacks some
of the resolution that the CTA has in terms of documenting stenosis. The MRA may
overestimate the degree of occlusion and may not visualize the IMA sufficiently,
although the IMA isn’t usually a clinically significant vessel in acute mesenteric
ischemia. So, for all these reasons CTA is the preferred imaging study. In this case,
a CTA is reasonably safe and appropriate to properly evaluate for AMI.
A CTA is performed that reveals an embolus in the SMA confirming the diagno-
sis of AMI.

J. Nelson (*)
Department of Medicine, Division of Gastroenterology and Hepatology, Medical College of
Wisconsin, Milwaukee, WI, USA
e-mail: [email protected]

© The Author(s), under exclusive license to Springer Nature 101


Switzerland AG 2023
W. H. Sobin et al. (eds.), Managing Complex Cases in Gastroenterology,
https://doi.org/10.1007/978-3-031-48949-5_16
102 J. Nelson

Is there a predilection for emboli to occlude the SMA?


Yes, there is a predilection for emboli to occlude the SMA, which relates to the
anatomy of the SMA [2, 3]. Many, if not most of these cases of AMI, are embolic in
etiology. The SMA, being a large caliber vessel, allows emboli to more easily pass
into it. Also, compared to the 90-degree takeoff of the celiac artery, the takeoff of the
SMA is a less acute angle. Emboli, therefore, generally bypass the celiac artery
takeoff and pass into the SMA much more readily. So, for those reasons there is a
predilection for emboli to occlude the SMA.
How long do you have in AMI before you need to intervene?
Several studies indicate that if there’s 100% occlusion of the SMA, you have
approximately 6 h before bowel infarction will occur. If you have a 75% occlusion,
you have about 12 h before you would need to intervene before infarction occurs
[3]. While this may seem like time is on your side when attempting to make a diag-
nosis of AMI, it’s not and any delays increase mortality and can complicate surgery.
Bottom line: “AMI is a medical emergency.”
How would you manage this?
Any patient that has acute mesenteric ischemia, whether it’s embolic or throm-
botic, should receive systemic heparinization [1]. Therefore, the initial approach,
when there is a concern for AMI, should be to immediately start full anticoagulation
with IV heparin. They should be NPO and placed on broad-spectrum antibiotics to
cover bacteremia secondary to bacterial translocation and the potential for sepsis.
Definitive treatment is ultimately going to require revascularization, either radio-
logic or surgical.
When do you decide to use surgery as opposed to interventional radiology?
Decisions regarding revascularization require consultation with both vascular
surgery along with interventional radiology [4]. If a patient is showing signs of
peritonitis or advanced ischemic changes/infarction such as worsening abdominal
pain, fever, or leukocytosis, surgical intervention is indicated. Otherwise, radiologic
intervention, specifically embolectomy or angioplasty/stenting, is the preferred ini-
tial approach.
Do you know how useful the finding of thumbprinting on KUB is? Is it sensi-
tive, specific?
It’s not very sensitive and you don’t see it very often, but when you do it usually
indicates severe ischemia or infarction. Therefore, plain X-rays of the abdomen are
generally not very helpful in the diagnosis or management of AMI.
The patient has a mesenteric angiogram and thrombectomy but afterward devel-
ops hypotension with a BP 90/60.
Is there any role for pressors in a hypotensive patient with mesenteric
ischemia?
Yes, but with care. If you do decide that a patient needs pressor support, you
should, in general, avoid alpha-adrenergic pressors, such as vasopressin, as they can
cause vasoconstriction of the splanchnic arteries and worsen the ischemia. While
low-dose vasopressin might be safe to use, more often dobutamine or dopamine are
used since they don’t cause vasoconstriction of the splanchnic arteries [2].
A follow-up X-ray shows jejunal distension with wall thickening, pneumatosis,
and air in the portal vein.
16 Vascular Disorders of the Intestine 103

How would you manage this?


These are worrisome signs of bowel infarction. The patient needs surgical inter-
vention for possible resection of the infarcted bowel. A second look operation is
many times done during the same hospitalization to evaluate and remove any
remaining, non-viable bowel. Short bowel syndrome is an unfortunate outcome of
large resections.
The patient goes to surgery and a section of necrotic small bowel of 100 cm is
removed distal jejunum and proximal ileum.
Is this the most common etiology for short bowel syndrome?
Yes, the main cause of short bowel syndrome is surgical resection for ischemia.
Crohn’s disease is the second most common cause. Recurrent chronic obstruction
from adhesive disease from prior surgeries is another etiology.
Following resection, the patient has a slow but consistent recovery.
Case 2: Mesenteric Vein Thrombosis (MVT) A 73-year-old man with a history of
metastatic renal cell carcinoma presents to the ER with a history of acute onset of
mid-abdominal pain. There is moderate abdominal tenderness and decreased bowel
sounds. Laboratories reveal leukocytosis and mild metabolic acidosis. A CT scan
reveals thrombosis of the SMV and portal vein with edema and dilation of small
bowel loops. The impression is that this patient has acute mesenteric vein thrombo-
sis. A surgical consult is called, but the decision is made to treat with
anticoagulation.
Should you do a thrombophilia work-up in all of these patients?
Statistically, the majority of patients who develop mesenteric venous thrombosis
have a primary hypercoagulable disorder as the cause. A smaller percentage have a
malignancy causing a secondary hypercoagulable state. So, in this case, although he
has a malignancy, it’s not unreasonable to do a limited hypercoagulable work-up.
The patient is found to have factor V Leyden deficiency.
Is it sufficient to treat with anticoagulation? Is surgery ever indicated? If
so, when?
Anticoagulation is the primary therapy [1]. If they don’t improve or if the patient
progresses to peritonitis and presumed infarction, surgery would be necessary.
Is there usually a different clinical presentation in acute SMV thrombosis vs
acute SMA thrombosis?
SMV thrombosis is more common in males, while SMA thrombosis is more
common in females. Many patients with SMV thrombosis have a history compati-
ble with or have a diagnosis of chronic mesenteric ischemia. They often have a his-
tory of postprandial abdominal pain and weight loss.
In comparison with SMA thrombosis which presents acutely, patients with SMV
thrombosis have a more insidious course with chronic or subacute symptoms. SMA
thrombosis tends to present with more acute, severe pain, while SMV thrombosis
typically presents with a dull pain that develops over days to weeks.
Which veins tend to be involved in acute MVT? How does this lead to organ
damage, does gangrene occur?
Most of the time it’s the SMV, which leads to damage of the distal small bowel
[2]. Rarely is it the IMV. The pathophysiology is like any form of venous outflow
104 J. Nelson

obstruction. The bowel wall gets edematous causing increased intramural pressure
resulting in a reduction in arterial flow through the bowel wall. If the reduction in
submucosal arterial blood flow is severe, bowel infarction can occur.
How successful is anticoagulation?
Very successful. Anticoagulation almost eliminates in-hospital mortality in these
patients. Overall mortality is less than 5% in patients who have been treated quickly
and adequately anticoagulated. Most of these patients will need to remain on long-­
term anticoagulation since their thrombotic risk continues indefinitely.
Case 3: Chronic Mesenteric Ischemia A 75-year-old man presents with a 1-month
history of abdominal pain and weight loss. The pain is periumbilical and tends to
occur about a half hour after meals. The patient also experiences mild nausea after
meals. To prevent nausea and abdominal pain, he has been avoiding eating and lost
15 lbs. over the past month. The abdominal examination is benign without a bruit.
The CBC, chem panel, and liver enzymes are all normal. EGD is negative. US abdo-
men is normal. CT abdomen and pelvis reveal a normal-appearing pancreas, but
there is extensive atherosclerosis of the mesenteric vessels. There is no suggestion
of acute ischemia. Bowel loops appear normal. Because of these findings in the set-
ting of abdominal pain and weight loss, the diagnosis of possible CMI is raised.
Is there any way to make a clear-cut positive diagnosis of CMI or do you
have to exclude other causes first and then chronic ischemia becomes a diagno-
sis of exclusion if the CTA or angio are consistent?
In most cases, it’s hard to come to a definite diagnosis of CMI. Only a small
percentage of patients have the classic clinical trial of abdominal pain, weight loss,
and abdominal bruit. In general, you first want to rule out other potential causes of
postprandial pain, like biliary tract disease, that can mimic the same clinical presen-
tation as CMI. Once other causes of postprandial abdominal pain and weight loss
have been excluded, and a CTA shows significant stenosis in at least 2 of the 3
mesenteric arteries, then there is a high likelihood of CMI.
What percentage of patients in this age range will have vascular abnormali-
ties simulating CMI, and yet be asymptomatic?
About 60% of older patients have radiologic findings of mesenteric vascular dis-
ease in one or two vessels but are asymptomatic. Another 15–20% will have athero-
sclerotic disease in all three vessels and are asymptomatic.
What is the role, if any, of duplex Doppler?
I think a lot of people would gravitate to this test first to screen for CMI but a
CTA should be done to confirm the diagnosis. Duplex Doppler is more of a physi-
ologic test that is highly sensitive and specific at detecting abnormally high mesen-
teric arterial velocities suggesting CMI [1]. A peak systolic velocity > 275 cm/sec
in the SMA suggests significant stenosis [2].
If you’ve convinced yourself this is CMI, how do you manage it?
The first approach usually involves radiologic revascularization, specifically
angioplasty and stenting.
The patient is evaluated by IR and surgery, and the decision is made to perform
endovascular revascularization. Following this, the patient’s pain resolves and he
gains back the weight lost over the next 2 months.
16 Vascular Disorders of the Intestine 105

Case 4: Nonocclusive Mesenteric Ischemia (NOMI) A 72-year-old man with a


history of CHF and the recent diagnosis of chronic cholecystitis is scheduled to
have an elective cholecystectomy. When presenting to the pre-op area of the hospi-
tal, he was noted to be hypotensive BP-90/50. IV fluids were started, but the patient
started complaining of increased pain across his abdomen, different from his usual
gallbladder pain. An urgent CT was performed, which revealed small bowel edema
with probable pneumatosis. There were scattered plaques in the abdominal aorta
and mesenteric vasculature but no occlusive disease. There was also a spasm of the
arterial cascade emanating from the SMA. Because there was concern for bowel
ischemia, urgent laparoscopy was performed. A section of dusky appearing distal
jejunum was identified and resected. Adequate pulsations to the rest of the bowel
were ascertained. There was no evidence of thrombosis or embolism to any blood
vessels. The post-op impression was nonocclusive mesenteric ischemia.
What are the risk factors for NOMI? Which patients are most at risk?
This patient was hypotensive on presentation possibly due to evolving sepsis
from cholecystitis. Most often these are critically ill patients in ICUs who have
some form of hemodynamic instability [3], whether it’s septic shock, hemorrhagic
shock, or cardiogenic shock [2, 4]. Be alert to a critically ill ICU patient with hypo-
tension who develops abdominal pain, distension, or signs of a GI bleed as NOMI
would be a concern.
Was surgery mandatory? Can you make a case for pre-op vasodilators?
Yes, especially with the finding of possible pneumatosis on CT. Other findings
concerning severe bowel ischemia are fever, leukocytosis, and an elevated lactate
level. If NOMI is suspected, vasodilator treatment should be promptly initiated [4].
Vasodilators, like PGE1 or papaverine, infused directly into the SMA or IV are
options. Equally important and what is still primary therapy is treating the underly-
ing cause of the hemodynamic instability.
Could the diagnosis be ascertained by CTA alone with confidence?
Yes, the classic finding is the irregularity of the vascular arcade suggesting spasm
[3]. There may also be hypo-enhancement of the intestinal wall. Many of these
patients also have some degree of atherosclerotic narrowing of their SMA without
occlusion. The degree of narrowing, however, is generally not sufficient to explain
the severity of ischemia and symptoms.
Case 5: Colon Ischemia (CI) A 65-year-old man presents to the ER with a history
of rectal bleeding. He has no past history of cardiovascular disease, hypertension,
etc. He does have occasional constipation. His last screening colonoscopy 2 years
earlier was negative except for mild diverticulosis.
He is on no meds. He was fine until that afternoon when he felt a sudden weak-
ness, sweating, and nausea causing him to sit down and compose himself. Shortly
after the episode, he developed abdominal cramping, following which he had two or
three bowel movements that were loose and mixed with blood. In the ER, he was
normotensive, his HR was 90 and regular, and he was afebrile. On the abdominal
examination, there was minimal abdominal tenderness and no rebound, and bowel
sounds were normal without a bruit. His laboratories showed a Hgb of 12 and a
106 J. Nelson

WBC of 13,000. There was no metabolic acidosis, but a CRP was 30. A CT scan was
obtained and showed colitis involving the splenic flexure. CI is suspected. Overall,
the patient appears quite stable.
How would you manage this patient, other than starting IV fluids? Would
you start antibiotics? Would you do a colonoscopy?
In general, you don’t need to start antibiotics if the patient appears stable and
doesn’t have any worrisome clinical findings, such as a high white count, fever,
significant abdominal tenderness, or more evidence of significant damage on
CT. Current ACG guidelines indicate you should consider starting antibiotics in
patients with moderate or severe disease, which is based primarily on colonoscopy
findings. Findings of severe CI include deep ulcerations and cyanosis of the mucosa.
If these abnormalities are found on colonoscopy, then antibiotics should be started.
The vast majority of these patients have mild disease and do well with supportive
care alone. As to whether you perform a colonoscopy on a patient that has sugges-
tive findings such as segmental colitis in the watershed regions on CT depends on
whether there are concerns for other causes of segmental colitis such as Crohn’s
disease. A colonoscopy can be helpful in differentiating between CI and other
causes. A finding that is pathognomonic for CI [5] is longitudinal ulcerations, the
so-­called stripe sign, which extend down the long axis of the colon. It’s generally
safe to do a colonoscopy as long as you don’t advance the scope past the area
of injury.
Colonoscopy is performed to the splenic flexure where there is a region of typical
ischemic colitis noted with longitudinal ulcers, dusky mucosa, and bluish blebs.
Because these findings are consistent with severe CI, the patient was started on IV
antibiotics.
CI seems to occur in a number of patients who have no risk factors.
Comments?
Because most cases of CI occur in the 60–80-year age group, they often have
undiagnosed atherosclerotic mesenteric vascular disease, especially if they have
other risk factors such as a history of hypercholesterolemia, hypertension, coronary
artery disease, or peripheral vascular disease.
Another risk factor is constipation which the patient reported. Yes, based on sev-
eral studies, constipation is a risk factor for ischemic colitis [5, 6]. The reason that
constipation is a risk factor, along with other causes of bowel distension such as
colonoscopy [7] or prepping for a colonoscopy, is that distension can lead to com-
pression of submucosal vessels resulting in ischemia.
There are some medicines that have been associated with precipitating ischemic
colitis [5, 2] including nasal decongestants, methamphetamines, digoxin, cocaine,
Tamiflu, rizatriptan, diuretics, alosetron, estrogens, and clozapine.
Some patients have microvascular thrombotic disease or a previously undiag-
nosed hypercoagulable state, but still most patients with CI have small or large ves-
sel atherosclerotic vascular disease.
The patient’s clinical course is unremarkable and he goes home after a few days’
hospitalization.
16 Vascular Disorders of the Intestine 107

The outcome in most of these CI cases is almost uniformly excellent in my


experience. Has that been your experience as well?
Yes, usually it takes longer for patients with more severe damage to improve,
often several days, as in the example case, but the majority who have mild disease
improve in 2–3 days.
Do you do any follow-up colonoscopy?
No, not routinely. If there is a concern, for instance, for a neoplasm based on a
history of constipation or IBD, then a follow-up colonoscopy would be appropriate.
It seems to me that a number of these patients present with nausea or weak-
ness prior to developing abdominal pain, diarrhea, or rectal bleeding, just like
this patient. Any comments?
It may be due to a vagal response to some degree of pain or they’re simply not
eating or drinking enough fluids due to a mild ileus that potentially may develop
preceding the ischemic event.
Case 6: Median Arcuate Ligament Syndrome (MALS) A 53-year-old woman with
a history of epigastric pain and postprandial nausea and early satiety presents with
20 lb. weight loss. There is no history of smoking or cardiovascular risk factors. She
is placed on a PPI without relief. She has an EGD that is normal and a gastric
emptying scan that reveals mild delay. A gastroparesis diet does not help much. A
CT scan is negative except for mild small bowel dilation that is no longer present on
CTE. She visits the ED after an episode of more severe pain where she is found to
have mild abdominal tenderness, an abdominal bruit, slightly elevated lipase and
lactate. A CT scan is ordered and suggests MALS.
Is MALS a real entity? Do some of these patients have functional symptoms,
and do they really require surgery?
The European Society of Vascular Surgery guidelines consider this to be one of
the most common causes of mesenteric ischemia that is not associated with athero-
sclerosis or thrombosis. In other words, they consider it a real entity.
One of my patients who had surgery for MALS did well for about a year, but then
started having recurrent abdominal pain likely due to untreated depression. I realize
now that part of her disease process was probably untreated depression. There are
studies showing that patients who have untreated psychiatric problems frequently
have poorer outcomes after surgical treatment of MALS [8]. While this was a mixed
result for my patient, MALS surgery should still be considered after a thorough
evaluation ruling out other causes and first treating associated conditions such as
depression.
How does duplex Doppler US help you?
A duplex Doppler US has good sensitivity (80%) and even better specificity
(95%) when used to screen for MALS. There are specific velocity patterns that are
seen in MALS including a change in pulse velocity with respiration. A dynamic
MRA or preoperative angiography is also sometimes done to see if there is com-
pression of the celiac artery during the respiratory cycle.
108 J. Nelson

Why are you interested in a dynamic MRA? Why is there a change during
the respiratory cycle?
A hallmark of MALS is expiratory compression of the celiac artery by the liga-
ment of the diaphragm crux with an increase in flow velocity that can be found on
Doppler US. Most of the time it is related to an anatomic variation in the location of
takeoff of the celiac artery that is higher than normal.
How would you manage this patient?
I would refer the patient to a vascular surgeon who has experience treating
patients with MALS.
Pre-op angiogram shows narrowing of the celiac with expiration. Surgery is per-
formed, releasing the ligament and decreasing vascular resistance. The patient
remains asymptomatic 1 year after surgery.

References

1. Clair DC, Beach JM. Mesenteric Ischemia. N Engl J Med. 2016;374:959–68.


2. Ahmed M. Ischemic bowel disease in 2021. World J Gastroenterol. 2021;27(29):4746–62.
3. Gnanapandthan K, Feuerstadt P. Review article: mesenteric ischemia. Curr Gastroenterol Rep.
2020;22:17.
4. Blaser AR, et al. A clinical approach to acute mesenteric ischemia. Curr Opin Crit Care.
2021;27:183–92.
5. YuShuang X, et al. Diagnostic methods and drug therapies in patients with ischemic colitis. Int
J Color Dis. 2020;36:47–56.
6. Fitzgerald JF, Hernandez LO III. Ischemic colitis. Clin Colon Rectal Surg. 2015;28:93–8.
7. Sadalla S, et al. Colonoscopy-related colonic ischemia. World J Gastroenterol.
2021;27(42):7299–310.
8. Goodall R, et al. Median arcuate ligament syndrome. J Vasc Surg. 2020;70:2170–6.
Chapter 17
Polypectomy

Zachary Smith and Matt Mohorek

Cold vs. Hot Snare for Flat Polyps

Case 1 A 52-year-old male comes in for his first screening colonoscopy and is
found to have a 1.2-cm flat polyp in the sigmoid colon.
How do you decide whether to use cold snare polypectomy or hot snare?
I will start by saying that I frequently favor cold snare over hot snare polypec-
tomy, but anyone doing a polypectomy should evaluate three things. The first is the
anatomic location of the polyp, the second is the polyp morphology, and the third is
whether to do en bloc vs. piecemeal resection.
In this case, most gastroenterologists would resect the polyp en bloc using hot
snare polypectomy. While hot snare polypectomy is more likely to lead to delayed
bleeding, in this case, we’re dealing with a polyp in the left colon and we know that
polyps in the left colon are less likely to have delayed bleeding compared to polyps
in the right colon [1]. The second thing is that if you take a 12-mm polyp out with a
cold snare, the chance of complete en bloc resection is far less compared to taking
that polyp out with a hot snare. So, for those reasons hot snare polypectomy may be
favorable in this situation.
Unfortunately, we don’t have good data to evaluate polyp recurrence rates in
comparing cold snare piecemeal polypectomy with hot snare en bloc polypectomy.
Now intuitively, one would think that en bloc resection is going to have less of a

Z. Smith (*)
Division of GI and Hepatology, Department of Medicine, Medical College of Wisconsin,
Milwaukee, WI, USA
e-mail: [email protected]
M. Mohorek
Department of Gastroenterology, GI Associates, Wauwatosa, WI, USA
e-mail: [email protected]

© The Author(s), under exclusive license to Springer Nature 109


Switzerland AG 2023
W. H. Sobin et al. (eds.), Managing Complex Cases in Gastroenterology,
https://doi.org/10.1007/978-3-031-48949-5_17
110 Z. Smith and M. Mohorek

chance of recurrence compared to piecemeal and hot is going to have less chance of
recurrence compared to cold. Per the guidelines, if you have a single 10 mm + ade-
noma removed en bloc, that’s a 3-year interval for repeat colonoscopy, whereas if
you take it piecemeal, you might feel less comfortable waiting 3 years.
So, there are a lot of things to consider in this case, but I think, putting together
all of the available data, hot snare polypectomy is still probably the preferred modal-
ity in this case.
Let’s say you are referred a patient with a large, laterally spreading granu-
lar tumor. You favor cold snare polypectomies. Do you have a size limit beyond
which you will not use cold snare?
No, I do most of my EMRs cold and I have taken out polyps that occupy >50%
of the circumference of the right colon cold. What cold snare EMR has done is that
it has changed the conversation and allowed us to have very safe options for remov-
ing large polyps in high-risk patients. We are frequently referred patients who were
scoped by community gastroenterologists after being found to be anemic, and large
polyps were discovered. Some of them are in their eighties, on Eliquis, and they’re
poor surgical candidates. The referring doctor is uncertain how to manage the situ-
ation. Whereas previously I might have been reluctant to offer one of these high-risk
patients conventional (hot snare) EMR due to the inherent risk of delayed bleeding,
etc., I am now much more willing to proceed, using cold snare EMR. It expands the
patient pool where we are able to offer therapy with polypectomy while maintaining
a risk/benefit ratio that is still favorable. After all, it’s hard to assess whether a 7 cm
laterally spreading granular tumor without high-grade dysplasia will turn into colon
cancer during a patient’s lifetime. It’s much more satisfying if we can safely remove
the polyp.
With hot snare EMR it is advised to ablate the margins of the resection site.
Do you ablate the margins when you do a cold snare polypectomy?
No, I don’t. The technique is still in its infancy where we don’t have a lot of data
on the optimal way to do things. Conventional, hot EMR has been around for
decades, and we just found out in 2019 that ablating the edge of the polyp is a good
way to minimize recurrence [2]. I don’t know if we are going to have cold EMR
ablation data for quite a while so my practice personally is to do more of a wide-­
field resection where you are taking several millimeters on the circumference of
normal-looking mucosa.
When you are doing this, you are thinking of a couple of things. First, the safety
profile of cold EMR is favorable, but it is also much more cost-effective. We were
able to show this in a cost-effectiveness study published in Endoscopy [3]. The
main determinant of cost-effectiveness for cold EMR is the lack of need for prophy-
lactic clipping. On the sensitivity analysis, when you are looking at various factors
determining cost-effectiveness, including the cost of clips, cost related to polyp
recurrence, and cost related to complications such as bleeding or perforation, it
turns out that the number one determinant of cost-effectiveness is the cost of clips
and the reason is that Medicare and most commercial insurances don’t reimburse
the cost of endoscopic clips. So, if you take out a 6-cm polyp and you use 8 clips
and each clip is $200, that is a significant cost to the patient.
17 Polypectomy 111

Therefore, consistent with this, when I perform a large polyp EMR cold, I try to
keep my device usage as low as possible. If you are going to ablate the edges of a
polyp that would require opening up some other device, either a hot snare (using the
tip for ablation) or an APC catheter. Until we have evidence that it helps tremen-
dously, it is not my practice to ablate the edges when using cold EMR technique.
Can you discuss the different types of snares on the market and the advan-
tages and disadvantages of each type?
There are a host of snare sizes, shapes, and designs. One can get very over-
whelmed with the available options. My suggestion is to get comfortable with at
most five and know how to use those. There are a lot of snare options, such as duck-­
billed snares and rotatable snares, and in my opinion, a lot of this is overly nuanced
and not clinically useful. So, I think that if you have a cold snare that you are com-
fortable with, and a couple of hot snares of various sizes, you can do most anything.
Certainly, be willing to try new snares, but folks are going to be most successful by
knowing their own arsenal and what they are most comfortable with.
Now, for cold snares specifically, there are a handful of dedicated cold snares out
on the market. Boston Scientific and STERIS are probably the two most well-­
known. Boston makes the Captivator cold, which is a 10-mm oval snare. Steris
makes the Exacto, which is a 9-mm hexagonal snare. Both of these function simi-
larly. There are some other companies that have dedicated cold snares with various
shapes and sizes. Diversatek has both a 10-mm and 15-mm cold snare that is
rotatable.
What you want with a cold snare is something that is going to capture tissue and
cut reliably. Without the use of diathermy, you need to rely on the mechanical force
of the snare to get through tissue. There are some technical nuances to that as well,
but what you don’t want to do is use a snare designed for hot polypectomy for cold
snare EMR. The manufacturers of hot snares don’t put a premium on using mechan-
ical force to cut through the polyp without heat, and so it is going to be much more
challenging to do cold EMR with those devices.

Anticoagulation After Polypectomy

Case 2 A 65-year-old woman who is on apixaban for atrial fibrillation (held for
2 days prior to colonoscopy) is found to have a 1.5-cm sessile polyp in the right colon.
Would you prefer to use cold snare or hot snare to remove this, why?
Personally, if anybody is on an antithrombotic, I am using a cold snare. Post-­
polypectomy and post-EMR bleeding can be morbid, and over 50% of the time, it
will require a second colonoscopy, so there is a significant cost associated with that
complication. Back in the days when I was doing more frequent conventional EMR,
I would instruct patients to avoid traveling for 2 weeks because if they are going to
have a delayed bleed, they need to be where good health care is accessible. We’re
seeing very low bleeding rates with cold EMR, the delayed bleeding risk is
112 Z. Smith and M. Mohorek

negligible. Therefore, I almost always place the patient back on their antithrombotic
agent on post-procedure day 1, unless there is an extenuating circumstance, I have
never had a delayed bleed and am well over 200 cold EMRs currently. The impetus
for delayed bleeding is diathermy-induced ulcers, and with cold, we just don’t
have that.
After resection would you routinely place clips to close the polypectomy site?
Cold EMR does not require clips. I think that this is a tough pill for people to
swallow sometimes. You take a big polyp out from the right colon and then leave it
unclipped, which can be a little unnerving for people. I think in all of the cold EMRs
I have done, I have placed two clips total and they were for vessels that were persis-
tently bleeding. The bottom line is that the observational data available suggest that
clips are not needed with cold EMR [4, 5].
Now for hot EMR the clinical trial data obviously suggest otherwise. The most
robust data for this are from LPS, a study published in Gastro in 2019 where they
randomized polyps larger than 2 cm to clips vs. no clips following hot EMR [1]. In
patients who had right-sided polyps proximal to the splenic flexure, the bleeding
risk in the clip arm was reduced by about two-thirds from 9% to 3%. The caveat to
this is that not every defect can be closed completely. In the LPS trial, only about
two-thirds of defects were able to be closed with clips, either because they were too
big, or in a difficult location. So, current evidence tells us that if you take out a large
polyp in the right colon using hot EMR you should attempt to close the defect
with clips.
How long would you hold apixaban afterward?
My anecdotal evidence suggests that if the polyp was removed with cold EMR,
anticoagulation can be restarted on day one after polypectomy. Every once in a
while, I will wait a little bit longer if I have a gut feeling that a patient is at low risk
for a cardio-embolic event but higher risk for bleeding. I may wait an extra day, but
otherwise I will restart the next day.
Would any of the above three answers differ if the patient was on clopido-
grel instead of apixaban?
Not based on my experience. Remember, most people on antiplatelet therapy are
receiving it for prophylaxis of cardiovascular stent thrombosis or stroke prophy-
laxis. Major cardiovascular and cerebrovascular events are a larger threat to these
patients’ lives, and therefore, the importance of promptly restarting anticoagulation
far outweighs the risk of having a delayed bleed from cold EMR. So, I would abso-
lutely resume these medications on day one.
Do you hold anticoagulation or antiplatelet agents prior to an elective
screening colonoscopy?
I do, and the reason is that you never know what you are going to find. Personally,
I don’t do a lot of screening colonoscopies, but because I do EMR and I am comfort-
able doing higher-risk endoscopic maneuvers on an index colonoscopy, I want the
patient to be prepared for possible EMR. For example, if I find a 5-cm polyp on a
screening colonoscopy I would remove it right there rather than bringing the patient
17 Polypectomy 113

back. With that being said, there are data from the Annals of Internal Medicine that
looked at cold snare vs. hot snare in patients on antiplatelet therapy and cold snare
was significantly safer. So, we can do cold snare polypectomy on anticoagulation/
antiplatelet agents, and I would not hesitate to do that if the need arose, but gener-
ally, we don’t find many situations where a patient is too sick to hold anticoagula-
tion/antiplatelet therapy for an appropriate length of time but is still appropriate for
screening colonoscopy. Having said that, if I’m called in to assist on a case where I
encounter an 8 mm sessile polyp in a patient on anticoagulation I would have no
hesitation to remove it with a cold snare.

EMR

Case 3 A 68-year-old man comes in for his first screening colonoscopy with his
community gastroenterologist and is found to have a 3-cm flat adenomatous appear-
ing polyp in the transverse colon.
If a gastroenterologist in the community finds a large flat polyp on an index
colonoscopy, should he generally remove it then, or come out and discuss it
with the patient after the procedure and set it up for another day? Or, for that
matter, should he refer it to someone who specializes in EMR?
This entirely revolves around the skill set of the endoscopist. This is a recurring
theme and something that I am very passionate about. For me personally, as some-
one who is a high-volume EMR provider and comfortable with techniques that
minimize the risk profile of EMR, I would have no hesitation taking this out at the
time of the index procedure. That being said, there are some medical-legal things to
consider. For example, if you are very cursory with your consent process and you
have not specifically talked about high-risk interventions, it is very justifiable to
bring the patient back.
However, the thing that I can’t stress enough is that if someone finds a large
polyp on colonoscopy and they don’t do a lot of EMR, the last thing you want to do
is to start taking out the polyp and not finish the job. Doing an incomplete polypec-
tomy causes a lot of submucosal fibrosis, which makes any attempt at salvage resec-
tion markedly more challenging. So, if you start resecting and then determine the
polyp is in a difficult spot, or it’s bigger than you thought it was, or it starts bleeding
and you decide to just stop, and refer it out, or attempt it another day, realize that
you’re jeopardizing the success of the polyp resection. A good way to increase the
chance of a patient needing surgery for a benign polyp is to do an incomplete EMR
before referring the patient to an expert center. So, if you look at the polyp and you
can’t tell yourself with 100% certainty that you can remove it, then you should not
attempt it. So, for your standard community practice general GI doc who sees a
gambit of GI pathology and is not a high-volume EMR person I would not resect
this for the reasons I have mentioned above.
114 Z. Smith and M. Mohorek

Do you feel like there is any role for EMR in an ambulatory endoscopy cen-
ter not attached to a hospital?
The reason not to do an EMR in an ASC, everything else being equal, is lack of
resources in case something goes wrong. If I run into trouble at an ASC, an emer-
gency surgery consultation is just not available. Part of the reason that the rate of
surgery for benign polyps continues to rise (at least through 2014 according to
observational data) is that a lot of people are averse to using EMR technique, but if
people can get more comfortable with cold EMR technique, understand the tech-
nique, understand that it is adequate, and fully grasp that the risk profile is markedly
reduced, I think many more of these polyps will be removed by gastroenterologists,
rather than being referred to surgeons. I think that doing a cold EMR at an ASC
becomes much more justifiable because the likelihood of a complication, where you
need an emergency surgery consultation, is so negligible. Having an immediate
perforation from a cold EMR is a reportable case event, and the ones that have been
reported are near surgical anastomoses or in patients who have some sort of predis-
position. In a patient who has a native colon, no inflammatory bowel disease, or
other risk factors, the chance of a perforation on a cold EMR is basically zero. So, I
think that it is completely justifiable to use cold EMR techniques at an ASC.
What do you like to use to inject to lift polyps? Will you ever use saline for
lifting?
Typically, I do still lift polyps and will use one of the commercially available
viscous solutions. However, emerging observational data from Michael Bourke’s
group and others suggest that lifting for large cold resections may not be necessary
[5, 6]. However, I think the verdict is still out on that. LPS-2, which we are involved
in, is a 2 × 2 randomized trial comparing hot and cold EMR as well as commercially
available viscous injectate vs. saline and methylene blue. So, there should be some
comparative data on solutions from this trial. A lot of the data on injection solutions
have looked at durability of lift, completeness of resection, resection time, and
polyp recurrence, so establishing meaningful endpoints is important. Sometimes
these solutions can be cost-prohibitive for smaller centers. In those circumstances,
saline or hydroxyethyl starch and methylene blue is an appropriate alternative. For
me personally, with EMR, unless I am doing an underwater EMR technique, I am
still injecting a submucosal agent.
This is your partner’s case and he is doing a submucosal injection and is finding
that this relatively flat polyp is flattening out further with the injection making it
difficult to get a snare around the polyp and calls you in for advice.
How do you avoid this problem with your injection?
One benefit of the commercially available injectates is that for the most part they
don’t tend to spread laterally as much as saline does. With that said, flatness of
injection can happen for a couple of reasons. First, it can be a technique problem
and there are ways to potentially combat this. They include:
Inject Slowly. If the injectate seems to be spreading laterally and not lifting, stop
and consider re-inserting the needle in a different place.
Direct the tip of the colonoscope upward to create tension away from the colon
wall while injecting.
17 Polypectomy 115

If using saline and not using a viscous solution (commercial or otherwise), con-
sider changing.
Second, there are other situations, like ulcerative colitis, where you might get
lateral spread of the injectate. Even if these patients have quiescent colitis, they have
had some submucosal fibrosis over time and this interferes with lifting.
There are some endoscopic techniques to try to get the snare to capture, even
when the polyp remains flat and doesn’t lift. Some of those are more advanced tech-
niques that I do not recommend unless one is trained. One example is to do a muco-
sal incision as if you were doing an endoscopic submucosal dissection around the
polyp and then create a little notch around the mucosa where the snare is able to
latch on and grab the tissue. This is commonly referred to as precut EMR.
Sometimes, just changing your snare to a thinner wire snare is enough to combat
this problem. Sometimes approaching the polyp from a different angle will help.
Is there anything to do once flattening has occurred?
Mucosal incision is one suggestion. Another suggestion is to convert to an under-
water technique. Here, you take air out of the colon and then you immerse the colon
with water using your pedal. Water immersion has a buoyant effect on the submu-
cosa. This was first described by Ken Binmoeller in San Francisco when he was
doing endoscopic ultrasound. He noticed that when water was instilled while look-
ing at submucosal lesions, there was expansion of the submucosa and he parlayed
that into the world of endoscopic resection. Underwater EMR gives you a lot of
benefits, it allows you to work in a collapsed space, and tissue is much easier to grab
in a collapsed space. It allows you to work very close to a polyp, giving you a mag-
nified view of everything. So, in this situation I would probably quickly convert to
an underwater technique.
Do you always inject into the polyp? Do you ever inject just adjacent to it?
It depends on the size and location of the polyp. I will inject into the polyp if it is
2 cm. or less, especially with sessile serrated polyps, just to get one uniform injec-
tion. If it’s a big polyp or in a saddle distribution (meaning over a fold), I will typi-
cally start by injecting on the oral side of the polyp and then work toward the anal
side. I won’t try to inject a whole 5 cm polyp at once, I will inject the part that I am
going to cut out first and then I will re-inject and take the rest of the polyp. This
seems to keep things plump when you are doing the cutting and doesn’t allow injec-
tate to diffuse out while you are working on the other side of the polyp.
What is your technique for removing large polyps using a cold snare
technique?
The most important things are the same regardless of cold vs. hot technique. You
want the scope to be straight, reduced, and have the polyp at the 5:00 to 7:00 region.
I always lock my wheels. I want scope stability as much as possible.
The main difference is that cold snares are smaller and so typically you are tak-
ing less tissue out per bite. When I am teaching people, I compare it to mowing your
lawn. You start on the edge near your sidewalk and you are then going to have some
overlap every time you go back and forth. So, you put your snare down, you make
a cut, it doesn’t matter how big your first cut is, and then your second cut is going to
lay on the edge of the resection defect of your first cut and is going to grab mucosa.
116 Z. Smith and M. Mohorek

So, if you are working left to right, the left side of the snare is going to sit in the
submucosa where you made your first cut, the right is going to scoop tissue on the
right, you are going to bring it together, and move down the line. When the back side
is done from left to right you move a little toward the anal side and overlap a little
with the resected zone on the oral side and repeat the process until you are done.
When you remove bigger polyps, you are going to see some lumps and bumps of
adenoma that don’t come up with the cold snare and you can go back and take those
with snare or avulsion techniques. For the most part, it is a systematic process and
it goes a lot faster than people think it will.
When you’re removing a large flat polyp in the cecum would you manage it
differently to avoid complications?
Probably not. While cecal polyps can hide in funny places, like behind the ileo-
cecal valve or juxtaposed to the appendiceal orifice, making things more challeng-
ing, for the most part my technique is going to remain the same whether the polyp
is located in the cecum, ascending colon, or transverse colon.
What are the polyps that you will send to the surgeon?
Usually, surgeons send me polyps. It is common for patients to be referred to
surgery for “unresectable” polyps by general community gastroenterologists. We
now know that surgery is associated with higher morbidity and mortality, higher
rates of procedure intervention, higher cost, and longer hospital stays than endo-
scopic resection. For the vast majority of people, surgery is not the best option.
Some gastroenterologists don’t understand the full implications of surgical inter-
vention required to address the lesions detected. Take for example a case where two
large “unresectable” polyps are detected in the ascending colon and splenic flexure.
What they need to think about is that the surgery that is required to address this
would require a subtotal colectomy and that has a major impact on the patient’s
quality of life. There are data from Stanford and Australia that both suggest that in
patients who are referred to surgery for large, “unresectable” polyps, referral to an
expert endoscopist or a high-volume EMR center can avoid surgery 90% of the time.
Having said that, there are a few instances where I will send the patient to a sur-
geon. The first is recalcitrant polyps. What I mean by this is if we remove a large
polyp and bring the patient back 6 months later to take another look there is a sig-
nificant recurrence of polyp. We then resect the polyp regrowth. Then, on follow-up,
6-12 months later there’s significant recurrence and you realize that you are not able
to completely resect it. These situations are more common in patients who have
significant fibrosis, either because they had a previous, limited EMR attempt prior
to seeing me, or they have some other condition that predisposes them to submuco-
sal fibrosis like ulcerative colitis.
The second instance is when I am referred to a polyp that is presumed to be
benign but it turns out there is invasive cancer on the resection specimen.
Occasionally, this is apparent on initial inspection using careful optical observation
with electronic chromoendoscopy, near-focus imaging, and you see clear evidence
of deep submucosal invasion. You do targeted biopsies, it proves cancer. Those
patients go to surgery, or, the polyp may look benign, but once you start taking it
out, it becomes obvious that there is cancer by the way it is behaving. Either it’s not
17 Polypectomy 117

lifting well, there is a lot of tissue that is matted to the submucosa, or it is bleeding
more than you would expect. All of those are indicators that there is invasive cancer.
In those circumstances, they go to surgery as well.
Most of the time, if I am referred to a polyp that has not been intervened on, I am
successful in removing the polyp and keeping the patient out of the operating room.
It is usually these few extenuating circumstances that could lead to an operation.
The vast majority of time, if someone else doesn’t partially remove the polyp we are
usually able to durably resect and cure those lesions.
What are the endoscopic features that most strongly suggest deep submuco-
sal invasion?
I would encourage people to become familiar with a pit pattern classification
such as Kudo. I think the simplest one for me is the NICE classification. It is a
narrow-band classification validated on the Olympus platform. It is a way to distin-
guish hyperplastic and serrated lesions from traditional adenomas and from submu-
cosal invasive cancer. It is a very simple classification, and I think you just have to
look very carefully. If you are using a 190-series adult scope, do near-focus imag-
ing. Sometimes, imaging something underwater is another way to get a nice per-
spective of the pit pattern.
I do very careful optical examination before I remove a polyp, because if there is
submucosal cancer my approach is very different. Another important finding on the
optical examination is surface ulceration. The finding of surface ulceration, visible
on regular high-definition white light imaging, goes hand in hand with NICE type
III pit pattern classification. If something looks ulcerated on the surface, the chance
of submucosal cancer is high regardless of the size.
I just had a case where a patient was referred by a community gastroenterologist
for EMR of an 11 mm polyp in the cecum with high-grade dysplasia. Although the
polyp was not that large it had surface ulceration and I explained that I was con-
cerned about possible invasive cancer, based solely on the surface ulceration. I did
the EMR. The polyp lifted great, we did an en bloc resection, the base of the polyp
looked pretty reasonable, clipped it, and the pathology came back as margin-­positive
invasive adenocarcinoma. He ended up with a hemicolectomy anyway.
Another indicator of malignancy is any lesion that is near circumferential. The
chance is high that there is an invasive cancer somewhere in that lesion. Then, if
something is just bleeding or oozing more than it should, that is usually a good
indication that you might be dealing with cancer.
What about the use of the Paris classification to suggest the likelihood of
deep submucosal invasion?
In the Paris classification, lesions that have a central depression, either IIc or II
a + c lesions (something that is raised on the edge and depressed in the middle),
can harbor cancer. These are usually accompanied by surface ulceration. Lesions
that have a 1-s component (which is what we call a dominant nodule) tend to har-
bor high-grade dysplasia, not necessarily cancer per se. I always look for those
areas specifically to ensure that I’m concentrating on getting those sections out.
So, as far as the benign Paris lesions, those are the two that would indicate poten-
tial cancer.
118 Z. Smith and M. Mohorek

Pedunculated Polyp

Case 4 A 55-year-old man is found to have to have a large (about 1.8 cm) pedun-
culated polyp on a thick stalk.
Is there any role for cold snare polypectomy of a pedunculated polyp?
I would say no, it’s just going to turn into a bloody mess if you are able to tran-
sect it. If you are unable to transect it, which is the most likely scenario because you
are cutting through a huge stalk and big vessel, then you are just getting yourself in
trouble. These polyps invariably require hot snare.
I will take out pedunculated polyps less than 1 cm in size in the left colon with
cold snare, especially if I already have a cold snare out and have already been using
it. They will bleed a little bit more in the short turn, but they will thrombose and stop
quickly.
Do you like to pre-treat the stalk in patients with a thick stalk? If so, what
do you like to use?
I will if it is a big polyp and I am having difficulty getting a snare around it. If
you inject standard epinephrine into the stalk of the polyp that will diminish the flow
from the feeding vessel(s) and result in an ischemia-induced reduction in size. The
polyp will shrink in real time and that may make enough of a difference to change
something from not capturable to capturable. Stalk, injecting into the stalk.
If I can get a snare around a polyp comfortably, I’m usually not pre-treating the
stalk. When I’m removing a polyp on a thick stalk, I know beforehand that I will
probably clip the stalk afterward. I prefer to clip it following polypectomy, rather
than before, because if a stalk starts bleeding post-polypectomy and you clip it and
it stops, then you know your clip is in the right place and you can feel good about it.
Some people will place standard endoscopic clips at the stalk base prior to pol-
ypectomy. I have avoided doing that because the stalks are frequently thicker than
the standard 11 mm clips and you can’t be sure that you are effectively tamponading
the vessel. Now that there are larger, 18 mm clips, and there may be more of a role
for them, but so far that is not my practice.
If I encounter a very high-risk polyp with a thick stalk and I am tempted to clip
prior to polypectomy, I prefer to use a detachable snare. A lot of people are uncom-
fortable with the deployment of a detachable snare because the deployment systems
can be a little tricky. When you cinch down on a detachable snare, you run the risk
of cold-ligating the stalk causing heavy bleeding if you aren’t careful or you deploy
it and it isn’t tight enough, which makes it ineffective. So, there is a very systematic
way to open the snare, get it around the polyp, and get the cinch down to the point
where you know that there is tamponade of the vessel that is indicated by color
change of the polyp as you tighten the loop.
In those cases, where you opt to use hot snare discuss the various cautery
settings that are available and how you decide on the particular ones in a par-
ticular setting.
There has always been a blue-pedal-yellow-pedal debate (on the ERBE). I have
always been on team yellow, but there was a pretty well-done randomized control
17 Polypectomy 119

trial looking at this exact question, yellow vs. blue pedal for complex polypectomy,
and found no difference in resection success and complications [7], so it looks like
you can probably do either in those circumstances. Some people feel that the blue
pedal has more cautery associated with it so maybe your bleeding risk is going to be
less. Any source of blended current (if you are using an ERBE or similar device) is
going to have some cautery current built-in with the cutting current. There is usually
some tissue impedance that directly feeds back to the generator that allows it to alter
the settings as it cuts through. So, I still use the yellow pedal for pretty much every-
thing (ENDOCUT Q for ERBE systems). Then it’s just personal preference on cut
duration, cut interval, and effect. I think whatever you are comfortable with is what
you should stick with.
Your partner is performing the polypectomy and has not pre-treated the stalk.
There is heavy bleeding at the stalk immediately following polypectomy.
How do you like to manage this?
You can do a handful of things. I think mechanical tamponade is the best
approach. If you want to inject epinephrine to slow things down so you can better
see where the bleeding is coming from that is also completely appropriate. But
standard endoscopic clips are fine in this scenario, and I think that while heavy
bleeding does impair visualization, I always tell the fellows that bleeding is a good
thing because once it is treated and the bleeding stops you know that your clips are
in a good position.

Post-Polypectomy Bleeding

Case 5 Your partner performs a cold snare polypectomy on a 48-year-old patient


in good health. The polyp was described as 6 mm in size, located in the sigmoid
colon. That Friday night you get a call from the patient that he has had two bloody
bowel movements since the colonoscopy that morning, about 4 h apart.
How would you advise this patient?
Anytime the patient calls, regardless of how the polyp was taken out, you want
to clearly assess their symptoms. Are they lightheaded? If so, is this postural?
If a polyp was taken out in the left colon, it was under 1 cm in size, and removed
cold, the chance of having a bleed that requires intervention is well below 1%. What
I mean by that is not the proportion who have cold snare polypectomy, but the pro-
portion of those who call you with bleeding after a cold snare of a diminutive polyp
in the left colon. So, reassurance is the most common thing, but besides that, I want
to make sure we are keeping an eye on things, the patient knows when to go to the
emergency department, etc.
The next morning the patient is still having bloody bowel movements and
you decide to perform a repeat colonoscopy that weekend. There is ongoing
bleeding from the polypectomy site. How would you treat it?
I would clip it.
120 Z. Smith and M. Mohorek

There is frequently mild oozing of blood at the time of cold snare polypec-
tomy. How do you decide whether the bleeding will stop spontaneously? When
does the bleeding appear excessive?
It is very unusual for something to bleed persistently at the completion of a pro-
cedure. Most things have slowed down to a trickle or have stopped completely, even
after a big EMR. Every once in a while, you can hit a big vessel with a cold snare so
if something doesn’t look like it is slowing down and you have given it 5 min or so,
nobody is going to fault you for putting a clip or two on there, but this is rarely
needed with cold polypectomy.
Do you ever see delayed bleeds after a cold snare polypectomy starting up
days after the event?
It is very uncommon. Again, we think the impetus for delayed bleeding is dia-
thermy and ulcer formation so it just rarely happens.
Slightly different scenario is the polyp is now 1.5 cm in size and was located in
the descending colon and removed with hot snare. The bleeding starts up 7 days
post-procedure and is quite brisk. You admit the patient and perform a colonoscopy
the next morning. There is active bleeding from the polypectomy site.
What will you use to stop the bleeding in this case following hot snare
polypectomy?
I think clips are still my choice. Usually, with clips you can focus your treatment.
You will likely see a big ulcer and it’s going to look ugly, but the culprit vessel is
going to be very obvious and so typically one or two clips on the culprit vessel are
going to be enough for hemostasis to occur. I wouldn’t see a need to close the entire
defect 7 days later, just treat the bleeding site because it’s unlikely that anything else
in that ulcer is going to bleed at that point.

References

1. Pohl H, Grimm IS, Moyer MT, et al. Clip closure prevents bleeding after endoscopic resection
of large colon polyps in a randomized trial. Gastroenterology. 2019;157:977–984.e3.
2. Klein A, Tate DJ, Jayasekeran V, et al. Thermal ablation of mucosal defect margins reduces
adenoma recurrence after colonic endoscopic mucosal resection. Gastroenterology.
2019;156:604–613.e3.
3. Mehta D, Loutfy AH, Kushnir VM, et al. Cold versus hot endoscopic mucosal resection for
large sessile colon polyps: a cost-effectiveness analysis. Endoscopy. 2022;54:367–75.
4. Mangira D, Cameron K, Simons K, et al. Cold snare piecemeal EMR of large sessile colonic
polyps >/=20 mm (with video). Gastrointest Endosc. 2020;91:1343–52.
5. van Hattem WA, Shahidi N, Vosko S, et al. Piecemeal cold snare polypectomy versus conven-
tional endoscopic mucosal resection for large sessile serrated lesions: a retrospective compari-
son across two successive periods. Gut. 2021;70:1691–7.
6. Tate DJ, Awadie H, Bahin FF, et al. Wide-field piecemeal cold snare polypectomy of large
sessile serrated polyps without a submucosal injection is safe. Endoscopy. 2018;50:248–52.
7. Pohl H, Grimm IS, Moyer MT, et al. Effects of blended (yellow) vs. forced coagulation (blue)
currents on adverse events, complete resection, or polyp recurrence after polypectomy in a
large randomized trial. Gastroenterology. 2020;159:119–128.e2.
Chapter 18
Rectal Incontinence

Ling Mei and Krupa Patel

Case 1 A 73-year-old woman comes to your office complaining of rectal inconti-


nence that has occurred on five occasions. With each episode, she is aware of the
urge to defecate but can’t “hold it” long enough to get to the toilet. There were a
number of other episodes when she felt the urge to defecate, but was just barely able
to get to the toilet on time. This all started about 3 months previously, after a bout
of “food poisoning.”
Is this passive, urge, or seepage incontinence? What is the distinction
between the three forms, and why is it important?
This is urge incontinence. Urge incontinence is the discharge of rectal contents
despite active attempts to retain them. Passive incontinence is the involuntary dis-
charge of feces or flatus without awareness. Fecal seepage is involuntary seepage
with otherwise normal evacuation.
The patient has a history of a complicated pregnancy when she was 20 years old,
which required an episiotomy. She has had no other anorectal operations and no
abdominal surgery. Her bowel movements occur once or twice a day, which is
unchanged from her norm. She denies urinary incontinence. Her last screening
colonoscopy was 4 years ago, when she was 69, which was normal, and no biopsies
were taken. Her physical examination is unremarkable except for the rectal exami-
nation, where she has a slightly diminished rectal tone.

L. Mei (*)
Department of Medicine, GI/Hepatology Division, Medical College of Wisconsin,
Milwaukee, WI, USA
e-mail: [email protected]
K. Patel
Methodist Medical Group, Dallas, Texas, USA
e-mail: [email protected]

© The Author(s), under exclusive license to Springer Nature 121


Switzerland AG 2023
W. H. Sobin et al. (eds.), Managing Complex Cases in Gastroenterology,
https://doi.org/10.1007/978-3-031-48949-5_18
122 L. Mei and K. Patel

Can you describe how you do your rectal examination in someone with
incontinence?
The digital rectal examination is a very important aspect of the physical exami-
nation in a patient with incontinence. Here is a description of the full rectal
examination:
1. Ask the patient to lie in the left lateral position.
2. Inspect the perineum and external anus by spreading the buttocks after donning
a pair of gloves, to look for any dermatitis, scarring, skin tags, hemorrhoids,
rectal prolapse, fistulas, or fissures.
3. Stroke the skin around the anus using a Q-tip to assess the anocutaneous reflex.
When the reflex is acting normally, sensory stimulation of the skin around the
anus leads to anal sphincter contraction (anal wink). A normal anocutaneous
reflex indicates an intact sacral reflex arc and pudendal nerve innervation of the
external anal sphincter.
4. Ask the patient to strain to evaluate the presence of perineal descent, prolapsed
hemorrhoids, and rectal prolapse.
5. Assess the resting tone by gently inserting your index finger into the anal
canal. This is best measured after waiting a few seconds to allow for accom-
modation. The resting tone is predominantly attributable to the internal anal
sphincter. A reduced tone may indicate weakness of the internal anal sphincter,
while the presence of increased tone may contribute to difficulty with
defecation.
6. Have the patient perform a voluntary anal squeeze to evaluate the external anal
sphincter tone. Reduced anal squeeze pressure may suggest weakness of the
external anal sphincter.
7. Palpate the posterior rectal wall to evaluate for any pain. If this is present, it sug-
gests puborectalis muscle tenderness.
8. Have the patient strain and try to push your finger out. Normally, the anal sphinc-
ter should relax associated with perineal descent while bearing down. If the
sphincter muscles seem to tighten and there is no perineal descent, this may sug-
gest dyssynergic defecation.
What are the causes of anal incontinence we are most likely to encounter?
Fecal incontinence may be caused by altered bowel habits and a variety of condi-
tions that affect the ability of the rectum and anus to hold stool. Causes of fecal
incontinence may include diarrhea, fecal impaction, obstetric or surgical sphincter
muscle injuries, pelvic floor injury, neuropathy resulting from diabetes or stroke,
spinal cord injuries, and some inflammatory conditions [1].
What evaluations would you do? Would you do anorectal manometry? If so,
is it high resolution? Would you do an MRI? An endoscopic ultrasound? A bal-
loon expulsion test? A colonoscopy?
If incontinence occurs in the setting of constipation or diarrhea, we have to man-
age the bowel disturbance first. We usually begin with an abdominal X-ray to evalu-
ate stool burden.
18 Rectal Incontinence 123

If diarrhea is a concern, the work-up often includes a colonoscopy, with random


biopsies to assess for microscopic colitis.
If fecal incontinence persists despite managing bowel disturbance, the next step is
to perform anorectal manometry with assessment of rectal sensation and a rectal bal-
loon expulsion test. We prefer to do high-resolution manometry, which is a novel,
solid-state manometric system, which provides more accurate resolution of intralu-
minal pressure and greater anatomical detail compared to water-perfused manometric
systems. The balloon expulsion test is a simple test to screen for a defecation disorder.
If a weak anal sphincter is identified during anorectal manometry, further evalu-
ation with endoanal ultrasound or MRI can be considered to evaluate for muscle
damage, especially when surgery is being considered. EMG can also be considered,
depending on center availability.
MRI is superior for visualizing external sphincter defects, atrophy, and patulous
anal canal. This should only be ordered if there are concerns about defects.
How do you like to manage patients like this?
We would first treat underlying causes such as constipation or diarrhea. For
patients with fecal impaction with an overflow component, I would start a bowel
regimen with a fiber supplement (psyllium) and a daily laxative program. For
patients with diarrhea, we would initially start with dietary modifications, such as
eliminating foods with artificial sweeteners (e.g., lactose, fructose, and sorbitol),
spicy food, and caffeine-containing beverages. We will observe for any improve-
ment in bowel habits and stool leakage. Anti-diarrheal agents, such as loperamide
(non-prescription) or Lomotil (prescription), can be used to reduce loose stool.
Loperamide is usually taken 30 min before meals or after each loose stool. The
maximal dosage is 16 mg/day. Studies have shown that loperamide can also improve
incontinence by increasing resting anal tone [2]. Other medications such as bile acid
binders can be used in patients with IBS-D or post-cholecystectomy diarrhea.
Are there any surgical options you might recommend?
If the above measures have failed, then surgical options can be considered. I
would initially recommend a sacral nerve stimulator, in which the nerves that sup-
ply the rectum and anal sphincters can be activated by a stimulator [3]. In selected
patients, injection of bulking agents into the anal sphincter is another approach that
may improve the anal seal [3]. A surgical sphincteroplasty can be considered in
women with a sphincter defect resulting from vaginal delivery [3]. However, long-­
term outcomes with this procedure are mixed, with about 50% of patients having
recurrence of symptoms [4]. In severe cases of incontinence that are debilitating and
significantly affecting quality of life, a colostomy can be considered.
Can you describe the process of sending patients for sacral nerve stimulation?
Sacral nerve stimulation can be considered in all patients who fail conservative
management. This is a two-stage procedure. First, a temporary nerve stimulator is
attached and used for 2 weeks. Patients whose symptoms respond to the temporary
nerve stimulator are then sent for permanent subcutaneous implantation of the
device. There is about 80% therapeutic success and 40% complete continence at
3-year follow-up [5, 6].
124 L. Mei and K. Patel

Do you have any experience with biofeedback therapy?


This is the mainstay of treatment for patients who fail medical treatment. Pelvic
biofeedback training by combining pelvic floor exercises and visual feedback from
anal manometry or surface EMG can strengthen anal sphincters and improve rectal
sensation. For patients having fecal incontinence due to dyssynergic defecation,
biofeedback therapy, using a different technique, can improve the coordination
between abdominal muscles and the pelvic floor and subsequently improve rectal
emptying. Satisfaction has been reported in up to 70% of patients in prior studies [7].
Are there any particular areas you are personally researching in anal
incontinence?
We have been researching the ability to exercise the external sphincter and
puborectalis muscles beyond simple Kegel exercises with added resistance training
via an exerciser balloon in the anal canal. We first evaluated the ability of the mus-
cles to fatigue more with added resistance [8]. Muscle overload and subsequent
neuromuscular fatigue are necessary requirements for any successful strength train-
ing as shown in rehabilitative exercises [9, 10]. We then assessed the ability to
strengthen the muscles using pre- and post-anorectal manometry after 6 weeks of
resistance training in patients suffering from fecal incontinence. Our preliminary
results have shown improvement of symptoms and sphincter muscle contractility
demonstrated as an increased anal resting and squeeze pressure after 6 weeks of
resistance training [11].
Case 2 A 75-year-old woman has occasional “accidents” where she is unaware
that she is having a bowel movement until she smells stool in her panties, or feels
wetness.
What are some possible etiologies for this?
This patient has passive fecal incontinence. Passive incontinence can occur in the
setting of neuropathy resulting in loss of rectal sensation and weakness of the inter-
nal anal sphincter. This may be seen in systemic illnesses such as diabetes. It can
also be seen in patients with spinal cord injury, stretch injury, or obstetric trauma.
What might the rectal examination show?
Rectal examination might show poor resting sphincter tone.
What would you be looking for on a neurologic examination, and what other
systems might be involved?
Neurologic examination may reveal decreased perianal pin-prick sensation,
lower extremity weakness if related to an underlying spinal cord injury, or reduced
sensation in the setting of neuropathy. The urinary system may also be involved
with incontinence.
What findings would you expect on anorectal manometry?
Anorectal manometry may reveal reduced rectal sensation and/or resting sphinc-
ter tone. Squeeze pressure could be normal or reduced.
How would you manage this patient’s problems?
Management would be different based on the cause of symptoms. In most cases,
we recommend increasing stool consistency by adding fiber supplements and
18 Rectal Incontinence 125

having scheduled bowel movements. Enemas or suppositories may be helpful in


scheduling bowel movements.
We would also recommend biofeedback for rectal hyposensitivity if some degree
of rectal sensation is still preserved. Biofeedback is usually not effective for isolated
internal anal sphincter weakness; in which case, we might consider a bulking agent
(such as dextranomer injection) and/or sacral nerve stimulation.
In some patients, the Eclipse system (in which a vaginal balloon compresses the
rectum) or Renew insert (in which an anal plug is worn) can be considered on a PRN
basis to reduce fecal leakage.
Case 3 A 64-year-old woman has problems moving her bowels and will go several
days without a bowel movement. In spite of difficulties with constipation, she has
occasional episodes where liquid stools pass of their own volition. She has minimal
warning but then feels the stool in her panties or down the side of her legs. Prior
screening colonoscopy, 5 years earlier, was negative. There is abdominal fullness
on the examination, presumably due to increased stool. On the rectal examination,
there is adequate sphincter tone and some stool in the vault, although not a large
rectal impaction.
How would you recommend managing this situation?
It appears the patient has constipation with overflow diarrhea. We would first
recommend an abdominal X-ray to assess the stool burden. After confirming
increased stool, we would recommend a bowel regimen to regulate the bowel move-
ments. We often recommend starting with a colon purge followed by a daily bowel
regimen.
How do you respond when patients complain that the laxatives are only
worsening their incontinence?
You are less likely to have this problem if you start out with a colon purge,
attempting to get the colon cleaned out initially. In these patients, the incontinence
is likely related to fecal retention/impaction, and once you correct this, you’ve cor-
rected their incontinence. We would follow this with a daily laxative regimen. We
also encourage patients to consume adequate daily fiber (25–30 g/day for adults).
We encourage them to include psyllium in their daily bowel regimen. We might also
do anorectal manometry to evaluate the defecatory disorder.
Will you ever use a full colon lavage, like 2 or 4 liters of Golytely?
We do usually recommend a colon purge before starting a daily bowel regimen.
If choosing Golytely for the colon purge, we will start with 2 liters first. If the
patient does not feel adequately emptied after the half dose of Golytely, we will then
advise the patient to complete the second half. Alternatively, a colon purge using a
bottle of MiraLAX or magnesium citrate is also an option if the patient is unable to
obtain or tolerate the Golytely.
Case 4 A 67-year-old man has had a stroke with left-sided weakness. He tries to
schedule his bowel movements once a day in the morning and is successful about
half the time. Recently, he has had episodes of rectal incontinence.
126 L. Mei and K. Patel

What are some possible explanations for this?


There are multiple possible reasons for his altered bowel habits and inconti-
nence. His immobility can be associated with reduced GI motility and incomplete
evacuation/stool retention, which may subsequently result in overflow incontinence.
Since he is post-stroke, his neurologic deficit may be associated with reduced rectal
sensation and decreased sphincter strength, which may also contribute to his
incontinence.
What might you find on the rectal examination?
We may feel the retained stool in the rectum. He may have reduced perianal
sensation and reduced anal sphincter tone.
How would you manage his problems?
We will recommend a colon purge if the patient is able to cooperate. We will
recommend a routine bowel regimen including adequate daily fiber intake and add-
ing a laxative if his stool is hard (Bristol 1 or 2). We will recommend a scheduled
bowel movement using a suppository or enema in the morning if the patient does
not have a spontaneous bowel movement. If his symptoms do not show significant
improvement with the bowel regimen, we will obtain anorectal manometry to fur-
ther evaluate anorectal function.
Case 5 A 54-year-old female psychotherapist is embarrassed because of occa-
sional episodes where she has stool involuntarily soiling her pants. Her bowel
movements seem normal, and she feels like she is totally emptying her rectum with
each bowel movement. Afterward, she tries to clean herself very well. In spite of
that, as the day goes on, she notices stool soiling her underpants almost every day.
What are some possible explanations for this?
This patient has anal seepage, which is defined as staining or streaking of under-
wear. Anal seepage can occur if the rectum is not fully emptied after a bowel move-
ment, something which is seen in dyssynergic defecation, rectocele, radiation
therapy, and with prolapsed hemorrhoids. It responds to different treatments as
compared to fecal incontinence.
Are there dietary or behavioral precautions she could take?
She should maintain a high-fiber diet to help bulk up her stool. We recommend
avoiding excessive wiping after defecation and using alcohol-free wipes. We will
advise the patient to place a cotton ball at the anus to act as an occasional wick.
Some women may benefit from a splinting technique during defecation, particularly
if a rectocele is present. The splinting technique involves placing a finger in the
vagina to push against the rectum to help stool to empty.
Are there medications that might be helpful?
Anal suppositories can be tried to help with emptying the rectum. Patients with
hemorrhoids can try topical steroids to shrink the hemorrhoids and reduce the
irritation.
Are there any other sorts of intervention?
Patients with dyssynergic defecation may benefit from pelvic biofeedback
retraining. Patients with a rectocele may need surgical consultation.
18 Rectal Incontinence 127

References

1. Menees SB, Almario CV, Spiegel BMR, Chey WD. Prevalence of and factors associ-
ated with fecal incontinence: results from a population-based survey. Gastroenterology.
2018;154(6):1672–81 e3.
2. Read M, Read NW, Barber DC, Duthie HL. Effects of loperamide on anal sphincter function
in patients complaining of chronic diarrhea with fecal incontinence and urgency. Dig Dis Sci.
1982;27(9):807–14.
3. Whitehead WE, Rao SS, Lowry A, Nagle D, Varma M, Bitar KN, Bharucha AE, Hamilton
FA. Treatment of fecal incontinence: state of the science summary for the National
Institute of Diabetes and Digestive and Kidney Diseases workshop. Am J Gastroenterol.
2015;110(1):138–46.
4. Glasgow SC, Lowry AC. Long-term outcomes of anal sphincter repair for fecal incontinence:
a systematic review. Dis Colon Rectum. 2012;55(4):482–90.
5. Meurette G, Siproudhis L, Leroi AM, Damon H, Urs Josef Keller D, Faucheron JL, French
Faecal Registry Study Group. Sacral neuromodulation with the InterStim™ system for faecal
incontinence: results from a prospective French multicentre observational study. Color Dis.
2021;23(6):1463–73.
6. Rao SS. Current and emerging treatment options for fecal incontinence. J Clin Gastroenterol.
2014;48(9):752–64.
7. Norton C, Cody JD. Biofeedback and/or sphincter exercises for the treatment of faecal incon-
tinence in adults. Cochrane Database Syst Rev. 2012;7:CD002111.
8. Mei L, Patel K, Lehal N, Kern MK, Benjamin A, Sanvanson P, Shaker R. Fatigability of the
external anal sphincter muscles using a novel strength training resistance exercise device. Am
J Physiol Gastrointest Liver Physiol. 2021;320(4):G609–16.
9. Marques A, Stothers L, Macnab A. The status of pelvic floor muscle training for women. Can
Urol Assoc J. 2010;4(6):419–24.
10. Johnson VY. How the principles of exercise physiology influence pelvic floor muscle training.
J Wound Ostomy Continence Nurs. 2001;28(3):150–5.
11. Patel KML, Lehal N, Benjamin A, Sanvanson P, Kern M, Shaker R. External anal sphincter
strength training exercise using a novel continence muscles resistance exerciser device results
in improved anal sphincter contractility in patients with fecal incontinence. Gastroenterology.
2020;158(6):S-388.
Chapter 19
Diverticulitis

William Berger, Keely Browning, and W. Harley Sobin

Case 1 A 54-year-old man presents to the ER with LLQ pain of 6-h duration. He
has a temp. of 99, WBC is 14,000, and CRP is 30. A CT scan is performed that
reveals mild uncomplicated diverticulitis.
How would you manage this patient? Would you consider treating without
antibiotics?
Browning: Treatment considerations for uncomplicated diverticulitis include
antibiotics and patient disposition. Hospitalization should be considered for patients
who cannot tolerate oral intake, have a fever, or have excessive vomiting. Outpatient
management includes bowel rest, liquid diet, and oral antibiotics [1].
The AGA recommends consideration of liquid diet for patient comfort, but evi-
dence suggests it is not necessary, and if patients would like to advance their diet,
they should be allowed to. However, if they are unable to tolerate advancing the diet
in 3–5 days, they should be re-evaluated [2]. The two most common antibiotic regi-
mens are quinolone with metronidazole or amoxicillin-clavulanate for 4–10 days [1].
However, several studies have shown that antibiotics do not always accelerate
recovery, nor prevent complications or recurrence. Due to this, the AGA recom-
mends antibiotics be used selectively for patients with uncomplicated diverticulitis,
including those who have comorbidities, frailty, immunocompromised state,
refractory vomiting, CRP >140 mg/L, WBC >15 × 109 cells/L, or who have CT
findings of a fluid collection or longer segment of inflammation on CT [2]. Patients

W. Berger · W. H. Sobin (*)


Division of Gastroenterology and Hepatology, Medical College of Wisconsin,
Milwaukee, WI, USA
e-mail: [email protected]; [email protected]
K. Browning
Madison Medical Affiliates, Milwaukee, WI, USA
e-mail: [email protected]

© The Author(s), under exclusive license to Springer Nature 129


Switzerland AG 2023
W. H. Sobin et al. (eds.), Managing Complex Cases in Gastroenterology,
https://doi.org/10.1007/978-3-031-48949-5_19
130 W. Berger et al.

with uncomplicated diverticulitis without comorbidities do not necessarily require


antibiotics [1, 2].
Berger: I would only briefly consider treating without antibiotics. I know that the
new literature is supposed to show that there’s no difference, but when I have some-
thing that looks like a bacterial infection, I usually do start antibiotics and patients
seem to respond. When antibiotics aren’t started, the process seems to go off and on
for a longer period of time before it resolves itself. I have a preference for using
metronidazole, partly to avoid pseudomembranous colitis. Along with metronida-
zole, I use cipro. I am not a fan of Augmentin because of cases of pseudomembra-
nous colitis that I’ve encountered using that.
I give the patient a follow-up call after 5 days, and if they’re not better, I will give
them a longer course of antibiotics or occasionally switch to a different antibiotic.
If they’re still not improving, I usually choose to do some more imaging.
What if you’re dealing with a patient who has had repeated episodes of
diverticulitis treated with antibiotics as an outpatient, but hates taking the
antibiotics because of adverse symptoms, and comes in now with one more
typical recurrence (mild, uncomplicated). Would you consider managing with-
out antibiotics?
Sobin: I have had a few cases like this where a patient had repeated mild episodes
that we reflexively treated with antibiotics. After a couple of times, the patient
started pleading to skip antibiotics after another bout of diverticulitis, because of
severe intolerance. These were mild cases, and because the literature supports with-
holding antibiotics in cases like this, I managed the cases without antibiotics and the
patients did fine.
Berger: No. Actually, if this is a patient who is in his 40 s, 50 s, or 60 s, I would
give antibiotics and suggest elective surgery after resolution of the acute episode.
There is something focally wrong with that colon, and this person will be fighting
this problem for the rest of his life. They just need that sigmoid in a bucket. Now if
they’re 85, the risk of surgery will far exceed any potential benefit, but no, I would
rarely pass on giving antibiotics for recurrent/chronic diverticulitis to cool it down
so we can get on with the appropriate therapy.
Case 2 A 48-year-old man presents with an episode of sigmoid diverticulitis and is
started on outpatient metronidazole and ciprofloxacin. However, after a week of
antibiotics he still complains of pain and so a second 7-day course is administered.
After 2 weeks, he returns to his doctor’s office with ongoing LLQ pain and low-­
grade fevers to 100.5. Because his WBC is 18,000 and CRP is 75, a CT is ordered.
The CT shows ongoing inflammation in the sigmoid. How would you manage this?
Browning: Diverticulitis is the most common complication of diverticulosis.
Prior estimates suggested that diverticulitis occurred in 10–25% of patients, but
more recent studies suggest that the incidence is much lower, about 4% based on
population studies [1].
19 Diverticulitis 131

Smoldering diverticulitis is a complication of acute diverticulitis where patients


have continued abdominal pain with the presence of inflammation on imaging in
spite of treatment with antibiotics [1, 2]. Based on systemic signs of inflammation
like fever and elevated inflammatory markers, I would start IV antibiotics.
Berger: I would start IV antibiotics to cool things down. Now, we do IV antibiot-
ics at home, and I think that’s a very reasonable thing to try in an appropriate patient.
I would also call a surgeon because this patient is going to have trouble again.
As I get older and more experienced, my threshold for operating on these patients
becomes lower, but it is a balanced calculation, having to do with how old they are
and how many comorbid health conditions they have. This is weighed against how
severe, recurrent, and symptomatic the diverticulitis is.
However, on balance, I’m tipping a little bit more toward surgery. For one thing,
the complications using laparoscopic or robotic surgery are so much less, and the
recovery so much better than the old open surgical techniques.
Sobin: I agree. I would also bring the patient in for IV antibiotics and consult a
surgeon.
Case 3 A 45-year-old patient presents with his third episode of sigmoid diverticu-
litis in 12 months. He has been on a high-fiber diet and remains in good general
condition.
Would you recommend elective surgery at this point? If not, when?
Berger: After three episodes, you can be pretty sure they’re going to have more,
particularly at this young age. If you know you’re going to end up having surgery,
the earlier you have it done the better. You’re healthier, and your nutrition’s going to
be better. You don’t have to wait for complications of the disease. Everything just
works better with earlier surgery if you know you’re going to need it eventually. If
you wait until you’re older, you get less benefit out of it and are much more likely
to have complications.
Three is generally my magic number for recommending diverticulitis in the
younger patient, not the 84 years old.
There was a recent report in JAMA surgery [3], where they followed a group of
patients who had a sigmoid resection vs conservative therapy for recurrent compli-
cated, persistent, or painful diverticulitis. Mortality and complications of surgery
were virtually nil. The quality-of-life assessment after 2 years was much better in
those who had a sigmoid resection. It is interesting, though, that those with elective
sigmoid resection still had a 10–18% risk of recurrent diverticulitis.
Now, in the conservative management group, about 20% ended up needing sur-
gery anyway, but they had a much higher complication rate than the group that had
elective surgery.
Browning: Due to recent data showing that there were fewer deaths and colosto-
mies and better cost-effectiveness if surgical resection was delayed until the fourth
episode rather than the second, both the American Society of Colon and Rectal
Surgeons (ASCRS) and the AGA recommend a case-by-case basis for elective
132 W. Berger et al.

surgical resection. These factors include immune status, severity of diverticulitis,


patient preference, patient age, and operative risk [2]. For example, young patients
with diverticulitis tend to have more severe disease and fewer complications follow-
ing surgical resection and therefore may be considered for earlier surgical resection
[1]. In this case, I would refer to a surgeon given multiple episodes in a relatively
short time and relatively good health of the patient.
If you had a 30 years old who had their first episode of sigmoid diverticulitis,
would you recommend surgery?
Berger: Oh, no, because the first incident is a one-off, and the second is a coinci-
dence, but three is a pattern. By the time you hit three, you’ll know what you’re in for.
Sobin: I agree. I would hold off on surgery after the first episode. I might recom-
mend surgery after the second, if it is a severe attack, otherwise, probably after
the third.
How do you advise your patients on a way to avoid recurrent diverticulitis?
Any dietary or physical activity suggestions?
Browning: Diverticulosis is very common, and it is estimated that 2/3 of adults
will have diverticula by the ninth decade of life [1]. Most people are asymptomatic,
and thus, the prevalence of diverticulosis may be underestimated. The etiology of
diverticula is debated. Traditionally, diverticula were thought to form from low-fiber
diets leading to smaller caliber stools and constipation. This causes higher intralu-
minal pressures leading to herniation of colonic mucosa and submucosa [4].
Diverticula typically form between the taenia coli, which are the three bands of
longitudinal fibers of the colon. Diverticula form along the sites of penetration of
the vasa recta. Circular muscle layers thicken and taeniae shorten, causing the colon
to appear thickened with luminal narrowing [1]. Evidence for this can be seen when
people immigrate to regions with a Western diet, they develop a higher incidence of
diverticula [4]. Most diverticula in the western hemisphere are actually pseudodi-
verticula, as the herniation only reaches the submucosa. In Asian countries, diver-
ticula tend to be in the right colon and can be true diverticula involving all layers of
the colonic wall [1].
Other known risk factors include smoking, alcohol, vitamin D deficiency, high
red meat diet, physical inactivity, obesity, and genetic factors. Non-steroidal anti-­
inflammatory drugs (NSAIDs), steroids, hormone replacement therapy, and opiates
increase risk of diverticula and diverticulitis, while calcium channel blockers and
statins were found to be protective [1, 5].
Traditionally, it was recommended that patients with a history of diverticulitis
should avoid nuts and seeds. However, studies have shown that there is no increased
risk of diverticulitis and nuts and seeds may actually lower the risk. The AGA does
not recommend a particular diet [2]. Unfortunately, there is no strong evidence to
suggest medications to prevent recurrence of diverticulitis, and risk of diverticulitis
is largely due to genetic factors. Although dietary changes are debated, due to
absence of side effects, I would propose that this patient stick to a high-fiber diet or
consider daily Metamucil. I would counsel him to avoid NSAIDs if possible,
encourage regular exercise, and continue cessation from smoking.
19 Diverticulitis 133

Berger: I encourage everybody, whether they’ve had diverticulitis or not, over the
age of 30 to 35 to be taking two scoops of Metamucil every morning for the rest of
their lives.
That will reduce their chance of getting diverticulosis. It will also help reduce
their cholesterol and their chance of developing colon cancer or colitis. Yes, I’m a
big fan of fiber. Is it going to turn things back after you have somebody who already
has a significant diverticular disease? Not really, but it may minimize symptoms and
complications.
I think everybody ought to be on fiber anyway because of our diet. Do you
remember the story about Denis Burkitt of Burkitt’s lymphoma? He went to Uganda
and not only discovered Burkitt’s lymphoma, but he also noted that colon cancer,
diverticular disease, and colitis were rarely seen in Uganda, and he attributed all of
that to fiber. In Uganda, they eat huge amounts of very fibrous food, and he found
that they would have four to five soft bowel movements every day. In contrast, the
average Irish stool was round and hard and was produced once a day or even once
every other day, and he concluded that it was the fiber that made the difference in
their colonic health.
What do you think about NSAIDs increasing the risk of diverticulitis?
Berger: When I see NSAIDs causing an issue, it’s usually something in the ter-
minal ileum or proximal colon. I don’t usually see them causing a problem down-
stream, in the sigmoid. So, from my experience I’m not big on that one.
Sobin: In my experience, it has not been a significant contributing factor. In spite
of that, I recommend NSAID avoidance in patients with a history of diverticulitis.
Case 4 A 47-year-old man has had one episode of sigmoid diverticulitis. He is
treated, and all inflammatory markers resolve. However, months later, he has ongo-
ing pain in the same region. His WBC and CRP are normal. A repeat CT scan shows
no active inflammation.
Is this SUDD (symptomatic uncomplicated diverticular disease)? Is this a
real, distinct entity? If so, how would you treat it?
Browning: If abdominal pain is present in patients with diverticulosis, in the
absence of diverticulitis or clinical bleeding, they may have SUDD. Some think of
this as a continuum of irritable bowel syndrome (IBS), with benign physical exami-
nation and colonoscopy (other than diverticula), normal inflammatory laboratories,
presence of abdominal fullness symptoms, and symptoms that improve with defeca-
tion [1].
Several medications have been used for prevention of recurrent diverticulitis or
management of SUDD. Proposed regimens include mesalamine 800 mg twice daily
for 3 months to help prevent SUDD relapse, rifaximin 400 mg twice daily for
7 days, or combination therapy with mesalamine and rifaximin. Most studies
resulted in symptomatic improvement, but these studies were open-label. A
Cochrane meta-analysis in 2017 reviewed the use of mesalamine for prevention of
diverticulitis. They did not find evidence that mesalamine prevented recurrent diver-
ticulitis, although the trials were heterogeneous [6].
134 W. Berger et al.

Sobin: My bias is to manage these patients as if they have IBS, with visceral
hypersensitivity. In fact, some have suggested that diverticulitis may predispose to
IBS. There may be an entity of post-diverticulitis IBS, along the lines of post-­
infectious IBS. I would not be inclined to try mesalamine unless they were not
responding to IBS therapy (which could include rifaximin).
Case 5 A 47-year-old woman presents to the ER with severe LLQ pain, a fever of
101, and marked tenderness on examination. Her WBC is 21,000, and a CT scan
shows phlegmonous sigmoid diverticulitis with an adjacent abscess, 4 cm in size.
She has no past history of diverticulitis.
Are patients with diverticulitis more likely to have perforations, and other
complications, after their first episode or subsequent ones?
Browning: Complicated diverticulitis is defined by abscess, fistula, obstruction,
or free perforation. Complicated diverticulitis should be suspected when there is
persistent pain, fever, and leukocytosis despite IV antibiotics. Complications are
more likely to occur with the first episode than subsequent episodes [2]. The
Hinchey classification was developed to stratify patient management [1]. Stage I is
confined pericolic abscess, stage II is distant abscess, stage III is generalized perito-
nitis due to rupture of pericolic/pelvic abscess, and stage IV is fecal peritonitis due
to free perforation of diverticulum.
Complicated diverticulitis is treated with hospitalization, IV antibiotics, surgical
consultation, and co-management. When patients develop worsening sepsis or have
recurrent abscess, it means that they have failed antibiotic treatment and should be
considered for drainage [7]. In general, if an abscess is greater than 3 cm in size,
percutaneous drainage is recommended in addition to IV antibiotics [7].
Berger: My bias is that I tend to see complicated diverticulitis in elderly patients
who have had problems before. In my experience, the first episode may not be
the worst.
On a per-episode basis, first may be most likely to be complicated, but on a per-­
patient basis, I worry most about the elderly patients with multiple prior episodes.
Tick, tick, tick.
Sobin: I believe the literature that says perforation is most likely with the first
episode. I tell patients who have had multiple admissions for diverticulitis that I am
less worried about them needing emergency surgery for perforated diverticulitis.
Case 6 A 46-year-old man with a history of prior diverticulitis comes in for screen-
ing colonoscopy. He is found to have moderate inflammatory change limited to an
8-cm length of sigmoid with moderate diverticulosis, where the inflammation
seemed to spare the diverticula themselves. Biopsies did not suggest chronic colitis.
Do you think this may be SCAD—segmental colitis associated with
diverticulosis?
Browning: Segmental colitis associated with diverticulosis (SCAD) is an inflam-
matory process affecting regions of colon that have diverticula [8]; SCAD can
mimic the presentation of inflammatory bowel disease (IBD), with clinical features
including rectal bleeding, chronic diarrhea, cramping abdominal pain, and fever.
19 Diverticulitis 135

Systemic symptoms like fever, weight loss, or leukocytosis are rare. It is present in
1.5% of colonoscopies, with the majority of patients being male, with a mean age
of 63 [1].
Pathogenesis of this condition is thought to be the effect of excessive mucosa
from bowel shortening in the setting of diverticulosis, leading to mucosal prolapse
and inflammation associated with this [6]. However, when examined microscopi-
cally, there can be chronic crypt changes such as crypt distortion, that mimic
changes seen in ulcerative colitis. It has also been suggested that there may be bac-
terial stasis in diverticula leading to increased bacterial mucinase activity and
mucosal injury.
Lastly, given the overall older age of presentation in SCAD, there is also a
hypothesis of relative colon ischemia, leading to segmental mucosal inflammation.
Interestingly, despite chronic inflammation, there is not a higher risk of diverticulitis
or colon cancer [1].
There are four subtypes of SCAD: crescentic fold (A), mild-moderate UC-like
(B), Crohn’s disease-like (C), and severe UC-like (D). These are delineated at time
of endoscopy, but importantly all have sparing of the diverticula and regions of
colon without diverticula, which separates this disease from other conditions such
as IBD [6]. Type A is most likely to present with chronic diarrhea, has no crypt
distortion on histologic appearance, and appears as red round lesions 0.5–1.5 cm at
top of mucosal folds. Types B and D have loss of vascular pattern with severe ulcer-
ation in type D and have crypt distortion on biopsy with goblet cell depletion. Type
C mucosa has isolated aphthous ulcers and evidence of transmucosal inflammation
on biopsy. Types C and D are more likely to have rectal bleeding. Types B and D
tend to have a high risk of relapse [6].
Berger: Segmental colitis associated with diverticulosis? I have seen cases where
a patient has a single diverticulum exuding pus, and even though some of these folks
were asymptomatic, but others were mildly symptomatic, in these cases, there’s that
one tic and it’s red and there’s pus coming out of it, and I usually treat them with
antibiotics if they have symptoms to see if they get better.
Then, on occasion, it’s 5 years later and we’re scoping them again. They have the
same darn thing with the same pus coming out of the same tic, and they’ve never had
any symptoms, and in cases like that it’s hard to justify treating that, unless of
course you had somebody who was about to undergo chemotherapy.
And sometimes, I’ll be working with the fellows, and we’ll be coming through a
really tight sigmoid with a lot of tics and very thick muscular haustra, and there will
be these red splotches. Then, the fellow will say “Oh, that’s colitis,” and sometimes,
I even say, “Okay, go ahead and take a biopsy of it,” and it comes back with nothing,
and that’s because it’s prolapsed, and you usually see the prolapse in the setting of
a sigmoid that’s also full of tics, and presumably there’s a high-pressure zone that’s
causing the tics and the hypertrophy and the prolapse. Just because it’s red doesn’t
mean it’s inflamed. Mucosa isn’t skin.
Sobin: I’ve primarily seen these mucosal changes on routine surveillance colo-
noscopy, and I believe that it’s a residual change from prior diverticulitis. In a few
cases, I have seen this pattern that looks like IBD, but there’s sparing of the
136 W. Berger et al.

diverticula themselves, and the inflammation is limited to a short segment of sig-


moid diverticula. Generally, these patients were asymptomatic, and before the des-
ignation of SCAD came about, I thought this was mild ischemia or mild Crohn’s
with just an unusual sparing of the diverticula themselves.
Browning: For patients who have more severe SCAD, with significant symp-
toms, treatment regimens have been based on case series and extrapolation of expe-
rience from IBD.
One first-line regimen starts with ciprofloxacin 500 mg BID and metronidazole
400 mg TID for 7 days, followed by mesalamine 2.4–3.2 g/day for 4 weeks and then
long-term maintenance dosing of 1.6 g/day of mesalamine. For patients not respond-
ing, second-line therapy that uses beclomethasone dipropionate (BDP) with VSL
for 4 weeks with tapering dose for another 4 weeks has been tried, followed by
maintenance mesalamine. Third-line therapy with high-dose prednisolone has been
offered, with surgery for refractory cases [4].
However, in those cases where second- or third-line therapies have been neces-
sary, one must consider a misdiagnosis, and the patient has IBD instead. In fact,
some patients later develop IBD at the site of anastomosis when resection was
needed for SCAD [4].
Case 7 A 70-year-old man presents with his fifth episode of sigmoid diverticulitis.
He has tried everything and has failed all conservative measures and is ready to
have elective surgery. On colonoscopy and CT scan, it is clear that he has diffuse
diverticulosis involving all portions of the colon (except the rectum). However, the
documented episodes of diverticulitis have only involved the sigmoid.
With this history do you think it is sufficient to recommend a sigmoid colec-
tomy, or do you think the patient should have a subtotal colectomy?
Berger: I would recommend just removing the sigmoid colon. I look at the colon
as two different organs, three if you include the rectum. The proximal and distal
colons have different motility, different pressures, different blood supply, and nerve
supply. The proximal colon is supplied by the superior mesenteric artery and is
innervated by the vagus. It is more-thin walled, and its contractions are segmental
contractions designed to mix and churn, to increase exposure of the bolus to the wall
to help extract salt and water. The distal colon is designed to store and expel, and its
motility is propulsive with much higher pressure, and it’s innervated by the sacral
outflow and the vascular system is the inferior mesenteric artery, and so, they’re
actually two different organs and I don’t really see diverticulitis in the proximal
colon. I will see more bleeding from proximal colon diverticula, probably because
the superior mesenteric artery has more perfusion pressure than the inferior mesen-
teric artery.
Really, they are two distinct organs and if you have pandiverticulosis, the more
likely to bleed is the proximal, but the more likely to get diverticulitis is far and
away the distal, either the descending or the sigmoid, but the lower down you go, the
more likely it is to occur. So, I’m comfortable having him remove the sigmoid, or
the sigmoid and descending colon, leaving the rest intact.
19 Diverticulitis 137

References

1. Feldman M, et al. Sleisenger and Fordtran's gastrointestinal and liver disease. 11th ed. Elsevier -
OHCE; 2020.
2. Peery AF, Strate SA, LL. AGA clinical practice update on medical Management of Colonic
Diverticulitis: expert review. Gastroenterology. 2021;160:906–11.
3. Santos A, Mentula P, Pinta T, Ismail S, Rautio T, Juusela R, Lähdesmäki A, Scheinin T, Sallinen
V. Quality-of-life and recurrence outcomes following laparoscopic elective sigmoid resection
vs conservative treatment following diverticulitis: Prespecified 2-year analysis of the LASER
randomized clinical trial. JAMA Surg. 2023;158(6):593–601.
4. Sheth AA, Longo A, Floch MH. Diverticular disease and diverticulitis. Am J Gastroenterol.
2008;103:1550–6.
5. Strate LL, Modi R, Cohen E, Spiegel BMR. Diverticular disease as a chronic illness: evolving
epidemiologic and clinical insights. Am J Gastroenterol. 2012;107:1486–93.
6. Carter F, Alsayb M, Marshall JK, Yuan Y. Mesalamine (5-ASA) for the prevention of recurrent
diverticulitis. Cochrane Database Syst Rev. 2017;10:CD009839.
7. Hall J, Hardiman K, Lee S, Lightner A, Stocchi L, Paquette IM, Steele SR, Feingold DL. The
American Society of Colon and Rectal Surgeons clinical practice guidelines for the treatment
of left-sided colonic diverticulitis. Dis Colon Rectum. 2020;63:728–47.
8. Schembri J, Bonello J, Chrisodoulou DK, Katsanos KH, Ellul P. Segmental colitis associated
with diverticulosis: is it the coexistence of colon diverticulosis and inflammatory bowel dis-
ease? Ann Gastroenterol. 2017;30:257–61.
Chapter 20
Pancreatic Cysts and Recurrent
Pancreatitis

Phillip Chisholm

Case 1 A 49-year-old man has a CT scan for LLQ pain. The CT reveals mild sig-
moid diverticulitis and an incidental 3-cm cyst in the tail of the pancreas. The pan-
creas otherwise appears normal. He has no history of pancreatic disease, does not
smoke, and reports only occasional alcohol use. Once he completes antibiotic treat-
ment for his diverticulitis, he sees you about the pancreatic cyst. His CBC, LFTs,
and lipase were all normal..
Is a 3-cm cyst large enough to warrant a workup, and if so, how would you
proceed?
When a patient like this is referred to us, we always review the history trying to
tease out any symptoms that may have been missed and any significant social or
medication history. We want to know, was there an episode of pain earlier that might
signal unrecognized pancreatitis? In a young patient like this, we want to make sure
there wasn’t more significant alcohol intake. Is there a medication that might have
caused pancreatitis, or a family history of pancreatic disease? We try to identify any
old imaging of the pancreas to compare with the latest study. The differential for a
cyst like this includes non-communicating pancreatic cysts that might be mucinous
or serous, pancreatic pseudocysts, and branch-chain IPMNs.
After reviewing the history, physical and any laboratories previously performed
we will generally order a dedicated pancreatic radiologic examination, either a dedi-
cated pancreatic CT (which includes fine cuts, non-contrast, and timed contrast
administration) or an MRCP. In our institution, we generally prefer the CT because
a number of our patients have claustrophobia or can’t hold still long enough to get
a high-quality MRI. Another advantage of CT is that it can pick up fine pancreatic
calcifications. Pancreatic calcifications won’t be seen on MRI. On the other hand, an
MRCP has the advantage of lack of radiation, which is important in patients who
may have serial surveillance examinations. In addition, the MRCP is better at pick-
ing up IPMNs and is better at picking up subtle mural nodularity.

P. Chisholm (*)
Department of Medicine, GI/Hep Division, Medical College of Wisconsin,
Milwaukee, WI, USA
e-mail: [email protected]

© The Author(s), under exclusive license to Springer Nature 139


Switzerland AG 2023
W. H. Sobin et al. (eds.), Managing Complex Cases in Gastroenterology,
https://doi.org/10.1007/978-3-031-48949-5_20
140 P. Chisholm

Another laboratory we occasionally order is a serum CA 19–9. A high CA 19–9


may be found in high-grade dysplasia or cancer of the pancreas. However, we are
selective with which patients we order the test, since an elevated CA 19–9 is not
specific for pancreatic malignancy (elevated CA19–9 may be seen in cases of biliary
obstruction, pancreatitis, liver cysts, and hepatitis). Also, some pancreatic cancers
don’t produce CA 19–9 so a normal result doesn’t rule it out.
In most patients, we generally do an EUS and FNA on all cystic lesions larger
than 2–3 cm in size. With the FNA, we want to see if the fluid is mucinous, and we
order a CEA and glucose. A mucinous lesion is much more likely to be neoplastic,
while a serous lesion or pseudocyst is very unlikely to be neoplastic. An elevated
cyst fluid amylase suggests that the cyst is in communication with the pancreas—
either a pseudocyst or a branch-chain IPMN. An elevated CEA suggests that the
lesion is mucinous, either a dysplastic lesion or neoplasm. The cutoff for abnormal
CEA is >192 although the specificity of elevated CEA is much higher when we use
a cutoff of 1000. We also send off cytology, although the utility of cytology on these
aspirates is low, there are many false-negative results.
When we aspirate these lesions, we draw off as much fluid as possible—until the
cyst collapses. We make sure we have enough fluid for the CEA, glucose, amylase,
and mucin stain and then send off whatever is left for the cytology. Hopefully, a
larger volume of fluid may increase the sensitivity of the cytology examination. We
hope to draw off at least 3–5 cc, and part of the reason we don’t investigate smaller
cysts is that it is difficult to draw off sufficient fluid.
With a mucinous cyst greater than 3 cm, we generally call in a multidisciplinary
pancreatic team. Assuming the patient is a surgical candidate, most of these patients
should have surgical resection since the risk of cancer in a mucinous cyst is rela-
tively high. If the fluid is serous or suggestive of a pancreatitis related pseudocyst,
we assume this is a benign cyst that does not require any follow-up, and repeat
radiologic examinations are not indicated.
Case 2 A 42-year-old woman presents to her primary care physician complaining
of gradually increasing pain and fullness over the past 10 days. Her physician
orders an ultrasound followed by a CT scan that shows cholelithiasis and a 7-cm
cyst off the body of the pancreas. The pancreas otherwise appears normal. The com-
mon bile duct is not dilated. Her liver enzymes and lipase are normal. Five weeks
earlier, she had an episode of severe pain that woke her in the middle of the night
but resolved by morning.
She has no history of alcohol use, prior pancreatitis, or family history of pancre-
atic disease. The distinct possibility of an episode of gallstone pancreatitis 5 weeks
earlier is raised.
Is it necessary to do an EUS to evaluate this cyst?
In this case, we suspect the cyst could be a pancreatic pseudocyst. In considering
other etiologies, we want to make sure there wasn’t a prior history of weight loss,
abdominal pain, or any new-onset diabetes, all of which could suggest neoplasm or
chronic pancreatitis. If the symptoms continue to improve and the cyst gets smaller,
we would simply observe the patient and repeat imaging studies in 1–2 months.
20 Pancreatic Cysts and Recurrent Pancreatitis 141

Assuming any pancreatitis has resolved, you would want to order a cholecystec-
tomy to prevent any repeated gallstone pancreatitis.
If the cyst does not get smaller, and symptoms don’t abate, it would be reason-
able to do an EUS and FNA. A cyst aspirate fluid with an amylase greater than 1000
and CEA < 192 would be compatible with a pancreatic pseudocyst.
If the cyst did not decrease in size, draining the cyst, perhaps with an Axios stent,
would be called for.
Case 3 A 48-year-old man is seen for abdominal pain. He has a history of cigarette
smoking of 28-year duration and has 2–3 drinks a day, 4–5 on weekends. A CT scan
reveals a dilated pancreatic duct. MRCP is performed, showing cystic dilation of
the pancreatic duct without proximal stricture, suggesting a main duct IPMN in the
body of the pancreas.
How do you manage a patient with pancreatic IPMNs?
IPMNs are mucous-producing tumors of the pancreatic duct that result in cystic
dilation of either the main pancreatic duct, a side branch, or both. There is an impor-
tant distinction between main duct IPMNs, which have a much higher malignant
potential than branch-chain IPMNs. Diagnosis of IPMNs can be challenging, and
radiologic examinations may confuse main duct IPMNs and chronic pancreatitis,
both of which can cause cystic dilation of the pancreatic duct. MRCP is better than
CT at picking up IPMNs but can miss the calcifications seen in chronic pancreatitis,
which are well seen on CT. An EUS is best at identifying any solid component to
the cysts or mural nodules in the IPMNs, and you can often see mucous extruding
from the ampulla, the so-called fish-eye appearance. However, calcifications can
interfere with EUS evaluation. Therefore, we frequently employ multiple imaging
modalities if thee is clinical uncertainty. If the diagnosis of main duct IPMN is con-
firmed, we generally refer the patient to a multidisciplinary pancreatic team for
evaluation for surgery because of the high malignant potential.
Branch-chain IPMNs, on the other hand, have a much lower malignant potential.
These lesions are being identified more frequently in our senior population. In stud-
ies of patients who had an MRI for non-pancreatic problems, the frequency of pan-
creatic cystic lesions in patients over age 70 is 40%. In general, we don’t investigate
these lesions unless they are at least 2 cm in size, in which case we do our standard
EUS with FNA with mucous stain, CEA, glucose, and amylase. Surgery and amy-
lase. Surgery would only be recommended if there were high-risk findings on
EUS or FNA.

“Idiopathic” Recurrent Pancreatitis

Case 4 A 53-year-old man is admitted for his second episode of pancreatitis. His
first episode was 3 months earlier. Prior to the first episode, he was a social drinker
on weekends only. Since that episode, he has had no alcohol. During each episode,
his liver enzymes were normal. His lipase was 1100 on admission with the first epi-
142 P. Chisholm

sode and 700 for the second. His triglycerides were 350 during the first episode with
normal calcium. An US shows a normal gallbladder. There is no family h/o pancre-
atic disease, and he is on no meds that would cause pancreatitis.
How do you evaluate a patient with recurrent idiopathic pancreatitis?
Remember, the vast majority of cases of pancreatitis, 70%, are due to gallstones
(40%) or alcohol (30%). So, it’s important to investigate whether there actually is
significant alcohol use in the history, and if not, gallstones are going to be the most
common etiology. Of course, in a 53 years old presenting with idiopathic pancreati-
tis we always want to consider, and rule out, a pancreatic neoplasm, particularly if
the patient has a history of weight loss or new-onset diabetes.
Ultrasound is our first test looking for cholelithiasis. After that, we do our dedi-
cated pancreatic imaging. In this instance, MRCP might be more sensitive for evalu-
ating occult cholelithiasis and pancreatic ductal abnormalities. If these studies are
unrevealing, we would do an EUS to rule out an occult neoplasm and look for evi-
dence of microlithiasis, and sludge in the gallbladder (with the proviso that if the
patient has been NPO for a number of days’ sludge might form as a result of fasting
and be a red herring).
If, after these investigations, an etiology is not discovered, we would generally
recommend a cholecystectomy anyway, because many of these cases of recurrent
“idiopathic” pancreatitis are due to occult microlithiasis.
What about a genetic etiology for recurrent pancreatitis?
This is a 53-year-old patient, and genetic pancreatitis usually presents earlier in
life. If the patient was a 25 years old with recurrent pancreatitis, we would order
genetic tests early on. After discussing the risks and benefits of genetic testing, we
would order a genetic panel that includes CFTR, SPINK1, PRSS1, and CTRC. Most
patients with genetic pancreatitis don’t have a family history. If there is a family
history of pancreatitis, the more common genetic abnormality is PRSS, which is
autosomal dominant, while the other genetic diseases are autosomal recessive.
This patient has a negative MRCP and EUS, and cholecystectomy is performed.
He presents 2 months later with a third episode of pancreatitis.
How would you proceed now?
In the patient with negative imaging who has had a cholecystectomy, we would
order a secretin-stimulated MRCP to look for subtle ductal abnormalities. Following
that, we consider an ERCP for the possibility of sphincter of Oddi stenosis or pan-
creas divisum, which we discuss further in the next two cases.
Case 5 A 57-year-old woman has gone to the ER four times for acute pancreatitis
over the past 8 months. There is no significant alcohol use. She had gallstones, and
a cholecystectomy was performed after the first episode. In spite of that, she pres-
ents with two more episodes of pancreatitis.
After the third episode, she had an MRCP that suggested a pancreas divisum.
There are no medications that should produce pancreatitis, and there is no sug-
gestion of autoimmune pancreatitis.
20 Pancreatic Cysts and Recurrent Pancreatitis 143

How would you manage this patient?


MRCP is fairly accurate at picking up pancreas divisum, but the finding of pan-
creas divisum does not necessarily make it the culprit, the cause of the recurrent
pancreatitis. However, in this case, other etiologies appear unlikely—the patient had
a cholecystectomy, and there is no suggestion of autoimmune pancreatitis nor med-
ication-induced pancreatitis. Because of her age, we would first offer her an EUS to
rule out any subtle suggestions of malignancy. Assuming that is negative, we would
recommend proceeding to ERCP. The marked frequency of the episodes—four epi-
sodes in 8 months, dictates the more aggressive management. We would suggest an
ERCP with minor ampulla sphincterotomy. This would likely end the cycle of
recurrent pancreatitis.
Case 6 A 38-year-old woman presents with recurrent pancreatitis. One year ear-
lier, she had suspected biliary pancreatitis treated with cholecystectomy. She has
occasional alcohol and does not smoke. She presents in the ER with acute abdomi-
nal pain. Examination revealed marked abdominal tenderness, decreased bowel
sounds, and increased tympany. She had a lipase of 600, white cell count of 10,000,
AST of 220, ALT of 350, alkaline phosphatase of 240, and bilirubin of 1.4. Ultrasound
revealed a dilated CBD of 11 mm.
How would you manage this patient?
If available, you’d like to know the post-cholecystectomy ductal diameter as a
baseline. In this case, you’re questioning a possible sphincter of Oddi dysfunction
(SOD), most likely type 1, vs a retained stone. We would order a secretin-stimulated
MRCP. Secretin increases pancreatic secretions and relaxes the pancreatic sphinc-
ter. If there is a problem with the pancreatic sphincter or sphincter of Oddi, the
pancreatic duct dilates. Secretin-stimulated MRCP could help rule out choledocho-
lithiasis and provide evidence for any sphincter dysfunction. We no longer do bili-
ary manometry to evaluate for SOD because of the high risk of post-ERCP
pancreatitis, without much benefit. While in the past we did more isolated pancre-
atic duct sphincterotomies, it has been found that a routine biliary sphincterotomy
will benefit most patients whether the defect is in the pancreatic sphincter or biliary
sphincter.
Case 7 A 47-year-old man is referred with a history of intermittent abdominal pain
accompanied by episodes of elevated lipase. There is no significant history of alco-
hol use, smoking, or family history of pancreatitis. A CT scan reveals enlargement
of the head of the pancreas, a sausage-shaped pancreas, with atrophy of the pancre-
atic tail, and some surrounding lymph nodes that are mildly enlarged. The diagnosis
of autoimmune pancreatitis is questioned. You send off an IgG4, and it comes back
negative.
Do you think this is autoimmune pancreatitis?
If the IgG4 had returned positive, the treatment would be clear-cut. This would
almost certainly be type 1 autoimmune pancreatitis, and high-dose steroids would
144 P. Chisholm

be indicated. In this case, the IgG4 returns negative and management is more com-
plicated. We do look for other diseases that would suggest IgG4-associated disorder
like cholangiopathy, parotid disease, or retroperitoneal fibrosis. If those are absent,
we have two choices. One is an empiric high-dose steroid trial with repeat imaging
in 1–2 months. Alternatively, we could do an EUS with FNA looking for histologic
diagnosis of type 2 autoimmune pancreatitis and to rule out occult neoplasm.
With a large head of the pancreas and enlarged lymph nodes, neoplasm has to be
considered in the differential. While pancreatic adenocarcinoma is uncommon in
someone this young, these X-ray findings could be seen with pancreatic lymphoma.
The fact that pancreatic lymphoma is somewhat steroid-responsive can also compli-
cate diagnosis in this case.
If the decision was made to proceed with an FNA and the results were non-­
diagnostic, we would probably do the steroid trial. If there was no response to ste-
roids, we would repeat EUS to confirm the absence of neoplasm, but in the patient
with ongoing symptoms, elevated lipase, in whom we don’t have a diagnosis, it is
extremely tricky but we would probably have to refer the patient for surgery.
Chapter 21
Chronic Pancreatitis

Srivats Madhavan

Case 1 A 45-year-old man who is a chronic alcoholic is referred to your office with
a history of chronic abdominal pain and weight loss. He has a history of three
admissions to the hospital for pancreatitis over the past 6 months. He was drinking
heavily before the first admission, cut back drinking, and then after a second epi-
sode quit entirely. He had no alcohol at all for the 2 months leading up to his third
admission for pancreatitis. On each admission, he has characteristic pain, an ele-
vated lipase, and he has CT scans that show mild bile duct dilation and calcifica-
tions in the pancreas. Although his pain subsided after his first and second admission,
it has become chronic since his third admission. He has cut back on meals because
eating seems to worsen his pain. In addition, he is having increased frequency
bowel movements that appear foamy, with oil droplets. His examination is remark-
able for abdominal tenderness and some muscle wasting. His Hbg is 12, WBC 4.8,
lipase 180 (normal—60), ALT 50, AST 75, Alk phos 110, Bili 1.8, and glu 108
(fasting).
When you see a patient like this with a past history of significant alcohol
consumption do you consider other etiologies for exacerbation of pancreatitis
or just assume that it is alcohol?
I think it’s a good idea to consider other etiologies for pancreatitis especially
since he has had another episode in spite of quitting alcohol. At the very least, I
would want to rule out gallstone pancreatitis and make sure the patient is not on
medications that might cause pancreatitis. We want to review whether there is any
familial pancreatitis and review the scans to see if the patient has developed a pan-
creatic duct problem that might be causing recurrent pancreatitis.

S. Madhavan (*)
Department of Medicine, Division of Gastroenterology and Hepatology, Medical College of
Wisconsin, Milwaukee, WI, USA
e-mail: [email protected]

© The Author(s), under exclusive license to Springer Nature 145


Switzerland AG 2023
W. H. Sobin et al. (eds.), Managing Complex Cases in Gastroenterology,
https://doi.org/10.1007/978-3-031-48949-5_21
146 S. Madhavan

Are there other diagnostic tests you might order?


Occasionally, I will check IgG4 levels, and if there’s any family history of pan-
creatitis we do a genetic panel, although, in general, these are low yield. We may
also do a secretin-stimulated MRCP.
How does the secretin-stimulated MRI help you?
The secretin stimulation causes an increase in pancreatic ductal secretion, which
helps elucidate the morphology of the pancreatic duct and helps to demonstrate any
strictures on MRI. There is another scan, the quantitative secretin-stimulated MRCP,
which can be used to evaluate pancreatic function by measuring the amount of
bicarbonate secreted after secretin stimulation. This is a noninvasive way of demon-
strating pancreatic insufficiency.
Would you do an EUS in this patient?
The main reason for doing EUS would be to diagnose pancreatic cancer. At
forty-five, he is young to have pancreatic cancer although the risk is higher because
of his history of chronic pancreatitis, but in general we find the yield of EUS in
acute recurrent and chronic pancreatitis to be fairly low, and it doesn’t add much to
what we get from CT and MRI.
How often do you see recurrent pancreatitis/chronic pancreatitis in someone
who has quit drinking?
Yes, we do see this occasionally, just a few cases each year. Most patients do
fairly well if they stop drinking.
How do you manage the pain in chronic pancreatitis?
The patients who have chronic pain are referred to pain management. In terms of
a celiac axis block, I don’t advocate for it in chronic pancreatitis because there is
always recurrence of pain down the road. At best, pain relief may last for three to
6 months. However, I do advocate for celiac axis neurolysis for patients with chronic
pain from pancreatic cancer.
In patients with chronic pain from chronic pancreatitis what do you think
about managing pancreatic ductal strictures or stones with endoscopic
therapy?
The overall data suggest that long-term pain relief is better with surgical drainage
of the duct rather than endoscopic drainage. There are only a few, select patients, in
whom we recommend endoscopic therapy.
If I’m going to consider a patient for endoscopic therapy for a pancreatic stric-
ture, the stricture has to be unifocal, or else a couple of strictures close together.
They have to be located in the proximal pancreatic duct, near the ampulla, and the
duct needs to be large enough that drainage makes sense.
Treating pancreatic stones is more complicated than treating choledocholithiasis.
The stones in CP mold to the pancreatic duct and its branches, so it’s not something
that will slide out of the duct easily. These stones have a lot of extensions growing
into the side branches of the pancreatic duct and anchoring them in place like den-
dritic processes reaching into the side branches, which is why they are difficult to
remove. If a stone is more than 5–6 mm in size, it will require extracorporeal shock
wave lithotripsy (ESWL) to break it up before we can remove it. The idea of using
a spyglass scope as used in cholangioscopy is not feasible for pancreatoscopy.
21 Chronic Pancreatitis 147

Directed lithotripsy using this approach is usually not possible because the spyglass
is about ten French in diameter, making it difficult to introduce into the pancreatic
duct without causing a lot of trauma, since the proximal duct is only 2–3 mm in
diameter. If there is a stone located beyond the neck of the pancreas, it is not feasible
to remove it unless you are doing ESWL. Note that with ESWL you also have to do
a pancreatic duct sphincterotomy to get those fragments out.
Our preference, if the patient quits drinking and the surgical anatomy will allow,
is to send the patient to surgery. These patients are generally younger, in their forties
and fifties, and often are candidates to undergo surgery. So, I will refer them to a
surgeon that I trust, but some patients may not be good surgical candidates because
they have portal vein thrombosis, or have portal hypertension from alcoholic cir-
rhosis. In these patients, we may offer endoscopic treatment and this may involve
ESWL, but for lithotripsy to be successful the pancreatic stone has to be large
enough to be visualized on X-ray, usually at least 5 mm in size.
We have seen good results from the surgery. The choice of operation comes
down to the individual surgeon’s preference and the patient’s anatomy. The two that
are generally offered are the Frey and Puestow procedures. It really comes down to
how much disease there is in the head of the pancreas, and the Puestow is not as
beneficial if you are trying to drain areas in the head of the pancreas. Total pancre-
atectomy with islet cell transplant is available in a few centers across the nation, but
not at our institution. We will occasionally refer patients with chronic pain for this
procedure.
Here the patient has weight loss and it sounds like he may have steatorrhea.
How do you evaluate patients for pancreatic insufficiency?
We use stool elastase and spot fecal fat. If the stool fat is positive and the elastase
is <200 that is sufficient to diagnose pancreatic insufficiency. We do not ask patients
to collect 48-h stool fats. If the spot stool fat is negative but the elastase is low and
the patient has typical symptoms, I will give a trial of pancreatic enzyme replace-
ment. We usually give Creon because most insurances will cover it. I usually don’t
put them on a low-fat diet, since they are already struggling with weight loss. I
generally give 72,000 lipase units of Creon (two 36 K tablets) with each meal. If
there is a high-fat snack, then I recommend one Creon tab with the snack.
Do you do surveillance for pancreatic cancer in these patients?
They are at some increased risk because of chronic pancreatitis. However, they
tend to get scans quite frequently because of new symptoms or to monitor the course
of the pancreatitis. I have a low threshold for ordering scans if they have new symp-
toms, but I don’t put them in a specific pancreatic cancer screening program.
What are the biggest mistakes you see primary care doctors and some gas-
troenterologists in the community making?
I see doctors putting patients on Creon for unnecessary reasons. Say, they’ve
had one episode of acute pancreatitis, or they’ve had an episode of gallstone pan-
creatitis, and, without any evidence of pancreatic insufficiency, they’re sent home
on Creon. For those who actually have pancreatic insufficiency, there is a ten-
dency to send them home on too low a dose of Creon, so under-dosing is another
problem.
148 S. Madhavan

We used to give patients with painful chronic pancreatitis Viokase, an


uncoated pancreatic enzyme, to try to suppress CCK stimulation of the pan-
creas. Is that something you find efficacious to help relieve chronic pain?
When I’ve tried it, I have not found it to be helpful, and the latest studies bear that
out, it is no longer recommended.
Case 2 A 43-year-old man presents with abdominal pain and weight loss. He has
one or two beers a weekend but was never a heavy drinker. He has no history of
acute pancreatitis. However, his abdominal CT scan shows pancreatic calcifica-
tions suggestive of chronic pancreatitis, and his stool elastase is decreased at 55.
When you encounter a patient with apparent chronic pancreatitis who is not
a drinker what differential diagnosis comes to mind?
We occasionally see a patient who presents with pancreatic calcifications and
signs of chronic pancreatitis without any past history of acute pancreatitis. First, we
want to know if he is a never-drinker or if he used to drink heavily and then quit.
Second, it’s important to get a good family history and find out whether there is a
family history of pancreatitis. You want to know the patient’s ethnicity and where
they grew up. In South Asia, where I went to med school, we saw a number of
patients with tropical pancreatitis, and some of these patients have SPINK muta-
tions. With tropical pancreatitis, there is a wide spectrum of disease severity but a
number of patients develop chronic pancreatitis with pancreatic insufficiency and
also insulin-dependent diabetes. They also have an increased risk of pancre-
atic cancer.
In evaluating the patient with chronic pancreatitis with no history of significant
alcohol use, no family history, and born in the USA with no travel history, I get a
right upper quadrant ultrasound to rule out stones, and I order an IgG4, a lipid panel,
and a genetic panel. You want to know about medication and smoking history and
alcohol. Of course, gallbladder disease does not tend to lead to chronic pancreatitis
although it might cause acute recurrent pancreatitis.
Autoimmune pancreatitis can present as chronic pancreatitis. Note that you can
have autoimmune pancreatitis with a negative IgG4 and you can be IgG4 positive
and not have autoimmune pancreatitis (SLE, IBD, etc.).
Cystic fibrosis is often associated with chronic pancreatitis, and the CFTR gene
mutation can contribute to pancreatitis in patients without cystic fibrosis.
While we used to do EUS in idiopathic and chronic idiopathic pancreatitis, we
have shied away from it because it has a low yield. Of the past forty patients, I did
an EUS on with idiopathic pancreatitis and the only positive was one patient with
microlithiasis.
Does the finding of pancreatic calcifications make chronic pancreatitis
almost a certainty?
Pancreatic calcifications can occasionally be seen in patients with a history of
abdominal trauma and occasionally splenic artery calcification may be misinter-
preted as chronic pancreatitis. So, no, the reading of calcifications over the region of
the pancreas is not pathognomonic for chronic pancreatitis.
21 Chronic Pancreatitis 149

Do you see much recurring acute pancreatitis or chronic pancreatitis from


medications?
We are seeing a number of patients with checkpoint inhibitor-induced pancreati-
tis that acts in a similar fashion to autoimmune pancreatitis. Most of these patients
respond to steroids. After 2–3 months of steroid therapy, we try to taper the steroids
and transition to azathioprine or mycophenolate.
We have also seen patients with recurrent pancreatitis from ACE inhibitors and
ARBS, also from HIV drugs.
How do you address patients with chronic pancreatitis who are
asymptomatic?
We have some patients referred to us because in the evaluation of alcoholic cir-
rhosis they are found to have X-ray findings of chronic pancreatitis. They are often
asymptomatic and have no signs of pancreatic insufficiency. Of course, we encour-
age alcohol abstinence and want to know about any future symptoms, but in the
patient who is asymptomatic we generally don’t recommend any intervention.
Chapter 22
GI Oncology

Ben George

Case 1 A 67-year-old woman presents with iron deficiency anemia and has a colo-
noscopy that reveals adenocarcinoma of the ascending colon. Following right hemi-
colectomy, it is determined that she has stage 3 colon cancer.
Is it true that most stage 3 and some stage 2 colon cancers should receive
adjuvant chemotherapy and that the adjuvant therapy of choice is usually
oxaliplatin and a fluoropyrimidine?
All localized colon cancers at the time of surgery should be tested to see whether
the tumors are mismatch repair-deficient, or mismatch repair-proficient.
In general, all stage three colon cancers require adjuvant chemotherapy. Adjuvant
chemotherapy can be for a duration of 3 months or 6 months. The chemotherapy
agents most commonly utilized are a combination of oxaliplatin and a fluoropyrimi-
dine so that will be FOLFOX or CAPOX.
Looking first at stage 3, there are a number of decisions to be made. Should the
duration of therapy be 3 months or 6 months? Should everyone get the fluoropyrimi-
dine and oxaliplatin combination, or, for older people, are there ways in which we
can de-escalate the chemotherapy? How do we deal with chemotherapy side effects
in these patients? Are there patients in whom we should shorten the chemotherapy
program?
In general, patients who have early stage 3 disease-like T3N1 can get away with
3 months of chemotherapy. In general, the treatment of choice is CAPOX,
capecitabine, and oxaliplatin. Any patient with T3N2 or higher should receive 6
months of adjuvant chemotherapy, which can be either CAPOX or FOLFOX.

B. George (*)
Division of Hematology and Oncology, Department of Medicine, Medical College of
Wisconsin, Milwaukee, WI, USA
e-mail: [email protected]

© The Author(s), under exclusive license to Springer Nature 151


Switzerland AG 2023
W. H. Sobin et al. (eds.), Managing Complex Cases in Gastroenterology,
https://doi.org/10.1007/978-3-031-48949-5_22
152 B. George

While we are still discussing stage 3 disease, how do we manage older people?
Patients who are 70 and older may have problems handling doublet chemotherapy.
In these patients, I think it is reasonable to administer leucovorin-modulated fluoro-
pyrimidine alone for a total duration of 6 months.
In the younger patient who starts out with adjuvant chemotherapy including fluo-
ropyrimidine and oxaliplatin, if rate limiting peripheral neuropathy develops in 3–4
months, it is reasonable to de-escalate to fluoropyrimidine alone (if the neuropathy
is relatively severe, grade 3 or 4).
Adjuvant chemotherapy in stage 2 disease is generally restricted to those patients
who are deemed to have high-risk disease. It is high risk if you have a perforated
tumor, an obstructing tumor, inadequate lymph node dissection (less than 12 lymph
nodes), or a poorly differentiated tumor. Lymphovascular invasion and margin posi-
tivity would be considered high-risk features as well. Ideally, anyone with high-risk
stage 2 colon cancer should receive 6 months of adjuvant chemotherapy with either
single-agent fluoropyrimidine, or, preferably, combination of fluoropyrimidine and
oxaliplatin, FOLFOX. If you’re using CAPOX, you can get away with 3 months of
CAPOX in high-risk stage 2 colon cancers. If you’re using FOLFOX, you need
6 months.
Now, there are a couple of exceptions to the rules. We started out saying that all
tumors should be checked to see whether they are mismatch repair-deficient, or
mismatch repair-proficient. If the tumor is stage 2 and it is mismatch repair-­deficient,
then you can get away without any adjuvant chemotherapy simply because the risk
of metastasis is very low. In the stage 3 setting, even if the tumor is mismatch repair-­
deficient, we prefer to give the full prescribed course of chemotherapy.
There are other new ways in which you can risk stratify stage 2 colon cancers
over and beyond the pathologic features. One tool we are using is oncotype DX, a
genomic panel to help with risk stratification. If deemed high risk on oncotype DX,
it is better to receive chemotherapy but if low risk, surveillance alone following
surgery is appropriate. If you’re high risk, you’re better off with combination
chemotherapy.
Another tool available for risk stratification in the adjuvant setting is minimal
residual disease (MRD)assessment in the circulation. It is a blood test using DNA
from the patient’s tumor. If MRD assay is positive, the risk of relapse is high. For
stage 2, colon cancers with positive MRD adjuvant chemotherapy is a reasonable
option. How long should it be given to these patients? Should chemotherapy be
administered until the MRD assay returns negative? Should we change the adjuvant
chemotherapy agents if the MRD is not impacted? We still don’t know. There are
trials that are addressing this question, which summarizes the adjuvant chemother-
apy landscape for stage 3 and stage 2 colon cancer.
Can you discuss the differences between 5FU, leucovorin, and capecitabine?
5FU and leucovorin are administered together as an IV infusion. Capecitabine is
a pill formulation of 5-FU. 5FU, when it is infused, is a continuous infusion over a
duration of two days. Capecitabine is cycled as a 2-week on, 1-week off, regimen,
where you take the pills morning and night. There are some subtle differences
between their side effect profiles in that capecitabine tends to cause more hand-foot
22 GI Oncology 153

skin reaction than 5FU and leucovorin, but, otherwise, the side effect profile is fairly
similar. Secondly, with capecitabine, you need to do dose modifications for kidney
and liver dysfunction. With 5FU and leucovorin, you don’t need to make any dose
adjustments for liver or kidney dysfunction, though cautious use is recommended in
the setting of kidney/liver dysfunction. Capecitabine is more convenient, since it is
a pill, while 5FU needs to be administered IV through a port.
Is leucovorin always given as an adjunct to 5FU?
Yes, you don’t need leucovorin along with capecitabine, but leucovorin is given
as an adjunct to 5FU, more as a catalyst to stabilize the interaction of 5FU with
thymidylate synthetase, thereby facilitating the function of 5FU. It helps form a
ternary complex that enhances drug action.
Is there something about adenocarcinoma of the GI tract where it responds
better to fluoropyrimidines and platinum drugs? I know these are used in
esophageal and pancreatic adenocarcinoma and colon cancer.
Yes, but we don’t necessarily have a great explanation for why tumors in the GI
tract respond better to these agents. In fact, there are some subtle differences in that
among the platinums, the only platinum that is active in colon cancer is oxaliplatin.
The other platinum drugs used in oncology, cisplatin and carboplatin, are active in
both pancreas and gastroesophageal cancer, but not so much in colon cancer. We
don’t have a good explanation for why these agents, in general, are active in GI
cancers.
Is it true that patients with KRAS mutations respond poorly to EGFR
inhibitors?
First of all, KRAS mutations in general are thought to be negative prognostic
indicators in patients with colon cancer, but the predictive value of KRAS has been
rigorously validated in the metastatic setting, not so much in the adjuvant setting.
We don’t use anti-EGFR-directed therapy for stage 2 or stage 3 colon cancer, but we
use it in the metastatic setting. In the metastatic setting, patients with KRAS muta-
tions do not respond to anti-EGFR-directed therapy simply because the KRAS
mutation is downstream of EGFR and therefore that pathway is already constitu-
ently activated with KRAS mutation. So, blocking EGFR does not necessarily help
in that scenario.
Is it true that right-sided colon cancers respond better to VEGF inhibitors
and left-sided respond better to EGFR inhibitors?
Yes, and in addition, the sidedness of colon cancer is both prognostic and predic-
tive. Right-sided colon cancers in general tend to behave worse than left-sided colon
cancer. Several reasons have been postulated to explain this, but we don’t know the
exact reasons completely.
We think left-sided colon cancers may get diagnosed a little sooner because of
symptoms than right-sided tumors. The right-sided cancers may get diagnosed later
in the disease course because you don’t get into obstructive-type symptoms.
Secondly, patients with right-sided colon cancer are more likely to harbor BRAF
mutations, which have a negative prognostic effect as well.
Third, the embryonic origin of the right side of the colon is different compared
to the left side of the colon and that could predispose to the different behavior.
154 B. George

If you’re looking at a biologic therapy to be used in combination with chemo-


therapy, right-sided colon cancers respond better to VEGF inhibitors than anti-­
EGFR-­directed therapy. So, for tumors in the right side of the colon, when you’re
combining a biologic agent with chemotherapy, anti-VEGF agents work much bet-
ter than anti-EGFR-directed therapy. That may partly be because there is a substan-
tial proportion of patients with right-sided colon cancers that harbor BRAF
mutations, but even in tumors that are wild type for KRAS or BRAF we see that the
anti-EGFR-directed therapies just don’t work as well for the right-sided tumors
(compared to left-sided colon tumors) for reasons that are not very clear.
Conversely, for left-sided tumors, anti-VEGF therapies work well in combina-
tion with chemotherapy, but for KRAS wild-type tumors, the anti-EGFR therapies
may work a little better than anti-VEGF therapies. Anti-EGFR therapies for left-­
sided colon tumors are effective only in the KRAS and BRAF wild-type tumors.
In a patient with newly diagnosed metastatic disease would you use
bevacizumab?
If a patient develops metastatic disease and it starts in the right colon, bevaci-
zumab should be added to chemotherapy backbone (in the absence of contraindica-
tions to bevacizumab). For right-sided colon cancers that present with metastatic
disease, the chemotherapy backbone would be either FOLFOX (5FU and oxalipla-
tin), FOLFIRI (5 FU and irinotecan), or FOLFOXIRI (5FU, oxaliplatin, and irino-
tecan), which is a triplet of the 3.
Case 2 A 48-year-old man is found to have a right-sided colon cancer and is sus-
pected of having Lynch syndrome. Testing is ordered on tissue obtained during colo-
noscopy, and results show it is microsatellite unstable (MSI-H) and there is an
MLH1 mutation consistent with Lynch syndrome.
Having these results would you suggest the patient have a more extensive
operation (subtotal colectomy)?
Yes, first of all, when you see an MLH1 mutation in the tumor in a young man
you certainly worry about Lynch syndrome, but you need to coordinate germline
testing to make sure that this is truly a Lynch and not a sporadic MLH1 mutation. If
the patient harbors a germline MLH1 mutation (Lynch syndrome), I think it is very
reasonable to do a subtotal colectomy or even a proctocolectomy, after an informed
discussion between the patient and the surgeon.
Assuming the patient is found to have stage 3 disease would you avoid using
a fluoropyrimidine because they are less effective in Lynch syndrome?
That’s a great question. There was a time when we thought that single-agent fluo-
ropyrimidine was not effective in patients with mismatch repair deficiency.
Therefore, we always advocated for combination therapy—fluoropyrimidine and
oxaliplatin. As we see more data, it seems that fluoropyrimidines may actually be
effective in patients with Lynch syndrome.
Would you choose to use a checkpoint inhibitor here because they are more
effective in Lynch syndrome?
There are no mature data currently that suggest using checkpoint blockade in the
adjuvant setting in a patient with Lynch syndrome, and there are ongoing trials
22 GI Oncology 155

trying to answer this question. However, it has certainly been found to be effective
in the metastatic setting. In Lynch syndrome, the use of checkpoint blockade alone
as a single agent results in high response rates.
Do you agree that for the same stage of colon cancer, patients with microsat-
ellite unstable colon cancers have a better prognosis than patients with spo-
radic colon cancer?
It is very important to emphasize that mismatch repair deficiency/microsatellite
instability can be germline (Lynch syndrome) or acquired. The percentage of
patients with microsatellite instability incidence is about 15% in patients with local-
ized colon cancer, but in the metastatic setting, that percentage is much lower, about
5%. This is because microsatellite unstable tumors do not metastasize as much. In
addition, these patients are very often part of a screening program, and things get
picked up sooner, but, regardless, they tend to have better disease biology, and better
prognosis, stage for stage than sporadic colon cancer.
Case 3 A 68-year-old woman with alcoholic cirrhosis (abstinent for 5 years) was
having routine surveillance USA and AFP every 6 months but then missed follow-
up for 18 months. She is found to have two hepatomas, one 4 cm in size and
another one is 5 cm. AFP is 600. TACE is performed, and both lesions decrease
in size to 3 cm. However, the AFP rose to 700. The patient’s functional status
is good.
Can you discuss the use of systemic chemotherapy in this setting?
An important question here is whether this woman could potentially be a trans-
plant candidate. With tumor downsizing after TACE, the patient meets Milan crite-
ria—a set of criteria that determines transplant eligibility.
So, let’s think about it in two different ways. First, let’s say the patient is deemed
not to be a transplant candidate after medical evaluation by the transplant team. If
she is not a transplant candidate, the best systemic therapy options for this patient
are either a combination of bevacizumab + atezolizumab or tremelimumab (anti-­
cytotoxic T lymphocyte-associated antigen 4) plus durvalumab (anti-programmed
cell death ligand-1).
Prior to receiving bevacizumab + atezolizumab, patients need to undergo an
EGD to assess for varices and undergo variceal banding.
The other frontline systemic therapy options include lenvatinib and sorafenib,
but certainly the preference would be either bevacizumab + atezolizumab or treme-
limumab plus durvalumab on account of their efficacy and toxicity profile.
If the patient is a transplant candidate, immunotherapy should be avoided in
the pre-transplant setting. The optimal systemic therapy options would be lenva-
tinib or sorafenib but preferably lenvatinib due to the higher objective
response rates.
Has lenvatinib largely supplanted sorafenib in patients who have contrain-
dications to immunotherapy and get tyrosine kinase inhibitor therapy?
Yes, lenvatinib has for the most part supplanted sorafenib, because of improved
efficacy. That said, the side effect profile is not necessarily more favorable than
sorafenib, it’s just different.
156 B. George

Are checkpoint inhibitors felt to be less effective in HCC associated with


alcohol? associated with NASH?
We don’t have prospective data addressing that specific question. However, the
available data certainly seem to indicate that checkpoint inhibitors are more effec-
tive in viral-induced (hepatitis B and hepatitis C) HCC compared to sorafenib. The
magnitude of benefit with checkpoint inhibitors appears to be less compared to
sorafenib in patients with hepatocellular carcinoma due to non-viral etiologies.
Right now, we are not prioritizing treatment based on etiology of HCC, but this
may become part of routine clinical practice in the future as more data emerge.
Case 4 A 67-year-old man presenting with dysphagia is found to have adenocarci-
noma of the esophagogastric junction (EGJ). EUS shows positive lymph nodes.
Tumor markers are sent off and the tumor has HER-2 overexpression.
Do you manage adenocarcinoma of the esophagus the same as adenocarci-
noma of the stomach?
Yes and no. For all intents and purposes, the chemotherapy agents that are active
in adenocarcinomas of the stomach and the esophagus/EG junction are very similar.
However, there are some key differences between tumors of the distal esophagus/
EG junction, and tumors in the stomach, particularly as it pertains to treatment for
localized disease. From a staging perspective, a diagnostic laparoscopy to look for
peritoneal carcinomatosis is mandated for T2 or higher tumors in the stomach while
that is not done routinely for tumors of the distal esophagus/GE junction (dictated
by patterns of spread). In terms of neoadjuvant/peri-operative treatment, radiother-
apy is more commonly utilized in tumors of the distal esophagus/EG junction than
tumors of the stomach. Surgical approaches are different for these entities as well.
There are some biological differences between these entities as well. We know
that tumors in the EG junction are more likely to overexpress HER2/neu than tumors
in the stomach. Similarly, tumors that originate in the EG junction are more likely
to respond to immunotherapy than tumors that originate in the stomach based on
available data (this has not been prospectively validated).
At a molecular level, tumors in the distal esophagus/EG junction and stomach
are classified into four distinct subgroups: (a) chromosomally unstable (CIN), (b)
genomically stable, (c) microsatellite unstable, and (d) the EBV-positive.
The EG junction tumors tend to belong to the chromosomally unstable subtype
(characterized by alterations in the signal transduction pathway) and the EBV-­
positive subtype (characterized by alterations in thePI3 kinase pathway and greater
likelihood of response to immunotherapy).
Microsatellite unstable tumors tend to be present throughout the stomach, with a
preponderance in the distal stomach while the genomically stable subtype correlates
very closely with linitis plastica or diffuse gastric cancer.
Why might trastuzumab be useful in the above patient, with HER-2
overexpression?
Trastuzumab is a monoclonal antibody that binds to ERBB2 (Her-2) preventing
the dimerization with ERBB3 and inhibiting downstream signaling that contributes
to cancer progression. So essentially, it’s a targeted agent that improves treatment
22 GI Oncology 157

response and prolongs survival in patients with metastatic EG junction/gastric can-


cers with a very favorable toxicity profile.
Can you discuss the general approach to treatment of localized adenocarci-
noma of the esophagus (and stomach)?
For localized tumors of the esophagus and EG junction, we typically use neoad-
juvant chemotherapy and radiation, followed by surgery for curative intent treat-
ment. For tumors of the stomach, we use neoadjuvant or perioperative chemotherapy
and surgery for curative intent therapy. The role of radiation in tumors of the stom-
ach is controversial and it’s not routinely used.
Is there a role for checkpoint inhibitors?
Yes, there is a role for utilizing a checkpoint inhibitor in the treatment of meta-
static disease regardless of the PD-L1 expression. Typically, the frontline systemic
therapy program for tumors of the stomach and distal esophagus/EG junction
includes a fluoropyrimidine, a platinum, and a checkpoint inhibitor, if the tumor
does not over express HER2/neu. If HER2/neu overexpression is present, we utilize
a combination of fluoropyrimidine, platinum, trastuzumab, and a checkpoint
inhibitor.
Case 5 A 71-year-old woman presents with abdominal pain, weight loss, and new-­
onset diabetes. A CAT scan reveals probable cancer of the body of the pancreas.
EUS shows malignant appearing lymph nodes and FNA is positive for adenocarci-
noma. Genetic testing is performed that includes MSI testing and homologous
recombination deficiency (HRD) testing.
Is it true that when we talk about HRD testing we are checking for muta-
tions in all of these genes: BRCA1/2, PALB2, ATM, BAP1, BARD1, BLM,
BRIP1, CHEK2, FAM175A, FANCA, FANCC, NBN, RAD50, RAD51, RAD51C,
and RTEL1?
We are interested in assessing the HRD status of pancreas cancers to help make
treatment decisions. Assays that determine the HRD status of tumors use different
methods, and the parameters tested vary based on the assay that is utilized. It is true
that many of the above genes play a key role in DNA damage repair.
Is it true that if a pancreas cancer is found to have HRD you will turn to a
platinum-based therapy like FOLFOX or FOLFIRINOX?
Yes, we will utilize platinum-based chemotherapy if a pancreatic adenocarci-
noma harbors HRD.
Are all patients with pancreatic cancer getting this genetic testing done?
We offer germline testing to all patients with pancreatic cancer, and it has been
incorporated in national guidelines.
Is gemcitabine commonly used anymore-alone or with nabpaclitaxil?
Yes, gemcitabine is used quite a bit both in the treatment of metastatic and local-
ized pancreatic adenocarcinomas. Gemcitabine can be used as a single agent or in
combination (with Nab-paclitaxel or Nab-paclitaxel and cisplatin). Gemcitabine is
also utilized as a single agent for purposes of radio-sensitization in patients who
undergo concurrent chemoradiotherapy.
158 B. George

When do you use a PARP inhibitor and how does it work?


PARP inhibitors are used currently in patients with pancreatic adenocarcinoma
who have germline pathogenic alterations in BRCA1 and BRCA2. It is utilized for
maintenance therapy in patients with metastatic pancreas cancer who are platinum-­
responsive. Olaparib is the PARP inhibitor approved for use in pancreatic adenocar-
cinoma., The idea behind treatment with the PARP inhibitors is that they cause
synthetic lethality in the cancer cells of patients who have BRCA1 or BRCA2 germ-
line alterations. Normally, PARP helps in the repair of single-stranded DNA breaks
and PARP inhibitors prevent that. So, DNA in cancer cells that are exposed to DNA-­
damaging drugs undergo single- and double-stranded DNA breaks, and PARP
inhibitors prevent the cancer cells from repairing them, causing increased cancer
cell death.
Are you using checkpoint inhibitors in pancreatic cancer?
There are a lot of clinical trials in this space. Currently, there is no proven role
for checkpoint inhibitors in pancreas cancer other than in the 1% of tumors that have
a deficient mismatch repair gene status. There is also a very small percentage of
pancreatic adenocarcinomas with an elevated tumor mutational burden that may
respond to checkpoint inhibitors as well.
Are there any other points about pancreatic cancer management that you
would like to emphasize for the community gastroenterologist?
One point I’d like to stress is the importance of a metallic common bile duct stent
(as opposed to a plastic stent)—once tissue diagnosis is established—in patients
with pancreatic head adenocarcinomas that cause biliary compromise. This helps
decrease the incidence of stent occlusion and cholangitis in patients with pancreatic
adenocarcinoma while on treatment.
Chapter 23
Issues in Therapeutic Endoscopy

Kulwinder Dua

 anagement of Refractory Benign Esophageal Stricture


M
(RBES) Including Placement of Esophageal Stents

Case 1 A 70-year-old man has prolonged NG tube insertion after a complicated


colon resection for diverticulitis. Following that, he develops dysphagia. An EGD
shows a tight stricture in the mid-to-distal esophagus from 27 to 37 cm, and biopsies
are benign. He has received several endoscopic dilations but continues to re-­
stricture and is referred to you.
How do you manage a patient with a refractory benign esophageal stricture?
The definition of RBES is that one cannot achieve an esophageal luminal diam-
eter of ≥14 mm despite one dilation done every 2 weeks × 5 or if the patient requires
one dilation every 4 weeks to maintain a diameter of ≥14 mm. In this patient, the
stricture is due to prolonged trauma from the NG tube and damage from acid reflux-
ing alongside the tube (like capillary action) for prolonged periods. Some of the
other causes of RBES include corrosive ingestion, pill injury, radiation-related,
post-endoscopic mucosal resections and ablations, and surgery.
In patients with RBES, besides etiology, it is important to know the diameter to
which the patient was dilated previously, the frequency of dilations, and how soon
the patient developed recurrent dysphagia. Based on this, one can decide on a treat-
ment plan. Let’s say you are dilating every 4 weeks and you can only achieve a
diameter of 12 mm, then you need to dilate every 2 weeks. If you are going two
steps forward and then two steps backward every 2 weeks, then you need to dilate

K. Dua (*)
GI and Hepatology Division, Dept. of Medicine, Medical College of Wisconsin,
Milwaukee, WI, USA
e-mail: [email protected]

© The Author(s), under exclusive license to Springer Nature 159


Switzerland AG 2023
W. H. Sobin et al. (eds.), Managing Complex Cases in Gastroenterology,
https://doi.org/10.1007/978-3-031-48949-5_23
160 K. Dua

every week. One can also consider injecting steroids (triamcinolone) into the stric-
ture to delay fibrosis and recurrence. There is no limit to how many times you inject
triamcinolone. If the stricture keeps closing up after each dilation, I would inject at
every dilation.
At each dilation session, assess how much the stricture has regressed and then
decide whether you need to increase or decrease the frequency of dilations (two
steps forward and one step backward), but even though you are making progress
you never want to leave the patient for more than 2 weeks without repeat dilation
until you have achieved your goal. Once a diameter of 14–18 mm is achieved and
maintained at weekly or every 2-week dilations, lengthen the interval between dila-
tions. If the stricture continues to regress, one can consider other interventions, such
as stent insertion, electrocautery incision, or both.
What sort of stents do you place?
The self-expanding stent has to be a fully covered stent so that it can be removed
at a later date. If the stent is uncovered or partly covered, it will get embedded in the
esophageal wall and may not be easy to remove. The only fully covered stent that is
FDA approved and available in the USA for benign esophageal stricture is a plastic
expandable stent (Polyflex® stent). This stent is difficult to load, has a high migra-
tion rate, and tends to cause chest pain. As a result, most gastroenterologists are
using fully covered metal expandable stents, off-label.
For precise placement such as when the strictures are near (<2 cm) the UES, as
we sometimes see in head and neck cancers after radiation therapy, non-­
foreshortening stents (laser-cut) are preferred. More often, we tend to use electro-
cautery and avoid placing stents in this location if the stricture is short and like
a shelf.
When the stricture is in the lower esophagus and failed dilations, we will use
stents, even if they need to bridge the LES. However, patients are predisposed to
having significant gastroesophageal reflux that can create a considerable aspiration
risk. Therefore, we have to use strict anti-reflux measures in these cases.
Stents for RBES are usually left in for 2–4 weeks. After removing the stent, rein-
stitute regular esophageal dilations. For those who fail the above approach, we offer
them surgery (in many cases, this may not be possible) or train them to do
self-dilation.
How do you teach self-dilation?
All patients get apprehensive when told about self-dilation. Good discussion
with the patient and showing them videos of those who have successfully adopted
this approach help. Patient is initially dilated to a diameter of 16–18 mm endoscopi-
cally. A week or so later, the patient is taught to self-dilate holding a non-wire-­
guided weighted dilator (such as the Maloney® dilator) of around 15 mm diameter,
sitting on a chair, and watching the dilator go down under fluoroscopy (biofeed-
back), and immediately repeat without fluoroscopy. After that, the patient takes the
dilator home and dilates himself once or twice a day. Initially, patients like to use
some viscous lidocaine, but after a week or two it’s no longer necessary. If, after a
while, the patient finds it is once again becoming more difficult to pass the dilator
23 Issues in Therapeutic Endoscopy 161

because the stricture is tightening, we bring the patient back and dilate them endo-
scopically to 16–18 mm and after that they find the 15-mm self-dilation go eas-
ily again.
How do you decide what type of dilator to use in treating a benign esopha-
geal stricture?
In talking about the type of dilator used, there are balloon dilators that have good
radial force and there are bougies that have both radial and longitudinal force. In
expert hands, the safety profile is equivalent. It is a good idea to use a bougie in
cases where the stricture is long, or multifocal as bougies cause pan-esophageal
dilation. These kinds of strictures are usually encountered with corrosives, radia-
tion, and prolonged NG tube use. Balloon dilators are certainly acceptable for short
strictures, such as anastomotic strictures, or short peptic strictures.
Case 2 A 72-year-old smoker presents with dysphagia and is found to have a tight
stricture of the mid-esophagus caused by esophageal squamous cell cancer.
Chemoradiation is planned.
Is there a role for a stent in this malignant esophageal stricture?
Generally, no; the initial enthusiasm for stents as a bridge to surgery for those
receiving neoadjuvant therapy or even for those receiving palliative chemotherapy
has been dampened by the high rate of adverse events and side effects from stents
that may interrupt chemo-/chemoradiation treatments. With the tumor regressing
and becoming “softer” with chemo-/chemoradiation treatments, stents can migrate,
cause ulcers and bleeding, and perforate the esophagus.
Stents, however, are very effective in immediately relieving dysphagia. Therefore,
if we have a patient with severe dysphagia (barely able to swallow liquids or total
dysphagia), we will communicate with the oncologist and find out how long a delay
they anticipate before chemoradiation will be started and how long a lag period
between initiation of therapy and when tumor response is anticipated. If therapy is
being delayed for a month and response will probably take 2 weeks, we may decide
to place a temporary stent if the patient is suffering from severe dysphagia. In this
instance, we will consider placement of a fully covered metal stent as a temporary
bridge so that the patient can swallow and plan on removing it around the time
chemo-/chemoradiation therapy is being initiated to avoid delayed stent-related
adverse events. If the dysphagia is just for solids, I would not place a stent because
I am fine with the patient simply swallowing a soft diet or liquids, such as Ensure
and milkshakes.
Covered stents are the treatment of choice for those who have esophageal-airway
fistulae or perforations. In these patients, available options, including stenting,
should be discussed in a tumor-board or multidisciplinary meeting.
What about using an esophageal stent for a malignant stricture where the
patient refuses chemoradiation?
It is important to discuss the expectations the patient may have about stents. The
best one can eat with a stent is a soft-liquid diet, not a normal diet. So, do not place
stents in those who are eating a soft diet at baseline.
162 K. Dua

In patients receiving a stent for palliation, it is important to counterbalance the


benefits of the stent vs the side effects/adverse events. If the patient does not want
any palliative chemoradiation, then optimally one can place a partially covered stent
where the upper and lower ends of the stent are uncovered. This will allow tissue
ingrowth and embed the stent, which will prevent migration, but if the patient
changes his/her mind and decides to go for chemoradiation, removing the stent will
be difficult (if not impossible). Therefore, one should have a frank discussion about
this with the patient before placing the stent. Alternatively, fully covered stents with
fixing devices to prevent migration can also be used in these patients for potential
removal in the future if needed.
The other problem with the partly covered stent, along with the fact that it is not
easily removed, is that you tend to get tumor ingrowth, which may eventually lead
to secondary dysphagia.
In most cases of malignant stricture, we are reserving stents for patients who can
barely handle liquids. We are placing stents in patients with severe, grade 3 or 4
dysphagia, but will not consider them with mild (grade 1 or 2) dysphagia.
Case 3 A 67-year-old man receiving radiation therapy for metastatic lung cancer
with enlarged mediastinal lymph nodes has a tight extrinsic compression of his mid-­
esophagus. The esophageal mucosa itself appears normal.
What is the role of an esophageal stent in a case of extrinsic, malignant
obstruction?
The best approach for extrinsic compression will be to treat the underlying cause,
which may not always be easy or possible. However, if there is hope that planned
chemoradiation or surgery will resolve the extrinsic pressure, one can consider plac-
ing a removable stent for the patient with grade 3 or 4 dysphagia.
A concern about stenting an extrinsic compression is that the mass may be press-
ing on the left or right main stem bronchus or trachea, and the moment the stent
expands, it may cause occlusion of the airways. With upper esophageal cancers or
obstruction from a mediastinal mass, it is very important to get and review a good
quality CT scan of the chest to ensure that there is no impending airway
compression.
If you have a patient who will be getting treatment and is not at risk of airway
obstruction, one can place a fully covered stent and fix it in place, or place a partially
uncovered stent based on long-term outcomes as discussed above.

Nutrition Needs

Although stents relieve dysphagia and allow feeding, often patients don’t eat enough
secondary to either chest discomfort or anorexia from chemotherapy. In these
patients, one can consider alternatives such as placing a G- or a J-tube to supple-
ment nutrition, while the stent improves quality of life, allowing the patient to take
oral feeding.
23 Issues in Therapeutic Endoscopy 163

Pancreatic Cancer

Case 4 A 68-year-old man comes in with painless jaundice. He has no history of


alcohol use, diabetes, or gallbladder disease. On examination, he is jaundiced;
examination is otherwise unremarkable. His laboratory findings include: Bili 5.6,
Alk phos 320, ALT 70, AST 55, and lipase 300.
An ultrasound shows a dilated common bile duct, a normal gallbladder and liver,
and a questionable mass in the head of the pancreas.
In a case of painless jaundice, what imaging studies do you like to start with?
If the patient has no contraindication (renal insufficiency or dye allergy), the
preferred study will be a pancreas protocol CT scan with IV contrast. This provides
accurate staging of the disease and must be done before other procedures such as
EUS-FNA/B or ERCP, as these other procedures can result in pancreatitis, and then,
CT staging can get clouded. If CT with contrast cannot be done, then MRI can be
considered. One would also like to order a CA 19-9 on the patient.
The community gastroenterologist plans on doing an ERCP on this patient
to decompress the bile duct with a plastic stent and do diagnostic brushing of
the bile duct for tissue diagnosis. What do you think of this approach?
No, this is the wrong approach. This decision is based on two faulty premises.
The first is the fear that if a patient is jaundiced from presumed pancreatic cancer,
the patient is at a high risk of getting cholangitis. Patients with malignant obstruc-
tion of the bile duct rarely develop cholangitis. The second is that doing an ERCP
and cytology brushing is an efficient way of making a tissue diagnosis, but it is not.
An EUS-FNA is far superior. Moreover, EUS-FNA-ERCP-metal stent (“one-stop
shop”) all can be done in one setting with rapid on-site rapid cytology (ROSE)
evaluation by a cytopathologist. Hence, if one does not have EUS-FNA and ROSE
available, one should not do ERCP and, instead, refer the patient to a tertiary care
center. Moreover, placing plastic biliary stents (because metal stents for durable
biliary drainage should not be placed without tissue diagnosis) carries an increased
risk of recurrent cholangitis. In addition, the patient will have to have another ERCP
to exchange for a metal stent if the brushing came back positive for cancer, hence,
all the more reason to refer these patients to centers with expertise.
Even in those with resectable pancreatic cancer, this approach of “one-stop shop”
is being widely used, as more and more patients with resectable pancreatic cancer
are getting neoadjuvant therapy rather than early surgery. Tissue diagnosis and dura-
ble biliary drainage are essential prerequisites for neoadjuvant therapy, and if,
indeed, the patient is being considered for early surgery, biliary drainage before
surgery is not required and can be detrimental. Hence, in this patient, ERCP should
not be done.
Case 5 A 65-year-old man presents with an episode of acute pancreatitis. There is
no history of diabetes. He does not drink much alcohol. An ultrasound shows a
normal gallbladder. A pancreas protocol CT shows pancreatitis but no mass or no
stone. A CA 19-9 is sent off and returns 50 (normal is up to 35).
164 K. Dua

What do you do when you suspect a patient may have pancreatic cancer but
there is no mass on US or CT?
Not every case of obscure pancreatitis is pancreatic cancer. Some patients may
have microlithiasis, etc., but when we see a patient who comes in out of the blue
with pancreatitis we worry about pancreatic cancer, while keeping the rest of the
differential in mind. Also, there can be a nonspecific rise in CA 19-9 with
pancreatitis.
In the face of active pancreatitis, an EUS is less helpful as pancreatitis will show
up as diffuse or focal hypoechogenicity of the pancreas, and a cancer can be missed.
So, in a number of these cases, where one is concerned about possible pancreatic
cancer, it is best to wait and let the inflammation settle down in 4–6 weeks and then
repeat the pancreas protocol CT scan and CA 19-9. Then, one could consider EUS.
We recently saw a patient referred by one of our pancreatic surgeons because the
patient had a dilated upstream pancreatic duct that stopped abruptly at the neck of
the pancreas. This patient did not have pancreatitis and had undergone two MRIs
and three CT scans in the last 5 months, and no mass was identified. On EUS, there
was a half-cm hypoechogenic mass at the site of pancreatic duct obstruction that on
FNA turned out to be a mucinous adenocarcinoma.
Here’s another scenario. A patient with elevated liver enzymes is seen who
doesn’t have pancreatitis, but does have a bile duct that is mildly dilated. A CA 19-9
can be elevated in patients with bile duct obstruction. There is no mass in the pan-
creas in this patient. Could the dilation be due to stones? Could this be a cholangio-
carcinoma along the wall of the bile duct? In these situations, we do an EUS, even
if there is no mass identified on imaging studies.
Are there any red flags that make you worry that someone with some subtle
findings could have pancreatic cancer when others aren’t suspecting it?
I worry about this in an elderly person with unexplained weight loss or an unex-
plained attack of pancreatitis. I also worry if I see someone with an elevated CA
19-9. If normal is around 35 but I see a patient who is smoldering around 110, but
nothing is showing up on the pancreas protocol CT scan, I still worry. Then, there
are cases where someone has a family history with two members who had pancre-
atic cancer or a family member who had breast or ovarian cancer and was BRCA
positive. In all these cases if pancreatic protocol CT is negative, we still do an
EUS. In families with a worrisome history, we may do EUS and MRI in alternate
years as part of our screening process.
Similarly, patients with pancreatic cysts/dilated main pancreatic duct with worri-
some or high-risk features do merit an EUS (at times, these patients get operated
upon based on these worrisome features).
Sometimes, we have the opposite problem, where the CT scan and EUS defi-
nitely show a lesion that looks like cancer but repeated FNA is negative for cancer.
A recent patient presented with jaundice and a mass on CT scan; CA 19-9 was 900,
and EUS with FNA/B was repeatedly negative. Everyone thought this was pancre-
atic cancer, but the oncologist would not give neoadjuvant therapy without a
23 Issues in Therapeutic Endoscopy 165

positive tissue diagnosis. Whipple surgery was being considered. However, prior to
surgery a therapeutic trial of oral steroids was given, and sure enough, the “tumor”
melted away and the CA 19-9 dropped precipitously. The jaundice resolved without
a stent. Apparently, this patient has autoimmune pancreatitis, but had there been no
response to steroids early Whipple would have been appropriate.
Chapter 24
GI Pharmacology:

Elizabeth Pieper

Eosinophilic Esophagitis

Case 1 A 26-year-old man is diagnosed with presumed eosinophilic esophagitis


(EOE) after an EGD is performed to remove a food impaction. Biopsies are done
which confirm the physician’s impression of EOE. The patient is started on PPIs
empirically. A repeat EGD is performed 2 months later because of ongoing dyspha-
gia, and repeat biopsies show a decrease in the eosinophil count from 90/hpf to 50/
hpf, but because of the continued eosinophilia the request is made to start steroids.
What steroid preparations do you have available for your patients, and what
are the problems with insurance access?
The drug we’ve used the longest is fluticasone with patients swallowing the
inhaled form. With fluticasone, the standard dose is 440 mcg twice a day. The hard-
est part though is to ensure the patient understands how to administer it, because the
natural response is that everyone wants to inhale the contents. It’s difficult to con-
ceptualize that you are actually spraying it into your mouth and putting up your
tongue to block it from going into your lungs so that you can swallow the liquid.
Regrettably, most pharmacists in the community who are filling these prescrip-
tions are not necessarily familiar with EOE because this condition is not regularly
taught in pharmacy school. Therefore, staff may inappropriately educate the patient
thinking this is being used for asthma. I advise that the provider includes the indica-
tion for EOE and that the patient is supposed to swallow contents and not inhale it
to prevent administration errors.

E. Pieper (*)
Department of Gastroenterology and Hepatology, Froedtert Hospital, Milwaukee, WI, USA
e-mail: [email protected]

© The Author(s), under exclusive license to Springer Nature 167


Switzerland AG 2023
W. H. Sobin et al. (eds.), Managing Complex Cases in Gastroenterology,
https://doi.org/10.1007/978-3-031-48949-5_24
168 E. Pieper

Additionally, the fluticasone inhaler is a branded medication; therefore, insur-


ance coverage can sometimes be a problem. When this is an issue, we may use
budesonide slurry, which is available as a generic (as a respule) and works well.
Once again, we are taking a product that is normally inhaled and putting it in a form
that can be swallowed. We do this by opening up budesonide respules and then mix-
ing the liquid with a vehicle that allows it to adhere to the esophagus. We ask the
patient to mix 1mg of budesonide suspension with ten Splenda packets so that it
creates a slurry that is very thick and syrupy and have the patient swallow that.
Otherwise, you can mix the budesonide in one or two teaspoons of agave nectar, or
chocolate syrup, or applesauce. Basically, you are looking for things that are thicker
and will just stick to the esophagus. I have also used honey in a few cases. The team
ultimately needs to work with the patient to determine what they can best tolerate.
So, due to insurance or out-of-pocket cost, many of our patients have easier
access to generic budesonide respules than branded fluticasone inhalers. Some
patients may opt to use a compounded form of budesonide suspension if they do not
like to mix the slurry themselves and it is not cost-prohibitive. Unfortunately, most
compounded medications are not covered by insurance, or each individual ingredi-
ent has to be billed separately through the insurance, which can cause financial
strain for the patient so we use compounded medications infrequently.
How long are patients able to continue taking PPIs and steroids in EOE?
In EOE, patients are generally refilling prescriptions for PPIs and steroids indefi-
nitely. It is our experience that when the patient stops the steroids or the PPI, patients
typically have a recurrence of symptoms if they were previously responsive to the
medication(s). By maintaining them on PPIs and steroids long term, we are trying
to prevent prolonged inflammation and damage to the esophagus. Ultimately, con-
tinuing medication(s) will improve the quality of life for those who are able to toler-
ate these medications. However, we may discontinue PPIs if a patient develops a
contraindication, say recurrent C. difficile infections, or atypical fractures. Topical
steroids may be discontinued if the patient is transitioned to a biologic for EOE or
if the patient is unable to tolerate steroids (such as having systemic side effects).
Otherwise, patients will be maintained on these drugs indefinitely.

Helicobacter Pylori

Case 2 A 35-year-old woman was found to have a gastric ulcer on endoscopy that
was helicobacter pylori-positive. She is sent to you to help manage her antibiotic
regimen.
Historically triple-drug regimens have been prescribed most commonly in
the USA. However, the latest ACG guidelines suggest quadruple therapy. What
do you prefer?
The major argument for using quadruple therapy is HP resistance to clarithromy-
cin. We would like to study H. pylori resistance in our area, but currently antibio-
grams are not available.
24 GI Pharmacology: 169

I still prefer to use triple therapy as the first line because patient compliance is
much better than with quadruple therapy. Triple therapy treatment is so much easier
for patients to take since it is only twice daily versus quadruple therapy, which may
need to be taken up to four times a day. Triple therapy also seems to be better toler-
ated in terms of GI side effects. Therefore, it is commonly prescribed first line as
long as there are no contraindications to any of the treatment components. However,
if a patient has a true penicillin allergy, is on medications that interact with clarithro-
mycin, has cardiac issues, or if a patient failed an earlier case of triple therapy with
clarithromycin, I commonly switch to quadruple therapy. So far, most patients who
have taken triple therapy (from our clinic) and were adherent seem to have a good
response and are able to clear the bacteria.
Will you use triple therapy in a patient who has received clarithromycin, or
other macrolides for some other indication in the past?
If a patient has received clarithromycin, or other macrolides, for an URI or some
other short-term indication, I will still try clarithromycin again, because I don’t
know how long the HP infection has been present, and whether this bacterium has,
or has not been previously exposed to macrolides.
If there is previous exposure but I think that triple therapy will suit the patient
best, I typically have a discussion with the patient regarding the pros and cons of the
treatment including the potential of drug resistance and the possibility that the
patient might require retreatment with a different regimen should the infection
persist.
If a patient has a real penicillin allergy (not just GI discomfort with amoxicillin),
l may use quadruple therapy instead. I do not switch out metronidazole for amoxi-
cillin in the triple regimen, because the combined resistance to both metronidazole
and clarithromycin is too high in the community and the estimated success rate is
less than 70%.
What is your preferred method for providing quadruple therapy?
First, although prescribing Pylera may be easier, this combination is a branded
product and is generally not covered by insurance, or it requires prior authorization
and causes marked delays. For example, Medicare recipients who have to pay a
percentage of the drug cost may have higher out-of-pocket copay compared to if the
individual medications were ordered.
The second thing is that I prefer to avoid prescribing tetracycline. I live in a dairy
state, and people love their milk, cheese, and yogurt. You are supposed to space out
the use of dairy products (and other calcium-containing products) at least 2 h from
tetracyclines, and since tetracycline has to be taken four times a day, it becomes a
logistical nightmare. Therefore, I commonly use doxycycline instead of tetracy-
cline. It only needs to be taken twice a day, and we see much better compliance than
taking tetracycline. In addition, it is much cheaper than tetracycline. Two weeks of
doxycycline might cost $20, in comparison with $100 for tetracycline.
In terms of bismuth, I order bismuth subsalicylate. Generally, patients have to
buy this over-the-counter since insurances normally exclude coverage of OTC
items. There are multiple OTC products that contain bismuth including Pepto-­
Bismol and Kaopectate. You have to check the label and verify that it is truly
170 E. Pieper

bismuth subsalicylate and to verify the dose of the product. Different Pepto-Bismol
products might have different ingredients, but I educate patients that they will need
to take 524 mg per dose. This may be two 262 mg tabs or generally 30cc of the
262 mg/15 ml liquid. Of note, there is a new formulation of Pepto-Bismol available
that is branded, called Pepto-Bismol Extra, which has 524 mg in one tablet. So,
when I provide education, I always stress that they have to check the labeling so that
they take the right amount, and not too much, of the medication.
Have you used any regimens containing levofloxacin or rifabutin?
I typically use levofloxacin (with amoxicillin and PPI) as second-line therapy if
the patient fails triple therapy (with clarithromycin) or quadruple therapy. The data
suggest that levofloxacin-based regimens work better when it is used as second-line
therapy. I have not had too many problems with patients tolerating the medicine, but
we typically check an EKG prior to initiation to ensure that there is no evidence of
QT prolongation if there are any risk factors (older age, h/o cardiac issues, females,
on other QTc prolonging medications).
I also have prescribed rifabutin-containing regimens, these are usually the third
line when patients have failed other regimens or if there are contraindications for the
other therapy options. I usually prescribe rifabutin with amoxicillin, but may also
combine rifabutin with moxifloxacin depending on the patient’s situation.
Do you test for post-treatment clearance?
Yes, and we also call and check in with the patient about halfway through treat-
ment to make sure he/she/they is taking the medicine appropriately. Nonadherence
is a huge problem with HP treatment so we try to prevent it and, if needed, correct
issues before therapy is finished. Once I verify that they have been taking the medi-
cine correctly, we will arrange to get a Helicobacter pylori stool antigen (HPSA)
6–8 weeks after finishing treatment. Our nurses order the test and call to remind the
patient to get the testing done when it is due.
Besides HPSA, we occasionally order the breath test for patients who have dif-
ficulty dropping off their stool expeditiously. In the case where patients need a fol-
low-­up endoscopy, then we will get biopsies to verify eradication.
The most important counseling point for these tests is to ensure that patients are
off PPIs for at least 2 weeks prior to testing and antibiotics for at least 4 weeks prior
to testing; otherwise, you may get a false negative. In terms of the breath test, there
are more requirements to get a valid result, such as fasting and not smoking for more
than 1 h prior to the appointment.
In using PPIs in treating HP, GERD, etc., do you think it matters which PPI
is used?
I do not think all PPIs are created equal. While they may have the same mecha-
nism of action, they differ in terms of enzymatic metabolism, excretion, and potency.
I think that most people get prescribed omeprazole first, because in terms of potency
it’s smack dab in the middle, and it is widely used and accepted. In addition, it is
available over-the-counter and it’s not terribly expensive.
However, if a person has intolerance to omeprazole you can switch to another
PPI. If omeprazole starts to lose effect, you might want to switch to a higher potency
PPI such as esomeprazole or rabeprazole. However, these may involve some insur-
ance hurdles.
24 GI Pharmacology: 171

Prescribers need to consider CYP interactions and drug metabolism for PPIs. For
example, there are many patients on clopidogrel, which is metabolized by CYP2C19.
If omeprazole is used concomitantly, it can decrease the serum levels of clopidogrel,
thereby increasing the risk of a cardiovascular event. In this specific case, pantopra-
zole is preferred due to less CYP interactions. However, because of pantoprazole’s
lower potency, a higher dose may be needed to get the same therapeutic effect as
omeprazole. In addition, if you know that someone is a CYP2C19 rapid metabolizer
you would want to switch to one of the PPIs where CYP2C19 is a minor substrate-
including pantoprazole, lansoprazole, or rabeprazole. Rabeprazole is the strongest
of these three. Normally, patients have to fail other PPIs before their insurance will
authorize rabeprazole.

IBS-C, IBS-D, and Chronic Constipation

Case 3 A 32-year-old woman is diagnosed with IBS-C. She has been taking
metamucil and miralax with only minor relief.
Which of the IBS-C and IBS-D medicines do your practitioners tend to
order, and what are the issues with accessibility for these drugs?
Linaclotide, plecanatide, and lubiprostone are the most commonly prescribed
medications for IBS-C. Linaclotide tends to be used first, it has very good evidence
to support its use, patients respond very well to it, and there are also good patient
assistance programs available, so financial issues tend to be pretty minimal.
However, if patients don’t tolerate linaclotide, or they do not get the desired effect,
then we will try lubiprostone or plecanatide. While tegaserod is approved for IBS-C,
I do not see it commonly prescribed due to its limitation for use in females only and
those without multiple cardiac risk factors.
For IBS-D, eluxadoline is approved, but we only put in a few requests for it, in
part because it’s difficult for patients to get coverage for the drug. In addition, elux-
adoline should not be prescribed to patients who have had a cholecystectomy or who
drink more than three alcoholic drinks a day because of the risk of pancreatitis.
Another medication approved for IBS-D is rifaximin. It is an incredibly expen-
sive medication and a number of doctors are reluctant to use it for IBS-D because of
its expense and limited supporting data. Now, if the patient also has small intestinal
bacterial overgrowth (SIBO) in addition to IBS-D, there is more evidence to support
its use. Additionally, the use of rifaximin for IBS-D involves cyclic treatment. The
patient takes the medicine for 2 weeks and then stops it, and then, even if it works,
you usually get a recurrence of symptoms, and you need to restart it. Therefore, in
my mind, it may be better to give a different medicine that a patient can take consis-
tently, rather than having to endure cyclic treatment with recurrent symptoms.
Alosetron is another drug available for females with IBS-D who have tried and
failed other drugs. I have not seen it used very often because of problems with insur-
ance coverage of this medication and that the prescriber must follow REMS
requirements.
172 E. Pieper

Prucalopride is FDA approved for idiopathic chronic constipation and is a 5HT-4


agonist. I see prucalopride prescribed often since it not only helps with CIC, but
there is evidence for its use in patients who have both upper and lower GI motility
issues. While it is not yet approved in gastroparesis, a number of our practitioners
have prescribed it for patients who have gastroparesis along with chronic constipa-
tion. I hope that it will be FDA approved for gastroparesis in the future given its
tolerability and our currently limited treatment options in this population.
In the past, when I had patients with chronic constipation who responded
incompletely to linaclotide I would occasionally add misoprostol (excluding
women of reproductive age) off-label as an add-on. Do you see any misoprostol
being used?
We do not see it used much for constipation. However, if a prescriber did want to
use it, I think the indication for use should be included on the prescription. Due to
rulings from the Supreme Court and state laws, pharmacists will need to pay special
attention to how the drug is being prescribed. If the indication is not on the prescrip-
tion, the pharmacist will likely need to call the prescriber before dispensing it since
misoprostol may be illegal if being used for abortive purposes depending on the
pharmacy’s state.
The antispasmodics, dicyclomine and hyoscyamine, have been commonly
used in IBS, particularly IBS-D. However, the 2021 ACG guidelines said they
no longer recommend the use of these drugs, because evidence of their benefit
is weak. The latest 2022 AGA guidelines, just published, gave a conditional
recommendation for their use. Are these able to be prescribed?
We have no problems getting these antispasmodics for our patients. I’m aware of
the literature but it’s one thing to be looking at guidelines and another thing when
you are talking to a patient with IBS who is in a lot of pain and you are trying to
improve their quality of life. With IBS, you generally cannot identify a specific trig-
ger that will make the symptoms disappear and prescribing these drugs can help
allow a patient to live his or her life with less suffering.
What about use of PAMORAs the peripherally acting mu-opioid receptors,
used in patients with chronic constipation related to chronic use of opioids?
I have seen mostly naloxegol and methylnaltrexone prescribed. Naloxegol is
only available orally, while methylnaltrexone is available both orally and SQ, so for
a patient having problems with oral intake it may be beneficial to have an injectable
medicine.
You also have to think about drug interactions when prescribing. Naloxegol is a
CYP3A4 substrate and so there are many potential drug interactions. On the other
hand, methylnaltrexone does not have as many interactions. Therefore, if you are
dealing with a patient on a number of medications for different health consider-
ations, Relistor might be preferable. I have yet to see naldemedine prescribed. In
part, it is because it is so new, but it is also a CYP3A4 substrate, so it will have simi-
lar drug interactions as naloxegol.
24 GI Pharmacology: 173

Hepatitis C

Case 4 A 26-year-old man with a history of IV drug abuse is found to have elevated
liver enzymes. A hepatitis panel is positive for HCV antibody, and a PCR test is
positive with a viral RNA of 1.5 million.
After years of struggling with trying to clear HCV using PEG interferon,
ribavirin, and protease inhibitors, now it would appear to be very simple treat-
ing HCV with the oral agents. Are there any complexities involved?
We tend to give almost every patient pibrentasvir/glecaprevir or sofosbuvir/vel-
patasvir. Both are pan-genotypic agents, and insurance generally covers these well.
Occasionally, we will have an insurer insist that we prescribe sofosbuvir/ledipasvir
in a patient who is genotype 1A, but this is becoming less common.
The two major barriers to treating hepatitis C are adherence and drug interac-
tions. Overall, it is expected that >75% adherence should allow for viral eradication.
However, missing consecutive doses (vs. sporadic misses) may also have a greater
chance for lack of response. I always stress the importance of adherence at my
teaching appointments and the risk of resistance if noncompliance occurs.
My other major role to support patients being treated is to review medication lists
and make sure there are no drug interactions. In my experience, patients often take
over-the-counter medications or herbal supplements and do not necessarily consider
these true “medications”; hence, they may not disclose them to their doctors to add
them to the medication list. Therefore, if medication lists are not adequately
reviewed, drug interactions may occur causing treatment failure or safety issues. In
terms of drug interactions, one of the most common adjustments that I make is acid-­
reducing medications when sofosbuvir/velpatasvir is prescribed. SOF/VEL requires
gastric acid for absorption. During my consultations, I specifically screen for
famotidine, PPIs, and/or antacids, all of which may decrease the effectiveness of
sofosbuvir/velpatasvir if not dosed or spaced appropriately. Additionally, strong
CYP3A4 inducers, such as carbamazepine, primidone, and phenytoin, can decrease
the drug levels of hepatitis C treatment, thereby preventing viral eradication. So,
prior to starting HCV treatment, prescribers need to weigh the risk versus the benefit
of their continued use.
Hepatitis C medications may also increase the risk for toxicity. For example, we
have heard of patients on pibrentasvir/glecaprevir who have developed myopathy
because pibrentasvir/glecaprevir can significantly raise the concentration of statins
in the blood. Additionally, the use of oral ethinyl estradiol combined with a protease
inhibitor (in pibrentasvir/glecaprevir) can increase the risk of potential liver injury.
Therefore, it is important to screen whether women are taking oral contraceptives
before starting pibrentasvir/glecaprevir.
There is some ongoing controversy about whether an HCV genotype is nec-
essary since we are generally prescribing pan-genotypic medications. What do
you think?
I still find that most insurance companies will not cover HCV meds without a
genotype. In addition, a number of patients who we are treating for HCV are still
174 E. Pieper

struggling with IV or IN drug use, and if they show up with an HCV infection after
receiving treatment, a genotype may help establish if it is a new infection versus a
recurrence of the old infection.

Hepatitis B

Case 5 A 38-year-old Asian American man was being followed for immune-­tolerant
HBV for a decade. However, his latest liver enzymes show an elevation, marking a
transition to immune active disease. His ALT is 150, AST is 130, and he is HBeAg
positive, with an HBV DNA of 100,000.
How are you treating your patients with chronic HBV?
Our patients are prescribed either entecavir or tenofovir because of the higher
barrier to resistance. If we are using tenofovir, I prefer tenofovir alafenamide (TAF)
because the side effect profile is much more preferable than tenofovir disoproxil
fumarate (TDF). Entecavir is generic while TAF is still brand only. TDF is also
generic. A lot of times, however, insurance will dictate what therapy we are
able to use.
However, if I have a young patient, I do not like to use TDF because of the long-­
term side effects. Many of these patients will eventually need to be changed because
of renal problems or osteopenia. So, if we are having problems with insurance
authorizing TAF we will generally use entecavir, although we acknowledge the fact
that tenofovir appears to be associated with lower rates of HCC development.
We are not using much lamivudine or other older HBV antivirals because of the
low barrier to resistance and their side effect profiles.

Pancreatic Insufficiency

Case 6 A 54-year-old man with a past history of years of alcohol abuse carries the
diagnosis of chronic pancreatitis. He presents with weight loss and a history of
frequent fatty stools, and work-up reveals steatorrhea and pancreatic insufficiency.
What agents are available for treating pancreatic insufficiency?
Typically, our first-line agent is Creon, it works well, and, in addition, the manu-
facturer has a robust financial assistance program for those patients who have prob-
lems paying. Pancreatic enzymes can be quite expensive. Our second line is Zenpep.
Often, the insurance will dictate what option we use though. Occasionally, if a
patient is having side effect to one, we can try a different pancrelipase option to see
if the patient tolerates it better
In the past, we would use uncoated pancreatic enzymes like Viokase to treat
the pain of chronic pancreatitis. Are you seeing much of that done currently?
No, we are not seeing much Viokase used, and when it is used, it is generally
because patients have an intolerance to Creon or ZENPEP. I do see a group of
24 GI Pharmacology: 175

patients who do not tolerate these enteric-coated preparations, and it seems like it
more commonly occurs in patients who are also taking PPIs. Creon and ZENPEP
have a pH-mediated release and perhaps the patients on PPIs have more problems
tolerating them because of the more proximal release of the enzymes causing gas-
tric upset.

Inflammatory Bowel Disease

Case 7 A 28-year-old man, who recently quit smoking, presents with severe diar-
rhea and rectal bleeding and is found to have severe ulcerative colitis. Your doctor
wants to start an anti-TNF.
Are you generally pressured to start with biosimilars?
There are an increasing number of insurance companies that do request that we
start with a biosimilar. Additionally, there is potential for cost savings for the patient
and the infusion center when biosimilars are administered because of the lower
acquisition costs compared to originator formulations.
What is your regimen for premedicating patients before they’re given an
infusion?
We have a premedication protocol before administering specific biologics that
includes IV steroids, famotidine, and acetaminophen for patients who had a drug
holiday (for IFX) or who had a history of infusion reaction. Patients on infliximab
receive acetaminophen prior to each infusion. For those with other reactions on
other infusions, it is a case-by-case basis what we may use for premedication going
forward (if the patient continues therapy)
Are there issues, from a coverage standpoint between choosing a drug given
via infusion vs. injection?
Insurance coverage always needs to be considered when choosing a high-cost
medication. Depending on whether the medication is billed through medical versus
pharmacy insurance, it could mean a thousand-dollar difference in out-of-pocket
expenses. For example, patients with Medicare commonly find that infusions (billed
through part B ± supplement) may be less expensive compared to medications billed
through part D (pharmacy benefits). Medicare supplements usually cover the
remaining cost after Medicare part B pays for 80%. However, for SQ drugs such as
adalimumab, the patient needs to pay for their deductible and then must pay up to
25% of the drug cost, which can be thousands of dollars. Additionally, patients with
government-provided insurance do not qualify for copay cards. Depending on the
patient’s income, the patient may not qualify for patient assistance and thus be
unable to get access to a medication.
What about the use of some of the other IBD medications?
We are seeing vedolizumab used in our institution, particularly in patients with-
out extra-intestinal manifestations. Because vedolizumab is gut-specific, I think it is
a worthy drug to use in cases where patients have isolated GI tract Crohn’s or ulcer-
ative colitis. Our doctors have generally turned to vedolizumab after trying
176 E. Pieper

anti-­TNFs first, but it may be used first line depending on specific patient factors.
We very rarely use natalizumab due to the risk of progressive multifocal leukoen-
cephalopathy, especially in those who are JC (John Cunningham) virus-positive.
We are also using more small molecules for IBD, especially in those who may
want to avoid injectable therapies. Tofacitinib also has been used first line as rescue
therapy to avoid colectomy in patients who are hospitalized for severe ulcerative
colitis. There is evidence that high-dose tofacitinib, being given in the hospital over
three days followed by maintenance dosing, may be useful to prevent colectomy.
In terms of one of the newer drugs, ozanimod, we educate and monitor for car-
diac complications since bradycardia is a potential side effect. However, prescribers
also have to be on the lookout for signs of a hypertensive crisis, particularly when
patients eat high amounts of tyramine-containing foods. Patients on ozanimod
should avoid eating things like aged cheeses or consuming dark beers. So, with
ozanimod, a lot of educating and monitoring has to be done, particularly if the
patient has a history of heart issues. You also need to check for macular edema in
those who have diabetes. Ultimately, prescribing ozanimod may require more effort
on behalf of the provider to ensure safety of the patient.
The new Jak-Stat inhibitors (tofacitinib and upadacitinib) are good oral options
for those patients with ulcerative colitis (and Crohn’s disease for upadacitinib) and
want to avoid injectable medications. However, it is important to screen for any his-
tory of thrombosis or cardiovascular issues prior to starting these medications since
there is a warning for thrombosis and myocardial infarctions with this class of
therapy.

Autoimmune Hepatitis

Case 8 A 33-year-old woman presents with weakness, and anorexia and is found to
have an ALT of 340, AST 280, ANA + 1:640, and ASMA + 1:160. A liver biopsy is
consistent with autoimmune hepatitis.
Are most of your providers using prednisone or budesonide to treat AIH?
Are they using azathioprine as well? In what doses? For patients who don’t
tolerate azathioprine, what are they using second-line?
Most of our providers are using prednisone as their first-line agent. Our doctors
are using budesonide more in patients who are having problems being weaned off
prednisone or having intolerable side effects from it.
Most are also adding azathioprine to prednisone for its steroid-sparing effects.
We are seeing most patients receiving azathioprine 50 mg a day. However, if liver
enzymes are elevated or trending up, the dose of azathioprine may be increased. We
may also check 6 TGN and 6 MMP to make sure that the rising LFTs are unrelated
to the medication via the presence of 6 MMP metabolite.
For patients who do not tolerate azathioprine, or do not get the appropriate thera-
peutic response, our second-line agent tends to be mycophenolate mofetil. In terms
24 GI Pharmacology: 177

of problems with mycophenolate, the most important one is that it’s absolutely con-
traindicated in pregnancy. The other is that some patients receiving the mycopheno-
late mofetil may be more prone to side effects (i.e., diarrhea). In these patients,
switching to the other formulation (mycophenolic acid) may eliminate the GI issues.
What are the questions the gastroenterologists in the clinic ask you
most often?
The most common thing the gastroenterologists contact me about is drug interac-
tions. A pharmacist is going to have a wealth of resources to help with navigating
interactions. At the hospital that I work at, there are complex patients with multiple
comorbidities and long lists of medications and supplements. Therefore, choosing
the best treatment can be difficult to ensure efficacy but also minimizing safety
issues. Pharmacists can screen for drug interactions, dose adjustments, and safety
concerns that may affect other health conditions for the patient. Doctors also want
to know which drugs are most likely to be covered or will be most affordable for
patients. I also get questions about supportive care guidelines such as immunization
recommendations since we commonly work with immunosuppressed patients.
Part II
IBD Compendium
Chapter 25
Introduction to the Inflammatory Bowel
Disease Compendium

W. Harley Sobin

There are multiple management decisions we deal with in caring for our IBD
patients. The cases in the chapters that follow cover many complicated issues.
Topics that are discussed include the following: How best to evaluate the remainder
of the small bowel when ileitis is found on colonoscopy? How long should you
continue budesonide? The use of agents such as ozanimod, ustekinumab, risanki-
zumab, and vedolizumab in patients who do not want to be on anti-TNFs; the use of
anti-TNFs in a patient with a history of MI; the use of JAK inhibitors when patients
fail anti-TNFs; whether to anticoagulate a hospitalized patient with IBD and rectal
bleeding; the management of C. difficile in a patient with IBD; how to manage
severe IBD in a patient not responding to anti-TNFs; managing post-op Crohn’s;
managing multiple pseudopolyps and the finding of dysplasia on a random biopsy
during surveillance colonoscopy; options in the management of acute severe ulcer-
ative colitis; treating refractory proctitis; how best to administer vedolizumab?
Managing functional diarrhea in IBD; when is it best to postpone infusions? How to
interpret and manage musculoskeletal complaints in a patient on anti-TNFs for
IBD? How to treat pyoderma gangrenosum? How to manage patients who are
squeamish about self-injection or taking rectal medications? How to treat a patient
with UC who has a colon stricture? How to approach nonspecific ileal ulcers?
Whether to prophylax against Pneumocystis jirovecii in patients on several immu-
nosuppressants; management of microscopic colitis; and help with many other
long-term decisions that need to be made.

W. H. Sobin (*)
Internal Medicine, Division of Gastroenterology and Hepatology, Medical College of
Wisconsin, Milwaukee, WI, USA
e-mail: [email protected]

© The Author(s), under exclusive license to Springer Nature 181


Switzerland AG 2023
W. H. Sobin et al. (eds.), Managing Complex Cases in Gastroenterology,
https://doi.org/10.1007/978-3-031-48949-5_25
Chapter 26
Crohn’s Ileitis

Preetika Sinh

A 25-year-old married woman whose family came to the US from Vietnam when she
was 5 years old presented to her community gastroenterologist with a history of
diarrhea of 2-month duration. Stool cultures were negative for C+S and C diff. She
does smoke cigarettes. She is on no meds and takes no NSAIDS. Fecal calprotectin
was elevated at 600, and CRP was elevated at 30. Colonoscopy was performed and
revealed mild inflammation of the terminal ileum over 5–6 cm. Findings were felt to
be consistent with Crohn’s ileitis.
If you find Crohn’s ileitis, how do you like to evaluate the remainder of the
small bowel?
The options for imaging are CT enterography, MR enterography, and a capsule
study. There are data comparing CT enterography to a capsule exam, and overall,
they are pretty equivalent in making a diagnosis. However, with milder disease, the
CT enterography will occasionally miss smaller ulcerations that will be detected on
capsule study. But, with more severe penetrating, or fistulizing, disease CT is diag-
nostic while the capsule may miss these changes. It is always helpful so see whether
the patient has ever had any cross-sectional imaging previously to serve as a
baseline.
If you decide to do a capsule study in your Crohn’s patients, will you always
do a patency capsule first?
We have to consider whether there are risk factors for capsule retention, which
include any prior history of abdominal surgery, or any obstructive symptoms. If the
answer is no and there has been prior cross-sectional imaging done, then I don’t
think a patency capsule is necessary. Another factor is whether the terminal ileum
(T.I.) was looked at during colonoscopy. If the T.I. has never been looked at, then I

P. Sinh (*)
Division of Gastroenterology and Hepatology, Medical College of Wisconsin,
Milwaukee, WI, USA
e-mail: [email protected]

© The Author(s), under exclusive license to Springer Nature 183


Switzerland AG 2023
W. H. Sobin et al. (eds.), Managing Complex Cases in Gastroenterology,
https://doi.org/10.1007/978-3-031-48949-5_26
184 P. Sinh

would be a little wary of doing a capsule because you are unsure whether there is a
longstanding disease, causing narrowing that might be missed on a CT scan. In this
particular case, the T.I. did not appear strictured, and there was acute onset of symp-
toms, all things that make me believe a capsule should pass without difficulty.
The patient has CT enterography that reveals ileitis but is otherwise negative.
Her community gastroenterologist starts her on entocort (budesonide) 9 mg a day.
Do you agree with the use of budesonide here?
In this setting, I think that is very appropriate because, first, the inflammation
was mild as seen on the colonoscopy. Second, the entocort will help us define how
much response we will get with steroid induction. And third, the entocort, while
working as an induction regimen, sets us up to discuss with the patient what the next
step, maintenance therapy, will be.
The patient does well on 9 mg a day of entocort but every time you try to taper it
the symptoms get worse.
How would you manage this scenario?
I generally plan on using entocort for only 8 weeks. I get the patient back in the
clinic before then to start the discussion about what maintenance drug to use, either
a biologic or immunomodulator, as our long-term treatment for Crohn’s. We are
only using entocort as the induction agent to get the patient into remission. If the
patient remains in remission on the maintenance drug, then entocort can be simply
stopped, it does not have to be tapered.
You would never think about keeping a patient chronically on budesonide
alone, 3 mg, 6 mg, or 9 mg alone if it controls symptoms?
In a Crohn’s patient, I would not maintain them on entocort alone. I have some
patients who are on 3 mg or 6 mg of budesonide in addition to a biologic, who I
can’t wean off of steroids. No, whenever I start entocort, I have an exit strategy that
includes switching to a different drug for maintenance.
Entocort is a steroid, but only 10% gets into the system after absorption and liver
metabolism. Its side effect profile is much better than prednisone. But we still need
to keep in mind that it’s a steroid, and Crohn’s disease is a long-term disease and
assuming that we confirm Crohn’s in this 25 year old, we need to use a steroid-­
sparing medication.
You suggest to the patient that she quit smoking. Have you had much suc-
cess getting your Crohn’s patients to quit smoking?
It depends on the patient population and it requires counseling, plus support with
medication like Chantix or nicotine gum. Overall, I’ve had about 50% success.
When I’m doing all the work myself, I’ve had very little success. Part of the time I
work in the VA, and I’ve had less success with that population.
Is it almost invariably true that your smoking patients who present with
IBD have Crohn’s rather than UC?
I’d like to frame the question a little bit differently. I think there is something of
a referral bias. We get referrals for Crohn’s patients who are doing poorly, and fre-
quently, it is those people who continue smoking. And then, there are patients who
have UC who quit smoking, flare up, do poorly, and then get referred to us. So,
partly it’s because we’re a referral center that we tend to see a lot of Crohn’s patients
who continue to smoke and so their Crohn’s doesn’t get better.
26 Crohn’s Ileitis 185

The patient is unable to quit smoking. She recounts that her mother smoked her
entire life, including through her pregnancy. In addition, she relates that she wishes
to get pregnant within the next 2 years.
Would you consider surgery as a preferred option in this patient?
If you asked me 7–10 years ago, the answer would have been no, we should treat
it medically. If we went back 30 or 40 years ago, the answer would definitely have
been yes, there were not great medical options. But now there are new data that sug-
gest that if a patient has short-segment T.I. disease, confirmed on CT enterography
(like the 5–6 cm this patient has), with no evidence for other disease involvement,
patients may do better with surgery. It has been shown that a surgical resection in
this setting decreases the likelihood of requiring an anti-TNF or other biologic agent
in a patient at a 1-year mark.
However, in this case, surgery would not be my first option. With an acute onset,
no past history of symptoms, and a colonoscopy that shows more inflammatory than
fibrotic disease, I’m going to discuss medical therapy primarily and keep surgical
options much lower on my list.
Every case is unique. This patient’s symptoms seem to be of a short duration.
Another patient may present with mild symptoms, but have a history of GI problems
that have been present for their entire lives. That patient may have had subclinical
Crohn’s for years, and colonoscopy may show more fibrotic change in the T.I. In
that case, the fibrotic component is not going to improve much with a biologic
agent. So, in those situations, I consider short segment T.I. resection and refer them
to a surgeon for their input.
What medical option would you prefer?
Since this is a young patient, given the long-term risks of other biologics, I think
vedolizumab would be a reasonable first choice. There is just one dose for vedoli-
zumab, 300 mg, and we start induction dosing at weeks zero, two, and six and then
continue maintenance dosing every 8 weeks.
After I start vedolizumab, I aim to continue budesonide for another 8 weeks. At
that point, I try to stop budesonide. We either stop it without tapering or decrease to
6 mg for a week then 3 mg for a week, then stop it completely.
Are you checking vedolizumab levels on these patients?
I tend to check vedolizumab levels in patients at the 14-week mark, which is the
first maintenance dose. The reason I check them is because in Crohn’s disease the
induction response to vedolizumab can be slow, and I want to make sure there is
adequate drug available. In Crohn’s, there’s only a 13% response at the end of
induction, it’s much higher in ulcerative colitis. But, after 1 year, the Crohn’s
patients became as responsive to vedolizumab as the ulcerative colitis patients. The
explanation for this is that it takes longer for vedolizumab to act in Crohn’s disease
because it is a transmural inflammation and the mechanism of action of vedoli-
zumab is as an anti-integrin molecule that starts working at the mucosal level.
Previously, I used to evaluate the response at 3 and 6 months with fecal calprotectin
or colonoscopy. If the response was inadequate, I would check vedolizumab levels
and, if levels were low, increase the frequency of the infusions. Now, I tend to check
proactively, rather than have to wait 6 months. In this case, I would repeat fecal
calprotectin, and get vedolizumab levels and a CRP at 14 weeks.
186 P. Sinh

Then, if the vedolizumab level is adequate, and the patient is doing well, all is
good. But if the vedolizumab level is low, the fecal calprotectin is elevated, and the
patient is symptomatic I increase the vedolizumab frequency to every 6 weeks or
4 weeks, depending on the levels and inflammatory markers.
The patient has adequate vedolizumab levels at 14 weeks but as budesonide was
stopped her diarrhea started worsening, and she developed drainage by her rectum,
with a perianal fistula. Colonoscopy is repeated and now she has proctitis along
with inflammation of the cecum and the terminal ileum as well.
How would you manage her now?
It’s important to know that the vedolizumab levels were adequate. On the one
hand, the disease has progressed on vedolizumab; however, it doesn’t necessarily
indicate vedolizumab failure, since it may take longer to see a clinical response to
vedolizumab.
There are several different management options. A shared decision making is the
best way forward. One option is to give vedolizumab a while longer to act. The
patient has an adequate vedolizumab level at 3 months, so you might just continue
the same therapy a while longer. However, the disease has progressed to a point
where the disease is not just mucosal, there is actually a perianal fistula. So, I would
like to switch to a biologic, like an anti-TNF drug. There are much more robust data
for treatment of Crohn’s perianal fistulae with anti-TNFs.
How do you evaluate and manage these perianal fistulae?
I like to get an MRI of the pelvis to assess if there is any abscess. If there is an
abscess, I always start antibiotics-usually ciprofloxacin and metronidazole. I also
ask a surgeon to place a seton in patients with Crohn’s and perianal fistulae. The
management combines medical management of Crohn’s disease, antibiotics, plus
surgical intervention with seton placement or abscess drainage, if needed.
When you see a perianal fistula in a patient with Crohn’s, is it usually an
association with proctitis?
The inflammation associated with a perianal fistula tends to extend into the distal
rectum, right above the dentate line, very close to the anal canal. This really isn’t
Crohn’s proctitis. But you can certainly see perianal fistulae in association with
isolated Crohn’s ileitis or proximal colitis.
The decision is made to place the patient on an anti-TNF for the more
aggressive disease. If you’re starting an anti-TNF, would you like to add an
immunomodulator?
I would probably add an immunomodulator. I would prefer to avoid methotrex-
ate because of the teratogenic risk, seeing how she wishes to become pregnant
However, she is not currently pregnant, so, we could start methotrexate and if we
can get the patient into remission switch to another immunomodulator, like azathio-
prine, which we are comfortable using in pregnancy [1]. We could also consider a
different biologic with evidence in treating Crohn’s fistulizing disease that does not
require an immunomodulator.
However, you don’t always need to add a second drug when you’re using an anti-­
TNF. Although the initial Sonic trial showed that combination drug therapy is better
than anti-TNF alone, when they looked at drug levels of those patients who were on
26 Crohn’s Ileitis 187

optimized anti-TNF dosage, they did as well as the combination therapy. Therefore,
you can also argue for using an anti-TNF alone, with optimized dosing, closely fol-
lowing drug levels.
There is another new blood test that we can use to predict how patients will do
on anti-TNF monotherapy. It is called RiskImmune (Prometheus lab), which uses
the HLA phenotype of the patient. It’s been shown to predict, at the 3-year mark,
which patients with the genetic variant have a higher risk of progressing and requir-
ing surgery on anti-TNF monotherapy.
So, I would consider sending this test, with the disclaimer that it is a very new
test, and if the patient does have that risk profile, then we will definitely add a sec-
ond agent. It has been shown that if you use combination therapy in those patients,
they have a better outcome. They have a higher risk of immunogenicity on
monotherapy.
Because the disease has progressed quickly, I would be more inclined to add an
immunomodulator, particularly since the patient is young. If I do add an immune
modulator, I generally keep that on for about 1–2 years. I have a plan of switching
over to monotherapy at that point, because of the risk of lymphoma and skin cancer,
with the immune modulator. Those are the main, big words that people get scared of,
but they don’t come into the picture until the patient has been on immunosuppres-
sant medications like azathioprine for 2–3 years. So, I would consider checking the
RiskImmune, start an anti-TNF, and add an immunomodulator like Imuran for now.
If you’re going to add azathioprine, would you do genetic testing for NUD 15
because she is Asian?
If I was practicing in Asia, I would definitely check a NUD-15 because any
abnormality can cause toxicity at the 6TG metabolite level. I’m not sure how com-
mercially available the test is. If it is not available, I would certainly still check the
TPMT phenotype, and closely monitor the CBC and liver panel.
The medical team suggests treatment with infliximab and azathioprine but the
patient does not wish to come in for infusions.
Would you allow the patient to get adalimumab or would you push the
patient to receive infliximab?
I think that shared decision making is very important. I tell the patient that I
would rather she be on the medication which she will stay on, rather than me trying
to tell her to take a different drug which she won’t comply with, which will only
make her disease worse. For some patients, there are other priorities that come
before treating their disease, so, we always try to be flexible, to help the patient stay
on track. So yes, I would be inclined to work with adalimumab if infusions are not
going to work with her lifestyle. In addition, the original Humira studies do show
that adalimumab is a good drug for perianal fistulizing disease.
However, another important point to make is that in perianal Crohn’s disease we
need to have higher anti-TNF trough levels. The usual desired level is 5 mg but with
perianal disease we want to get levels of 10–15 or even 20 ug/ml. It can be more
challenging to achieve these drug levels using adalimumab. With infliximab, we can
go up to 10 mg/kg every 4 weeks. With adalimumab, all we can do is increase the
frequency to q weekly.
188 P. Sinh

Reference

1. Mahadevan U, Robinson C, Bernasko N, Boland B, Chambers C, Dubinsky M, Friedman S,


Kane S, Manthey J, Sauberan J, Stone J. Inflammatory bowel disease in pregnancy clinical care
pathway: a report from the American Gastroenterological Association IBD Parenthood Project
Working Group. Inflamm Bowel Dis. 2019;25(4):627–41.
Chapter 27
Ulcerative Colitis Refractory
to Mesalamine

Daniel Stein and Salina Faidhalla

A 35-year-old woman with mild ulcerative colitis is not responding to mesalamine.


The patient is a non-smoker with no family history of IBD and no other medical
problems.
How do you manage a patient with mild UC who is not responding to
mesalamine?
When there is a concern for no response/loss of response, there must be objective
evidence of disease activity since many IBD patients can have nonspecific symp-
toms due to IBS, rectal scarring, etc. Loss of response can be defined by a worsen-
ing of clinical status, and evidence of active disease based on colonoscopy or at least
inflammatory markers. After confirming disease activity, we always confirm medi-
cation adherence and rule out other possible etiologies, like infection.
Other options for mild UC include maximizing mesalamine dose if not already
done, adding topical mesalamine or topical steroids (in enema, rectal foam, or sup-
pository form), to induce remission prior to escalating to a biologic. In patients with
only mild UC, we can also consider adding curcumin or fish oil.
If the above fails, then we can proceed with evaluation for other treatment options
like anti-TNFs, other biologics, or the new small molecules.
The dosage of mesalamine is maximized and mesalamine enemas are added. In
spite of this, there is no response and alternative therapy is needed. She has heard
about the anti-TNFs and wants to avoid starting those.

D. Stein (*)
Internal Medicine, Division of Gastroenterology and Hepatology, Medical College of
Wisconsin, Milwaukee, WI, USA
e-mail: [email protected]
S. Faidhalla
Department of Medicine Division of Gastroenterology and Hepatology, Medical College of
Wisconsin, Milwaukee, WI, USA
e-mail: [email protected]

© The Author(s), under exclusive license to Springer Nature 189


Switzerland AG 2023
W. H. Sobin et al. (eds.), Managing Complex Cases in Gastroenterology,
https://doi.org/10.1007/978-3-031-48949-5_27
190 D. Stein and S. Faidhalla

Since she wants to take anti-TNFs off the table, can you discuss the choice
between ozanimod, Ustekinumab, and vedolizumab in UC?
Vedolizumab (VDZ) binds α4β7 integrin on blood monocytes, inhibiting their
ability to enter the intestinal epithelium. It is more gut selective. VDZ is an infusion
administered every 8 weeks and an effective choice in UC patients who are anti-­
TNF naïve.
Ustekinumab (UST) is an interleukin inhibitor, targeting IL-12 and IL-23.
Induction is an infusion followed by self-administered injections every 8 weeks. It
is an effective medication in UC patients who have failed anti-TNFs, some data sug-
gest that it is more effective in inducing and maintaining remission than VDZ.
Ozanimod is a selective sphingosine-1-phosphate receptor modulator that has
been used for relapsing MS and was recently approved in patients with moderate to
severe UC. The medication is in pill form with once-daily dosing. This is a newer
medication with some limited data. However, it seems effective in patients with
mild to moderate UC who are anti-TNF naïve. Some of the known side effects of
ozanimod include infection, malignancy, macular edema, bradycardia, and ele-
vated LFTs.
For the three treatment options outlined, there is no head-to-head comparison
study, but a meta-analysis suggests that ustekinumab could be more effective than
vedolizumab in inducing and maintaining remission in UC patients [1]. Limited
data are available about ozanimod especially when comparing it to other biologics.
When it comes to choosing a treatment, we recommend discussing with the
patient in detail available treatment options, their efficacy, side effects, and route of
administration to reach a mutual decision.
In this young patient with mild UC, no previous anti-TNF exposure, and no
chronic medical problems, ozanimod would be a reasonable choice. The fact that it
is an oral med is a benefit. Vedolizumab would also be a reasonable option to start
with, given its efficacy and favorable side effect profile. Ustekinumab is also a
valid option.
What is involved with starting a patient on ozanimod?
Ozanimod is a safe, effective pill that can be used in mild-moderate UC patients
who are TNF naive. During the induction and maintenance period of the ozanimod
trial, the incidence of clinical remission was significantly higher in the ozanimod
group when compared to the placebo group [2]. Improvement in the incidence of
histologic remission also occurred with ozanimod therapy.
When deciding to start ozanimod, you need to document vaccination or immu-
nity to varicella zoster virus prior to starting the medication. In addition, you need
to obtain an EKG, and ask about any history of uveitis or DM, as these patients
might need to see an ophthalmologist.
In women of reproductive age, it’s important to remember that there are no data
about ozanimod and pregnancy, and there is possible risk of teratogenicity in animal
studies. We must discuss if there are any plans for pregnancy and if this patient is
planning to become pregnant, we recommend against using ozanimod.
Ozanimod dosing: starter pack (7 days), Days 1-4 (0.23 mg), Days 5-7 (0.46 mg).
Day 8 patients start the maintenance dose of 0.92 mg daily.
27 Ulcerative Colitis Refractory to Mesalamine 191

Some of the reported side effects of ozanimod are infections, malignancy, macu-
lar edema, bradycardia, and elevated LFTs. When it comes to bradycardia, ozani-
mod has been shown to have a lower risk of bradycardia on long-term follow than
some other drugs in this class. This may be due to the selective nature of the drug
since it targets S1P1 and S1P5 (notice that S1P 1-3 is highly expressed in the heart).

References

1. Welty M, Mesana L, Padhiar A, Naessens D, Diels J, van Sanden S, Pacou M. Efficacy of


Ustekinumab vs. advanced therapies for the treatment of moderately to severely active
ulcerative colitis: a systematic review and network meta-analysis. Curr Med Res Opin.
2020;36(4):595–606.
2. Sandborn WJ, Feagan BG, D’Haens G, Wolf DC, Jovanovic I, Hanauer SB, Ghosh S, Petersen
A, Hua SY, Lee JH, Charles L. Ozanimod as induction and maintenance therapy for ulcerative
colitis. N Engl J Med. 2021;385(14):1280–91.
Chapter 28
Managing Crohn’s in a Patient with Prior
MI

Preetika Sinh

Patient is a 58-year-old male smoker with a history of myocardial infarction 2 years


earlier, who presents with bloody diarrhea. There is a strong family history of coro-
nary artery disease. Stool cultures are negative. Colonoscopy is performed and
demonstrates moderately severe colitis with ulcers involving the entire colon. The
terminal ileum is normal. It is interpreted to be an indeterminate colitis.
Is there any role for using ANCA or ASCA antibodies in the diagnosis of
indeterminate colitis?
I do not use ASCA/ANCA for diagnosis. I think we get more information from
the clinical picture alone. But there is a role for ASCA and ANCA, in those patients
who will require colectomy. The risk of pouchitis and pouch failure is much higher
in patients who are ANCA positive.
Although it was an indeterminate colitis, it was felt clinically to be favoring
Crohn’s colitis.
Is there any role for mesalamine or sulfasalazine in milder Crohn’s colitis?
I have only one or two patients on these drugs, I rarely start Crohn’s patients on
mesalamine. However, if I had a patient with mild Crohn’s colitis who was not yet
ready to start a biologic, I might start them on mesalamine. If someone is already on
mesalamine and in remission, I would not insist on making changes.
The patient starts developing increased diarrhea and bleeding and is hospital-
ized for LGI bleeding, presumably related to colitis alone. His hemoglobin drops
from 12 to 10.

P. Sinh (*)
Division of Gastroenterology and Hepatology, Medical College of Wisconsin,
Milwaukee, WI, USA
e-mail: [email protected]

© The Author(s), under exclusive license to Springer Nature 193


Switzerland AG 2023
W. H. Sobin et al. (eds.), Managing Complex Cases in Gastroenterology,
https://doi.org/10.1007/978-3-031-48949-5_28
194 P. Sinh

You have a patient with Crohn’s disease, and a history of an MI who’s in the
hospital with GI bleeding. Would you be willing to use lovenox for DVT
prophylaxis?
Yes, we face this problem frequently. I will place the patients on lovenox unless
they develop a more significant bleed, which we define as a drop in hemoglobin of
two grams. In the case outlined here, I would hold the lovenox because there was a
2-point hemoglobin drop. In any other situation, it's very important to stay on
lovenox, even if they have some amount of bleeding, because there is a sixfold
increased risk of venous thromboembolism in patients who have inflammatory
bowel disease, and the risk is even higher in those who have an active flare
So, in those other cases I will start lovenox, while keeping a close eye on the
hemoglobin. If there was a significant Hbg drop, I would hold the lovenox but oth-
erwise emphasize very emphatically to the primary team that patients should remain
on lovenox.
Would you be willing to start an anti-TNF in this patient with a history
of an MI?
Yes. The contraindication to using an anti-TNF in heart disease is heart failure.
There’s a black box warning from the FDA saying that anti-TNFs should not be
used in patients with NYHA classification three or four heart failure.
This is based upon the ATTACH study, which looked to see if anti-TNFs could
be used to treat heart failure because TNF levels have been shown to be increased in
heart failure. However, in patients taking 10 mg/kg of the anti-TNF there was a
higher mortality in CHF, leading to this black box warning.
However, this patient has a history of MI, but no history of clinical heart failure.
Studies have not shown any increased mortality in MI so; therefore, this would not
be a contraindication. Overall, the data on anti-TNFs in other chronic inflammatory
disorders show that they might be beneficial to reduce cardiovascular events over
time, in those patients who don’t have advanced CHF.
You are thinking this indeterminate colitis is favoring Crohn’s disease. How
would you like to manage that?
So, for Crohn’s colitis I would go with steroid induction first to get the disease
into remission. And then, I would probably use an anti-TNF for maintenance.
How would you feel about using ustekinumab instead of an anti-TNF in this
patient?
The patient is 58. If he was 65, or a 70-year-old, I would be more concerned
about a higher risk of infection with anti-TNFs compared to ustekinumab in this
older age range. But this patient is younger and is admitted for acute care and there
is a lot of experience using anti-TNFs as the first line in hospitalized patients.
Have I started patients on ustekinumab in the hospital with an induction? Yes,
primarily in patients who have had a prior anti-TNF failure.
The patient is placed on an anti-TNF, has quit smoking, and does very well for
about a year. But then the patient starts developing diarrhea and bleeding. You
check stool cultures for C diff and his NAAT is positive, but his toxin is negative.
28 Managing Crohn’s in a Patient with Prior MI 195

How would you manage this?


First, I would try to go back and check if the patient ever had a c. diff check
before, and usually we do check c. diff at the time of diagnosis. So that means that
this patient, at some point in time, was not colonized by c diff.
But, now the c diff NAAT is positive. This could either mean colonization or
active disease. But c diff in IBD behaves differently than in non-IBD patients. I
think you have to look at the clinical picture here. We have a situation where the
patient flares and now the c diff NAAT is positive. So, even though the toxin is nega-
tive, I would treat this patient as if he has active c diff.
The next question is how would we categorize his c diff? Is it fulminant, severe,
or not severe? In severe c diff, the white blood cell count is >15,000 and the creati-
nine is >1.5. Fulminant infections are associated with severe hypotension or shock
or toxic megacolon. Non-severe infections can be treated with vancomycin 125 mg,
four times a day for ten days or fidaxomicin 200 mg, twice a day for ten days.
Fulminant infections are treated with vancomycin 500 mg qid PO and IV metroni-
dazole. Severe infections fall between the two.
The patient is clinically deteriorating and is hospitalized. Vancomycin 500 mg po
QID and IV metronidazole are started. Colonoscopy is performed and he has a
moderately severe diffuse colitis, looking like ulcerative colitis.
The patient had an indeterminate colitis that was originally diagnosed as
favoring Crohn’s disease. Since then, he quit smoking and clinically worsened.
Do you ever change your categorization, your diagnosis, from Crohn’s to UC?
When I said that the colitis was indeterminate but favored Crohn’s over UC, it
was with the understanding that there were never any granulomas on biopsy, there
were never skip lesions, and there was never any extra-colonic disease. Had these
things been present, it wouldn’t be labeled as indeterminate colitis. So, it was never
definite Crohn’s disease. And now, the patient has quit smoking, and the fact that the
disease flared when he quit smoking is more consistent with an ulcerative colitis
phenotype. What is the significance of changing the diagnosis? This largely relates
to the drugs you would use and the prognosis if you were to perform a colectomy.
So, the patient has been relapsing in spite of being on an anti-TNF. Do you
think he is a candidate for a JAK inhibitor, is this permissible in this setting?
After all, the original diagnosis was Crohn’s, where JAK inhibitors are not
approved, and he has a coexisting c. diff infection. Do we have any idea how
JAK inhibitors perform in this setting?
He is a complicated patient, and we do have to keep the whole concept of active
c diff infection in mind. I would add to the management options possibly treating
his c diff with an FMT. It is not at all unusual that when a patient with IBD develops
c diff the c diff flares up the colitis. We are frequently faced with bumping up the
treatment regimen for UC as well as trying to get the c diff under control. We have
to be cautious with our management of the UC, however. There have been patients
maintained on high doses of steroids who were not tapered off prednisone at the
right time, and their c diff infections flared and they underwent colectomies with a
poor outcome.
196 P. Sinh

But if we are treating the c diff and doing steroid induction and the patient is not
responding, we can say that anti-TNF treatment is a failure. Then, we can switch to
a JAK inhibitor. The benefit of JAK inhibitors is that they are small molecules and
their response is not dependent on the albumin. Acutely ill patients in the hospital
are more likely to have a low albumin level. There are data showing that when there
is a low albumin level those patients are less responsive to anti-TNFs. This does not
hold true for JAK inhibitors. And in treating acute (fulminant) UC, the data are bet-
ter for using a higher dose of tofacitinib, 10 mg, three times a day for induction and
as a rescue therapy, when anti-TNFs don’t work.
Chapter 29
Managing Post-Op Crohn’s

Amir Patel

A 57-year-old male is admitted with a bowel obstruction. He has no past history of


IBD and has not had a prior colonoscopy. He ends up requiring surgery for the
obstruction and is found to have an ileal stricture, which is resected, and found on
pathology to be Crohn’s. The patient is a non-smoker. There is no family history of
Crohn’s.
Does small bowel obstruction in Crohn’s usually occur at the terminal ileum?
Yes, it is usually at the terminal ileum. I’ve had one or two where they were more
proximal, usually in a younger patient who had a duodenal or jejunal stricture. But
that’s exceedingly rare. About 33% of patients with Crohn’s will have just isolated
ileal disease.
He’s never had a colonoscopy. When would you perform that?
I would do it a month post-op. You really want to assess what their phenotype is.
You want to make sure they don’t have colonic involvement. For someone who has
had prior colonoscopies, we would not repeat it at 1-month post-op; instead, we
typically repeat the colonoscopy about 6 months to a year after surgery, because
that’s when we would see some type of recurrence.
One-month post-op you do a colonoscopy and find a normal appearing colon
and distal 20 cm of ileum.
Would you put this patient on any medication for Crohn’s at this time?
I would say he’s probably low risk. He is a non-smoker and he’s older. This is his
first operation and theoretically it’s a short, stricture. I don’t know how long he’s
had his Crohn’s disease; he might have had smoldering Crohn’s disease since his
twenties and it’s just progressed to this point.

A. Patel (*)
Dept. of Medicine, Division of Gastroenterology and Hepatology, Medical College of
Wisconsin, Milwaukee, WI, USA
e-mail: [email protected]

© The Author(s), under exclusive license to Springer Nature 197


Switzerland AG 2023
W. H. Sobin et al. (eds.), Managing Complex Cases in Gastroenterology,
https://doi.org/10.1007/978-3-031-48949-5_29
198 A. Patel

The high-risk individuals who I think really need biologics and thiopurines to be
started postoperatively are the smokers, the patients that are younger than 30, those
with a history of fistulizing disease, and those with a history of two or more surger-
ies or who have a shorter duration of disease prior to surgery.
If I have a patient who says, “I started having diarrhea about a month ago, and all
of a sudden, I have a bowel obstruction,” then I would say that person’s at very high
risk. For our patient, I’m not that concerned. Some practitioners do order genetic
testing to see if their patient has a higher risk for postoperative recurrence, checking
the NOD-2 or CARD-15 genes. Some of us will do that to see if the patient is at a
higher risk for a recurrence. But for this gentleman, I think that his risk of recur-
rence is pretty low. You could potentially get away with not starting him on therapy
right away and wait to see what the postoperative colonoscopy shows at 6 months
to a year.
Chapter 30
Managing Pseudopolyps

Poonam Beniwal-Patel

A 45-year-old male with a 12-year history of ulcerative colitis is referred to you. His
personal gastroenterologist performed a recent colonoscopy which once again
revealed dozens of pseudopolyps scattered through the colon (last colonoscopy was
3 years earlier—with similar finding). None of these polyps looked like a conven-
tional adenoma. Random biopsies were done throughout the colon and a few pseu-
dopolyps were removed. The patient is referred to you because biopsies done in the
right colon and a pseudopolyp in the transverse colon both showed dysplasia. The
referring doctor did not anticipate these biopsy results and is referring the patient
asking you how to manage this.
What is your standard approach to surveillance in patients with chronic UC?
In general, I begin dysplasia surveillance after a patient has had pan-ulcerative
colitis for 8 years or left-sided colitis for 10 years. In patients who have been in
sustained deep remission, without a history of colon dysplasia or large post-­
inflammatory polyps (PIP), I generally survey the colon every 3–5 years. In con-
trast, if a patient has had extensive disease or large PIPs, I bring them back in 2–3
years. Finally, those patients with concurrent primary sclerosing cholangitis are
brought back annually for dysplasia surveillance.
In patients with multiple pseudopolyps are there any tips you have to detect
which are true adenomas? Do you bring them in more often for colonoscopic
surveillance?
I use a combination of endoscopic appearance and narrow-band imaging (NBI) to
decide whether I am encountering a PIP versus an adenoma. The endoscopic appear-
ance of a fibrin cap along with distinct appearances such as a mucosal bridge formed
by a long polyp are more consistent with a PIP and do not need to be biopsied.

P. Beniwal-Patel (*)
Medicine, Gastroenterology/Hepatology Division, Medical College of Wisconsin,
Milwaukee, WI, USA
e-mail: [email protected]

© The Author(s), under exclusive license to Springer Nature 199


Switzerland AG 2023
W. H. Sobin et al. (eds.), Managing Complex Cases in Gastroenterology,
https://doi.org/10.1007/978-3-031-48949-5_30
200 P. Beniwal-Patel

One of the first natural history of PIP studies was published in 2019 [1]. About
500 patients with PIPs were followed for a median of about 5 years. This group was
compared with patients without PIPs to evaluate the rate of developing colorectal
neoplasia. There was no difference between these 2 groups. A more recent study
confirmed these findings [2]. I survey patients with large PIPs closer and generally
bring them back in 2–3 years for repeat colonoscopy.
It is important to carefully survey for flat polyps because these adenomatous
polyps represent a more aggressive type of polyp in the setting of IBD. I utilize NBI
to evaluate the pit pattern of a polyp and to delineate the borders of flatter lesions. It
is important to biopsy the mucosa around such a polyp and place it in a separate
bottle to evaluate for active IBD.
If you’re removing one of these larger pseudopolyps, will you use cold snare
or hot snare?
I generally do not remove PIPs because of the increased bleeding risk. If I do
need to biopsy one, I often need to apply a hemoclip to achieve hemostasis.
What do you do when a random biopsy in a patient with chronic UC shows
dysplasia?
First, I look at the pathology report to determine the type of dysplasia: low vs
high grade vs. indeterminate. Low-grade/indefinite dysplasia can be indistinguish-
able from active inflammation in ulcerative colitis. In this scenario, if there is con-
current active disease, I would adjust therapy and bring the patient back in 6 months
for a repeat colonoscopy.
In the case of high-grade dysplasia, if these were truly random biopsies without
a focal lesion, I would repeat a colonoscopy with chromoendoscopy.
So, you are generally using NBI to look at the pit pattern. When will you
actually do chromoendoscopy where you are spraying the mucosa?
The pendulum on whether and when to use chromoendoscopy keeps changing,
each year. It is certainly warranted when there is high-grade dysplasia on random
biopsies to better delineate if there’s a focal lesion that can be intervened upon. In
patients with concurrent PSC, chromoendoscopy can also be used to especially sur-
vey the right colon.

References

1. Lewis AE, Kirchgesner J, Dray X, Svrcek M, Beaugerie L. 305 Clinical significance of pseudo-
polyps for patients with inflammatory bowel disease. Am J Gastroenterol. 2019;114:S179–80.
https://doi.org/10.14309/01.ajg.0000590752.06064.e8.
2. Wolf T, Lewis A, Beaugerie L, Svrcek M, Kirchgesner J, Network S-AIBD. Risk of colorectal
neoplasia according to histologic disease activity in patients with inflammatory bowel dis-
ease and colonic post-inflammatory polyps. Aliment Pharmacol Ther. 2023;57(12):1445–52.
https://doi.org/10.1111/apt.17495. Epub 2023 Mar 31.
Chapter 31
Ulcerative Colitis Refractory to Anti-TNF

Daniel Stein and Salina Faidhalla

A 38-year-old man with moderately severe ulcerative colitis is not responding to


infliximab. You are considering switching to a JAK inhibitor. He is mildly obese, a
non-smoker and has no other medical problems.
How effective have you found the JAK inhibitors to be in patients who have
failed anti-TNFs?
Currently, the FDA has approved two different JAK inhibitors, tofacitinib, a JAK
1-3 inhibitor, and upadacitinib, a JAK 1 inhibitor for use in moderate to severe UC
patients who failed anti-TNFs. Both medications have been found to be effective in
induction and maintenance of remission in patients with moderate to severe UC
who failed other biologics including anti-TNF agents,
With tofacitinib, there is a rapid decrease in stool frequency and rectal bleeding,
generally within days. The recommended induction dose is 10 mg BID for 8 weeks,
which can be continued for another 8 weeks if there is no initial response. This is
then followed by a maintenance dose of 5 mg BID. A maintenance dose of 10 mg
BID can be used in patients with severe disease and in patients having failed anti-­
TNF therapy.
Upadacitinib has been recently approved for moderate to severe UC. It also has
a rapid clinical response. The recommended induction dose is 45 mg daily for 8

D. Stein (*)
Department of Internal Medicine, Division of Gastroenterology and Hepatology, Medical
College of Wisconsin, Milwaukee, WI, USA
e-mail: [email protected]
S. Faidhalla
Department of Medicine Division of Gastroenterology and Hepatology, Medical College of
Wisconsin, Milwaukee, WI, USA
e-mail: [email protected]

© The Author(s), under exclusive license to Springer Nature 201


Switzerland AG 2023
W. H. Sobin et al. (eds.), Managing Complex Cases in Gastroenterology,
https://doi.org/10.1007/978-3-031-48949-5_31
202 D. Stein and S. Faidhalla

weeks followed by 15 mg daily maintenance dose. A higher maintenance dose of


30 mg daily can be used in patients with a higher disease burden or in those that
have failed other advanced therapies.
Both medications are very effective in patients with moderate to severe disease
who have failed TNF inhibitors, making them valuable treatment options in UC
patients. Overall, the efficacy of both medications has been shown to be dose depen-
dent with higher doses associated with higher response rates.
In our practice, tofacitinib efficacy has been comparable to the efficacy in clini-
cal trials reaching up to 20% remission rates. The response rates for upadacitinib
have been even higher, presumably because the medication is more selective.
In the OCTAVE induction trials, remission rates with tofacitinib were up to
18.8% compared to 8.2% in the placebo group [1]. And in the OCTAVE sustain
trial, maintenance of remission rates at 52 weeks was 34% in the 5 mg group and
40% in the 10 mg group compared to only 11.1% in the placebo group.
For upadacitinib, in the UC1 trial, remission at 8 weeks was 26%, and in the UC2
trial, 8-week remission rate was 33% (higher than rates of tofacitinib). In the UC 3
trial [2], clinical remission at week 52 was achieved in 42% of patients receiving
upadacitinib 15 mg once daily and in 52% of patients receiving upadacitinib 30 mg
once daily.
When it comes to positioning these new medications, it appears that upadacitinib
is more effective than tofacitinib, but this has yet to be validated in a head-to-­
head trial.
With JAK inhibitors there is no immunogenicity. There is an FDA black box
warning about MACE events and thrombosis risk, based on observations in an RA
population. MACE and VTE events were not seen more commonly in the trials
studying ulcerative colitis.
In clinical practice, lowering the dose sometimes results in flare-ups, which we
can usually overcome by increasing the dose. Although the lower maintenance dose
is preferred, patients have been comfortable with continuing higher doses when
needed to achieve and maintain remission.
Tofacitinib has been found helpful in treating acute severe ulcerative colitis
(ASUC) in inpatients. We know that the most studied and effective options for
ASUC are infliximab and cyclosporine. When a patient with ASUC has failed inf-
liximab therapy in the past, tofacitinib has worked as a rescue therapy, knowing that
despite rescue therapy up to 30% of patients with ASUC end up requiring colectomy.
In a retrospective case control study by Berinstein et al. [3], 40 patients, 85% of
whom failed IFX in the past, received tofacitinib in addition to IV steroids for
ASUC. These patients had a significantly lower risk of colectomy at 90 days when
compared to the controls, with no increased risk of infection, VTE, or cardiovascu-
lar events in the 90-day period.
This suggests that high-dose tofacitinib plus IV steroids may be considered for
treatment of high-risk biologic exposed patients admitted with ASUC.
31 Ulcerative Colitis Refractory to Anti-TNF 203

If insurance will allow either drug, would you prefer to go with tofacitinib
or upadacitinib?
There have been no head-to-head trials yet, but upadacitinib has shown higher
absolute clinical remission rates during induction and maintenance in cross trial
comparisons. In terms of side effects, upadacitinib was found to cause nasopharyn-
gitis, and we see an increase in lipid levels, although usually not significant enough
to stop treatment. Cardiovascular risks and increased VTE/PE risks in patients over
age fifty have been documented in RA patients but not yet in the IBD population.
Other risks include malignancies, particularly non-melanoma skin cancers.
We suggest using caution with these medications in older patients who have
underlying cardiovascular disease or a history of VTE. However, we would still
consider using them in patients with severe ulcerative colitis who failed other bio-
logics because the risk of complications from active disease usually outweighs the
risk of medication side effects. Also, if patients do have a history of VTE/PE and
they are already anticoagulated, then we feel that you can safely start JAK inhibitors.

References

1. Sandborn WJ, Su C, Sands BE, D’Haens GR, Vermeire S, Schreiber S, Danese S, Feagan BG,
Reinisch W, Niezychowski W, Friedman G. Tofacitinib as induction and maintenance therapy
for ulcerative colitis. New Engl J Med. 2017;376(18):1723–36.
2. Danese S, Vermeire S, Zhou W, Pangan AL, Siffledeen J, Greenbloom S, Hébuterne X, D’Haens
G, Nakase H, Panés J, Higgins PD. Upadacitinib as induction and maintenance therapy for
moderately to severely active ulcerative colitis: results from three phase 3, multicentre, double-­
blind, randomised trials. Lancet. 2022;399(10341):2113–28.
3. Berinstein JA, Sheehan JL, Dias M, Berinstein EM, Steiner CA, Johnson LA, Regal RE,
Allen JI, Cushing KC, Stidham RW, Bishu S, Kinnucan JAR, Cohen-Mekelburg SA, Waljee
AK, Higgins PDR. Tofacitinib for biologic-experienced hospitalized patients with acute
severe ulcerative colitis: a retrospective case-control study. Clin Gastroenterol Hepatol.
2021;19(10):2112–2120.e1. https://doi.org/10.1016/j.cgh.2021.05.038. Epub 2021 May 25.
PMID: 34048936; PMCID: PMC8760630.
Chapter 32
Acute Severe Ulcerative Colitis

Amir Patel

A 34-year-old man presents with severe exacerbation of his ulcerative colitis. He


was diagnosed with ulcerative colitis at age 30 and responded well to infliximab. He
moved out of state for a new job, which involved doing a lot of traveling. He did not
want to travel home and miss work to have infusions, and so he let them lapse.
You see him for the first time after he gets admitted. He has been having bloody
diarrhea about 15 times a day. The diarrhea started 2 weeks earlier but is getting
progressively worse. He is acutely ill, with a temp of 101, a BP of 90/60, and his
heart rate is 110. His WBC is 19,000; his CRP is 85. Stool cultures are negative. You
place him on IV steroids. An unprepped flex sig confirms severe colitis. After 3 days
of steroids, he is still doing poorly. You get a surgical consult but decide to try a
rescue therapy.
How do you manage acute severe UC in this case?
For this patient, there are three options. If he’s in the hospital setting you can
retry the infliximab at a high dose and see if he responds after 72 h. You could try
cyclosporine. However, many of us are not using cyclosporine as much because
there is a lot of monitoring to do and you can't give it to patients who have high
blood pressure, renal disease, seizures, electrolyte abnormalities, or low albumin.
Another problem with cyclosporine is that you can use it in the hospital setting,
but then you have to have some type of end game. What are you going to put him on
once he gets out of the hospital? For this particular patient, I would probably try the
infliximab, and if it doesn't work, then I would consider another therapy or surgery.

A. Patel (*)
Dept. of Medicine, Division of Gastroenterology and Hepatology, Medical College of
Wisconsin, Milwaukee, WI, USA
e-mail: [email protected]

© The Author(s), under exclusive license to Springer Nature 205


Switzerland AG 2023
W. H. Sobin et al. (eds.), Managing Complex Cases in Gastroenterology,
https://doi.org/10.1007/978-3-031-48949-5_32
206 A. Patel

The other therapy that has good data and a lot of promise is high-dose tofacitinib.
This is for in-patients with severe UC, you give them 10 mg of tofacitinib three
times a day. The tricky part with that is that after they leave the hospital, you need
to make sure that they can continue on the Xeljanz on an outpatient basis.
But there are some robust data on using tofacitinib 10 mg three times a day,
which is a high dose, to treat acute severe ulcerative colitis. We’ve had a couple of
patients in the hospital recently where we were able to use it, and my colleague
recently had a case where she tried to give Remicade rescue therapy but the repeat
flex sig failed to show improvement. She gave high-dose tofacitinib, and miracu-
lously, after 2 days, there was significant mucosal healing.
If you're giving the infliximab rescue, what dose would you give, would you
pretreat with steroids, and would you give azathioprine along with the
infliximab?
You're concerned about antibodies, but this patient’s already on steroids. So, you
don’t really need to pretreat them. In terms of the dose of infliximab, it depends on
the patient’s albumin. If the albumin is low, I would probably give the 10 mg per
kilogram dose of Remicade in the hospital setting.
I would plan on adding an immune modulator in the future, but we all know that
it takes some time for that to build up in the system. But if it works for them, I would
keep them on combination therapy going forward.
I would not necessarily start the immune modulator in the hospital. We’re not
sure which way he’s going to go with the treatment, with the infliximab. I would
probably start it a few days later, if they do respond to the infliximab.
If they respond to infliximab, when are you giving the second dose and the
third dose?
It depends how they do clinically. If they do fine, and you’re able to get them off
of IV steroids, then I would just restart the induction doses at 0, 2, and 6 weeks. If
they’re in the hospital and they’re having a partial response, some reduction in stool
frequency and bleeding, sometimes I will give the second dose earlier. I might give
it a week later or even within the same week.
The tricky part always is insurance coverage once they leave the hospital. But a
lot of times we have a good prior authorization team that helps us with that.
Chapter 33
Resistant/Refractory Proctitis

Poonam Beniwal-Patel

A 38-year-old man is sent to you for a second opinion regarding proctitis. The
patient started having bloody diarrhea and tenesmus 1 year earlier. His gastroen-
terologist performed a colonoscopy that revealed moderate severity proctitis, with a
normal appearing colon proximal to 15 cm. Biopsies were consistent with ulcer-
ative colitis. He was started on oral mesalamine and rowasa enemas and had only
minor clinical improvement and is disappointed and frustrated. He wants to talk to
an academic expert regarding his case.
In treating proctitis how do you decide whether to use suppository,
enema, foam?
I actually tend to use suppositories very little for two reasons. One is that I’m not
convinced that they have great distribution in the rectum, certainly compared to
enemas or foams. And, anecdotally, I think the patient sometimes has a harder time
holding in a small suppository versus the enema with the tip, or the foam where you
spray it in.
The one occasion where I do use a suppository is if someone has a pouch, and
they have cuffitis. Otherwise, I tend to use either an enema or the foam. I particu-
larly like the foam; I think it’s very user friendly. Patients can just spray the foam
into the rectum and hold it and that way they don’t have to mix the solution, hold the
catheter in there, and distribute the solution the way you do with your tradi-
tional enema.
Therefore, if I have a patient who is having a flare, I like to use the foam. However,
it is a steroid product, so it’s fine for short-term use, for a flare. But if someone is
going to be taking a long-term maintenance rectal treatment, I will use mesalamine
enemas. I would rather not use the steroid product long term.

P. Beniwal-Patel (*)
GI and Hepatology Division, Medical College of Wisconsin, Milwaukee, WI, USA
e-mail: [email protected]

© The Author(s), under exclusive license to Springer Nature 207


Switzerland AG 2023
W. H. Sobin et al. (eds.), Managing Complex Cases in Gastroenterology,
https://doi.org/10.1007/978-3-031-48949-5_33
208 P. Beniwal-Patel

Since the patient’s proctitis clinically is not responding remarkably well to


rectal medications, what would you do next?
I would restage him with a flexible sigmoidoscopy because the last scope was
done a year ago, in order to assess whether there has been a change in severity or
extent. If the repeat endoscopy shows persistent limited Mayo 2 disease, I some-
times consider adding oral mesalamine-4.8 g, along with enema/foam therapy. In
select cases, we also try an oral budesonide or prednisone taper. If the disease has
progressed to Mayo 3, if there’s more extensive distribution, or if the patient is clini-
cally not doing well, I would start biologic therapy. In that case, we might start with
vedolizumab versus an anti-TNF.
I think a common misconception with proctitis, when I see patients who are com-
ing in for a second or third opinion, is that, oh, it’s “just proctitis”, how bad can it
be? But it can really progress rapidly, and I’ve had several patients who’ve had
Mayo three of just the distal 30, 40 centimeters who ended up in the hospital and got
quite sick and almost needed surgery.
I think the key thing with resistant proctitis is making sure that we pull the trigger
early enough, but not too early. We will usually start something like vedolizumab or
an anti-TNF and that usually does the trick.
Chapter 34
Vaccinations in Newly Diagnosed IBD

Preetika Sinh

A 57-year-old woman, non-smoker, is seen for bloody diarrhea and found to have
moderately severe UC involving most of the colon. She has not had any vaccinations
in recent memory, perhaps none since she finished school.
How would you go about vaccinating this lady and does her lack of recent
vaccinations alter your choice of therapy?
We always discuss the necessity of having up-to-date vaccinations, and in most
cases, we can get patients to go along with that. We are talking about yearly flu vac-
cine, pneumonia vaccine, and tetanus every 10 years. We need to check hepatitis
serology and vaccinate if not immune. In this patient, HPV is not clinically indi-
cated. Shingrix is very important, particularly when JAK inhibitors are used.
IBD itself does not interfere with the ability of patients to respond to vaccina-
tions but immunosuppressive drugs may. Ideally, we should wait for at least 4 weeks
after vaccination to start our immune-suppressing drugs. But we often don’t have
time to wait, we have to start the drugs right away, and so we just get the patient
rolling on their vaccinations. We do try to give the first Shingrix dose prior to start-
ing an anti-TNF and particularly a JAK inhibitor.
But, no, I would not veto the use of anti-TNFs because the patient has not been
vaccinated. The major infections associated with anti-TNFs are TB and fungal
infections, and we can’t vaccinate against those anyway. We do take age into
account, however. There is an increased risk, with anti-TNFs, of infection, including
serious infections, in those patients over 65.

P. Sinh (*)
Division of Gastroenterology and Hepatology, Medical College of Wisconsin,
Milwaukee, WI, USA
e-mail: [email protected]

© The Author(s), under exclusive license to Springer Nature 209


Switzerland AG 2023
W. H. Sobin et al. (eds.), Managing Complex Cases in Gastroenterology,
https://doi.org/10.1007/978-3-031-48949-5_34
210 P. Sinh

In talking about JAK inhibitors, how does the new upadacitinib compare
with tofacitinib?
Upadacitinib is a selective JAK inhibitor while tofacitinib is non-selective. There
does not appear to be a different safety, side effect profile with either one currently.
It is possible, in the long run, that the selective JAK inhibitor may have a better
safety profile.
Upadacitinib appears to have a more rapid induction, the number of patients
responding in the first 2 weeks is compelling. And these excellent results were seen
not only in those who are biologic naïve, but also those who had been exposed to
anti-TNFs previously. Would I switch a patient from tofacitinib to upadacitinib? No,
not unless a new mechanism of action was present. But, if a patient is first being
started on a JAK inhibitor, I would probably opt for the use of upadacitinib.
Chapter 35
Ustekinumab vs Risankizumab in Crohn’s

Daniel Stein and Salina Faidhalla

A 28-year-old female smoker is found to have mild moderately severe Crohn’s coli-
tis without obvious small bowel involvement. She does not want to go on an anti-­
TNF and is wondering about going on ustekinumab or the new drug risankizumab.
Can you discuss how you would choose between ustekinumab and risanki-
zumab if insurance coverage was not an issue.
Ustekinumab is an IL-12 and IL-23 inhibitor while risankizumab is an IL-23
inhibitor alone. Risankizumab was recently approved for use in moderate-to-severe
Crohn’s disease. It has endoscopic and histologic data supporting its efficacy.
Both medications start with IV infusion for induction followed by SQ injections.
They have good efficacy and comparable side effect profiles; however, there are not
enough data comparing the two medications since risankizumab was only recently
approved. The safety profile overall appears to be similar; there might be a small
risk of DILI with risankizumab since one patient developed liver injury in the trial.
Risankizumab may have increased efficacy compared with ustekinumab and
clinically may be comparable to infliximab. Our clinical experience with risanki-
zumab in Crohn’s patients who failed multiple biologics has been similar to what
was seen in the clinical trials. If insurance is not an issue, then we would generally
choose risankizumab.

D. Stein (*)
Department of Internal Medicine, Division of Gastroenterology and Hepatology, Medical
College of Wisconsin, Milwaukee, WI, USA
e-mail: [email protected]
S. Faidhalla
Department of Medicine Division of Gastroenterology and Hepatology, Medical College of
Wisconsin, Milwaukee, WI, USA
e-mail: [email protected]

© The Author(s), under exclusive license to Springer Nature 211


Switzerland AG 2023
W. H. Sobin et al. (eds.), Managing Complex Cases in Gastroenterology,
https://doi.org/10.1007/978-3-031-48949-5_35
212 D. Stein and S. Faidhalla

In comparing vedolizumab, ustekinumab, and risankizumab in Crohn’s


disease, what is your first choice, or which drug do you prefer in which
situation?
Many factors, including age, disease severity, disease behavior (inflammatory,
stricturing or fistulizing), location, the presence of perianal disease, extraintestinal
manifestations, prior biologic exposure, history of malignancy, serious infections,
and patient preference, play a role in choosing a biologic and must all be considered
carefully.
VDZ is usually used to treat moderate-to-severe Crohn’s patients with no extrain-
testinal manifestations, and no perianal disease since it is more gut selective. This is
also a good choice in patients with a history of malignancy or serious infections
given its favorable side effect profile. However, it does have a slower onset of action
and possible delayed clinical response when compared to anti-TNF or UST.
That being said, there are some data supporting VDZ use in perianal disease. The
exploratory analysis from the GEMINI 2 trial showed a greater percentage of
Crohn’s disease patients with draining fistulae achieving fistula closure by week 52
with vedolizumab compared to placebo [1]. There were similar findings in the
ENTERPRISE study where >50% of patients with fistulizing CD treated with
vedolizumab (with concurrent seton until week 14 and antibiotics until week 6) had
>50% decrease in the number of draining perianal fistulae [2].
UST has been effective in Crohn’s patients including those patients who are
refractory to infliximab. While the data for use of UST for treating EIMs didn’t
show statistical significance, UST is effective in treating psoriasis and psoriatic
arthritis. There are some data that suggest that UST can be effective in patients with
perianal disease in CD. Notably, infliximab is the most effective and most studied
drug for treatment of perianal disease in CD, but UST may be useful in CD patients
not responding to infliximab who have refractory perianal CD. Being a self-­
administered injection is more convenient for patients.
As mentioned above, there is no head-to-head trial comparing ustekinumab and
risankizumab. However, the absolute remission rates in separate trials were higher
with risankizumab [3]. If we have the freedom of choosing between the two, we
would start with risankizumab since it appears to be more effective than UST. We
also have some clinical experience in using it in patients with TNF-resistant perianal
disease with very good response. The safety profile for UST and risankizumab is
comparable. They are both effective for perianal disease but not the first choice.
However, if we have a patient with Crohn’s disease and EIM, we usually start
with an anti-TNF; if the patient fails the anti-TNF, then we would consider UST or
risankizumab.
If the patient was amenable to an anti-TNF, would you choose that over the
other 3 drugs previously mentioned? How do you decide?
Anti-TNFs are long established with good efficacy and known safety profile in
treating CD. Treatment with anti-TNF agents appears to be more effective when
given earlier in the course of disease; rates of response and remission are higher if
given within 2 years of onset of disease.
Anti-TNFs also target extraintestinal manifestations (EIMs) of CD and should be
considered in such patients. IFX is very effective in patients with perianal and fistu-
lizing disease.
35 Ustekinumab vs Risankizumab in Crohn’s 213

In this young patient with mild-moderate disease, no EIM or perianal CD, and no
previous biologic exposure, all options are reasonable and should be discussed
in detail.
We recognize that choosing therapy in patients with Crohn’s disease can be com-
plicated. To shed some light on positioning therapy, a recent meta-analysis by Singh
et al. [4] assessed the comparable efficacy and safety of biologics in patients with
Crohn’s disease and this may be a guide.
This study highlighted that treatment with anti-TNFs: infliximab or adalimumab,
consistently ranked high for induction and maintenance of clinical remission. And
infliximab in combination with azathioprine was the highest ranked treatment for
induction of clinical remission and maintenance of long-term remission in biologic-­
naive patients.
It is also important to remember that anti-TNFs still play a crucial role in the
treatment of patients with high-risk phenotypes such as fistulizing, penetrating, and
stricturing disease and also TNF-sensitive extraintestinal manifestations.
IL-23 blockade might be the preferred mechanism of action in patients who have
been previously exposed to TNF antagonists. Overall data suggest that both
ustekinumab and risankizumab are effective after either primary or secondary loss
of response to other biologics. Risankizumab and ustekinumab also appear to be
more effective than vedolizumab in patients who have lost response or have intoler-
ance to anti-TNF therapy.
It is also crucial to keep in mind that when you are dealing with a primary non-­
responder, the best strategy is to switch the class of drugs. In cases of secondary
non-response, you can still try another medication within the same class.

References

1. Sandborn WJ, Feagan BG, Rutgeerts P, Hanauer S, Colombel JF, Sands BE, Lukas M, Fedorak
RN, Lee S, Bressler B, Fox I. Vedolizumab as induction and maintenance therapy for Crohn’s
disease. N Engl J Med. 2013;369(8):711–21.
2. Schwartz DA, Peyrin-Biroulet L, Lasch K, Adsul S, Danese S. Efficacy and safety of 2 vedoli-
zumab intravenous regimens for perianal fistulizing Crohn’s disease: ENTERPRISE study.
Clin Gastroenterol Hepatol. 2022;20(5):1059–67.
3. Ferrante M, Panaccione R, Baert F, Bossuyt P, Colombel JF, Danese S, Dubinsky M,
Feagan BG, Hisamatsu T, Lim A, Lindsay JO. Risankizumab as maintenance therapy for
moderately to severely active Crohn’s disease: results from the multicentre, randomised,
double-­blind, placebo-controlled, withdrawal phase 3 FORTIFY maintenance trial. Lancet.
2022;399(10340):2031–46.
4. Singh S, Murad MH, Fumery M, Sedano R, Jairath V, Panaccione R, Sandborn WJ, Ma
C. Comparative efficacy and safety of biologic therapies for moderate-to-severe Crohn’s
disease: a systematic review and network meta-analysis. Lancet Gastroenterol Hepatol.
2021;6(12):1002–14. https://doi.org/10.1016/S2468-­1253(21)00312-­5. Epub 2021 Oct 22.
PMID: 34688373; PMCID: PMC8933137.
Chapter 36
Use of Vedolizumab in UC

Daniel Stein and Salina Faidhalla

A 67-year-old man who quit smoking 6 months earlier is diagnosed with new-onset
left-sided ulcerative colitis that is mild to moderate in severity. He is started on oral
and rectal mesalamine but does not respond. He continues to move his bowels 4–5
times a day with a small amount of blood mixed in his stool. He would like to try
vedolizumab as his next course of therapy.
Is vedolizumab a reasonable option? If so, would you start steroids as well?
In patients with mild-moderate left-sided UC with active disease despite being
on an adequate dose of oral and rectal mesalamine, we usually add budesonide 9 mg
daily for 12 weeks for induction of remission. We prefer budesonide to prednisone
due to its lower systemic side effects. We would start budesonide prior to escalating
to biologics and assess response.
If the decision was made to proceed with VDZ, then we would start budesonide
that clinic visit while waiting for the first infusion of VDZ. We would then continue
budesonide for 12 weeks and stop without tapering.
Do you check vedolizumab levels early on?
There is some evidence that higher VDZ levels are associated with higher clini-
cal response and mucosal healing especially in UC. In a recent systematic review
and meta-analysis by Singh et al. [1], they reported that in UC patients, median
vedolizumab trough concentrations were consistently higher in patients achieving

D. Stein (*)
Department of Internal Medicine, Division of Gastroenterology and Hepatology, Medical
College of Wisconsin, Milwaukee, WI, USA
e-mail: [email protected]
S. Faidhalla
Department of Medicine Division of Gastroenterology and Hepatology, Medical College of
Wisconsin, Milwaukee, WI, USA
e-mail: [email protected]

© The Author(s), under exclusive license to Springer Nature 215


Switzerland AG 2023
W. H. Sobin et al. (eds.), Managing Complex Cases in Gastroenterology,
https://doi.org/10.1007/978-3-031-48949-5_36
216 D. Stein and S. Faidhalla

clinical remission or endoscopic remission. Based on their meta-analysis, they sug-


gested that vedolizumab trough concentration >20 μg/mL at week 6 and >12 μg/mL
during maintenance may be associated with better outcomes.
Some providers do proactively check VDZ levels but we don’t do this routinely.
We only do reactive drug monitoring in the case of primary non-responders or sec-
ondary loss of response. We don't think that there is enough evidence supporting
proactive monitoring of the VDZ levels. We know what to do with anti-TNF levels
but with vedolizumab you have completely different mechanisms of action. How
high a level do you need to saturate those receptors? We don’t know. We don't think
that we necessarily know the full significance of our vedolizumab levels.
The patient is responding to combination budesonide and vedolizumab but when
you stop the budesonide after 8 weeks his disease starts to flare. His vedolizumab
levels are adequate.
How would you manage this?
Usually, when drug levels are adequate, and the patient’s clinical response has
been steroid dependent, the best next step is to switch therapy. However, in the case
of VDZ it has been recognized that it can take longer to achieve the desired clinical
response (6–8 weeks in UC, 10–14 weeks in CD).
In this patient who has been on the medication for 8 weeks, we would discuss the
options of giving the drug more time and continuing budesonide vs switching to
another agent. If at 10–12 weeks the patient still has no response, then we should
switch to another agent.
Do you ever suggest to your patients who develop UC after quitting smoking
that they go back to smoking a cigarette or two a day?
We wouldn’t recommend that the risks of smoking outweigh the benefits, espe-
cially since we have so many options for treatment.
Are patients who develop UC after quitting smoking more difficult to get
under control?
There are no data to suggest that.
If you decide to give vedolizumab more time to “kick-in” how much longer
would you continue budesonide?
Maybe 10–12 weeks if they are primary non-responders, if they have partial
response then up to 6 months. If there is no clinical response and active inflamma-
tion is confirmed with fecal calprotectin and/or CRP, then switching therapy to
another agent is reasonable. If the patient is doing poorly, we would switch medica-
tion earlier.
The patient continues to show a lack of response to VDZ
What is your next choice for patients who choose vedolizumab but fail to
respond to it?
In this patient with mild-moderate left-sided UC who failed mesalamine and
VDZ, available options include anti-TNFs and UST. Ozanimod is another option for
UC patients who are anti-TNF naive.
In this case, since this patient is older and has mild-moderate UC with no EIM
we would probably start UST. Obviously, a discussion with the patient regarding
treatment options is essential in choosing the next drug.
36 Use of Vedolizumab in UC 217

Reference

1. Singh S, Dulai PS, Vande Casteele N, Battat R, Fumery M, Boland BS, Sandborn WJ. Systematic
review with meta-analysis: association between vedolizumab trough concentration and
clinical outcomes in patients with inflammatory bowel diseases. Aliment Pharmacol Ther.
2019;50(8):848–57. https://doi.org/10.1111/apt.15484. Epub 2019 Sep 4. PMID: 31483522;
PMCID: PMC7083298
Chapter 37
Functional Diarrhea in IBD

Amir Patel

A 32-year-old woman with a history of Crohn’s involving the terminal ileum and
right colon has been well controlled on Humira. She now comes in complaining of
frequent watery diarrhea and rectal urgency. Her exam, CRP, stool cultures, stool
calprotectin, and colonoscopy are within normal limits. Are there other tests you
would want to run?
It sounds like her Crohn’s is well controlled and the symptoms are either
functional or bile acid related. How do you manage those patients?
We see that a lot. Many of our patients with Crohn’s disease will have overlap-
ping irritable bowel syndrome, and if they have ileal disease, we also think about
bile acid diarrhea. In this particular patient, you would also like to know their under-
lying medical history, do they have a history of depression, anxiety?
But I probably would start them on cholestyramine first, to treat bile acid diar-
rhea. And if that doesn’t help, I would wonder whether this could be irritable bowel
syndrome. That, to me, is a little bit more difficult to treat. But some studies in our
inflammatory bowel disease literature show that patients may do well with a low
FODMAP diet.
The other thing you think about, if this patient has ever had any stricturing dis-
ease, is the possibility of small intestinal bacterial overgrowth. So, it would be fair
to try a course of antibiotics as well.

A. Patel (*)
Dept. of Medicine, Division of Gastroenterology and Hepatology, Medical College of
Wisconsin, Milwaukee, WI, USA
e-mail: [email protected]

© The Author(s), under exclusive license to Springer Nature 219


Switzerland AG 2023
W. H. Sobin et al. (eds.), Managing Complex Cases in Gastroenterology,
https://doi.org/10.1007/978-3-031-48949-5_37
220 A. Patel

When you use cholestyramine, do you tend to use it once a day or several
times a day?
I typically use it once a day, mostly in the morning. I think that’s when they get
the most benefit from it. So, I start it in the morning with a low dose, and if they
notice some improvement, I’ll increase the dose incrementally, but I just give it once
in the morning.
Chapter 38
When to Postpone Infusions

Poonam Beniwal-Patel

A 32-year-old man has UC well controlled on infliximab. He has a history of sinus


infections in the past and feels like he has one now. He is due for his infusion
tomorrow.
Would you postpone the infusion? In which situations do you feel it is neces-
sary to postpone an infusion and why?
In the setting of a mild infection where a patient has been afebrile for 48 h, the
infusion is given. If a patient is hospitalized with an infection, we will delay therapy
given that source control is needed.
But this raises an interesting question. It used to be that the only biologic agents
available were the anti-TNFs, and we would have to struggle with this dilemma. But
now, if I have a patient who keeps getting recurrent sinus infections, in the absence
of any anatomic defects, and we are really thinking that it’s the infliximab that is
increasing their risk of infections, I actually have changed them over to vedoli-
zumab if they’re in deep remission. It’s nice that now, we have other options, other
medications that may not increase the risk of things like sinus infections or
folliculitis.
What are some other examples of patients where you’ve told them it’s not a
good idea to go ahead with their infusions?
With COVID, if someone was diagnosed within the last couple of days, we'll
have them wait a week and make sure they haven’t spiked a fever for a few days
before administering an infusion. But outside of having a fever or a prolonged infec-
tion where source control with antibiotics hasn’t been achieved there really are very
few scenarios where we will postpone an infusion.

P. Beniwal-Patel (*)
Medicine, Gastroenterology/Hepatology Division, Medical College of Wisconsin,
Milwaukee, WI, USA
e-mail: [email protected]

© The Author(s), under exclusive license to Springer Nature 221


Switzerland AG 2023
W. H. Sobin et al. (eds.), Managing Complex Cases in Gastroenterology,
https://doi.org/10.1007/978-3-031-48949-5_38
Chapter 39
Musculoskeletal Complaints in a Patient
on Anti-TNFs

Amir Patel

A 35-year-old woman has been receiving infliximab for 2 years for Crohn’s disease.
Her ileocolitis has been under good control, but she comes in complaining of joint
pain in her knees and fingers which is something new for her. On exam, her joints
seem tender but not acutely inflamed.
When do you consider the possibility that a patient with IBD who develops
new musculoskeletal symptoms might be experiencing an adverse reaction
from their anti-TNF?
Arthropathy is common in inflammatory bowel disease. There are two types.
Type one is generally peripheral, acute, and involves less than six joints. That type
of arthropathy typically occurs at the beginning of the disease. It’s self-limiting,
non-erosive, and correlates with flares. So, if someone has active colitis, they typi-
cally complain of joint pains, commonly affecting the knees and elbows in my expe-
rience. Type two is more polyarticular. The MCPs, the small joints, are commonly
involved. These do have active synovitis but this type of joint pain does not correlate
with IBD flares and can happen years after the disease commences. That’s one thing
I think about, but if someone’s been on an anti-TNF for a couple of years and now,
they start having arthralgias, I do start wondering if this could be a side effect of the
Remicade.
Usually, I see arthralgias as an adverse reaction to Remicade occurring early
after initiation of therapy. If it starts up years later, then I do get concerned about
conditions like drug-induced lupus, where I consider referring the patient to the
rheumatologist.

A. Patel (*)
Dept. of Medicine, Division of Gastroenterology and Hepatology, Medical College of
Wisconsin, Milwaukee, WI, USA
e-mail: [email protected]

© The Author(s), under exclusive license to Springer Nature 223


Switzerland AG 2023
W. H. Sobin et al. (eds.), Managing Complex Cases in Gastroenterology,
https://doi.org/10.1007/978-3-031-48949-5_39
224 A. Patel

Drug-induced lupus is pretty low risk with your anti-TNFs compared to other
medications, but it can happen. This usually occurs months or years after they’ve
been exposed to the drug. If I am thinking about drug-induced lupus, I do get some
blood work before I send them to the rheumatologist, including a CBC, CMP, anti-­
double-­stranded DNA, anti-histone antibodies and complement levels, C3 and C4.
And if any of those antibodies are elevated, I have them see the rheumatologist to
determine whether they have to come off anti-TNF and go on some other medication.
The rheumatologist reports that it is possible these symptoms are related to the
infliximab and she would like you to stop infliximab and switch to a different class
of drug,
Which drug would you choose?
In a patient with arthralgias like this, I’d probably go with an IL-12,23 or an
IL-23 inhibitor, so that’s ustekinumab (Stelara) or risankizumab (Skyrizi). Those
are the types of therapies I’d probably go with in a patient with Crohn’s disease.
Obviously, it also depends on the phenotype of their disease. If it’s more severe and
stricturing, then obviously I prefer the IL-12,23, and IL-23 over something like anti-­
integrin therapy, vedolizumab (Entyvio). If a patient has a lot of extraintestinal
manifestations, I'll also go with Stelara or Skyrizi over Entyvio. Data suggest that
Entyvio might not be the best drug for extraintestinal manifestations, including
arthralgias.
With arthralgia and other extraintestinal manifestations, particularly psoriasis,
we have found that the IL-12,23s and IL-23s have really helped. However, the one
exception is in ankylosing spondylitis. In ankylosing spondylitis, the anti-TNFs are
clearly superior and the anti-IL 12,23s and anti-IL23s have not been that effective.
Now, if it were ulcerative colitis and not Crohn’s disease you could also consider
switching to a JAK inhibitor like tofacitinib or upadacitinib. So far, these drugs are
just approved for UC not Crohn’s (upadacitinib was recently approved for Crohn’s).
But they are very effective second-line therapies.
Chapter 40
Use of Immune Modulators in Patients
Being Started on Anti-TNFs

Daniel Stein and Salina Faidhalla

You have a patient with moderately severe ulcerative colitis who you want to start
on an anti-TNF. He is a 32-year-old man who is a social drinker, having about 2
drinks on Saturday night and Sunday during ball games. No other medical problems.
Do you think immune modulators should be used in all patients on anti-­
TNFs or are you doing more anti-TNF monotherapy with close dose monitoring?
Combination therapy with IFX and AZA is superior to monotherapy. The benefit
of adding an IM to adalimumab or a different anti-TNF is not well established. We
are less likely to add AZA to Humira unless we are treating a patient with Humira
who has had secondary loss of response and antibody to IFX.
On the other hand, close dose monitoring with monotherapy can be very effec-
tive in patients on anti-TNFs. We are more likely to employ this when using IFX in
a patient over 60, to decrease the risk of bone marrow suppression and lymphoma.
We know that very high drug levels of the infliximab are as protective against anti-
body formation than immunomodulators, if not more. So that's a different approach.
If you decide to use azathioprine, how would you use it in this young male,
and for how long, noting concerns that exist about hepatosplenic T-cell lym-
phoma (HSTCL)?
Men under 35 years of age receiving long-term thiopurines (>2 years) are at a
significantly increased risk of HSTCL, although the true incidence remains low. The
risk of HSTCL in IBD patients on combination therapy is about 1:3500. This

D. Stein (*)
Department of Internal Medicine, Division of Gastroenterology and Hepatology, Medical
College of Wisconsin, Milwaukee, WI, USA
e-mail: [email protected]
S. Faidhalla
Department of Medicine Division of Gastroenterology and Hepatology, Medical College of
Wisconsin, Milwaukee, WI, USA
e-mail: [email protected]

© The Author(s), under exclusive license to Springer Nature 225


Switzerland AG 2023
W. H. Sobin et al. (eds.), Managing Complex Cases in Gastroenterology,
https://doi.org/10.1007/978-3-031-48949-5_40
226 D. Stein and S. Faidhalla

usually doesn’t affect our decision to start the medication. But we always discuss
the risk with the patient beforehand.
While we feel this is a small risk, one strategy to reduce the risk even further is
to use a lower dose of azathioprine. We can target lower 6TG levels, a 6TG level of
100–120 is enough to prevent antibody formation.
We can also stop the azathioprine earlier. We think that once you get two years
out that the benefit with combination therapy, probably isn’t there. If you look at
when people are going to get antibodies, they usually get it, within the first year. The
cases of HSTCL have occurred in patients taking azathioprine for >2 years.
You can also do monotherapy with anti-TNF alone with close dose monitoring.
Do you ever use methotrexate as your immune modulator in treating IBD?
How do you administer it?
In UC, there are limited data on using methotrexate in combination with anti-­
TNFs. Currently, MTX is not commonly used in adult patients with UC. However,
in CD there are data supporting use of methotrexate to reduce immunogenicity and
improving drug concentrations when used in combination with an anti-TNF agent.
This may be the preferred immunomodulator for combination therapy in those at
higher risk of adverse effects of thiopurines such as young men or those with mul-
tiple skin cancers.
We do use methotrexate occasionally in our practice for select patients.
For combination therapy, we use an oral dose of 12.5 mg per week. This low dose
works to prevent antibody formation. The only time we use MTX IM or SC is when
we use it as monotherapy in CD at a dose of 25 mg weekly.
Alcohol use in this case would make using methotrexate risky.
Chapter 41
Pyoderma Gangrenosum

Poonam Beniwal-Patel

A 27-year-old female presents with acute ulcerative colitis and a skin rash on her
legs, which is diagnosed as pyoderma gangrenosum. Colonoscopy revealed a mild-
moderate severity pancolitis.
Does someone have to have severe ulcerative colitis to develop pyoderma
gangrenosum? What therapy would you use to treat the UC and the pyoderma
in this patient? Is a dermatologist always involved?
You can see pyoderma gangrenosum in someone with mild IBD. We had one
patient who had very severe PG and never had any GI symptoms. He had several
colonoscopies that looked normal and revealed only minimal histologic evidence of
ulcerative colitis.
PG severity does not have to correlate with bowel disease activity. I would say
that about half of the patients I’ve seen with PG have severe colitis and half do not.
The best treatment for PG is anti-TNF drugs. I prefer infliximab. So, even if they
have just mild UC, I would treat them with an anti-TNF if they have biopsy-­
proven PG.
There is a dermatologist involved in the beginning to do a skin biopsy and make
the diagnosis. But after that, they generally leave the management up to us, since
treating the patient with anti-TNFs usually heals the skin lesions as well as the
IBD. Occasionally, the dermatologist will use topical steroids, but the mainstay is
the anti-TNF.

P. Beniwal-Patel (*)
Medicine, Gastroenterology/Hepatology Division, Medical College of Wisconsin,
Milwaukee, WI, USA
e-mail: [email protected]

© The Author(s), under exclusive license to Springer Nature 227


Switzerland AG 2023
W. H. Sobin et al. (eds.), Managing Complex Cases in Gastroenterology,
https://doi.org/10.1007/978-3-031-48949-5_41
Chapter 42
UC in a Patient Who Failed Mesalamine
and Anti-TNF

Daniel Stein and Salina Faidhalla

A 33-year-old female has moderate left-sided UC. She has tried and failed mesala-
mine and Humira (with azathioprine) and needs a change of therapy to treat her
symptoms.
How would you feel about each of these options in a patient with moderate
UC who has failed an anti-TNF? (a) Vedolizumab (b) Ustekinumab (c)
Ozanimod (d) Tofacitinib
UST is one option. It has proven effective in patients who have failed an anti-­
TNF. It is an injectable medication, which is convenient in patients who have work
or school.
UST has been found to be more effective than vedo in inducing and maintaining
remission as a second-line therapy in Crohn’s disease in a network meta-analysis.
So, vedo is not our first choice in patients who have failed an anti-TNF.
We would not recommend ozanimod in a patient who failed anti-TNF, since the
original study population had only a small percentage of patients who failed
anti-TNF.
Tofacitinib is a Jak1/JAK3 inhibitor that is approved for moderate to severe UC
patients who have failed anti-TNFs. Upadacitinib is a specific JAK1 inhibitor, which

D. Stein (*)
Department of Internal Medicine, Division of Gastroenterology and Hepatology, Medical
College of Wisconsin, Milwaukee, WI, USA
Department of Medicine Division of Gastroenterology and Hepatology, Medical College of
Wisconsin, Milwaukee, WI, USA
e-mail: [email protected]; [email protected]
S. Faidhalla
Department of Medicine Division of Gastroenterology and Hepatology, Medical College of
Wisconsin, Milwaukee, WI, USA

© The Author(s), under exclusive license to Springer Nature 229


Switzerland AG 2023
W. H. Sobin et al. (eds.), Managing Complex Cases in Gastroenterology,
https://doi.org/10.1007/978-3-031-48949-5_42
230 D. Stein and S. Faidhalla

is also effective. Our clinical experience suggests that it is possibly more effective
than tofacitinib, and there is some consensus that it has a more favorable side-effect
profile, given that it’s more selective. However, since this is a young female who
may have plans for pregnancy, we would want to avoid any JAK inhibitor and ozani-
mod at this time, and we would probably choose UST.
IFX can be considered if the patient failed adalimumab; however, most of us are
turning to the JAK inhibitors in patients who have failed anti-TNFs.
Chapter 43
Patients with IBD Who Are Squeamish
About Rectal Meds and Self-Injection

Amir Patel

A 20-year-old college sophomore presents with chronic UC that previously involved


the left colon to the splenic flexure. She had been doing well, taking mesalamine for
a year but over the past month started having increased bowel frequency, urgency,
and rectal bleeding. Her community gastroenterologist repeated her colonoscopy
and noted active inflammation in the rectum, with the rest of the colon looking fairly
normal. The woman is going to school 1000 miles away on a fairly rural campus.
Her doctor recommends that she try rectal meds – either suppository or enema – but
she gets squeamish thinking about this and doesn’t want to try. He also discusses
possibly using Humira, but she is frightened about self-injection.
This gastroenterologist, who frequently sends you patients for a second
opinion, calls and curbsides you with these questions:
Do you have any tips for convincing a patient to try taking a rectal med? If
so, are you more successful in getting them to take suppositories, foams,
or enemas?
Do you have any tricks for dealing with patients who are reluctant to do
self-injection?
A lot of our patients with proctitis don’t want to take rectal suppositories for
several reasons: first, they’re uncomfortable; second, there’s a stigma of putting
something in the rectum; and third, sometimes it’s really hard to retain suppositories
or enemas, if there’s significant inflammation, and they have a lot of urgency and
diarrhea already.

A. Patel (*)
Dept. of Medicine, Division of Gastroenterology and Hepatology, Medical College of
Wisconsin, Milwaukee, WI, USA
e-mail: [email protected]

© The Author(s), under exclusive license to Springer Nature 231


Switzerland AG 2023
W. H. Sobin et al. (eds.), Managing Complex Cases in Gastroenterology,
https://doi.org/10.1007/978-3-031-48949-5_43
232 A. Patel

Injections are another problem. We have a lot of young individuals diagnosed


with IBD who don’t want to self-inject or they’re afraid of needles. This is some-
times very difficult to manage. What we typically do in the clinic is discuss with
them the risks if their disease goes uncontrolled and what the benefits of our medi-
cations are. So, we have that risk and benefit talk. Because the truth is that if they
don’t get their proctitis under control, there’s always a chance the disease may prog-
ress. There’s an increased risk of flares. And, obviously, there’s concern for the
increased risk for cancer.
So, we have that discussion and see if they’re willing to try the rectal supposito-
ries or the self-injections. But unfortunately, I don’t have any other tricks to con-
vince them otherwise.
In terms of choosing between suppositories, foam, and enemas, if it’s just proc-
titis involving the last 5 cm of the rectum, I think that suppositories are the ideal
route. Typically, they can be easier to retain than enemas. If you’re talking about
more left-sided colitis, where there’s involvement to the splenic flexure, then I
would use enemas. And here I’m talking about mesalamine. Now, we do have
hydrocortisone foams as well. And some patients feel that they can retain those better.
There is also a small subset of patients who may have a paradoxical response to
mesalamine. So, if they try Canasa suppositories or Rowasa enemas, which are both
mesalamine products, and they get a paradoxical response, you’re typically going to
switch them to hydrocortisone enemas or foams as a substitute.
Do you have any experience with tacrolimus suppositories?
Yes, tacrolimus suppositories have been around, and they’re usually dosed at
2 mg twice a day. We typically reserve those for patients who are refractory to mesa-
lamine or steroid suppositories. I use them rarely. There have been several studies,
one was a head-to-head study looking at tacrolimus versus steroids, and while that
study didn’t find tacrolimus superior in controlling symptoms, there was another
study of about 40 patients that did show a benefit. So, I am using it for those who
are willing to take suppositories but haven’t had a robust response to mesalamine or
steroid suppositories. And I typically give it for about a month to see if it helps. But
obviously, a lot of times, this is just a bridge until you find them an appropriate
maintenance therapy.
Chapter 44
Colon Stricture in UC

Poonam Beniwal-Patel

A 58-year-old female with chronic ulcerative colitis managed with infliximab comes
to see you for a surveillance colonoscopy. Her last gastroenterologist just retired,
and her last colonoscopy was 3 years ago. Colonoscopy and random biopsies were
unremarkable, showing chronic UC in remission. Now on colonoscopy, you find
mild inflammation in the rectum, descending colon and transverse colon. But when
you get to the ascending colon, there is narrowing, and you cannot advance the
colonoscope to the cecum. There is minor inflammation in the ascending colon but
no suggestion of a tumor or dysplasia. Biopsies distal to the narrowing and blind
biopsies within the narrowed segment are unrevealing; there is no dysplasia found.
Do you feel compelled to investigate this “stricture” in some other way? If
so, how?
Anytime a stricture is encountered, the diagnosis needs to be reevaluated. If there
truly is a stricture present, this suggests a diagnosis of possible Crohn’s disease
instead of ulcerative colitis. This finding would warrant a review of prior colonos-
copies and cross-sectional imaging. The next step would be a CT or MR enterography.
Can you really delineate fibrosis on an MR enterography?
There have been research studies showing that MRE can distinguish between
active inflammation versus fibrotic lesions; however, this is not done as standard
clinical practice.
Assuming there is a true colon stricture in a patient with IBD, do you feel
compelled to send the patient to surgery? And, if you do, do you think it is suf-
ficient for the surgeon to do a right hemicolectomy or is a more extensive resec-
tion indicated?

P. Beniwal-Patel (*)
Medicine, Gastroenterology/Hepatology Division, Medical College of Wisconsin,
Milwaukee, WI, USA
e-mail: [email protected]

© The Author(s), under exclusive license to Springer Nature 233


Switzerland AG 2023
W. H. Sobin et al. (eds.), Managing Complex Cases in Gastroenterology,
https://doi.org/10.1007/978-3-031-48949-5_44
234 P. Beniwal-Patel

I have encountered a number of patients with ascending colon strictures, and this
is in the setting of people feeling well. So, then the question becomes what do you
do? And in those patients, I actually have sent them to surgery, because you don’t
know what’s on the other side of the stricture. You can’t survey it for disease activity
or, more importantly, dysplasia. And so, if someone truly has an ascending colon
stricture, I would send them to surgery.
To answer the question how extensive a resection is necessary, there are cases
(e.g., limited colonic Crohn’s involvement), where a right hemicolectomy can be
considered over a total colectomy. It really is decided on a case-by-case basis and
includes consideration of several factors, such as which medications a patient has
tried, disease trajectory, and overall health status.
Chapter 45
Nonspecific Ileal Ulcers

Amir Patel

A 35-year-old nonsmoking male is referred with chronic diarrhea. All stool cultures
are negative. Diarrhea is watery and non-bloody. Stool calprotectin is 250. You
decide to do a colonoscopy. The colon appears normal, but you enter the terminal
ileum and find three or four small ulcers that are fairly nondescript. Biopsies of the
colon are negative, and biopsies of the ulcers are nondiagnostic, with no granulo-
mas present. You think these findings are not convincing for IBD. The patient does
not routinely take NSAIDS but did take some one weekend, a month earlier, after a
basketball injury.
What is the level of calprotectin that you will pursue?
It depends on the range at the institution, but at our institution, anything greater
than 50 can be considered elevated. Now that being said, I think the range between
50 and 80 is a gray area. So, if it's in that low range, right below a hundred, I might
consider repeating the fecal calprotectin in a few weeks just to see if it's still ele-
vated. Some of the literature suggests that we'll see higher fecal calprotectin when
there’s significant colitis versus ileitis.
The value generally does not correlate with disease severity; however, if I see
numbers in the thousands, that’s when I get really concerned, and that result is prob-
ably more sensitive for inflammatory bowel disease.

A. Patel (*)
Dept. of Medicine, Division of Gastroenterology and Hepatology, Medical College of
Wisconsin, Milwaukee, WI, USA
e-mail: [email protected]

© The Author(s), under exclusive license to Springer Nature 235


Switzerland AG 2023
W. H. Sobin et al. (eds.), Managing Complex Cases in Gastroenterology,
https://doi.org/10.1007/978-3-031-48949-5_45
236 A. Patel

When you see a patient with ulcers in the terminal ileum that you think are
probably not Crohn’s, how do you approach them? Will you do further small
bowel imaging (capsule, SBFT, or CTE)? Will you give a trial of budesonide?
Just follow them?
Although we’ve always been taught to look for granulomas, the actual finding of
granulomas is pretty low, they’re only found in a minority, perhaps 30% of patients
with Crohn’s disease. When you think about the different causes of ileal ulcers, it
could be NSAIDs, or it could be Crohn’s disease, and you always think about your
bacterial infections, like yersinia and tuberculosis, but we’re not seeing those in
developed countries.
But when I see ulcers in the small bowel, I do want to consider further testing,
just to make sure that we’re not missing anything. So, your options include capsule
endoscopy, CT, MRE, or the old-school, small bowel follow-through. We do know
that capsule endoscopy is the most sensitive test because it can actually get direct
visualization.
In Crohn’s disease, we use scoring systems, like the Lewis scoring system, where
we look at the villous appearance on a capsule, look for ulcers, and look for stric-
tures. Capsule endoscopy has a pretty high negative predictive value; when you’re
looking at Crohn’s disease, it’s almost a hundred percent; I think it’s around 95%.
So, in this case, if the patient has never had bowel obstructions or any abdominal
surgeries, I’d probably go with a capsule endoscopy to evaluate the remainder of the
small bowel.
Now, if capsule endoscopy is not available, CTE and MREs have become more
popular. And there have been studies looking at our ability to diagnose or define
Crohn’s disease using enterography studies. What you’re looking for, on CTE, is
mucosal enhancement, fat stranding, and hyper-enhancement as well.
Dynamic MRIs have also been used in our institution. When they’re looking at
the contrast in the ileum, they can actually see how well the ileum is moving. So, not
only can they determine if there’s an inflammatory component of the Crohn’s, but
they can actually see if there’s any fibrotic appearance. So that’s some exciting, new
technology we can apply to our patients with Crohn’s disease. But typically, I would
do a capsule endoscopy in this case, but I think a CT or MR would be acceptable
as well.
The patient has a capsule endoscopy, which is otherwise negative.
Would you do any further testing, do any treatment, or simply follow the
patient at this point?
If I’m not concerned about Crohn’s disease, I would simply tell the patient to
avoid NSAIDs. NSAID damage can happen pretty quickly, within 10 days, accord-
ing to the Crohn’s disease literature. If I’m not concerned about Crohn’s disease, I
will not treat them. Some practitioners will give a course of Entocort, 9 mg a day,
and try it for a couple of months. But I would only do this if I think the patient might
have mild Crohn’s disease and has ongoing symptoms.
Chapter 46
Concern About Pneumocystis jirovecii

Poonam Beniwal-Patel

A 32-year-old female presents with acute ulcerative colitis, requiring admission and
a course of IV steroids. She improves markedly within a couple of days, and she is
able to be discharged on prednisone 40 mg a day. You begin therapy with infliximab
after sending off hepatitis serology, a QuantiFERON assay, and TPMT. A few days
after the TPMT returns normal, azathioprine is added. The patient’s dose of predni-
sone has been dropped to 20 mg.
Would you agree with adding azathioprine in a patient still receiving pred-
nisone along with infliximab? Would you wait until prednisone is tapered off?
If you do start azathioprine, along with infliximab and prednisone, would you
start prophylaxis for pneumocystis (in the face of triple immunosuppressive
therapy?)
The SONIC trial showed that starting patients on azathioprine when beginning
infliximab led to higher steroid-free remission. Additional data has shown that the
use of an immunomodulator with antitumor necrosis factor therapy decreases the
risk of drug antibody formation [1]. Therefore, in a patient with aggressive disease
who is starting infliximab, I would start azathioprine as an outpatient without wait-
ing for prednisone to be completely tapered off.
However, over the last year or two, there have been studies that show that if you
treat someone with infliximab monotherapy and then they develop antibodies, it is
possible to add azathioprine at that time and usually be able to recapture them by
eliminating their antibodies [1, 2].
That’s really interesting data, because the question arises whether you really
have to start azathioprine at the outset. Certainly, I will say that for patients with
severe ulcerative colitis who are at high risk for a colectomy, I will definitely start

P. Beniwal-Patel (*)
Medicine, Gastroenterology/Hepatology Division, Medical College of Wisconsin,
Milwaukee, WI, USA
e-mail: [email protected]

© The Author(s), under exclusive license to Springer Nature 237


Switzerland AG 2023
W. H. Sobin et al. (eds.), Managing Complex Cases in Gastroenterology,
https://doi.org/10.1007/978-3-031-48949-5_46
238 P. Beniwal-Patel

azathioprine at the same time as infliximab. These patients, especially if they’ve


been hospitalized, have proven that they have aggressive disease. We don’t have
time for them to develop antibodies and then reverse them.
So, in this patient, I would start azathioprine, start it at the same time that she
receives infliximab, and not wait until prednisone is tapered off. With the long half-­
life of azathioprine, it actually takes about 8 weeks for it to reach its steady state. So,
even though we’re starting the azathioprine at the same time as the other two, it
really doesn’t kick in full effect until the prednisone dose is leveling off.
As to whether you need prophylaxis for PJP, there have been some small cohort
studies, looking at the rate of PJP in patients who have IBD but do not have
HIV. There was one study, about 10 years ago, that found the absolute risk to be
only about 0.07% of developing PJP in non-HIV IBD patients [3].
In general, it is high-dose, long-duration prednisone scenarios in older patients
where you want to prophylax against PJP. But in our patients, we really start taper-
ing them after the first week. So, if it’s a patient in whom I’m tapering off steroids,
I do not prophylax them. On the other hand, if I have a patient who’s older than 55
who I’m meeting for the first time and they’ve been on steroids for 3–4 months, then
that’s someone who I may consider doing a PJP prophylaxis, especially if they have
underlying lung disease. So, it’s really on a case-by-case basis. But most of the time,
I do not prophylax. them.

References

1. Villareal EM, Yarur AJ. Better late than never: adding thiopurines after loss of response
to infliximab monotherapy. Dig Dis Sci. 2021;66(9):2851–2. https://doi.org/10.1007/
s10620-­020-­06681-­w. Epub 2020 Oct 31.
2. Zeze K, Hirano A, Torisu T, Esaki M, Moriyama T, Umeno J, Kawasaki K, Fujioka S,
Fuyuno Y, Matsuno Y, Kitazono T. Adding thiopurine after loss of response to infliximab
versus early combination in treating Crohn’s disease: a retrospective study. Dig Dis Sci.
2021;66(9):3124–31. https://doi.org/10.1007/s10620-­020-­06600-­z. Epub 2020 Sep 13.
3. Sierra CM, Daiya KC. Prophylaxis for Pneumocystis jirovecii pneumonia in patients with inflam-
matory bowel disease: a systematic review. Pharmacotherapy. 2022;42(11):858–67. https://doi.
org/10.1002/phar.2733. Epub 2022 Oct 25. PMID: 36222368; PMCID: PMC9828113.
Chapter 47
Bloating in Crohn’s

Poonam Beniwal-Patel

A 42-year-old female with a history of Crohn’s disease well-controlled on infliximab


and azathioprine comes in for complaints of diarrhea and bloating. She had a prior
terminal ileal resection (20 cm) for her Crohn’s. Her CRP and fecal calprotectin are
normal. A colonoscopy to the neoileum is normal.
Is there any treatment you would consider?
First, if there’s significant diarrhea or bloating, and this is relatively new, I would
do cross-sectional imaging, like a CTE or MRE, to make sure there’s no upstream
disease activity that we’re not capturing on a colonoscopy.
If that comes back negative, then I’d be interested in knowing what kind of anas-
tomosis they have, because patients who have a side-to-side anastomosis do have a
higher rate of SIBO compared to those with an end-to-end anastomosis.
If there’s no active disease on cross-sectional imaging, you could try cholestyr-
amine. But be judicious with it; I would just give it once a day. Also, you could try
treating with rifaximin, just empirically for SIBO, given that they have had an
abdominal intestinal surgery, which is a risk factor for SIBO.
The patient is started on cholestyramine, 4 g, once daily, and diarrhea improves
but bloating gets worse. A CTE shows some areas of small bowel dilation without
significant stricture.
Would you consider doing a capsule in someone like this?
I think it would be reasonable to do, but I certainly would do a patency capsule
before that. The interesting thing is that to have dilation, there’s one of three things
that could cause that. There might be a mechanical stricture, there could be

P. Beniwal-Patel (*)
Medicine, Gastroenterology/Hepatology Division, Medical College of Wisconsin,
Milwaukee, WI, USA
e-mail: [email protected]

© The Author(s), under exclusive license to Springer Nature 239


Switzerland AG 2023
W. H. Sobin et al. (eds.), Managing Complex Cases in Gastroenterology,
https://doi.org/10.1007/978-3-031-48949-5_47
240 P. Beniwal-Patel

aperistalsis at that point, or it could be that we’re viewing the x-ray in the middle of
a peristaltic wave and it’s not truly a dilation. I would also consider a small bowel
follow-through in order to get a functional assessment of the area.
Would you bother doing a breath test looking for SIBO or would you just
treat with rifaximin?
In this case, because rifaximin is relatively low risk, I’d treat it empirically. If this
occurs again, then I’d perform testing for SIBO before retreating.
Chapter 48
Severe Diarrhea in a Patient
on Chemotherapy

Amir Patel

A 57-year-old male is admitted to the oncology service with severe diarrhea. He has
been having diarrhea for weeks. The patient is on chemotherapy for metastatic lung
cancer but is not receiving a checkpoint inhibitor. You look up the drugs he is receiv-
ing and see that they may cause diarrhea but do not tend to cause colitis. His chemo
has been on hold since his diarrhea started, but his oncologists are anxious to
restart it. His stool cultures are negative. He had his last screening colonoscopy 4
years ago, just before his lung cancer was first diagnosed, and it was
unremarkable.
Obviously if you’re consulted on a patient receiving checkpoint inhibitors
and having bad diarrhea, you’re going to do a colonoscopy, looking for the
inflammatory bowel disease-type phenotype. Do you ever do that with patients
who are on other chemotherapy?
We've been seeing a lot of checkpoint inhibitor therapy causing colitis in the last
decade, but we have also found that some other chemotherapy medications that are
not checkpoint inhibitors can cause an immune-mediated colitis as well.
So, a lot of times the oncologists will send these patients to us, and we will do a
colonoscopy just to make sure that there's no immune-mediated colitis occurring. If
that does happen, we treat it like checkpoint inhibitor colitis. The oncologists are
pretty vigilant about this, and a lot of times when we see these patients, they’re
already on steroids. If they have a response to prednisone, that tells you that this is
likely some type of autoimmune-mediated colitis, even if the biopsies are not
specific.

A. Patel (*)
Dept. of Medicine, Division of Gastroenterology and Hepatology, Medical College of
Wisconsin, Milwaukee, WI, USA
e-mail: [email protected]

© The Author(s), under exclusive license to Springer Nature 241


Switzerland AG 2023
W. H. Sobin et al. (eds.), Managing Complex Cases in Gastroenterology,
https://doi.org/10.1007/978-3-031-48949-5_48
242 A. Patel

Together, the oncologists and gastroenterologists have generated some guide-


lines and have good data on the use of anti-TNFs as well as anti-integrins. We tend
to see a robust response to anti-TNFs and Entyvio for treatment of these medication-­
induced autoimmune immune-mediated colitis cases.
To treat checkpoint inhibitor colitis, usually it takes one or two doses of an anti-­
TNF, at the standard 5 mg per kg dosing of infliximab, 2 weeks apart. Sometimes,
one dose alone is enough to knock out the colitis; sometimes you need two. And for
vedolizumab, 300 mg IV, one to two doses can be quite effective.
I think the oncologists are actually looking at what they can do preventatively.
There are no good studies on this, but they’re studying this particular question: if
someone is going to be put on a checkpoint inhibitor, should they be started on an
anti-TNF or anti-integrin therapy prophylactically?
You decide to do a colonoscopy to evaluate the diarrhea, and you find mild dif-
fuse erythema and some minimal friability involving portions of the colon. The
appearance is nonspecific. Although it might be a milder IBD, the appearance is
consistent with infection or possibly a drug-induced colitis. Biopsies are not diag-
nostic for IBD and are rather nonspecific.
Would you treat for IBD?
With these findings, I would not give therapy for IBD if the patient was not on a
checkpoint inhibitor. We’ll give them antidiarrheals, but a lot of times, if the diar-
rhea is really debilitating, the oncologist will be changing the choice of
chemotherapy.
Chapter 49
Microscopic Colitis

Poonam Beniwal-Patel

A 67-year-old female presents with diarrhea. She had a screening colonoscopy 2


years earlier that was negative. Her only medical problem is hyperlipidemia. Her
meds include simvastatin and baby ASA. A repeat colonoscopy is performed, which
is grossly normal, but biopsies are positive for lymphocytic colitis. She is started on
budesonide 9 mg a day, and her diarrhea resolves. After a month, her gastroenter-
ologist tapers her dose to 6 mg, and she does well. But every time she tries to drop
the dose further, the diarrhea worsens. She is referred to you for a second opinion.
What would be your initial approach upon seeing her, and would you stop
her statin, since statins have been associated with microscopic colitis?
The first thing I think about, even though it sounds like she responded to
budesonide initially, is to check a celiac panel, because patients with microscopic
colitis are at increased risk of developing celiac disease.
Her celiac panel returns negative. Her IgA level is normal.
I would not stop her statin, because even though there is an association between
statins and microscopic colitis, the statin clearly has cardiovascular benefits that are
more important.
Would you keep a patient on budesonide 3 mg or even 6 mg long term?
Yes, sometimes I do; if we need to, then we need to. With lymphocytic colitis,
sometimes they’re flaring, and I’ll keep them on 6 mg for 6 months, bring them
back, and then try to drop them to 3 mg. So, it’s definitely not a rapid taper. And I’ve
actually had pretty good success with that. I find that keeping them on the higher
dose for longer periods, not just a month, makes it much more likely they will stay
in remission and are not as likely to rebound.

P. Beniwal-Patel (*)
Medicine, Gastroenterology/Hepatology Division, Medical College of Wisconsin,
Milwaukee, WI, USA
e-mail: [email protected]

© The Author(s), under exclusive license to Springer Nature 243


Switzerland AG 2023
W. H. Sobin et al. (eds.), Managing Complex Cases in Gastroenterology,
https://doi.org/10.1007/978-3-031-48949-5_49
244 P. Beniwal-Patel

The other thing I will sometimes do in these scenarios is drop the budesonide
dose and use loperamide for breakthrough diarrhea. And then, if this works, I’ll wait
6 months before trying to bring the dose down further. I've had pretty good success
with this strategy. However, there are some patients who develop bad diarrhea and
electrolyte abnormalities, because their diarrhea is so severe. Those patients I’ll just
leave on higher-dose budesonide.
The good news is that there have been studies looking at metabolic bone disease
with budesonide, and thankfully, budesonide really does not impact bone health
significantly [1]. In patients with concurrent osteoporosis, I’ll loop in an endocri-
nologist, and generally I’ve never had pushback regarding budesonide use.
Have you ever had cases where you’ve had to go to stronger medications?
Not for purely lymphocytic colitis. I can only think of one patient who had a
family history of ulcerative colitis and was initially diagnosed with lymphocytic
colitis. But then years later, things evolved. We repeated the scope, and now, it
looked like it was ulcerative colitis. So, we ended up putting that person on a bio-
logic. Rarely, there are indications for placing patients on a biologic, such as
vedolizumab.

Reference

1. Reilev M, Hallas J, Thomsen Ernst M, Nielsen GL, Bonderup OK. Long-term oral budesonide
treatment and risk of osteoporotic fractures in patients with microscopic colitis. Aliment
Pharmacol Ther. 2020;51(6):644–51. https://doi.org/10.1111/apt.15648. Epub 2020 Jan 30.
Chapter 50
Miscellaneous Questions About IBD

Daniel Stein, Salina Faidhalla, and Amir Patel

1. Do you ever try a second anti-TNF if the first one didn’t work? What if the
first one was adalimumab? What if it was infliximab?
In a primary nonresponder to adalimumab, we would try infliximab but not the
reverse. The thought is that the IV infusion and high-peak drug levels that we can
achieve with infliximab are beneficial. In general, Humira seems to be underdosed
in comparison with infliximab. We have much bigger dosing flexibility; with inflix-
imab, you can really ramp those levels up. So, in general, if they failed infliximab,
we won’t really consider Humira. That being said, we do have some patients that are
on Humira after failing infliximab and are doing well, but it’s not something I would
do today.
Now, if a patient has had a response to one agent and then developed antibodies,
(a secondary nonresponder), we would try another anti-TNF, because there is no
overlap with the antibodies. However, we would suggest adding an IM with the
second anti-TNF.

D. Stein (*)
Division of Gastroenterology and Hepatology, Department of Internal Medicine, Medical
College of Wisconsin, Milwaukee, WI, USA
e-mail: [email protected]; [email protected]
S. Faidhalla
Department of Medicine Division of Gastroenterology and Hepatology, Medical College of
Wisconsin, Milwaukee, WI, USA
e-mail: [email protected]
A. Patel
Dept. of Medicine, Division of Gastroenterology and Hepatology, Medical College of
Wisconsin, Milwaukee, WI, USA
e-mail: [email protected]; [email protected]

© The Author(s), under exclusive license to Springer Nature 245


Switzerland AG 2023
W. H. Sobin et al. (eds.), Managing Complex Cases in Gastroenterology,
https://doi.org/10.1007/978-3-031-48949-5_50
246 D. Stein et al.

2. If patients are responding well to biologics, do you ever try to get them off
these drugs?
We would consider dose de-escalation, if possible, for a patient who has been in
deep remission for a long time (clinical remission is not sufficient). However, stop-
ping the medication completely will risk flaring up disease, and the medication may
not be effective if restarted.
The recurrence rate after stopping biologics is about 85% in 5 years.
In a large meta-analysis by Torres et al. [1], to evaluate the effect of de-­escalation/
stopping immunomodulators or anti-TNFs in IBD patients, relapse rates after cessa-
tion appear high across all therapeutic classes. After stopping IM, only 15%–37%
of patients maintained clinical remission after 5 years.
That meta-analysis also showed that approximately 40%–50% of patients who
discontinue anti-TNFs will experience a relapse within 2 years, and studies with
longer follow-up of 7 and 10 years after withdrawal show only 35% and 12%,
respectively, remain in remission at those time intervals [2, 3].
You can consider stopping biologics in patients who are in remission and have
low drug levels, because their remission might not be drug related. This can be con-
sidered if they are in deep remission. On the other hand, for dose de-escalation, we
usually let drug level guide that.
If the patient is insisting on stopping medication: then we need deep remission,
at least two normal colonoscopies, with normal histology 2 years apart before stop-
ping medication. Once the drug is stopped, we closely follow fecal calprotectin
every 6–12 months and follow up colonoscopies every 2 years.
In general, we do not proactively stop anti-TNFs. But if a patient has concerns
and wants to stop it, or if there’s some borderline reason to stop it, for example, if
someone is getting recurrent minor infections or recurrent UTIs where you wouldn’t
otherwise stop it, it might be worth trying.
But the caveat is that anytime you stop medications, it has to be somebody who
you trust will follow up and be willing to have surveillance colonoscopies. So, in
short, we avoid stopping biologic agents proactively and generally attempt to get
patients off immune modulators first.
Amir Patel
It is accepted dogma that adding azathioprine to infliximab enhances bene-
fit. But has adding azathioprine to adalimumab been shown to be beneficial?
The original Sonic study looked at the combination of azathioprine and inflix-
imab and showed efficacy. They did not study adalimumab and azathioprine. But,
more recently there was a study looking at Humira with azathioprine, which did
show superiority compared to Humira alone [4].
When I use combination therapy, I’m typically combining anti-TNFs and immu-
nologics in someone who has severe disease. So, for someone with fistulizing dis-
ease or severe colitis, we use combination therapy to reduce the risk of developing
antibodies, and we know that adding the immunologics to the anti-TNFs can also
boost the drug levels.
50 Miscellaneous Questions About IBD 247

If a patient fails an anti-TNF in ulcerative colitis, it is my impression that


ustekinumab is a good second choice, JAK inhibitors are a good second choice,
but vedolizumab doesn’t work. Do you agree?
I would ask: what is the severity of disease and what is the context? Entyvio is a
great drug. It just doesn’t have a lot of good induction data. It works slower. So
that’s why, if speed is of the essence in treating someone’s disease, a lot of us will
probably go toward an IL-23 or a JAK inhibitor as our second line.
Is your opinion about Entyvio as second-line treatment for patients who fail
anti-TNFs in Crohn’s the same as for UC?
A lot of us have the opinion that Entyvio works better for colonic disease. So, if
someone has a stricture in the ileum or fistulizing disease, then Entyvio would not
be our preferred choice.
Is infliximab superior to adalimumab in UC and in Crohn’s?
There’s been no head-to-head study looking at that question, but I would say that
for ulcerative colitis, Remicade is probably superior to Humira. A lot of that is
because with Humira, you’re just giving a flat dose, while with Remicade, you can
adjust the dose by weight. And some of these patients will have a protein-losing
enteropathy, where they’re losing albumin and protein and potentially the drug
itself. So, for ulcerative colitis, I would say that there’s no question Remicade is
superior.
For Crohn’s disease, I guess it would depend on the context of the disease itself.
So, if someone came to me and they had fistulizing disease and had multiple
surgeries, then I would say, yes, Remicade is probably going to be a superior drug
for that same reason and that you can give them a dose based on their weight and
adjust it easier. The problem with Humira has always been that the only adjustment
you can make is giving it weekly or every 2 weeks.
And the data on therapeutic drug monitoring with Humira is not as well validated
as it is with Infliximab. That’s why Remicade, in our opinion, has always been supe-
rior to Humira, in which you can always adjust the dose. They can get 10 mg every
4 weeks, or every 8 weeks. Some of us have gone as high as 15 mg every 4 weeks
just to get them at this nice therapeutic drug range. A lot of us would probably say
that Remicade is superior to Humira.
Do you feel relatively safe using AZA in young males for periods under
2 years?
Yes, I think I feel safe doing that. And a lot of that relates to the context of the
disease. So, if a young man comes to me and he’s had multiple surgeries and a his-
tory of fistulas, there’s no question; I’m going to give him combination therapy,
because the risk of the untreated disease far outweighs the small risk of that rare
hepatosplenic T cell lymphoma.
Another question, and it’s along a similar narrative, is when can we de-escalate
therapy. So, for someone on combination therapy, when can we stop the azathio-
prine? And a lot of us would do it after 2 years, but there have been some recent
studies on de-escalating therapy within a year.
248 D. Stein et al.

So, someone’s on combination therapy, and they’re doing great. The colonos-
copy shows deep remission, where they don’t have any activity endoscopically or
under the microscope. Then, the question is, can we just stop the azathioprine? And
some of us will do that and that’ll be based on drug levels.
Where do you stand on the newer drugs? How much do ozanimod, risanki-
zumab, and upadacitinib add to the mix? Do you think they are a good
addition?
Yes, a lot of the experts are very optimistic, particularly about risankizumab and
upadacitinib. As a result, we have been using them a lot. In one of your earlier
cases—the young lady with proctitis who didn’t want to take suppositories—I was
going to say that if she’s away at college, you can offer her some of these new oral
therapies, like ozanimod (Zeposia), tofacitinib (Xeljanz), or upadacitinib (Rinvoq).
She would’ve been a perfect candidate for ozanimod because she was just failing
mesalamine. And the studies actually looked at a subset of patients with colitis who
had failed mesalamine, and patients on Zeposia did very well.
The reason why we’re enthusiastic about Skyrizi and Rinvoq is because a lot of
the studies these days are not only looking at how these patients respond clinically
but how they respond histologically and endoscopically. So, more of the studies
over the last decade have looked at mucosal healing as an endpoint, because we do
know that if you can achieve that deep remission, it reduces the risk of flares, hos-
pitalizations, and colon cancer risk, and some of the data on Skyrizi and Rinvoq
have shown early induction data and the ability to heal the mucosa.
A lot of IBD experts are considering the use of Skyrizi first line for Crohn’s dis-
ease and Rinvoq first line for ulcerative colitis. But we have to remember, Rinvoq
right now is being marketed only for patients with colitis who have failed anti-TNF
Is vedolizumab a godsend because of its lack of side effects and concerns? Is
it strong enough?
Yes, vedolizumab is a great drug because of its safety profile, but I think that the
induction therapy is what concerns a lot of us. It takes a certain type of patient to
consider Entyvio, someone with moderate ulcerative colitis or moderate Crohn’s
disease who isn’t on the verge of needing surgery. Also, older patients who typically
don’t present with a severe disease are candidates for Entyvio. But the question is,
will the therapy work? And I don’t think we have an answer for that, but we’re look-
ing into data and doing studies, where maybe we can do a blood test or have a stool
sample or a mucosal sample that could inform us whether this person is going to
respond to anti-TNF or that person’s going to respond to anti-integrin therapy.
But we’re not there yet. Hopefully, we’ll get there soon. But yes, I think Entyvio
is a great drug. It just depends on the person that I’d use it on. So, for someone who
has severe disease, heading for surgery, I probably wouldn’t use it. But if their dis-
ease is on the more moderate side, then, yes, I would.
Daniel Stein, Salina Faidhalla
Do you see any differences in the ways you manage IBD from the way gas-
troenterologists in the community do? Are there any general recommendations
you would make?
50 Miscellaneous Questions About IBD 249

Yes, there are a few. First, we see too much use of mesalamine in patients with
Crohn’s disease. Second, we find that community practitioners are occasionally too
conservative with their treatment. Part of this may be an under-recognition of the
severity of the disease. In addition, community practitioners tend to be hesitant to
use advanced therapies. Although there are some risks with the use of advanced
therapies the risk/benefit ratio remains quite low. Biologics, including the small
molecules, should be strongly considered in patients with IBD who have moderate
to severe disease.
We also see many UC patients treated with adalimumab, which is usually not a
first choice for us; we prefer infliximab.
And, finally, we think there is some under-recognition of primary nonresponse,
defined as no significant improvement without the aid of steroids. Too often patients
are continued on agents for long periods of time despite being ineffective or requir-
ing repeated doses of steroids.

References

1. Torres J, Boyapati RK, Kennedy NA, Louis E, Colombel JF, Satsangi J. Systematic review
of effects of withdrawal of immunomodulators or biologic agents from patients with inflam-
matory bowel disease. Gastroenterology. 2015;149(7):1716–30. https://doi.org/10.1053/j.gas-
tro.2015.08.055. Epub 2015 Sep 14.
2. Steenholdt C, Molazahi A, Ainsworth MA, Brynskov J, Østergaard Thomsen O, Seidelin
JB. Outcome after discontinuation of infliximab in patients with inflammatory bowel disease
in clinical remission: an observational Danish single center study. Scand J Gastroenterol.
2012;47(5):518–27. https://doi.org/10.3109/00365521.2012.660541. Epub 2012 Mar 1.
3. Waugh AW, Garg S, Matic K, Gramlich L, Wong C, Sadowski DC, Millan M, Bailey R,
Todoruk D, Cherry R, Teshima CW. Maintenance of clinical benefit in Crohn’s disease patients
after discontinuation of infliximab: long-term follow-up of a single Centre cohort. Aliment
Pharmacol Ther. 2010;32(9):1129–34.
4. Matsumoto T, Motoya S, Watanabe K, Hisamatsu T, Nakase H, Yoshimura N, Ishida T,
Kato S, Nakagawa T, Esaki M, Nagahori M. Adalimumab monotherapy and a combination
with azathioprine for Crohn’s disease: a prospective, randomized trial. J Crohn's Colitis.
2016;10(11):1259–66.
Part III
Disorders of Gut–Brain Interaction
Chapter 51
Introduction to Disorders of Gut-Brain
Interaction

W. Harley Sobin

The challenges of treating IBS-D, IBS-C, functional constipation, functional dys-


pepsia, bloating, and functional abdominal pain are discussed in multiple case pre-
sentations. A thorough review of medications, dietary therapy, and the interplay
between anxiety, depression, and severity of symptoms is presented.
The use of eluxadoline, alosetron, rifaximin, IBgard, linaclotide, lubiprostone,
prucalopride, buspirone, mirtazapine, and other drugs is reviewed. Tests involved in
the diagnosis of these functional syndromes are discussed. Bloating, distension, and
the interplay with the viscerosomatic reflex are explained. Tips for managing pain-
ful gas are outlined as well as the use of antidepressants in treating these patients
and how to manage abdominal cramping.
Other issues covered in these case discussions include the following: Is it neces-
sary to perform a colonoscopy to diagnose IBS-D? What other tests should be
ordered in the patient with presumed IBS-D? What tests may be helpful in a patient
with severe constipation? How best to manage constipation in a patient on chronic
opiates, or a patient with possible outlet dysfunction. How to manage functional
dyspepsia both postprandial distress syndrome (PDS) and epigastric pain syndrome
(EPS) and how to treat abdominal wall pain.
Multiple causes of bloating are discussed along with potential therapies. The use
of central neuromodulators in treating patients with hard-to-manage disorders of
gut-brain interaction is discussed at length.

W. H. Sobin (*)
Division of Gastroenterology and Hepatology, Department of Medicine, Medical College of
Wisconsin, Milwaukee, WI, USA
e-mail: [email protected]

© The Author(s), under exclusive license to Springer Nature 253


Switzerland AG 2023
W. H. Sobin et al. (eds.), Managing Complex Cases in Gastroenterology,
https://doi.org/10.1007/978-3-031-48949-5_51
Chapter 52
IBS-D

W. Harley Sobin and Patrick Sanvanson

Case 1 A 32-year-old female presents with a history of weekly recurrent loose


bowel movements over the past year, along with associated abdominal cramps. She
works as a paralegal in a high-pressure law firm. She is on no medicines. Her fre-
quent bathroom trips during the work day are now embarrassing her, so she sees
her primary care physician. Her exam is unremarkable. Labs reveal a normal CBC,
CRP, TSH, iron panel/ferritin, and fecal calprotectin. Her primary care doctor
thinks she has IBS-D but is worried she might have inflammatory bowel disease
(IBD) and sends her for a second opinion, asking whether she should have a
colonoscopy.
Is a colonoscopy necessary to rule out IBD and diagnose IBS?
No, the normal lab results all weigh against the diagnosis of IBD. The history,
physical exam, and labs are all consistent with the diagnosis of IBS-D. A colonos-
copy is not necessary at this time to confirm this diagnosis.
Are there other labs necessary to evaluate the patient before treating her?
Celiac serology is indicated in the evaluation of patients with suspected IBS-D
[1, 2]. On occasion, we will obtain an abdominal X-ray upon initial presentation to
confirm there is not an excessive stool burden and that we are not dealing with an
overflow diarrhea situation.
Celiac serology returns negative. Abdominal X-ray demonstrates no increased
stool burden. You suggest dietary manipulations, including avoiding caffeine, lac-
tose, artificial sweeteners, and processed meats, which does not help much. You
discuss a low-FODMAP (fermentable oligosaccharides, disaccharides, monosac-
charides, and polyols) diet, which she tries but also gets no benefit from this

W. H. Sobin (*) · P. Sanvanson


Division of Gastroenterology and Hepatology, Department of Medicine, Medical College of
Wisconsin, Milwaukee, WI, USA
e-mail: [email protected]; [email protected]

© The Author(s), under exclusive license to Springer Nature 255


Switzerland AG 2023
W. H. Sobin et al. (eds.), Managing Complex Cases in Gastroenterology,
https://doi.org/10.1007/978-3-031-48949-5_52
256 W. H. Sobin and P. Sanvanson

intervention. You have prescribed Imodium (loperamide), which helps a little bit
with the diarrhea but not the cramping. She is worried about taking this on a chronic
basis, even though you assure her that it is okay.
What other treatments can you consider?
We may try psyllium-based fiber to see if it helps with regulating stools.
Antispasmodics, like dicyclomine and hyoscyamine, have been part of our general
armamentarium for decades. However, studies proving their efficacy are limited.
The latest ACG (American College of Gastroenterology) guidelines [1] did not sup-
port their use, although more recent AGA guidelines [3] did. In our practice, we
continue to use antispasmodics in younger patients. We worry more about their
anticholinergic side effects in the elderly and are reluctant to use them in those
patients. Other drugs that we will try include eluxadoline (Viberzi) [4], peppermint
oil in the form of IBgard [5], rifaximin [6], cholestyramine [7, 8], and alose-
tron [9–11].
Eluxadoline [4] works on opioid receptors to decrease bowel frequency and
abdominal pain, and we find that it helps a few of our patients with IBS-D. It is
contraindicated in patients who have had a cholecystectomy. If we have a patient
with presumed IBS-D who has had a cholecystectomy, we will always try cholestyr-
amine first or obtain a 48-h stool collection for bile acids. Many of these patients
will have bile-salt-related diarrhea and respond to this treatment [7, 8]. However, if
the patient has an intact gallbladder, no past history of pancreatitis, and is not a
heavy drinker, we will try eluxadoline.
IBgard is another drug worth trying [5]; it is an OTC medication containing pep-
permint oil. Rifaximin [6] will work to relieve symptoms in some patients with
IBS-D. This is particularly true if your patient has had reason to have a hydrogen
breath test and the results were consistent with SIBO. A breath test is not required
before trying rifaximin, but it is those patients with IBS-D and a positive breath test
who are most likely to respond. However, rifaximin is quite expensive, requires
repeat treatment, and is often not covered by insurance. When starting any new IBS
therapy, we ask patients to give us an update in 2–4 weeks.
The patient tries dicyclomine, eluxadoline, IBgard, and even rifaximin without
much benefit. She describes her frustration to you and expresses feelings of hope-
lessness and anxiety and says she is starting to feel depressed.
What is the association between anxiety, depression, and IBS, and how do
you manage it?
For patients with debilitating IBS, we let them know that we do not think their
physical symptoms are simply manifestations of depression. There are several
points we emphasize. First, it is natural to experience feelings of anxiety and even
depression when physical symptoms continue unabated. Second, as physical symp-
toms continue, they tend to get embedded in the patient’s psyche and lead to soma-
tization, where a patient becomes obsessed with those physical symptoms. It is also
common for patients to catastrophize, thinking in terms of gloom and doom regard-
ing their ailments. We will typically try getting these patients to see a mental health
provider, who provides the tools to help with these unhelpful behavioral patterns.
52 IBS-D 257

Cognitive behavior therapy is particularly useful in many of these patients [12].


Learning that they can control their thought processes can also benefit the level of
their IBS.
Will you ever prescribe antidepressants for these patients?
We are comfortable using tricyclics in patients with IBS-D [13] as central neuro-
modulators, not as antidepressants. Using a relatively low dose of nortriptyline
helps decrease diarrhea and abdominal pain [14]. The dose we’re using is much less
than the usual antidepressant dose. We usually don’t prescribe an SSRI in IBS-D,
because SSRIs tend to increase diarrhea (with the exception of paroxetine (Paxil),
which is somewhat constipating but has a number of problems associated with its
use). Our tricyclic of choice is nortriptyline. We think it is better tolerated than ami-
triptyline. But in patients with severe diarrhea, amitriptyline may be preferable.
When we prescribe these drugs, we tell the patient that we are not using them to
treat depression; we are giving them to treat the brain-gut connection that is out of
kilter in IBS. Prior to starting a tricyclic, an EKG is obtained to confirm the absence
of baseline QTc prolongation and is subsequently monitored with dose adjustments.
The patient is placed on nortriptyline 25 mg at bedtime, which is then increased
to 50 mg. Her symptoms do improve. She is pleased that she is feeling somewhat
better but wonders if there are any other specific GI drugs that might benefit her.
One other medicine worth considering is alosetron [9–11]. This is approved only
for females with IBS-D who have failed other therapies. We have had some great
success using alosetron in women with intractable symptoms of IBS-D. We do
review the background history of the drug with our patients. It was taken off the
market years ago because of cases of severe constipation and cases of bowel isch-
emia. But, at that time, the drug was being given in higher doses and being used in
older patients who probably should not have been receiving it in the first place. So,
now, we use lower doses and avoid its use in anyone who is at increased risk of
ischemic colitis.
After trying alosetron, at the dose of 0.5 mg twice daily, the patient quickly feels
marked relief of her diarrhea and abdominal pain. After a month on the medicine,
she feels back to normal. You gradually taper her off of the nortriptyline while main-
taining on alosetron. After tapering nortriptyline off, she has some increased diar-
rhea that rapidly responds to an increase in alosetron to 1 mg bid.
Case 2 A 43-year-old female presents with increased abdominal cramps and diar-
rhea over the past 8 months. She gives a long history of frequent, somewhat soft,
bowel movements since she was in her 20 s. The fact that she moved her bowels 2–3
times a day never bothered her. One year ago, she divorced her husband of 22 years
after learning of an affair he was having. Since then, she has been having increased
abdominal cramps, along with even looser stools. There is no blood in her stool. She
takes no medications. She has no surgical or other past medical history. On the
physical exam, she winces when you press on her abdomen, but you think it is
abdominal wall pain. Indeed, you do a Carnett test and it is positive. Her exam is
otherwise negative. You decide to order a CBC, CRP, fecal calprotectin, iron panel/
ferritin, and celiac panel. These tests all return negative.
258 W. H. Sobin and P. Sanvanson

How would you manage her abdominal wall pain?


We have tried lidocaine patches in many, gabapentin in a few, and have even sent
a couple of patients to anesthesia pain specialists for local nerve root injections for
presumed anterior cutaneous nerve entrapment [15].
The patient’s abdominal wall pain improves after you explain its benign nature
and after starting lidocaine patches. However, her abdominal cramping increases,
generally accompanying her loose bowel movements.
How would you manage her loose stools and abdominal cramps?
This patient has a long history of frequent bowel movements, and now they are
associated with increased cramping, which started at the time of her divorce. The
history is very consistent with IBS-D. Many patients have long-standing symptoms
of IBS and never see a physician, because they are not particularly alarmed or both-
ered by them. Those who do, however, often have associated anxiety or depression,
and a life-altering stress can often precipitate an exacerbation of symptoms. Mental
health counseling can really benefit a lot of patients.
Other things that could cause these symptoms include IBD, but the normal CBC,
CRP, iron studies, and fecal calprotectin make that unlikely. You could potentially
have normal labs with microscopic colitis, but she is young for this diagnosis and is
not taking medications that might precipitate microscopic colitis. However, micro-
scopic colitis may be considered, if she fails to respond to management of IBS-D.
So how would we manage her IBS-D? We might start with psyllium-fiber-based
products [1, 3, 8]. If these are not totally effective, we might next try dicyclomine
[3]. There is less concern about anticholinergic side effects in younger patients.
Besides helping with cramps, dicyclomine can slow diarrhea. We like to start with
10 mg before meals and at bedtime, although many recommend 20 mg. We find the
10 mg dose to be much better tolerated and will occasionally titrate up to 20 mg
if needed.
Loperamide is another option. In some people, loperamide will not only decrease
stool frequency but also decrease abdominal cramping. We start slowly with loper-
amide because occasional patients may have severe constipation with even one dose.
The patient is started on dicyclomine but stops it because of side effects, such as
dry mouth, and worry about altered vision. You have her try loperamide, which she
finds slows her bowel frequency but does nothing for her cramping.
What would you try next?
We like to try eluxadoline [4] in those with an intact gallbladder and cholestyr-
amine [7, 8] in those without. We will also have them try IBgard [5]. Occasionally,
we will order rifaximin [6], particularly in those who might be predisposed to hav-
ing small bowel bacterial overgrowth. We have had more problems getting rifaximin
approved and, because of its expense, do not order it that often. Occasionally, we
will try cholestyramine in someone with an intact gallbladder. Some patients may
have a defective bile salt transporter in their terminal ileum [8]. But patients with an
intact gallbladder are much less likely to benefit from cholestyramine than those
who have had a cholecystectomy.
Unfortunately, response to any one medication can be unpredictable, and trial
and error is required. As mentioned above, alosetron is our go-to in women who do
not respond to other remedies.
52 IBS-D 259

The patient fails to improve with eluxadoline and IBgard. Rifaximin is not tried
because it is not covered and is too expensive. She refuses to try alosetron because
of fears of ischemic colitis even after being reassured that it is extremely uncommon.
Her bowel frequency is reasonable with use of loperamide, and her main complaint
is severe cramping. The cramping is now worse whenever she moves her bowels,
whenever she feels stressed by things like deadlines, and whenever she feels
like crying.
How would you manage her pain?
If she would agree to it, we would give her a trial of the SNRI (serotonin and
norepinephrine reuptake inhibitor) duloxetine. Duloxetine has been found helpful in
treating functional abdominal pain, and the fact that it is mildly constipating would
benefit her IBS-D [14, 16]. We would educate her about the gut-brain connection
and the possibility that she is suffering from a disorder of gut-brain interaction. The
use of central neuromodulators has been found useful in patients with IBS, even
with those who do not suffer from depression or anxiety, although this patient prob-
ably does.
In using drugs like SNRIs, TCAs, or SSRIs, it is necessary to explain to patients
that it generally takes a month for them to be truly operational, but unfortunately,
side effects may occur immediately [14]. We start with a lower dose and then titrate
up to the desired dose. We also strongly recommend that the patient see a mental
health specialist for cognitive behavioral or other therapy as an adjunct in treating her.
She agrees to take duloxetine, starting at 30 mg a day and then, after 2 weeks,
going up to 60 mg a day. After 6 weeks, she is feeling better, and as a result, her
mood is also improved. But she is still having some breakthrough pain and com-
plains of insomnia.
She is doing better, but do you have other recommendations at this point?
An excellent augmenting drug, added to duloxetine, would be low-dose quetiap-
ine (Seroquel) [17]. A nighttime dose of 100 mg works to help decrease pain and
almost always reverses insomnia.
She adds quetiapine to duloxetine and feels great. She is seeing a counselor and
feels the best she has in years. She is also ready to start dating again.

References

1. Lacy BE, Pimentel M, Brenner DM, Chey WD, Keefer LA, Long MD, Moshiree B. ACG
clinical guideline: management of irritable bowel syndrome. Off J Am Coll Gastroenterol.
2021;116(1):17–44.
2. Smalley W, Falck-Ytter C, Carrasco-Labra A, Wani S, Lytvyn L, Falck-Ytter Y. AGA clinical
practice guidelines on the laboratory evaluation of functional diarrhea and diarrhea-­predominant
irritable bowel syndrome in adults (IBS-D). Gastroenterology. 2019;157(3):851–4.
3. Lembo A, Sultan S, Chang L, Heidelbaugh JJ, Smalley W, Verne GN. AGA clinical practice
guideline on the pharmacological management of irritable bowel syndrome with diarrhea.
Gastroenterology. 2022;163(1):137–51.
4. Lembo AJ, Lacy BE, Zuckerman MJ, Schey R, Dove LS, Andrae DA, Davenport JM, McIntyre
G, Lopez R, Turner L, Covington PS. Eluxadoline for irritable bowel syndrome with diarrhea.
N Engl J Med. 2016;374(3):242–53.
260 W. H. Sobin and P. Sanvanson

5. Cash B. Novel peppermint oil formulation for dietary management of irritable bowel syn-
drome. Gastroenterol Hepatol. 2015;11(9):631.
6. Pimentel M, Lembo A, Chey WD, Zakko S, Ringel Y, Yu J, Mareya SM, Shaw AL, Bortey E,
Forbes WP. Rifaximin therapy for patients with irritable bowel syndrome without constipation.
N Engl J Med. 2011;364(1):22–32.
7. Wedlake L, A’hern R, Russell D, Thomas K, Walters JR, Andreyev HJ. Systematic review:
the prevalence of idiopathic bile acid malabsorption as diagnosed by SeHCAT scanning in
patients with diarrhoea-predominant irritable bowe syndrome. Aliment Pharmacol Ther.
2009;30(7):707–17.
8. Camilleri M. Diagnosis and treatment of irritable bowel syndrome: a review.
JAMA. 2021;325(9):865–77.
9. Bardhan, Bodemar, Geldof, Schütz, Heath, Mills, Jacques. A double-blind, randomized,
placebo-­controlled dose-ranging study to evaluate the efficacy of alosetron in the treatment of
irritable bowel syndrome. Aliment Pharmacol Ther. 2000;14(1):23–34.
10. Camilleri M, Chey WY, Mayer EA, Northcutt AR, Heath A, Dukes GE, McSorley D, Mangel
AM. A randomized controlled clinical trial of the serotonin type 3 receptor antagonist alos-
etron in women with diarrhea-predominant irritable bowel syndrome. Arch Intern Med.
2001;161(14):1733–40.
11. Krause R, Ameen V, Gordon SH, West M, Heath AT, Perschy T, Carter EG. A randomized,
double-blind, placebo-controlled study to assess efficacy and safety of 0.5 mg and 1 mg
alosetron in women with severe diarrhea-predominant IBS. Off J Am Coll Gastroenterol.
2007;102(8):1709–19.
12. Everitt HA, Landau S, O'Reilly G, Sibelli A, Hughes S, Windgassen S, Holland R, Little
P, McCrone P, Bishop FL, Goldsmith K. Cognitive behavioural therapy for irritable bowel
syndrome: 24-month follow-up of participants in the ACTIB randomised trial. Lancet
Gastroenterol Hepatol. 2019;4(11):863–72.
13. Ford AC, Lacy BE, Harris LA, Quigley EM, Moayyedi P. Effect of antidepressants and psy-
chological therapies in irritable bowel syndrome: an updated systematic review and meta-­
analysis. Off J Am Coll Gastroenterol. 2019;114(1):21–39.
14. Sobin HW, Heinrich TW, Drossman DA. Central neuromodulators for treating functional GI
disorders: a primer. Off J Am Coll Gastroenterol. 2017;112(5):693–702.
15. Srinivasan R, Greenbaum DS. Chronic abdominal wall pain: a frequently overlooked problem:
practical approach to diagnosis and management. Am J Gastroenterol. 2002;97(4):824–30.
16. Drossman DA, Tack J, Ford AC, Szigethy E, Törnblom H, Van Oudenhove L. Neuromodulators
for functional gastrointestinal disorders (disorders of gut− brain interaction): a Rome founda-
tion working team report. Gastroenterology. 2018;154(4):1140–71.
17. Grover M, Dorn SD, Weinland SR, Dalton CB, Gaynes BN, Drossman DA. Atypical antipsy-
chotic quetiapine in the management of severe refractory functional gastrointestinal disorders.
Dig Dis Sci. 2009;54:1284–91.
Chapter 53
IBS-C

W. Harley Sobin and Patrick Sanvanson

Case 1 A 32-year-old female presents for worsening constipation and abdominal


pain. She has a long history of constipation since her menarche, at about age 12.
She takes a tablespoon of Metamucil on a daily basis as well as MiraLAX several
times a week to manage to have a bowel movement, about every other day, generally
hard pellets (Bristol score 1–2). Over the past 6–9 months, she is also experiencing
abdominal cramping that is generally relieved with a bowel movement. She has no
family history of colon cancer. Her physical exam and basic lab panel are normal.
Do you think a colonoscopy is warranted to rule out organic pathology, in a
case like this?
Indications for colonoscopy would be if the patient was 45 or older or if there
were any alarm features, like blood mixed with stool, unintentional weight loss, and
iron deficiency anemia. In a case like this, the diagnosis of IBS-C is likely, and
doing a colonoscopy to rule out organic pathology does not seem indicated. We
would go ahead with managing the patient’s symptoms after providing education to
the patient about her disorder and reassurance that her symptoms can be managed.
How beneficial is fiber for patients like this?
We find the response to fiber is very variable. It seems to work well in about a
third of our patients with IBS-C; in another third, there is a neutral response, and
another third note worsened constipation or pain after taking fiber. But if fiber ther-
apy works, we think that is the best approach.
In this case, where the patient continues to have hard stools despite taking a fiber
supplement, we drill down on the importance of drinking more water. First, we
would encourage more liquid intake. If the patient does not have a great response,

W. H. Sobin (*) · P. Sanvanson


Division of Gastroenterology and Hepatology, Department of Medicine, Medical College of
Wisconsin, Milwaukee, WI, USA
e-mail: [email protected]; [email protected]

© The Author(s), under exclusive license to Springer Nature 261


Switzerland AG 2023
W. H. Sobin et al. (eds.), Managing Complex Cases in Gastroenterology,
https://doi.org/10.1007/978-3-031-48949-5_53
262 W. H. Sobin and P. Sanvanson

we might have her try to increase the amount of fiber she is taking along with even
more fluid. In a number of cases, this will work to enhance the bulk of the stool and
may relieve cramping. Patients with IBS-C should be educated on soluble fiber
intake, as insoluble fiber may increase abdominal bloating. Soluble fiber including
psyllium-based products have been shown to improve both constipation and IBS
symptoms. If the patient tries to increase fiber intake and does not improve, we
would go on to other measures.
Is it safe to use MiraLAX on a long-term basis? Should the dose be increased?
It is safe to use MiraLAX (polyethylene glycol) on a chronic basis and the dose
should be increased. We would certainly go to daily use of MiraLAX. If there is a
limited response, we would increase the dose to twice daily. If we overshoot and
start getting diarrhea, we would titrate the dose down. The only potential problem
from using MiraLAX is if excessive doses are used and diarrhea ensues. Patients
should be educated on how to titrate the dose of MiraLAX as needed.
Do you believe that MiraLAX works simply to relieve constipation or does it
decrease pain as well?
In studies, MiraLAX improves constipation, but its benefit on abdominal pain is
not statistically significant. However, in practice, we find that simply relieving con-
stipation does improve underlying pain in many patients.
The patient does not benefit from increased fluid or fiber and doesn’t tolerate the
increased dosing of MiraLAX, complaining of increased bloating and gas.
What other regimens might you turn to?
We are looking for something to use on a chronic basis (at least in the near term).
We turn first to the laxatives that increase small bowel secretion [1] by their action
on the guanylate cyclase receptor (linaclotide or plecanatide) or the chloride chan-
nel receptor (lubiprostone). We usually start with linaclotide (Linzess) [2, 3] because
it is well tolerated, works well, and is available in three different doses (72 mcg,
145 mcg, and 290 mcg), which offers very useful flexibility. Once patients start on
this, it is usually taken daily. If a patient develops diarrhea after trying the lowest
dose of linaclotide, then it would be reasonable to trial lubiprostone.
Does linaclotide work primarily to relieve constipation or does it work on
pain as well?
Linaclotide works quickly to relieve constipation, but it also works to relieve
abdominal cramping. However, the full effect on pain relief takes longer to kick in;
it may take a month for the full benefit to be seen.
For patients who do not respond to the secretagogues like linaclotide or find
it insufficient, what other medications are you using?
Another class of drugs is the 5-HT4 receptor agonists, which act as promotility
agents. Prucalopride (Motegrity) [4] is the one we would try first. The combination
of a secretagogue and a promotility agent can be efficacious. Prucalopride has also
been found helpful in treating gastroparesis, although it’s not yet approved for this
indication. If we have a constipated patient with delayed gastric emptying, we will
turn to prucalopride earlier. Tegaserod is another 5HT4 agonist, which is approved
in women under age 65. If prucalopride is not approved or tolerated, we might
try this.
53 IBS-C 263

Another drug that we occasionally add-on is misoprostol. This is a prostaglandin


analogue, which we will use off-label because it is known to cause diarrhea. Used
in constipated patients on a chronic basis, we have found it helpful in some of our
refractory patients. Of course, it cannot be given to females of reproductive age
without appropriate contraception as it is an abortifacient.
This patient is started on linaclotide starting at 145mcg a day and then goes up
to 290 mcg a day. She gets marked relief but still feels like she is not emptying nor-
mally. After adding prucalopride to the linaclotide, she feels that she is empty-
ing better.
Case 2 A 67-year-old female with severe osteoarthritis is seen in the ER because of
severe abdominal pain and constipation. She is on chronic opiates for arthritis pain.
She has not had a bowel movement in about 10 days. On exam, her abdomen is
protuberant, and X-ray confirms a large amount of stool throughout the colon. On
the rectal exam, she has a large stool impaction.
How would you manage this?
First, you need to evacuate the colon. If necessary, we would start out with a
rectal disimpaction. Since this is often very painful, we may give an IV sedative at
the bedside just prior to disimpaction. Following that, we would give the patient a
bowel lavage. If possible, we would have the patient take a gallon of GoLYTELY
(polyethylene glycol-3350) over a number of hours. If the patient cannot tolerate
GoLYTELY, we would try to get them to at least take a bottle of magnesium citrate
or a MiraLAX purge. After that, you will have addressed the acute constipation, but
now it is a matter of addressing the chronic constipation. Obviously, you would like
to get the patient off opiates, but that is often not possible. We do have regimens for
managing constipation in chronic opioid users. The PAMORAs (peripherally acting
mu-opioid receptor antagonists) were designed for patients on chronic opiates. We
have used naloxegol (Movantik), which is an oral agent, and methylnaltrexone
(Relistor) is something we have given subcutaneously in the hospital, although it is
now available in oral form. Linaclotide has also been found effective in treating
constipation from chronic opioid use, and it may be easier to access and potentially
as effective.
Case 3 A 27-year-old female comes in with a history of constipation that started
within the last year. The constipation began after she started a new office job that
requires a lengthier commute. She is very rushed getting out of the house in the
morning. She does not have time to eat breakfast and then go to the bathroom to
defecate before work, which was her normal pattern. Her workplace has many
employees, and there are always women in adjacent stalls when she tries to defecate
at work. Now, when she tries, she cannot relax enough to allow her bowels to “open
up” and empty normally. She has been working for 3 months and things have not
improved. Even on weekends or when she tries to go in the evening, she finds that
she cannot evacuate normally. There is a lot of struggling and straining with only
small amounts coming out and a continued sensation of incomplete evacuation. The
mechanics of moving her bowels are feeling totally foreign to her.
264 W. H. Sobin and P. Sanvanson

In taking a history, what questions do you ask to evaluate for outlet


dysfunction?
For an assessment of outlet dysfunction, we tend to focus on obtaining a history
that includes the following: “Do you strain with trying to have a bowel movement?”
“Do you feel empty after having a bowel movement?” “Do you spend a lot of time
on the toilet to have a bowel movement, how long on average?” “Do you have the
urge/sensation when you want to have a bowel movement?” “Do you have to wipe
excessively to get clean when you have a bowel movement?” “Any maneuvers on
the toilet that help you have a bowel movement (elevating your legs, digital extrac-
tion of stool)?” “Do you have to press against the vagina and/or the space between
the vagina and rectum to have a bowel movement?” “Do you have to return to the
bathroom in a short time frame to have another bowel movement?” [5]
How would you approach this patient’s outlet dysfunction?
We would take two different approaches here. First, we would suggest seeing a
psychotherapist with an approach to relaxation exercises and cognitive behavior
therapy. But if she doesn’t respond to that and the constipation continues, we would
send her for anorectal manometry. We suspect the patient now has dyssynergic def-
ecation. If this is confirmed on anorectal manometry, she might do well with bio-
feedback therapy.
Case 4 A 35-year-old woman travels frequently to Europe and Asia for her work.
She experiences bouts of constipation repeatedly during these excursions. She has
mild constipation at home, which responds to a dose of MiraLAX taken as needed.
She takes packets of MiraLAX with her on these trips but is afraid to take anything
before her flight, fearing possible diarrhea on the airplane.
What is your approach to the occasional use of different laxatives for people
with intermittent constipation?
The one we recommend most frequently is MiraLAX. It is safe and flexible, and
you can titrate the dose. Patients need to know that MiraLAX usually does not work
immediately; it may take hours to kick in. While patients can take more than one
dose in a day, we warn them that the full effect may not demonstrate itself for 24 h
(although in some, it can work in just a couple of hours), so they need to give the
first dose some time to work before they take more.
For patients who want a more acute purge, we do like bisacodyl (Dulcolax) or
senna tabs or bisacodyl suppositories. Some patients prefer the “kick” of a stimulant
laxative, particularly those who experience more colonic inertia. Patients should be
aware of the potential for abdominal cramping with use of stimulant laxatives.
Another agent some patients prefer is milk of magnesia, which has a gentler
effect, and some do well taking this on a chronic basis. We do not suggest the use of
docusate, as we do not think there is much benefit, particularly if used as a sole agent.
Case 5 A 46-year-old female presents with a history of chronic constipation. She
had a recent screening colonoscopy that was negative, except for mild diverticulo-
sis. She is already taking fiber and occasional MiraLAX. You suggest daily MiraLAX,
but her response is limited. You order linaclotide, but she doesn’t like it. You switch
to lubiprostone (Amitiza), which she tolerates, and it gives her some relief, but the
response is still incomplete.
53 IBS-C 265

Are there any other tests you will run to evaluate constipation?
In this case, the patient had a colonoscopy for screening purposes. We do not see
any need to repeat the colonoscopy. Another simple test we like is a Sitzmarks study
[6]. The patient swallows a capsule in the office that contains a number of tiny radi-
opaque markers, and then, we follow up with an abdominal X-ray 5 days afterward.
Most, if not all, of the markers should be gone by day 5 (>80% of markers being
passed on day 5 is considered a normal study). If the abdominal X-ray shows
increased markers, it can be useful information. If there is a diffuse pattern of
retained markers throughout the colon, it suggests general colon inertia. If they are
all retained in the rectum it suggests more of an outlet problem and, possibly, dys-
synergic defecation. If dyssynergic defecation is being considered, anorectal
manometry is ordered. The Sitzmarks study may be performed on or off the patient’s
current bowel regimen, depending on the clinical situation: performed on regimen
to determine efficacy of treatment while performed off regimen to determine diag-
nosis. We frequently use abdominal X-rays and Sitzmarks studies to objectively
assess the degree of constipation and the response to therapy. This is especially
helpful when patients state that they have tried and failed numerous therapies.
You end up ordering anorectal manometry, which is normal. A Sitzmarks test
reveals increased markers throughout the colon – suggestive of generalized colon
inertia. You try prucalopride with no response.
What other strategies might you add?
We might add tegaserod to enhance colon motility, but since she did not respond
to prucalopride, another 5HT4 agonist, we would not be too optimistic. Tenapanor
[7] is a sodium hydrogen exchange (NH3E) inhibitor that works in IBS-C and
chronic constipation and may be considered.
Something that we have found useful in a number of patients is the off-label use
of misoprostol as an add-on to a secretagogue. We always warn patients not to use
misoprostol, which is an abortifacient, if they are intending to get pregnant, and not
to share it with others.
The patient is on oral contraceptive pills and not intending to get pregnant.
Misoprostol is added to lubiprostone, starting at 200 mcg bid and then increased to
400 mcg bid. She has an excellent response to combined lubiprostone and
misoprostol.

References

1. Chang L, Sultan S, Lembo A, Verne GN, Smalley W, Heidelbaugh JJ. AGA clinical practice
guideline on the pharmacological management of irritable bowel syndrome with constipation.
Gastroenterology. 2022;163(1):118–36.
2. Videlock EJ, Cheng V, Cremonini F. Effects of linaclotide in patients with irritable bowel syn-
drome with constipation or chronic constipation: a meta-analysis. Clin Gastroenterol Hepatol.
2013;11(9):1084–92.
3. Chey WD, Lembo AJ, Lavins BJ, Shiff SJ, Kurtz CB, Currie MG, MacDougall JE, Jia XD,
Shao JZ, Fitch DA, Baird MJ. Linaclotide for irritable bowel syndrome with constipation: a
26-week, randomized, double-blind, placebo-controlled trial to evaluate efficacy and safety.
Off J Am Coll Gastroenterol. 2012;107(11):1702–12.
266 W. H. Sobin and P. Sanvanson

4. Sajid MS, Hebbar M, Baig MK, Li A, Philipose Z. Use of prucalopride for chronic consti-
pation: a systematic review and meta-analysis of published randomized, controlled trials. J
Neurogastroenterol Motil. 2016;22(3):412.
5. Rao SS, Patcharatrakul T. Diagnosis and treatment of dyssynergic defecation. J
Neurogastroenterol Motil. 2016;22(3):423.
6. Metcalf AM, Phillips SF, Zinsmeister AR, MacCarty RL, Beart RW, Wolff BG. Simplified
assessment of segmental colonic transit. Gastroenterology. 1987;92(1):40–7.
7. Chey WD, Lembo AJ, Rosenbaum DP. Efficacy of tenapanor in treating patients with irritable
bowel syndrome with constipation: a 12-week, placebo-controlled phase 3 trial (T3MPO-1).
Am J Gastroenterol. 2020;115(2):281.
Chapter 54
Functional Dyspepsia

W. Harley Sobin and Patrick Sanvanson

Case 1 A 34-year-old female complains of feeling full about 10 min after starting
a meal. She has begun to eat very slowly and will quit eating earlier to avoid feeling
sick. These symptoms have been going on for over 6 months. They started after hav-
ing a meal at a Mexican restaurant. Both she and her husband got sick with some
sort of gastroenteritis. After that, her stomach never felt better. She has no other
medical problems and no history of diabetes. She has come to loathe that full feeling
and avoids eating to prevent feeling nauseated. Her family doctor saw her several
months ago and started her on omeprazole, which initially helped her but then quit
helping, and she subsequently stopped it. Her community gastroenterologist then
sent off a stool test for H. pylori, which returned negative. He did an EGD, which
was unremarkable with no evidence of gastritis, no ulcers, and no retained food.
Biopsies of the stomach were negative for H. pylori and eosinophilic gastroenteritis.
Duodenal biopsies were negative for celiac disease. He ordered gastric scintigra-
phy, which showed mild delayed gastric emptying at 4 h. He ordered metoclo-
pramide for the patient, but she developed dystonia with the very first dose, leading
to an ER visit to reverse the side effects of the metoclopramide. Because of her
weight loss, he also ordered a CT scan of the abdomen and pelvis to rule out other
organic pathology, which returned negative. He is sending the patient to you for a
second opinion.
What is your diagnosis and how would you manage the patient?
With this patient’s postprandial fullness going on for more than 6 months, the
diagnosis appears to be functional dyspepsia-postprandial distress type. Considering

W. H. Sobin (*) · P. Sanvanson


Division of Gastroenterology and Hepatology, Department of Medicine, Medical College of
Wisconsin, Milwaukee, WI, USA
e-mail: [email protected]; [email protected]

© The Author(s), under exclusive license to Springer Nature 267


Switzerland AG 2023
W. H. Sobin et al. (eds.), Managing Complex Cases in Gastroenterology,
https://doi.org/10.1007/978-3-031-48949-5_54
268 W. H. Sobin and P. Sanvanson

the diagnosis of functional dyspepsia, it is important to rule out H. pylori [1], which
was done here. A trial of PPIs is usually initiated [2], which was ultimately unsuc-
cessful here.
Left with a patient who is still quite symptomatic, with the diagnosis of
FD-postprandial distress type, we like to try buspirone [3], which has been found to
increase gastric accommodation in these patients. What we tell the patient is that the
stomach is overly stiff and doesn’t expand appropriately to allow more food.
Buspirone allows for better relaxation and expansion. We start with 5 mg TID before
meals and then go up to 10 mg before meals if needed.
Doesn’t the finding of delayed gastric emptying dissuade you against the
diagnosis of FD, and isn’t this more likely gastroparesis?
No, delayed gastric emptying is common in FD, present in about a third of
patients. The two diagnoses are quite similar and may coexist or be indistinguish-
able in some [4]. However, the diagnosis of gastroparesis does not require symp-
toms to be present for 6 months, as they do in FD. Also, buspirone is generally not
considered in gastroparesis, which is generally considered a disorder of delayed
gastric emptying rather than decreased gastric accommodation. Promotility drugs
have been tried in FD and may be effective; however in the USA, we do not have
promotility agents that are safe to use long term.
Isn’t buspirone used as an antianxiety drug? Doesn’t it lead to sedation,
tolerance, and addiction?
Some patients may find buspirone slightly sedating, but unlike the antianxiety
benzodiazepines, buspirone does not cause tolerance or addiction and can be
abruptly discontinued without withdrawal symptoms.
What would you use if the patient doesn’t tolerate buspirone or it isn’t
effective?
We have tried STW-5 [4] (Iberogast), an OTC herbal preparation, which has been
beneficial in a couple of cases. FDgard [5] is another OTC preparation. Others have
used Rikkunshito [6], a Japanese herbal product.
If her symptoms and weight loss were to continue in spite of these measures, we
would try mirtazapine [7] or olanzapine [8], which are two central neuromodulators
that we generally reserve for patients with chronic nausea and vomiting. We gener-
ally try mirtazapine first at 15 mg at bedtime. While some people start with 7.5 mg
first, it is reported that the likelihood of side effects is the same at 7.5 mg or 15 mg,
but the efficacy is lower with the 7.5 mg dose.
Mirtazapine acts to decrease nausea and increase appetite, mostly through
anti-­5-HT3 blockade, and these are the reasons we would try it. It also happens to
be an antianxiety medication that is a safe antidepressant. We have found this drug
to be particularly effective in older patients, in their 50 s, 60 s, and 70 s, with chronic
nausea and vomiting. Psychiatrists [8] think mirtazapine is one of the safer antide-
pressants to use in the elderly, and it also has the benefit of not causing sexual side
effects. However, we have found that younger patients, like this 32-year-old, are
more likely to find the side effects of daytime sleepiness and increased appetite
problematic. In spite of this, we would try mirtazapine first. Prior to starting mir-
tazapine, we would recommend monitoring for evidence of QTc prolongation and
monitoring QTc about 4 weeks after starting mirtazapine.
54 Functional Dyspepsia 269

If she doesn’t tolerate mirtazapine, then we would try olanzapine [8]. This is an
atypical antipsychotic that has shown a lot of efficacy in patients with chronic nau-
sea and anorexia. The drug works via both 5-HT3 and D2 blockade. We would start
with 5 mg at bedtime. This drug may be better tolerated than mirtazapine in some
patients.
If she fails these drugs, we would also entertain getting a psychologist involved.
From the history, it sounds as if the patient may be experiencing a restrictive eating
disorder that might benefit from psychotherapy [9].
What about the use of cannabinoids?
We have not used these and are concerned about the development of cannabinoid
hyperemesis syndrome (CHS) [10] or further delay in gastric emptying with chronic
use. We do have a lot of experience managing CHS, where we always instruct com-
plete cessation of cannabinoids.
Case 2 A 35-year-old female has daily abdominal burning pain that has been going
on for 7 months. There are no exacerbating or relieving factors. She has been avoid-
ing spicy food, alcohol, and caffeine for months. She is not taking any aspirin or
NSAIDs. There has been no weight loss, and there are no alarm features. Her MD
sends off a stool H. pylori antigen, which returns negative. She takes omeprazole for
2 months without relief.
How would you manage this patient?
In a young patient like this, the risk of a GI malignancy is low, and the likelihood
of finding anything on endoscopy, particularly after taking PPIs for 2 months, is
very low. It is important that the stool for H. pylori was sent off prior to initiating
PPI therapy, since PPIs could lead to a false-negative H. pylori. In a young patient
who does not take NSAIDs, H. pylori is a more common cause of organic disease
that would lead to chronic abdominal burning pain.
In this patient, it is likely that we are dealing with functional dyspepsia-epigastric
pain syndrome type. In many patients, symptoms are relieved with PPIs. But if they
are not, we would try central neuromodulators. Nicholas Talley [11] demonstrated
that amitriptyline benefitted patients with FD-EPS type. We prefer the use of nor-
triptyline, which has fewer anticholinergic side effects, particularly less constipa-
tion. We would start with nortriptyline 25 mg at bedtime for 1 month and then go up
to 50 mg at bedtime if tolerated. For patients who are more anxious, we might start
with 10 mg at bedtime for a week before going up to 25 mg. In using the tricyclics,
we explain to patients that it takes at least a month to see clinical efficacy, but side
effects will commence right away. Prior to starting tricyclics, we would recommend
monitoring for evidence of QTc prolongation and monitoring QTc about 4 weeks
after starting tricyclics.
For patients who do not respond to any of these therapies, we will perform an
EGD to rule out something like a PPI-resistant peptic ulcer, gastroduodenal Crohn’s,
or eosinophilic gastroenteritis. However, the likelihood of finding anything in these
young patients is low.
In addition, we generally suggest working closely with a dietician to help under-
stand patients’ dietary patterns and what or when they eat, maintaining a food diary
to assess for food triggers, and considering tailoring a diet that minimizes symptoms.
270 W. H. Sobin and P. Sanvanson

References

1. Ford AC, Mahadeva S, Carbone MF, Lacy BE, Talley NJ. Functional dyspepsia. Lancet.
2020;396(10263):1689–702.
2. Moayyedi PM, Lacy BE, Andrews CN, Enns RA, Howden CW, Vakil N. ACG and CAG clini-
cal guideline: management of dyspepsia. Off J Am Coll Gastroenterol. 2017;112(7):988–1013.
3. Tack J, Janssen P, Masaoka T, Farré R, Van Oudenhove L. Efficacy of buspirone, a
fundus-­relaxing drug, in patients with functional dyspepsia. Clin Gastroenterol Hepatol.
2012;10(11):1239–45.
4. Von Arnim U, Peitz U, Vinson B, Gundermann KJ, Malfertheiner P. STW 5, a phytopharmacon
for patients with functional dyspepsia: results of a multicenter, placebo-controlled double-­
blind study. Off J Am Coll Gastroenterol. 2007;102(6):1268–75.
5. Lacy BE, Chey WD, Epstein MS, Shah SM, Corsino P, Zeitzoff LR, Cash BD. A novel
duodenal-­release formulation of caraway oil and L-menthol is a safe, effective and well toler-
ated therapy for functional dyspepsia. BMC Gastroenterol. 2022;22(1):1–9.
6. Suzuki H, Matsuzaki J, Fukushima Y, Suzaki F, Kasugai K, Nishizawa T, Naito Y, Hayakawa
T, Kamiya T, Andoh T, Yoshida H. Randomized clinical trial: rikkunshito in the treatment
of functional dyspepsia—a multicenter, double-blind, randomized, placebo-controlled study.
Neurogastroenterol Motil. 2014;26(7):950–61.
7. Ly HG, Carbone F, Holvoet L, Bisschops R, Caenepeel P, Arts J, Boeckxstaens G, Van
Oudenhove L, Tack J. Mirtazapine improves early satiation, nutrient intake, weight recov-
ery and quality of life in functional dyspepsia with weight loss: a double-blind, randomized,
placebo-controlled pilot study. Gastroenterology. 2013;144(5):S.
8. Sobin HW, Heinrich TW, Drossman DA. Central neuromodulators for treating functional GI
disorders: a primer. Off J Am Coll Gastroenterol. 2017;112(5):693–702.
9. Orive M, Barrio I, Orive VM, Matellanes B, Padierna JA, Cabriada J, Orive A, Escobar A,
Quintana JM. A randomized controlled trial of a 10-week group psychotherapeutic treatment
added to standard medical treatment in patients with functional dyspepsia. J Psychosom Res.
2015;78(6):563–8.
10. Allen JH, de Moore GD, Heddle R, Twartz J. Cannabinoid hyperemesis: cyclical hyperemesis
in association with chronic cannabis abuse. Gut. 2004;53(11):1566–70.
11. Talley NJ, Locke GR, Saito YA, Almazar AE, Bouras EP, Howden CW, Lacy BE, DiBaise JK,
Prather CM, Abraham BP, El-Serag HB. Effect of amitriptyline and escitalopram on functional
dyspepsia: a multicenter, randomized controlled study. Gastroenterology. 2015;149(2):340–9.
Chapter 55
Bloating

W. Harley Sobin and Patrick Sanvanson

Case 1 A 35-year-old female complains of abdominal bloating and distension.


When she wakes up, she feels fine, but as the day proceeds, her belly pouches out,
and toward the end of the day, she feels like she is 7 months pregnant. These symp-
toms have been present for at least 10 months. She feels progressively more dis-
tended and uncomfortable, and her clothes feel tighter. Her bowel movements are
relatively normal, more on the harder side.
In cases like this, do patients usually have increased gaseous retention?
No, usually most patients do not retain excess gas. What is more common is
visceral hypersensitivity, increased sensitivity to normal levels of gas [1]. But
another thing that might be going on is an abnormal viscerosomatic reflex [2].
Normally, when we eat, the diaphragm rises and the abdominal muscles contract.
This is the viscerosomatic reflex, which prevents our stomachs from pouching out
after meals. Some patients have an abnormal reflex, where the diaphragm drops and
abdominal muscles don’t contract. These patients have increased abdominal girth
after meals when measured on MRI. There is actually an increase in abdominal
diameter, although the amount of measured gas is the same. We think this may be
going on with this patient.
She says that the gas is really painful and asks for something to alleviate the
pain. Are there any tips you have found helpful?
We’ve counseled countless patients about gas pain through the years. First, we
would prescribe a low-FODMAP (fermentable oligosaccharides, disaccharides,
monosaccharides, and polyols) diet [3], which should result in less fermentation.
Avoid raw fruits and raw vegetables. Stay away from carbonated beverages. Watch

W. H. Sobin (*) · P. Sanvanson


Division of Gastroenterology and Hepatology, Department of Medicine, Medical College of
Wisconsin, Milwaukee, WI, USA
e-mail: [email protected]; [email protected]

© The Author(s), under exclusive license to Springer Nature 271


Switzerland AG 2023
W. H. Sobin et al. (eds.), Managing Complex Cases in Gastroenterology,
https://doi.org/10.1007/978-3-031-48949-5_55
272 W. H. Sobin and P. Sanvanson

out for lactose sensitivity. Avoid artificially sweetened candies with sorbitol, xylitol,
or mannitol, all of which can produce a lot of gas.
In prescribing a low-FODMAP diet, we often have the patient work with a dieti-
cian to educate about the diet and also plan the reintroduction of food components
to identify triggers. Giving a complete low-FODMAP diet for an indefinite period
is usually not sustainable.
In terms of over-the-counter remedies, patients often try one of the products that
contains simethicone (Mylicon, Phazyme, Gas-X). This is in spite of the fact that
controlled studies have not shown a benefit from simethicone [4]. The results evi-
dently are more lackluster than when we instill it during colonoscopies, to help
dissolve gas bubbles. We have, on occasion, had patients try charcoal tables [5] to
help absorb gas (some patients praise the effects of eating burnt toast). Beano [6]
has been found effective in decreasing gas from beans and some other vegetables.
Pepto-Bismol [7] has been used to decrease hydrogen sulfide production, which is
helpful in the management of flatulence.
Once again, most of these patients do not really have increased gas, just an
increased sensitivity to the gas that is present. For patients who are very symptom-
atic, we will occasionally prescribe central neuromodulators [8]. We particularly
like the tricyclic nortriptyline to decrease painful bloating. Compared to the popular
tricyclic amitriptyline, nortriptyline is less constipating, and constipation can cer-
tainly exacerbate bloating. We generally start nortriptyline at 25 mg at bedtime and
after a couple of weeks go up to 50 mg at bedtime. We tell patients that side effects
(dry eyes, dry mouth, occasionally sexual side effects) will start right away, but that
full efficacy, with decreased visceral hypersensitivity, might take a month or so.
The SSRIs are not constipating (with the exception of paroxetine), but they don’t
have much effect on decreasing pain. The SNRIs are effective in decreasing pain
and tend to be less constipating than the TCAs. Of the SNRIs, we generally prefer
duloxetine [8]. We like to start with 30 mg at bedtime and then consider going up to
60 mg at bedtime after a couple of weeks.
In patients with underlying constipation who complain of bloating, we like to use
Linzess (linaclotide) [9]. Linzess relieves constipation quickly, but it also has an
effect on decreasing discomfort and bloating. However, it may take a month for this
neuromodulatory benefit to fully kick in.
Case 2 A 40-year-old female complains of painful abdominal distension. She has a
history of Raynaud’s. She has no clinical signs of systemic sclerosis. She is on diltia-
zem for the Raynaud’s and has mild constipation, for which she has tried MiraLAX
(polyethylene glycol). However, MiraLAX seems to exacerbate her bloating. On the
exam, her abdomen is tympanitic, but bowel sounds are normal. The abdomen is
mildly tender without guarding. Her doctor ordered a lactulose breath test to see
whether she might have small intestinal bacterial overgrowth (SIBO). The breath
test showed a baseline methane level of 25 and hydrogen of 10. Over the next
60 min, the methane rose to 50 and the hydrogen increased to 45. This was inter-
preted as a positive breath test.
55 Bloating 273

How would you interpret these findings and manage her bloating?
Constipation may be an exacerbating factor in anyone with bloating, and control-
ling constipation is always part of solving the bloating dilemma. While MiraLAX is
our most commonly used laxative, occasional patients complain that it actually
makes bloating worse.
Her history of Raynaud’s does raise the possibility of altered small bowel motil-
ity, as one would see in full-fledged systemic sclerosis. The use of the hydrogen
breath test (using lactulose or glucose) without synchronous scintigraphy has been
questioned as a definitive test for SIBO [10]. When the hydrogen breath test is being
considered, treatment of constipation prior to testing is essential.
So, the question is, does she really have SIBO and could this be causing bloat-
ing? The other interesting finding is the elevated methane level. There is some
debate about whether everyone is capable of producing methane, but elevated meth-
ane levels have been associated with delayed colon motility, which can exacerbate
bloating, as well as constipation [11].
If you wanted to treat this patient for SIBO, the treatment is more complicated
because of the elevated methane. To knock out the methanogenic archaea along with
the hydrogen producing bacteria, it is not sufficient to just use rifaximin. Generally,
it requires the addition of neomycin to rifaximin. Therefore, the combination of
rifaximin 550 TID and neomycin 500 mg BID for 2 weeks is indicated [12].
Metronidazole has been reported to inhibit growth of methanogenic archaea [13],
and its use in place of neomycin has been considered.
Case 3 A 33-year-old female was suffering from abdominal bloating and stomach
upset and decided to start taking the probiotic, Align (bifidobacterium infantis).
However, within a few weeks of starting Align, she started feeling lethargic and had
the sensation of brain fog. In addition, her bloating seemed much worse. As a result,
she stopped the Align, and her symptoms markedly improved.
How do you interpret these findings?
There have been reports [14] of probiotic bacteria metabolizing carbohydrates
into D-lactic acid. High levels of D-lactic acid have been associated with increased
“brain fog,” confusion, and bloating. If these patients stopped taking their probiot-
ics, the brain fog and abdominal bloating improved. This has been reported with
both lactobacillus and bifidobacterium species, which are both common compo-
nents of probiotics. While most patients won’t develop these problems from probi-
otics, it is important to be aware of the potential association. If your patient on
probiotics develops worsened bloating and “brain fog,” consider stopping the
probiotic.
Case 4 A 40-year-old female complains of abdominal bloating over the past month.
She has a long history of constipation and started taking Metamucil on her own
about 6 weeks earlier. She started on a low dose of Metamucil, mixing it with only
a small amount of fluid because she doesn’t like drinking fluids. She gradually
increased the dose of Metamucil but avoided increasing the amount of fluid that she
274 W. H. Sobin and P. Sanvanson

mixed it in. Because this caused increased cramping, she went to her family doctor,
who started her on dicyclomine (Bentyl). Her bloating got worse, and she was
referred to you.
How would you manage this situation?
While fiber laxatives benefit a lot of people with constipation, they have to be
taken with large amounts of fluid. We would say that these agents probably help
about 1/3 of our constipated patients, have a neutral effect on another third, and may
worsen constipation [15] in the others, usually in those who do not drink enough fluid.
Metamucil certainly may be a cause of gassiness, and it is not surprising that this
patient developed increased bloating. In addition, the use of dicyclomine can further
slow gut motility and worsen bloating. Dicyclomine is more beneficial in patients
with diarrhea.
In a patient like this, we would stop the dicyclomine and try switching from
metamucil to linaclotide (Linzess). Linzess should help the constipation without
causing increased gas. In addition, after about a month, Linzess [9] can help with
visceral hypersensitivity. So, there is an immediate benefit of Linzess in treating
constipation, but you may also see a decrease in bloating and cramps after about a
month of use.
Case 5 A 27-year-old female is referred to you with intractable abdominal bloat-
ing. She feels bloated several hours after eating and has modified her diet without
much success. She has avoided meat, gluten, and dairy and now simply tries not to
eat at all to try to relieve her discomfort. She has lost 10 pounds over the past
2 months. She has tried Gas-X and peppermint oil without much benefit. In going
over her history, you find that she seems mildly depressed. She notes that the bloat-
ing always occurs over the suprapubic/pelvic area. She says that she has seen a
gynecologist to make sure that there was nothing wrong with her uterus or ovaries
that is causing this discomfort. Her thyroid and basic CBC and chemistries have
been normal. When you ask whether she might be willing to try a central neuro-
modulator to work on visceral hypersensitivity, she is quite reluctant. She says that
when she was 15, she had a “nervous breakdown” and was on heavy duty antide-
pressants. The breakdown was precipitated by several episodes of sexual abuse by
an uncle who was briefly living with the family.
How do you manage patients with a history like this?
Douglas Drossman and others have shown that there is a high incidence of abuse,
both sexual and physical, in patients who have refractory GI functional symptoms
[16]. A case like this reveals how GI symptoms like bloating may be associated with
significant psychopathology, and relieving these symptoms will require collabora-
tion with a mental health specialist.
But while a psychiatrist might get involved in a case like this, they are not neces-
sarily tuned into the effect of psychiatric drugs on the GI tract. As gastroenterolo-
gists, we can occasionally advise psychiatrists, in order to benefit our patients.
Certain psychiatric agents can be used as central neuromodulators to decrease
abdominal pain and bloating. We can utilize low- to medium-dose tricyclics and
55 Bloating 275

full-dose SNRIs for their analgesic benefit [8]. These drugs can promote analgesia
by inhibiting norepinephrine reuptake (due to their action on the norepinephrine
transporter—NET), thereby increasing levels of norepinephrine.
Some of the atypical antipsychotics that may be used on our patients—like
aripiprazole, lurasidone, and ziprasidone—can cause undesirable GI side effects,
primarily early satiety and nausea [8]. If our patients develop these adverse symp-
toms on these agents, we can advise the psychiatrist to consider switching to the
atypical antipsychotics, like olanzapine or quetiapine, which are good treatments
for nausea.
Case 6 A 53-year-old female complains of persistent abdominal bloating and mild
constipation. She has a history of type 2 diabetes of 10-year duration. Her diabetes
is not tightly controlled. She is aware that she can’t eat large portions at meals;
otherwise, she will fill up. She can avoid feeling full if she eats multiple small meals,
but, regardless, as the day progresses, she does tend to get more bloated.
How would you evaluate and treat this patient?
In this case, disordered GI motility due to diabetes may be a contributing factor.
Diabetic neuropathy could contribute to decreased gastric emptying of food,
decreased small bowel emptying of gas, and increased colon retention [17]. While
small bowel motility measurement may not be widely accessible, measuring gastric
emptying with nuclear medicine testing and colon emptying with a Sitzmarks study
[18] are both inexact but broadly available tools.
Improved glucose control and dietary change (restricting fat and fiber and mul-
tiple small meals) can benefit the patient. Prucalopride [19] may be prescribed to
treat constipation while enhancing gastric motility at the same time.
Case 7 A 77-year-old male presents with the chief complaint of severe abdominal
bloating. This has been going on for months. He has mild constipation. He has a
past history of abdominal surgery and radiation therapy for a non-Hodgkin lym-
phoma. On exam, his abdomen is quite tympanitic and mildly tender. Bowel sounds
are high pitched.
An abdominal series shows dilated loops of small bowel but no suggestion of
obstruction.
How would you manage this patient’s bloating?
In this case, we think the patient probably has delayed small bowel motility and
is certainly predisposed to developing SIBO, so we would order a breath test. In
community-based practices, lactulose breath tests are frequently used. At the
Medical College of Wisconsin, Dr. Benson Massey is a proponent of the glucose
hydrogen breath test, done in combination with scintigraphy [10]. And, as he likes
to point out, we want to manage constipation before doing a breath test.
The patient is given laxatives to treat his constipation, and a glucose hydrogen
breath test is done 1 week later. He is placed on a low-fiber diet prior to testing, and
on arrival, he promises the lab nurse that he has absolutely adhered to the diet. In
spite of this, the breath test shows a markedly elevated baseline hydrogen of 120. He
is given glucose, and the hydrogen level rises to 180 over 30 min.
276 W. H. Sobin and P. Sanvanson

What do you make of the elevated baseline hydrogen?


Sometimes, we do not know whether a breath test is uninterpretable if the base-
line hydrogen is elevated. Did the patient really adhere to the diet? But in this case,
it is likely that the elevation is truly due to background fermentation related to
marked bacterial overgrowth. We then see the level rise appropriately after giving
the glucose load, confirming the impression of SIBO.
How would you manage this patient’s bacterial overgrowth?
We would start antibiotics and generally continue them for a couple of weeks. If
we can get rifaximin covered, that would be our first choice. Since there are often
barriers to prescribing rifaximin, we would try trimethoprim/sulfamethoxazole
(Bactrim), ciprofloxacin, or amoxicillin/clavulanic acid (Augmentin).
Do you expect that to cure the bloating?
I certainly think it can help. However, the patient will continue to have bowel
dilation, and it’s possible that the trapped air will contribute to the sensation of
bloating. Also, the patient will undoubtedly develop a relapse of bacterial over-
growth in the future. Antibiotics will generally improve SIBO, but recurrence in
3–4 months is common.

References

1. Malagelada JR, Accarino A, Azpiroz F. Bloating and abdominal distension: old misconcep-
tions and current knowledge. Official J Am Coll Gastroenterol ACG. 2017;112(8):1221–31.
2. Accarino A, Perez F, Azpiroz F, Quiroga S, Malagelada JR. Abdominal distention results from
caudo-ventral redistribution of contents. Gastroenterology. 2009;136(5):1544–51.
3. Halmos EP, Power VA, Shepherd SJ, Gibson PR, Muir JG. A diet low in FODMAPs reduces
symptoms of irritable bowel syndrome. Gastroenterology. 2014;146(1):67–75.
4. Friis H, Bode S, Rumessen JJ, Gudmand-Høyer E. Effect of simethicone on lactulose-induced
H2 production and gastrointestinal symptoms. Digestion. 1991;49(4):227–30.
5. Hall RG Jr, Thompson H, Strother A. Effects of orally administered activated charcoal on
intestinal gas. Am J Gastroenterol. 1981;75(3)
6. Ganiats TG, Norcross WA, Halverson AL, Burford PA, Palinkas LA. Does beano prevent gas?
A double-blind crossover study of oral a-galactosidase to treat dietary oligosaccharide intoler-
ance. J Fam Pract. 1994;39(5):441–5.
7. Suarez FL, Furne JK, Springfield J, Levitt MD. Bismuth subsalicylate markedly decreases
hydrogen sulfide release in the human colon. Gastroenterology. 1998;114(5):923–9.
8. Sobin HW, Heinrich TW, Drossman DA. Central neuromodulators for treating functional GI
disorders: a primer. Off J Am Coll Gastroenterol. 2017;112(5):693–702.
9. Rao SS, Quigley EM, Shiff SJ, Lavins BJ, Kurtz CB, MacDougall JE, Currie MG, Johnston
JM. Effect of linaclotide on severe abdominal symptoms in patients with irritable bowel syn-
drome with constipation. Clin Gastroenterol Hepatol. 2014;12(4):616–23.
10. Massey BT, Wald A. Small intestinal bacterial overgrowth syndrome: a guide for the appropri-
ate use of breath testing. Dig Dis Sci. 2021;66(2):338–47.
11. Chatterjee S, Park S, Low K, Kong Y, Pimentel M. The degree of breath methane produc-
tion in IBS correlates with the severity of constipation. Off J Am Coll Gastroenterol.
2007;102(4):837–41.
55 Bloating 277

12. Low K, Hwang L, Hua J, Zhu A, Morales W, Pimentel M. A combination of rifaximin and neo-
mycin is most effective in treating irritable bowel syndrome patients with methane on lactulose
breath test. J Clin Gastroenterol. 2010;44(8):547–50.
13. Khelaifia S, Drancourt M. Susceptibility of archaea to antimicrobial agents: applications to
clinical microbiology. Clin Microbiol Infect. 2012;18(9):841–8.
14. Rao SS, Rehman A, Yu S, De Andino NM. Brain fogginess, gas and bloating: a link between
SIBO, probiotics and metabolic acidosis. Clin Transl Gastroenterol. 2018;9(6):e162.
15. Ho KS, Tan CY, Daud MA, Seow-Choen F. Stopping or reducing dietary fiber intake reduces
constipation and its associated symptoms. World J Gastroenterol: WJG. 2012;18(33):4593.
16. Drossman DA, Talley NJ, Leserman J, Olden KW, Barreiro MA. Sexual and physical abuse and
gastrointestinal illness: review and recommendations. Ann Intern Med. 1995;123(10):782–94.
17. Prasad VG, Abraham P. Management of chronic constipation in patients with diabetes mel-
litus. Indian J Gastroenterol. 2017;36:11–22.
18. Alame AM, Bahna H. Evaluation of constipation. Clin Colon Rectal Surg.
2012;25(01):005–11.
19. Sajid MS, Hebbar M, Baig MK, Li A, Philipose Z. Use of prucalopride for chronic consti-
pation: a systematic review and meta-analysis of published randomized, controlled trials. J
Neurogastroenterol Motil. 2016;22(3):412.
Part IV
Hepatology Compendium
Chapter 56
Introduction to the Hepatology
Compendium

W. Harley Sobin

We gastroenterologists deal with many patients with liver disease in the community
setting. We get many requests to see patients for abnormal liver enzymes or for
questions about NAFLD. Managing complications of cirrhosis continues to con-
sume a lot of our time. Treating hepatitis C used to be burdensome, but since the
development of DAA treatment, this is rarely a problem.
While community gastroenterologists continue to spend a lot of time dealing
with liver issues, we frequently curbside our hepatology experts at the regional ter-
tiary care center for help with our complicated patients. There are numerous man-
agement decisions in hepatology that are confusing or controversial. In the sections
below, we highlight how our liver specialists deal with some of these problem cases.
Oftentimes, our hepatology dilemmas start with abnormal liver enzymes. The
workup is usually launched with a battery of serologic tests. At MCW, the standard
panel of tests for evaluation of abnormal liver enzymes includes a viral hepatitis
panel, an autoimmune panel that includes ANA, ASMA, and AMA, as well as ceru-
loplasmin, Fe/TIBC, ferritin, alpha-1-antitrypsin level, and celiac panel (occasion-
ally IgG4 and others). This panel is a good starting point.
While these results often lead us to a diagnosis, the results are sometimes unre-
vealing and other times confusing. In addition, liver enzymes may be normal in
cases where patients are consuming a lot of alcohol, cases where ultrasound reveals
extensive hepatic steatosis, or even in cases of clear-cut cirrhosis. If the workup for
abnormal liver enzymes is negative, what should the next steps be? These and many
more issues are discussed in the case presentations that follow.

W. H. Sobin (*)
Division of Gastroenterology and Hepatology, Department of Medicine, Medical College of
Wisconsin, Milwaukee, WI, USA
e-mail: [email protected]

© The Author(s), under exclusive license to Springer Nature 281


Switzerland AG 2023
W. H. Sobin et al. (eds.), Managing Complex Cases in Gastroenterology,
https://doi.org/10.1007/978-3-031-48949-5_56
282 W. H. Sobin

Below, we encounter a case of suspected hemochromatosis, where the transferrin


saturation and the ferritin are discordant. How should we proceed? And if hemo-
chromatosis is diagnosed, what is the end goal for phlebotomy?
The importance of Wilson’s disease is highlighted in two different cases. We
never want to miss Wilson’s, which can lead to severe liver disease and debilitating
neurodegenerative disease. In a case of suspected Wilson’s, how is the diagnosis
confirmed? The serum ceruloplasmin is a starting point in making the diagnosis,
which, if low, triggers investigations. But, what about a case where the ceruloplas-
min is normal? And, if Wilson’s is confirmed, how is it treated? In the community,
we may see one or two patients with Wilson’s throughout our career, but it is imper-
ative that we remain vigilant. If Wilson’s is diagnosed, we certainly need advice
about management.
A patient with abnormal LFTs and apparent alpha-1 antitrypsin deficiency is
discussed. Alpha-1 antitrypsin deficiency may be the primary cause of chronic liver
damage, or it can be a secondary, exacerbating factor in other cases. It is easily
missed, if not specifically investigated.
We are all familiar with patients with indirect hyperbilirubinemia who have
Gilbert’s. But a case is discussed where Gilbert’s is suspected but the patient also
has an elevated direct bilirubin. How is this interpreted?
Normally, the ALT > AST, except in cases of active alcohol abuse. How do we
interpret cases where the AST > ALT and alcohol abuse is apparently absent?
Several authors discuss this phenomenon.
A case of profound hyperbilirubinemia is presented. In cases where the bilirubin
is >20, it is usually not due to obstruction alone. What other factors may be
contributing?
Community gastroenterologists are seeing more and more referrals for managing
NAFLD. This is a confusing and controversial area, and it may be difficult to know
what course of action to take. Talking to the hepatologists, there are several recom-
mendations they all agree upon, but different viewpoints exist, even among the
experts. Below, we share various approaches from several advisors with different
management styles.
While this textbook was in production, the terminology-NAFLD was accepted
by consensus. But, just prior to printing, NAFLD was replaced by MASLD. As Dr.
Sourianarayanane says, “MASLD is the current terminology for non-alcoholic fatty
liver disease (NAFLD), which is an encompassing diagnosis of those with fatty liver
with metabolic risks. Similarly, metabolic dysfunction-associated steatohepatitis
(MASH) is the replacement term for prior non-alcoholic steatohepatitis (NASH)”.
Since most of our readers are more familiar with NAFLD and NASH, we have not
gone back to edit these terms that are still present in the majority of the chapters.
There is a case where we discuss the increased susceptibility women have to
alcohol-induced liver disease. In another case, we discuss management of acute
alcoholic hepatitis. Community gastroenterologists have managed patients with
alcoholic hepatitis through the decades. We have gone through periods where pent-
oxifylline was recommended and where steroids have been advocated. Now, even
the possibility of transplant has been raised. We hear about the management of
alcoholic hepatitis in the tertiary care setting.
56 Introduction to the Hepatology Compendium 283

Another problem we commonly deal with in the community is managing ascites.


In the initial evaluation of ascites, we know to examine the ascites albumin and cell
count. We measure the SAAG (serum albumin minus ascites albumin), expecting it
to be >1.1 in most cases, consistent with portal hypertension. We know that a PMN
count in fluid >250 requires treatment for infection. We discuss cases of low SAAG
ascites, ascites in a patient without apparent cirrhosis, and ascites in a patient with
cirrhosis of unclear etiology. There is also a case that explores the possible etiolo-
gies of worsening ascites in an apparently stable cirrhotic.
Another issue we deal with in the community is managing bleeding esophageal
varices. But what is the best approach to dealing with bleeding from post-banding
ulcers? The other problem that may lead to a tertiary center referral is management
of gastric varices. In the segments below, we discuss both of these challenging
scenarios.
In the community, we spend a lot of time managing the complications of cirrho-
sis. Unfortunately, our therapies are themselves fraught with multiple potential
complications. We present one such case. Ultimately, many of these patients with
end-stage liver disease require a liver transplant. However, as we highlight in our
section on transplantation, getting a liver may be frustratingly slow.
Portal vein thrombosis (PVT) is a known complication of cirrhosis. Decisions on
anticoagulation for patients with PVT, who may have varices, bleeding portal
hypertensive gastropathy, or GAVE, can be challenging. This is discussed in one of
the cases.
Another complication of cirrhosis that we need to screen for is hepatocellular
carcinoma (HCC). In our community hospital, once HCC is detected, the patient
usually gets referred to the tertiary care center, where specific therapies and poten-
tial liver transplant are available. There are several cases discussing manage-
ment of HCC.
While most patients with HCC have underlying cirrhosis, it is well-known that
you can develop HCC without having cirrhosis if there is underlying chronic hepa-
titis B or hemochromatosis. Now there are reports of HCC occurring in NASH
patients who don’t have cirrhosis. With the huge numbers of patients with NAFLD
in the USA, this is potentially a huge problem. This is also discussed.
We see quite a lot of PBC in the community, patients who are referred with an
asymptomatic elevation of alkaline phosphatase. Management is often uncompli-
cated, but sometimes questions arise. We see far fewer patients with advanced
PBC in the community, and this generally warrants referral to the transplant cen-
ter. How do they approach these patients? An extensive discussion of management
follows.
In the following chapters, we have several hepatologists who discuss cases of
autoimmune hepatitis, overlap syndromes, and PSC. In PSC, of course, we need to
be vigilant in screening for cholangiocarcinoma and colon cancer.
Chronic hepatitis B is not commonly seen by most community GIs. It can be a
confusing disease to understand and treat. There are cases discussing management
of immune-tolerant and immune-active chronic HBV. Another reviews the situation
where a patient with chronic HBV stops taking his antiviral medication and disease
relapses.
284 W. H. Sobin

The question posed in a different case asks whether a patient once infected with
HBV will continue to have lifelong stigmata of the infection.
The management of an exacerbation of chronic HBV differs from the treatment
of acute HBV. Sometimes, it is unclear which we are dealing with. What are some
clues to making this distinction and managing this?
While long-term management of HCV used to take up a lot of our time, the
development of DAAs has made management so easy that often gastroenterologists
in the community are not even consulted. However, there are occasional confusing
cases that are sent our way. One such case is presented.
Liver failure and liver transplant are explored in several cases. A previously
healthy patient who presents with acute liver failure always necessitates an urgent
call to the transplant center. Here are some insights into their management once
transferred. In the community, we get consulted on patients with a recent Tylenol
overdose, and their management is often straightforward. But how should we man-
age patients with a delayed presentation?
Deciding who to refer for a transplant can be confusing. Four different cases
highlight the controversies that occur. And then, once a patient goes on the trans-
plant list, it can still be very frustrating for the community gastroenterologist to have
to manage the recurring complications while waiting for the transplant to happen. It
seems like the wait for a liver goes on forever.
Finally, the transplant hepatologists are the ones who commonly deal with com-
plications that may occur post-liver transplant. However, community gastroenter-
ologists may be the first to encounter these problems and need to know what to be
on the lookout for. This is discussed in our last case.
Chapter 57
Liver Enzyme Elevation-negative Work-up

Francisco Durazo

A 47-year-old female is referred for abnormal liver enzymes. She has a normal
BMI, does not drink or take street drugs, and is on no medications. Her AST is 78,
ALT is 90, Alk phos is 100 (nl < 80), and bilirubin is 1.4, in which the direct is 1.0.
The US of the liver is unremarkable. ANA + 1:16, ASMA negative, AMA negative,
celiac panel, ceruloplasmin, and alpha-1 Antitrypsin are all normal, and Fe/
TIBC is 24%.
How do you approach the case of a patient with elevated LFTs where
workup is negative?
My first approach with this patient would be to corroborate the history and do a
detailed physical exam.
Is there any more information that you need to know?
It is helpful to know the time that the liver tests became abnormal and for how
long they have been abnormal. When was the last time that the patient had normal
liver tests? If we know this information, we can focus the interrogatory around that
time and look for events that may be related. Is the patient symptomatic? Any
changes in bowel habits that would suggest inflammatory bowel disease with pri-
mary sclerosing cholangitis or celiac sprue? Does the patient have itching that
would suggest cholestasis? Any symptoms that would suggest heart failure such as
paroxysmal nocturnal dyspnea or dyspnea on exertion? Any joint swelling and
hyperpigmentation suggesting hemochromatosis? Does the patient have upper
abdominal pain that would indicate biliary tract disease?

F. Durazo (*)
Division of Gastroenterology and Hepatology, Department of Medicine,
Medical College of Wisconsin, Milwaukee, WI, USA
e-mail: [email protected]

© The Author(s), under exclusive license to Springer Nature 285


Switzerland AG 2023
W. H. Sobin et al. (eds.), Managing Complex Cases in Gastroenterology,
https://doi.org/10.1007/978-3-031-48949-5_57
286 F. Durazo

I would check for risk factors for viral hepatitis: recent travel to endemic areas,
parenteral exposure, tattoos, blood transfusions before 1992, and eating under-
cooked pork (hepatitis E).
Other important information that is frequently not disclosed accurately is the use
of alcohol and illicit drugs. This patient apparently does not drink alcohol, but did
she drink heavily in the past? She can have alcohol-related liver disease from previ-
ous drinking. The same with street drugs. Did she take street drugs in the past? Even
if she did once, that’s enough to acquire viral hepatitis. We saw this in the baby
boomers that experimented with intravenous drugs in the 1970s and acquired hepa-
titis C. Many denied using intravenous drugs, but when asked more specifically,
they confessed. Many street drugs can cause liver test abnormalities, hepatitis, and
acute liver failure.
Frequently, patients do not offer the information we are looking for. Up to two
thirds of patients taking supplements do not disclose this information to their physi-
cian. Others don’t mention them when interviewed because they do not consider
supplements to be part of their medications. Drug-induced liver injury is a frequent
cause of liver test abnormalities. This patient was not taking medications at the time
she was seen, but a thorough history of medications can be the culprit. Some medi-
cations can cause drug-induced liver injury with liver test abnormalities that persist
even months after stopping them (intrahepatic cholestasis, i.e., amoxicillin/clavu-
lanic acid, estrogens, anabolic steroids).
A detailed physical exam is most helpful. A good exam can narrow or even give
you the diagnosis you are looking for. Does this patient have stigmata of chronic
liver disease, such as palmar erythema or vascular spiders? Does she have a palpa-
ble liver? Is the liver firm and nodular? Is there a hepatic bruit suggesting alcoholic
hepatitis or hepatocellular carcinoma? Is there a venous hum present (Cruveilhier-­
Baumgarten murmur, which is highly suggestive of portal hypertension)?
Are there other labs you would order?
I would like to rule out chronic viral hepatitis with hepatitis B surface antigen,
hepatitis B core antibody, and hepatitis C antibody. I would also like to check her
for type 2 and 3 autoimmune hepatitis with a liver-kidney microsomal antibody and
a soluble liver antigen. Thyroid disease can affect the liver tests, especially hyper-
thyroidism. A thyroid panel with thyroid stimulating hormone is another test I
would order.
A platelet count is helpful to assess for the presence of significant fibrosis. The
AST to platelet ratio index (APRI score) or the Fib-4 score can be obtained with
simple routine laboratory tests. They have good ability to differentiate patients with
significant fibrosis (F2 to F4) from those without significant fibrosis (F0 to F1).
Would you do FibroScan®? If so, how would it help you?
A FibroScan® can be used as the first-line assessment for the severity of liver
fibrosis in patients with chronic hepatitis. It performs best with regard to the ruling
out of cirrhosis. It is primarily used as an alternative to liver biopsy for the assess-
ment of hepatic fibrosis. It can also be used to predict complications in patients with
cirrhosis. However, in this patient, I would be more interested in finding out the
etiology of her liver test abnormalities. Depending on this information, I may or
may not request a FibroScan® and may or may not proceed with a liver biopsy.
57 Liver Enzyme Elevation-negative Work-up 287

In considering a liver biopsy, what are you looking for? What might it show,
and what are you hoping to rule out?
Doing a liver biopsy in a patient with a negative serologic workup is tempting.
However, a liver biopsy doesn’t always give you the answer. Pathologists base, to
some extent, their histologic interpretation on the clinical history and the results of
the serologic workup. If the work up is noncontributory, the histologic interpretation
of the biopsy will be less specific. A liver biopsy may help to definitively establish
the final diagnosis in some patients. However, the results rarely change your pre-
sumptive diagnosis or influence the patient management. On the other hand, if the
liver test abnormalities persist, then I would proceed with a percutaneous liver
biopsy. Occasionally, the liver biopsy may reveal an unsuspected diagnosis or dic-
tate a change in the patient management. In most of the cases, the liver biopsy will
reassure the patient and physician that there is no advanced liver disease.
Any other management decisions?
In the case of this patient, I would check the results of the viral hepatitis serolo-
gies and a thyroid panel. If these are noncontributory, like the rest of the workup, I
would repeat the liver tests in 1 month and look at the trend. If the liver test abnor-
malities continue to get worse, I would proceed with a percutaneous liver biopsy
(not a transjugular liver biopsy). If the liver test abnormalities would get better, I
would continue to follow up the patient until these normalize.
Chapter 58
Increased LFTs with Increased Iron

Jose Franco

A 35-year-old male is being elevated for abnormal LFTs: AST, 60; ALT, 78; and
ALK PHOS,100 (nl < 80). Hepatitis panel and autoimmune panel are negative, and
Fe/TIBC is 20% but ferritin is 800.
Do you think this is hemochromatosis?
The protocol that I follow is that if the transferrin saturation (TS) is > or equal to
45% and/or the ferritin is elevated, you should order a hemochromatosis genotype.
So, in this case, the ferritin is elevated, although the transferrin saturation isn’t. You
need to order the genotype, because one of them is elevated. I don’t think hemo-
chromatosis is very likely, with the TS being normal, but you do have to go that next
step. Of course, many chronic liver diseases, besides hemochromatosis, may be
associated with elevated ferritin.
What if the iron saturation was 55% and the ferritin was 250? What if the
blood draw was non-fasting?
Once again, if either the transferrin saturation or the ferritin is elevated, you need
to order a hemochromatosis genotype. In this instance, I also think it is unlikely to
be hemochromatosis, with a normal ferritin, but I would still check the genotype. In
terms of the iron level, this should be drawn fasting. Presumably, there can be some
artifactual rise in the iron level, if the draw is postprandial, although I've never been
able to find a good reason why (except where the patient is ingesting iron).
In what cases are you seeing elevated ferritin in patients with liver disease
who don’t have hemochromatosis?
Fatty liver, alcohol, chronic viral hepatitis, chronic cholestatic diseases, pretty
much any chronic liver disease. Back in the 1990s, there was actually a movement
to consider phlebotomy in patients with chronic HCV who had elevated ferritin

J. Franco (*)
Department of Medicine-Division of Gastroenterology and Hepatology, Medical College of
Wisconsin, Milwaukee, WI, USA
e-mail: [email protected]

© The Author(s), under exclusive license to Springer Nature 289


Switzerland AG 2023
W. H. Sobin et al. (eds.), Managing Complex Cases in Gastroenterology,
https://doi.org/10.1007/978-3-031-48949-5_58
290 J. Franco

prior to initiating interferon therapy. The ferritin is elevated in these diseases because
of ongoing inflammation, and it is an acute phase reactant.
What if the iron saturation is 55%, ferritin is 750, and the hemochromatosis
genotype is C282Y homozygote in this patient with elevated liver enzymes.
Would you bother with a liver biopsy? What if the ferritin was above 1000,
would you want a biopsy then?
In my practice, any patient who is a C282Y homozygote with a ferritin >1000
and/or elevated liver enzymes should have a liver biopsy. If the liver enzymes were
normal and the ferritin was <1000, a liver biopsy would not be required. The 1000
cutoff relates to the fact that patients with hemochromatosis and a ferritin <1000 are
unlikely to have cirrhosis. The main reason for doing a liver biopsy would be to
document the presence of cirrhosis.
It is important to look at the age in patients with suspected hemochromatosis. If
this were a 20-year-old male, he would be unlikely to have cirrhosis. At 35, it is
much more likely.
When you start your phlebotomies, what would be your goal level for TS or
ferritin? What are the numbers you aim for?
I like to follow the ferritin and I aim to get it down below 100. Some people go
as low as 50. It takes you a while to get that low, even with frequent phlebotomies,
and then once you do, you go into a maintenance phase, where the patient gets phle-
botomies quarterly. Sometimes, you are limited by the hemoglobin in some patients
who are anemic for some other reason, and then you can't be as aggressive.
Is there any significance to patients with NASH who have an elevated
ferritin?
Once again, chronic inflammatory conditions seem to be the common thread
here. We tend to do a full chronic liver disease workup in our patients with sus-
pected NASH, even when we're close to a hundred percent confident of our diagno-
sis, so we are routinely getting iron studies.
We do find that some patients with NASH have secondary iron overload, that's
how I describe it to patients. Secondary iron overload doesn't require phlebotomy.
However, is it possible to have NASH and hemochromatosis coexist? Absolutely.
You hate to miss a potentially treatable component, and so we always check ferritin
levels in these patients, and, once again, NASH doesn't really have a serologic test.
Even though I may be dealing with somebody with a BMI of 40 who’s diabetic and
has fatty liver on imaging, I still do the autoimmune markers, the viral markers, and
the iron studies.
Chapter 59
Increased LFTs with Low Ceruloplasmin

Jose Franco

A 27-year-old male is referred for elevated liver enzymes. His only complaint is
mild chronic fatigue and difficulty concentrating at work. His AST is 100, ALT is
120, Alk phos is 40, and bilirubin is 1.2. His Hbg is 12.2. and iron saturation is
25%. Hepatitis panel is negative, while ANA, AMA, and ASMA are all negative.
Serum ceruloplasmin is 17 (normal 20–40). The diagnosis of Wilson’s is consid-
ered. He is sent to an ophthalmologist, who does a slit lamp exam and does not see
K-F rings.
How would you evaluate this patient further?
In this young male with elevated liver enzymes and a decreased ceruloplasmin,
Wilson’s disease is a consideration. The absence of K-F rings does not rule out
Wilson’s. The next step would be to do a 24-h urine copper. I don’t think there is any
value in getting a serum copper or a spot urine copper; these tests are not useful.
Whatever the result of the 24-h urine copper, we should do a liver biopsy. In
Wilson’s, the 24-h urine copper should be high, > 40 μg. If it is high, this is likely
Wilson’s, and we do the biopsy for quantification of copper in liver tissue. More
than 250 μg of copper/g dry weight is diagnostic for Wilson’s. If, for some reason,
the level is lower, between 50 and 250 μg/g dry weight, then molecular/genetic test-
ing is indicated. If the 24-h urine copper is not elevated, I would do a liver biopsy
for histology, and if significant copper is present, do quantitative copper levels.
When you send quantitative copper studies, you need to send tissue in a dry tube to
an institution capable of performing this test.
One interesting thing in this case is the low alkaline phosphatase. No one seems
to know why this is low in Wilson’s, but the only condition you ever really see a low
alkaline phosphatase is in Wilson’s disease.

J. Franco (*)
Department of Medicine-Division of Gastroenterology and Hepatology, Medical College of
Wisconsin, Milwaukee, WI, USA
e-mail: [email protected]

© The Author(s), under exclusive license to Springer Nature 291


Switzerland AG 2023
W. H. Sobin et al. (eds.), Managing Complex Cases in Gastroenterology,
https://doi.org/10.1007/978-3-031-48949-5_59
292 J. Franco

If Wilson’s is present, would you do further neuropsychiatric evaluation?


I'm not sure that doing further neuropsychiatric evaluation would help if the
patient is already reporting difficulty concentrating at work. He’s 27, and, obvi-
ously, this is a genetic disorder, so he has already had a couple of decades worth of
disease, and I would attribute those symptoms to Wilson’s disease. I don't know
what further neuropsychiatric evaluation would tell me, and it's not going to alter
my treatment, which is, of course, to treat the condition.
If Wilson’s is diagnosed, how would you treat the patient?
Historically, penicillamine was the agent we used the most, but today, it is hardly
ever used because there are less toxic options. Trientine is another agent that works
like penicillamine, helping to increase the urinary excretion of copper. But my agent
of choice is zinc, which has lower toxicity than both penicillamine and trientine.
Zinc works to prevent absorption of copper from the GI tract. It is usually well toler-
ated. Occasionally, I'll use a combination of zinc and trientine because they work
via different mechanisms.
How do you monitor their response to therapy?
I mostly follow their liver enzymes, but you can also monitor their 24-h urine
copper. We tend to do that once a year. The urine copper should be low if the zinc is
doing its job. Some also follow the non-ceruloplasmin-bound copper (or “free cop-
per”), which can be calculated by subtracting ceruloplasmin-bound copper
(3.15 × ceruloplasmin in mg/L equals the amount of ceruloplasmin-bound copper in
μg/L) from the total serum copper concentration (in μg/L; serum copper in
μmol/L × 63.5 equals serum copper in μg/L). In order to simplify this, you can use
the following: total serum copper (in μg/L) − (3 × ceruloplasmin in mg/L) = non-­
ceruloplasmin-­bound copper (to be more accurate, you can use 3.15). This can be
cumbersome, and it is difficult when results return without specific numerical val-
ues (e.g., serum copper <3).
How many cases of Wilson's would you say you've seen over the years?
I would say 40–50, but right now, I’m following three quite actively.
Chapter 60
Normal Ceruloplasmin-Suspected Wilson’s

Jose Franco

A 37-year-old male is referred for elevated LFTs; AST is 120, ALT is 150, Alk phos
is 50, and Bili is 1.2.
The patient has had increased fatigue for the past 1 month. ANA, ASMA, AMA,
celiac, alpha-1-AT, and Fe/TIBC are all normal. Ceruloplasmin is 20. Slit lamp
exam is positive, and 24-h urine copper is elevated.
Can you comment on the finding of a normal ceruloplasmin in a case of
likely Wilson’s?
In this case, KF rings are present but ceruloplasmin is normal. It is important to
remember that ceruloplasmin is an acute phase reactant. The elevated urinary cop-
per level confirms the diagnosis of Wilson’s. If the 24-h urine is normal or only
mildly elevated, a liver biopsy is indicated.

J. Franco (*)
Department of Medicine-Division of Gastroenterology and Hepatology, Medical College of
Wisconsin, Milwaukee, WI, USA
e-mail: [email protected]

© The Author(s), under exclusive license to Springer Nature 293


Switzerland AG 2023
W. H. Sobin et al. (eds.), Managing Complex Cases in Gastroenterology,
https://doi.org/10.1007/978-3-031-48949-5_60
Chapter 61
Increased LFTs-Alpha 1 Antitrypsin
Deficiency

Jose Franco

A 49-year-old female is seen for a history of elevated liver enzymes. Her AST is 55
and ALT is 80; alk phos and bilirubin are normal. Also, she is mildly obese. She
notes a family history of cirrhosis in her father and uncle, neither of whom were
smokers or drinkers, but both also suffered from lung disease, presumed COPD. She
has not had pulmonary problems. Because of the family history, alpha-1 antitrypsin
levels and genotype were sent off. She was found to have mild AAT deficiency with
ZZ genotype.
What manifestations of A1AT have you seen in your liver patients?
I have seen symptoms of liver involvement including liver failure, more in the
pediatric patients. I have had one adult present with advanced liver disease requiring
liver transplant.
Do you find it more as a primary disorder or a contributing disorder to some
other underlying liver disease?
I would say it is more commonly a contributing factor in a patient with some
other underlying liver disease.
Any treatments you have used?
While there is treatment for lung disease due to alpha-1 antitrypsin deficiency,
there is none for liver disease. This is because of the different pathogenesis of the two.
The lung disease of alpha-1 antitrypsin deficiency is due to proteolytic damage from
PMNs, which is usually inhibited by alpha-1 antitrypsin. Synthetic alpha-1 antitryp-
sin is available to prevent this damage. The liver disease, on the other hand, is due to
the retention of abnormal A1AT-Z molecules in the hepatocyte leading to cell dam-
age. Synthetic alpha-1 antitrypsin is of no benefit here. Therefore, there is no treat-
ment for liver disease except transplant when end-stage complications have developed.

J. Franco (*)
Department of Medicine-Division of Gastroenterology and Hepatology, Medical College of
Wisconsin, Milwaukee, WI, USA
e-mail: [email protected]

© The Author(s), under exclusive license to Springer Nature 295


Switzerland AG 2023
W. H. Sobin et al. (eds.), Managing Complex Cases in Gastroenterology,
https://doi.org/10.1007/978-3-031-48949-5_61
Chapter 62
Suspected Gilbert’s

Jose Franco

A 37-year-old male is seen because of an elevated bilirubin. He is asymptomatic.


His total bilirubin ranges from 2.2 to 2.9 with a direct of 0.4–1.0 over the last
3 months. Physical exam is normal. An US of the liver with Doppler is normal.
Hepatitis panel, autoimmune markers, iron saturation, ceruloplasmin, and alpha-1
antitrypsin are all normal. His reticulocyte count and LDH are normal.
Would you diagnose this as Gilbert’s? How do you approach cases of sus-
pected Gilbert’s, where the direct bilirubin is higher than expected?
In this case, where you have an elevated unconjugated bilirubin, I think you’re
either dealing with Gilbert’s or hemolysis, and you never want to miss hemolysis. I
check the reticulocyte count and haptoglobin and have them look at the red blood
cells on the smear to make sure there is no hemolysis.
Here, the elevated bilirubin is mostly unconjugated and ranges from 2.2 to 2.9.
But, from personal experience, I will tell you that it can go into the mid-three range.
I know this because I have Gilbert’s, and I’ll frequently be in that 3.5 range when
they have me go for blood tests. They’ll always call me back with a critical lab
value, and I have to explain it to them. The unconjugated bilirubin is worsened with
physiologic stress, and when you’re fasting. My wife can look at me when I get
home and tell whether I have skipped lunch, seeing whether I look jaundiced. But if
you are under the weather, if you have any viral syndrome, many things will increase
it. You’re supposed to check the bilirubin in a fasting state.
Now in this case, there is a mild elevation of the conjugated bilirubin. In cases
where it is predominantly conjugated bilirubin, rather than unconjugated, you may
be looking at those rare disorders—Rotor and Dubin-Johnson. In these two
disorders, the basic defect is in the transporter that gets the conjugated bilirubin out

J. Franco (*)
Department of Medicine-Division of Gastroenterology and Hepatology, Medical College of
Wisconsin, Milwaukee, WI, USA
e-mail: [email protected]

© The Author(s), under exclusive license to Springer Nature 297


Switzerland AG 2023
W. H. Sobin et al. (eds.), Managing Complex Cases in Gastroenterology,
https://doi.org/10.1007/978-3-031-48949-5_62
298 J. Franco

of the hepatocyte, into the bile duct. These conditions are very rare, Gilbert’s on the
other hand is very common, 6–7% of the population. But in a case like this, where
the vast majority is unconjugated, I think the mild elevation of the direct bilirubin is
probably a red herring.
There are certainly cases where you can have other underlying liver disease in
association with Gilbert’s, but then you usually see a rise in the AST and ALT or
alkaline phosphatase, in addition to the bilirubin. If it’s just a rise in bilirubin and
it’s mostly indirect with a slight elevation of the direct, it’s usually going to be
Gilbert’s.
Chapter 63
The Significance of AST > ALT

Francisco Durazo

A 42-year-old female is referred for abnormal liver enzymes with an AST of 52 and
ALT of 38. She has a normal BMI but mildly increased waist circumference. She has
two glasses of wine a night. An US shows some steatosis but is otherwise negative.
Does the AST > ALT send off any alarm bells?
Yes, we wonder if she is actually drinking more than two glasses of wine a day;
is alcohol-induced liver injury responsible for these numbers? The normal pattern
for transaminases is ALT > AST, which also holds true for most liver diseases,
including viral hepatitis, autoimmune hepatitis, and even fatty liver disease. But we
tend to see AST > ALT in patients with alcoholic liver disease, and we see it in
patients who have gone on to develop cirrhosis. In cirrhosis, the production of ALT
is decreased and so the AST > ALT. Therefore, in this patient, what is the explana-
tion for the AST > ALT? Is it coming from alcohol or from occult cirrhosis, or might
the elevated AST be coming from muscle damage or hemolysis?
We also note that the ultrasound shows steatosis. Anyone with fatty liver deserves
a workup for chronic hepatitis, which includes serology for hepatitis B, hepatitis C,
autoimmune hepatitis, Wilson’s disease (in the right age cohort), alpha-1 antitrypsin
deficiency, and celiac disease.
If the workup for chronic hepatitis returns negative, would you pursue a
liver biopsy?
If all those labs return negative, we suspect the patient has alcoholic steatohepa-
titis. The only way to make this diagnosis is with liver biopsy. We would discuss
liver biopsy with the patient, elaborating on how it could alter management and help
with determining prognosis. In the meantime, we would suggest the patient stop

F. Durazo (*)
Division of Gastroenterology and Hepatology, Department of Medicine,
Medical College of Wisconsin, Milwaukee, WI, USA
e-mail: [email protected]

© The Author(s), under exclusive license to Springer Nature 299


Switzerland AG 2023
W. H. Sobin et al. (eds.), Managing Complex Cases in Gastroenterology,
https://doi.org/10.1007/978-3-031-48949-5_63
300 F. Durazo

drinking and recheck liver enzymes after 3 months. If the labs normalize, we would
probably cancel the biopsy. If they remain abnormal, we would proceed with biopsy.
An important pearl is that the biopsy appearance of NASH is identical to that of
alcoholic steatohepatitis, and the only way of distinguishing the two is by history.
But, as noted, the fact that the AST is higher suggests alcohol is the culprit; usually
in NASH, the ALT > AST, although the ratio has been noted to reverse in some cir-
rhotics with NASH.
Chapter 64
Marked Hyperbilirubinemia

Kia Saeian

A 75-year-old male presents to the ER with fever and elevated LFTs. An US shows
a stone in the CBD, with a very high bilirubin. His alk phos was 480, his AST was
160, and his ALT was 200. His total bilirubin is >30, and direct bilirubin was >20.
The patient had a normal bilirubin 1 month earlier. MRI of the liver shows multiple
stones in the gallbladder, a common duct stone, and no suggestion of biliary stric-
ture, hepatobiliary, or pancreatic neoplasm. PMH is positive for mild CHF and
mild renal insufficiency.
His abdominal pain started about a week before coming to the ER. He is mildly
hypotensive. His congestive heart failure is worsened. He is started on broad-­
spectrum antibiotics and has an urgent ERCP and stone extraction with clearing of
the bile duct and pus. His blood cultures return positive, growing gram-negative
rods. After the procedure, all his liver enzymes are improving except his bilirubin.
He remains mildly hypotensive. Four days after the procedure, his bilirubin remains
>30, although all the other liver enzymes are returning toward baseline.
How can you get such a high bilirubin in this setting?
A good rule of thumb that I use is that it is atypical to have a bilirubin over 20
and, particularly, a bilirubin of over 30 with pure bile duct obstruction. In these
circumstances, I always consider other potential contributors that may lead to intra-
hepatic cholestasis and hemolysis. In this particular case, the history provides a
number of clues to other potential contributors. The mild congestive heart failure
can lead to cholestasis due to passive congestion. Renal insufficiency can result in
more prolonged hyperbilirubinemia and slower clearance of the bilirubin. And,
most importantly, the positive blood cultures with gram-negative rods can result in

K. Saeian (*)
GI/Hepatology Division, Department of Medicine, Medical College of Wisconsin,
Milwaukee, WI, USA
e-mail: [email protected]

© The Author(s), under exclusive license to Springer Nature 301


Switzerland AG 2023
W. H. Sobin et al. (eds.), Managing Complex Cases in Gastroenterology,
https://doi.org/10.1007/978-3-031-48949-5_64
302 K. Saeian

cholestasis of sepsis, which can develop from impairment of bile transport in the
setting of sepsis. In this setting, continued hypoperfusion, either due to his low
blood pressure or passive congestion from his heart failure, can further prolong this
episode.
What are the different issues to keep in mind when you are confronted with
hyperbilirubinemia (in the 20 s or 30 s range) in patients who are septic, have
cardiac surgery, or have multisystem disease, often in an ICU setting?
By far, the most common scenario is due to infection or cholestasis of sepsis,
which, again, is believed to be, at least in part, due to a defect in bile transport due
to sepsis. The most severe form of this is cholangitis lenta, which has more com-
monly been reported in liver transplant patients and is commonly associated with a
poor overall prognosis. Cholangitis lenta itself is a histologic diagnosis in which
there is proliferation of dilated bile ductules and inspissated bile along with neutro-
phils and, on occasion, portal inflammation with a lymphoplasmacytic infiltrate. As
many physicians know, it is common to see episodes of cholestasis of sepsis, but
physicians rarely encounter cholangitis lenta because we rarely proceed with liver
biopsy in such settings. This is often because these patients are so critically ill and
the biopsy is not felt to alter management. This may also be why it is thought to be
more common in liver transplant patients, since those patients are much more likely
to undergo a liver biopsy when critically ill.
We already touched on a couple of scenarios outlined in the first question of this
section that can result in hyperbilirubinemia, often with aminotransferase and alka-
line phosphatase elevations. Entities that can result in more of an isolated hyperbili-
rubinemia with mild alteration of other liver enzymes, albeit not always in the 20–30
range, include heart failure/passive congestion, medications (ceftriaxone for
instance is a common contributor), hemolysis (in the setting of indirect hyperbiliru-
binemia), and the effects of anesthesia. The latter is particularly an issue in patients
with underlying cirrhosis. It is not uncommon for us to see patients without known
prior liver disease who were subsequently diagnosed as having cirrhosis, because
they developed hyperbilirubinemia in response to undergoing anesthesia, particu-
larly for an abdominal or cardiac operation.
Chapter 65
Suspected NAFLD

James Esteban

A 35-year-old Latin female is being evaluated for elevated liver enzymes. She has
an ALT of 160, AST of 120, Alk phos of 140 (nL up to 80), bilirubin of 1.2, platelet
count of 180,000, and albumin of 3.8. She has no alcohol and no family history of
liver disease. She is on no medications. Her BMI is 32, and her HbA1C is 6.2.
Hepatitis panel is negative, autoimmune markers are negative, and ceruloplasmin
and alpha-1 antitrypsin phenotype are normal.
How would you approach this patient?
She may have nonalcoholic fatty liver disease, or NAFLD, based on her risk fac-
tors of increased BMI and prediabetes. We need to get an abdominal or right upper
quadrant ultrasound for confirmation that she has hepatic steatosis. It is important to
test and exclude other causes of chronic liver diseases, such as chronic viral hepati-
tis; autoimmune liver diseases, like autoimmune hepatitis and primary biliary chol-
angitis; and inherited metabolic types of liver diseases, such as hemochromatosis,
Wilson’s disease, and alpha-1 antitrypsin deficiency. We should check that the
patient is not receiving known steatogenic medications, such as methotrexate, amio-
darone, or certain systemic chemotherapeutic agents.
Recognizing the primacy of metabolic risk factors on the pathogenesis of the
disease and the exclusionary and potentially stigmatizing verbiage of the current
nomenclature, an international expert panel recently published a consensus state-
ment renaming NAFLD to metabolic dysfunction-associated steatotic liver disease,
or MASLD [1].
Ultrasound shows steatosis without obvious signs of cirrhosis or portal
hypertension.

J. Esteban (*)
Division of Gastroenterology and Hepatology, Department of Medicine, Medical College of
Wisconsin, Milwaukee, WI, USA
e-mail: [email protected]

© The Author(s), under exclusive license to Springer Nature 303


Switzerland AG 2023
W. H. Sobin et al. (eds.), Managing Complex Cases in Gastroenterology,
https://doi.org/10.1007/978-3-031-48949-5_65
304 J. Esteban

What are your next steps?


First, I will stratify the patient’s risk for significant fibrosis. Inflammation (or
nonalcoholic steatohepatitis, NASH) and fibrosis are the most important predictors
of hepatic decompensation, hepatocellular carcinoma, and mortality [2]. This can
be done noninvasively through the use of serum biomarkers or through elastography
or invasively with liver biopsy.
Biomarkers from standard lab tests can be used to calculate risk scores, such
as NAFLD fibrosis score (NFS) or the Fibrosis-4 (FIB-4) index. Both are
accessible from various publicly available websites. There are also proprietary
serum biomarkers such as FibroMeter® (Echosens) and the Enhanced Liver
Fibrosis or ELF™ score (Siemens). Proprietary biomarkers are slightly more
accurate that nonproprietary biomarkers in diagnosing significant and advanced
liver fibrosis.
On the other hand, elastography measures the stiffness of the liver and uses this
as a surrogate for the stage of liver fibrosis. In the clinic, vibration-controlled tran-
sient elastography or FibroScan® (Echosens) is available and provides real-time
results at the bedside. Radiologists are also able to measure liver stiffness through
ultrasound elastography or magnetic resonance (MR) elastography.
Negative predictive values of these noninvasive markers are generally very good
and, thus, “low” or “low risk” values, in general, excludes advanced liver disease.
However, the positive predictive values of these tests for diagnosing significant
stages of fibrosis (i.e., stage 2 and above), and cirrhosis, are not as good (although
still reasonable). Patients with “indeterminate,” “high,” or “high risk” values on
noninvasive testing may be offered a liver biopsy to clarify fibrosis staging and
determine the presence of steatohepatitis. If the biopsy shows NASH and fibrosis,
then the patient may benefit from off-label use of pharmacotherapies, such as pio-
glitazone and glucagon-like peptide-1 (GLP-1) receptor analogs, or from enroll-
ment in a clinical trial.
Next, all patients with NAFLD/MASLD should be counseled on lifestyle inter-
ventions. We should counsel the patient to lose at least 5–7%, and ideally >10%, of
their body weight. Weight loss of at least 5%, 7%, and 10% is associated with,
respectively, reduction in hepatic steatosis, resolution of NASH, and stabilization
and potential regression of liver fibrosis [3]. Other lifestyle interventions that can be
recommended include:
• Reduce daily calories by 500–1000 kcal/day.
• Minimize saturated fats and refined carbohydrates.
• Avoid sugar-sweetened beverages.
• Mediterranean diet should be considered, although low-carb/low-fat diet and
intermittent fasting both appear to have comparable efficacy.
• Regular physical activity equivalent to 150–300 min weekly of moderate inten-
sity aerobic exercise.
• Some resistance and weight training should complement aerobic exercise.
65 Suspected NAFLD 305

While weight loss is very important, many of these other interventions can
improve hepatic steatosis and steatohepatitis, even without significant weight loss,
especially among patients with lean NAFLD/MASLD.
Patients always ask if they should avoid alcohol. Early data suggested that light
alcohol may be protective in NAFLD or MASLD. However, more recent data from
prospective studies indicate that “moderate” alcohol use reduces the likelihood of
clearing NASH while increasing the risk of fibrosis [4].
You have a FibroScan® in your clinic, are you getting a FibroScan® on all
patients like this? How accurate is the FibroScan® in NAFLD?
I routinely obtain a FibroScan® on all of my NAFLD/MASLD patients. FibroScan
has >90% negative predictive value for ruling out advanced fibrosis in NAFLD/
MASLD [5]. Thus, if liver stiffness on FibroScan® is low, or less than 7–8 kPa, I feel
comfortable and confident in excluding advanced fibrosis. I reassure the patient, and
we continue working on lifestyle interventions.
The positive predictive value for FibroScan® in diagnosing significant fibrosis
and cirrhosis in NAFLD/MASLD patients is modest, ranging from 40 to 70% [6],
especially if the patients are overweight and obese. The positive predictive value is
better in those with lean NAFLD/MASLD. For these patients, I offer liver biopsy
for staging purposes.
How do liver biopsies get done at your institution?
Liver biopsies can be done percutaneously, by a hepatologist or by a diagnostic
radiologist, or transvenously, by an interventional radiologist. In percutaneous biop-
sies, the proceduralist may elect to do the procedure under direct ultrasound guid-
ance (usually the radiologists) or “blindly” after identifying and marking suitable
intercostal areas via bedside ultrasound (usually the hepatologists).
In transvenous or transjugular liver biopsy, the interventional radiologist passes
a wire and needle through the internal jugular vein, down the vena cava, and into a
hepatic vein (usually the right), from where they collect cores of liver tissue. The
interventional radiologist can also measure free and wedged hepatic venous pres-
sures during the same procedure, which provides valuable information in diagnos-
ing portal hypertension.

References

1. Rinella ME, Lazarus JV, Ratziu V, Francque SM, Sanyal AJ, Kanwal F, et al. A multi-­
society Delphi consensus statement on new fatty liver disease nomenclature. Hepatology.
2023;29(1):101133.
2. Sanyal AJ, Van Natta ML, Clark J, Neuschwander-Tetri BA, Diehl A, Dasarathy S, et al.
Prospective study of outcomes in adults with nonalcoholic fatty liver disease. N Engl J Med.
2021;385(17):1559–69.
3. Vilar-Gomez E, Martinez-Perez Y, Calzadilla-Bertot L, Torres-Gonzalez A, Gra-Oramas B,
Gonzalez-Fabian L, et al. Weight loss through lifestyle modification significantly reduces fea-
tures of nonalcoholic steatohepatitis. Gastroenterology. 2015;149(2):367–78.
306 J. Esteban

4. Ajmera V, Belt P, Wilson LA, Gill RM, Loomba R, Kleiner DE, et al. Among patients with
nonalcoholic fatty liver disease, modest alcohol use is associated with less improvement in
histologic steatosis and steatohepatitis. Clin Gastroenterol Hepatol. 2018;16(9):1511–20.
5. Mózes FE, Lee JA, Selvaraj EA, Jayaswal ANA, Trauner M, Boursier J, et al. Diagnostic accu-
racy of non-invasive tests for advanced fibrosis in patients with NAFLD: an individual patient
data meta-analysis. Gut. 2022;71(5):1006–19.
6. Xiao G, Zhu S, Xiao X, Yan L, Yang J, Wu G. Comparison of laboratory tests, ultrasound, or
magnetic resonance elastography to detect fibrosis in patients with nonalcoholic fatty liver
disease: a meta-analysis. Hepatology. 2017;66(5):1486–501.
Chapter 66
Treatment of NAFLD

Francisco Durazo

An obese nondiabetic male, age 39 with a BMI of 34, is referred with an ultrasound
showing steatosis. He has one or two drinks each weekend. His platelet count is
170 K. His ALT is 75, AST is 48, and alk phos is 150. His hepatitis panel, ANA,
ASMA, alpha-1-antitrypsin, ceruloplasmin, and celiac panel are all normal.
How would you manage this patient?
We explain that this is not purely a liver disease but rather a systemic inflamma-
tory condition. Many people with fatty liver and normal glucose have insulin resis-
tance. Therefore, just having a fatty liver should be a red flag for insulin resistance,
present in as many as 95% of people with fatty liver.
Treatment is not aimed specifically at the liver but rather at the entire body. The
first thing we recommend is exercise. Studies have shown that consistent exercise
over a 2-year period significantly improves steatosis and fibrosis on liver biopsy. We
recommend a Mediterranean diet, avoiding fructose and avoiding alcohol. We rec-
ommend drinking 2–3 cups of regular coffee daily.
We recommend weight loss. Evidence shows that weight loss of 10% of total
weight leads to dramatic improvement in liver histology. In a patient who weighs
240, a 10% loss in weight requires only losing 1 pound a week over 6 months.
For patients who are diabetic, pioglitazone is beneficial. It is certainly a far better
choice than insulin for patients with NAFLD. We offer vitamin E to patients with
biopsy proven NASH. We prefer to use it in women because vitamin E may increase
the rate of prostate cancer.

F. Durazo (*)
Division of Gastroenterology and Hepatology, Department of Medicine,
Medical College of Wisconsin, Milwaukee, WI, USA
e-mail: [email protected]

© The Author(s), under exclusive license to Springer Nature 307


Switzerland AG 2023
W. H. Sobin et al. (eds.), Managing Complex Cases in Gastroenterology,
https://doi.org/10.1007/978-3-031-48949-5_66
308 F. Durazo

We recommend bariatric surgery for those patients with NAFLD and a BMI >40,
who can’t lose weight by other means, or in patients with a BMI >35, who have
other comorbidities, like obstructive sleep apnea, diabetes that is hard to control, or
severe hyperlipidemia.
Can you predict which patients with NAFLD will have a more ominous clin-
ical course?
Yes, we think the NAFLD fibrosis score is a good noninvasive predictor.
Chapter 67
Treatment of MASLD

Achutan Sourianarayanane

A 43-year-old female with a BMI of 31 is seen by her primary care doctor and found
to have an AST of 60 and an ALT of 90. She is prediabetic and has one or two drinks
on weekends. Alkaline phosphatase and bilirubin are normal. Other labs are sent
off, including hepatitis panel, autoimmune panel, ceruloplasmin, A1AT, and Fe/
TIBC, which return normal. Ultrasound shows steatosis. The doctor tells her to
abstain completely from alcohol and try to lose 20 lbs. over the next 3 months. If her
liver enzymes don’t improve, he says, he will refer her to a hepatologist, and she
might end up needing a liver biopsy.
1. Is this initial strategy of weight loss and abstention and then waiting and
seeing reasonable?
Her imaging suggests the presence of hepatic steatosis. Based on the pattern of
liver chemistry, along with the presence of metabolic risks and the absence of excess
alcohol intake, her fatty liver is secondary to metabolic dysfunction-associated stea-
totic liver disease (MASLD). MASLD is the current terminology for nonalcoholic
fatty liver disease (NAFLD), which is an encompassing diagnosis of those with fatty
liver with metabolic risks. Similarly, metabolic dysfunction-associated steatohepa-
titis (MASH) is the replacement term for prior nonalcoholic steatohepatitis (NASH).
Lifestyle changes are the most important components for the management of this
condition and should be incorporated in every patient with MASLD. Although she
does not consume excess alcohol, reduction or abstinence from alcohol will facili-
tate improvement in her liver disorder. Hence, the recommendation for abstinence is
appropriate within the current guidelines. Studies recommend lifestyle changes that
result in sustained weight loss of 5–7%, which has been found to be beneficial to
patients with MASLD. It also helps many patients with prediabetes improve their

A. Sourianarayanane (*)
Division of Gastroenterology and Hepatology, Department of Medicine, Medical College of
Wisconsin, Milwaukee, WI, USA
e-mail: [email protected]

© The Author(s), under exclusive license to Springer Nature 309


Switzerland AG 2023
W. H. Sobin et al. (eds.), Managing Complex Cases in Gastroenterology,
https://doi.org/10.1007/978-3-031-48949-5_67
310 A. Sourianarayanane

overall health and the metabolic risk associated with MASLD. Although a target
weight loss of 5–7% is considered appropriate, it may be difficult for most patients
to achieve consistently. Improvement in metabolic risks has been noticed, with con-
sistent lifestyle changes, including diet and exercise, even in the absence of weight
loss. This may need to be considered in subsequent clinical visits and recommenda-
tions [1–3].
2. If the numbers were reversed and the AST was 90 and ALT 60, would that
raise any alarm bells?
A liver chemistry pattern with an AST higher than ALT can suggest more than
one clinical possibility in this patient. The occurrence of AST higher than ALT in
patients with MASLD could raise concerns about advanced fibrosis or even the
presence of cirrhosis. However, her AST was 1.5 higher than her ALT. This could
also raise concerns about other causes, such as a relatively higher level of alcohol
consumption than could be safely metabolized by her liver. Her alcohol usage is
well within liver societies’ and the American Dietary Association’s recommenda-
tions. It is possible a person’s perception of the standard unit of alcohol measure
differs from those recommendations; besides, the effect of a given amount of alco-
hol may vary in an individual [2, 4].
She is only able to lose 5 lbs., and her repeat LFTs 3 months later are essentially
unchanged. She is referred to you, the hepatologist.
3. Is there any role for FibroScan®? Would you order one? What would be
the considerations in deciding on this or using any other noninvasive measure
for the presence of fibrosis or potentially classifying her as having steatohepa-
titis versus simple fatty liver?
Liver chemistries, commonly called liver function tests, do not reflect the sever-
ity of liver disease. She could have hepatic steatosis with an ongoing inflammatory
process and fibrosis. The MASLD fibrosis score and FIB-4 are commonly used
noninvasive biomarkers based on clinical and biochemical parameters. These bio-
markers stratify patients into a low, intermediate, or advanced stage of MASLD. These
tests are easily available and less costly to administer. Although these biomarkers
are sensitive enough to detect an advanced stage of the disease, they are less specific
in diagnosing them. Hence, a second test, such as FibroScan®, should be used to
confirm the diagnosis, when available [5, 6].
The duration of the disease was not known in this patient. She also has a meta-
bolic risk and elevated aminotransferases. As MASLD is a “silent disease” with
minimal or no symptoms until the onset of decompensated cirrhosis, assessing the
severity of the disease is appropriate. An evaluation by FibroScan® is appropriate.
4. How accurate is FibroScan® when evaluating fatty liver? Do you rely on
the CAP score?
FibroScan® can diagnose the presence of fat in the liver if it is more than 11%.
Following a FibroScan® (also called vibration-controlled transient elastography®),
two sets of results are obtained. One is a controlled attenuation parameter (CAP),
which correlates with the presence of hepatic steatosis or a fatty liver. It requires the
presence of hepatic steatosis of >11% to be detected and quantified by this method.
67 Treatment of MASLD 311

Studies have shown that the CAP value correlates with the histological grade of
steatosis. The cutoff ranges used to differentiate various histological grades of ste-
atosis may vary due to patient characteristics, such as BMI. The other value obtained
from FibroScan® is transient elastography measured in kilopascals, which indicates
the fibrosis stage of liver disease in MASLD. The elastography results may be less
reliable in patients with a BMI of >35 [7, 8].
The SAF score (steatosis, activity, and fibrosis) is a histological scoring system
for patients with MASH. It is used mostly in Europe and in MASLD research. A
similar histological scoring system called NAFLD activity score (NAS) proposed
by the NASH Clinical Research Network (CRN), which is often referred to as the
NASH-CRN, is used commonly in the United States.
5. Would you do a liver biopsy? Which patients do you think should have a
liver biopsy?
A liver biopsy to diagnose or stratify a patient with MASLD is becoming less
common. The use of liver biopsy is usually considered for patients in whom there is
a concern for an advanced stage of fibrosis. Noninvasive tests are recommended to
stratify MASLD patients initially as a screening test. These tests, such as the
MASLD fibrosis score, FIB-4, etc., are based on clinical and biochemical parame-
ters. These tests have a high sensitivity for detecting patients with an advanced stage
of MASLD. Individuals who are found to have a low probability of advanced
MASLD do not require further testing. A second test with higher specificity, such as
elastography (FibroScan®), is performed on patients with an intermediate or
advanced stage on initial screening tests. Patients who cannot be categorized and are
considered to have intermediate risk will benefit from liver biopsy staging. A liver
biopsy is also required if the patient is considered for a clinical trial [5].
If this patient has a BMI of less than 35, the FibroScan® test results could be
considered reliable. If FibroScan® elastography does not suggest advanced fibrosis
(<10 kPa), a liver biopsy is not recommended. A liver biopsy could be considered
otherwise [8].
6. Other than avoiding alcohol and weight loss, are there any other second-
ary, behavioral, or dietary measures that can be helpful? Would you have done
any other serologic or other testing at this point? Do you routinely check all
your patients for alcohol use even if they claim to drink minimally, such as in
this patient?
Significant sustained weight loss has been shown to improve many histological
parameters seen in MASLD. A 5–7% weight loss has been associated with a reduc-
tion in steatosis. With additional weight loss, other histological features, such as
inflammation and fibrosis reduction, have been documented. A weight loss of >15%
has been shown to improve fibrosis stages on histology. Avoidance of alcohol is an
important factor among those who have histological improvements with weight
loss. Significant and sustained weight loss is not easily achievable by most patients.
Other lifestyle changes, such as regular exercise outside work, improve the overall
health of the liver. Some of the biochemical parameters and insulin sensitivity have
improved with this measure, even in the absence of weight loss [2, 3].
312 A. Sourianarayanane

Different diets have been suggested for the management of patients with
MASLD. These include diet modification based on the reduction of macronutrients,
such as fat, and carbohydrates, including fructose, either individually or in combi-
nation and along with a variation in the time of meal consumption. These modifica-
tions have resulted in weight loss and an improvement in their body weight,
metabolic risks, and liver aminotransferases. Many of these modifications are diffi-
cult to maintain, and a long-term benefit is less often achieved. A portion- and
calorie-­controlled Mediterranean diet (without alcohol use) is beneficial in MASLD
and is also easier to adhere to in the long term. Studies have also found that caffeine
intake is associated with a reduced progression of MASH.
Some providers routinely recommend tests such as phosphatidylethanol (PEth)
to measure alcohol in patients with a presumed diagnosis of MASLD to corroborate
the role of alcohol in their disease. This is considered, because an individual’s per-
ception of the standard unit of alcohol measure may differ from the guidelines of
liver societies and the American Dietary Association. Additionally, many people
underestimate their alcohol use. Patients with MAFLD can include those with alco-
hol usage or viral infection. This is a move away from the use of MASLD, which is
a diagnosis of exclusion in those with an absence of other causes.
Scenario 1 Liver biopsy shows steatosis without any changes of steatohepatitis or
any fibrosis.
7. Is steatosis without steatohepatitis a benign entity? Is the prognosis that
much better? Do you have thoughts on what proportion of these patients prog-
ress to more severe liver injury? Does this depend on the risk factors?
Our initial understanding of the minimal or nonprogressive form of MASLD
patients with simple steatosis was mainly from retrospective studies. These studies
found that patients with hepatic steatosis without MASH had a benign course and
did not progress to MASH. Newer studies suggest that a portion of patients with
steatosis alone without MASH on initial biopsy had findings of MASH on subse-
quent histological evaluation. The proportion of patients with initial simple steatosis
who progress to advanced disease is unknown. Some studies suggest that nearly
25% of these patients may progress to MASH or may even have fibrosis later.
Insulin resistance plays a significant role in progressive disease. Among all the fac-
tors, the presence of fibrosis is significant and associated with poor outcomes in
patients with MASLD [9].
8. In this scenario, are there any medications that you would consider or
would initiate at this point?
Currently, there are no FDA-approved medications for patients with simple ste-
atosis. The only approved medication is vitamin E, which is considered for patients
with biopsy-proven MASH without type II diabetes. This patient has diabetes and
does not have MASH; hence, lifestyle changes without additional pharmacological
agents is appropriate for her [10, 11].
67 Treatment of MASLD 313

Scenario 2 Same patient but now the liver biopsy shows changes of steatohepati-
tis, including ballooning without any fibrosis.
9. What does it take to turn steatosis into steatohepatitis? Are there any
causative factors you can identify, modify, or even prevent?
Multiple pathophysiological processes are now considered to contribute to the
progression of MASLD. Type II diabetes, or insulin resistance, is an important fac-
tor associated with the progression of the disease. An inflammatory process second-
ary to lipid accumulation and a dysfunctional oxidative process, called lipotoxicity,
occur. Different chemicals are released because of these inflammatory processes.
This creates a subsequent event that results in neutrophil and lymphocyte accumula-
tion, altered cell death, fibrosis deposition by macrophages, and the progression of
the disease to steatohepatitis. Lifestyle changes that result in a consistent reduction
of metabolic risks, such as excess body weight, insulin resistance, and other modifi-
able factors, reduce the progression to steatohepatitis [2, 3, 10].
10. Once you’ve developed steatohepatitis, is this fully reversible? What is
the chance of sampling error on the biopsy?
Fortunately, this patient does not have fibrosis. It may be easier to reverse the
inflammatory process and steatosis in the absence of fibrosis. Both steatosis and
steatohepatitis reversal are possible with significant weight loss (possibly 7–10%),
along with metabolic risk reduction. However, complete improvement may not
occur in all patients. A liver biopsy is a reliable way to diagnose MASLD and stage
accurately, although it samples only 1/50,000 or 1/100,000th of the entire liver
organ. It has been suggested that there could be errors due to this. Studies have
shown variations between two samples taken on the same day and from different
lobes of the liver. Despite these variations, liver biopsy is the current gold standard
for the diagnosis of this condition.
11. Do you think you can “save” many of these patients from developing
advanced fibrosis?
Active research and clinical trials are being conducted to mitigate MASLD and
its progression. As more than one pathophysiological process is involved, a combi-
nation of medications is administered to reduce fibrosis formation and progression.
Lifestyle changes remain a consistent and valuable component of this disease’s
management. If possible, a combined holistic approach would be the key to improve-
ment for most of these patients.
12. What medications, if any, would you initiate in this particular case based
on the biopsy? Any other secondary measures?
Studies have shown that a sustained weight loss of 7–10% could reverse the
inflammatory process. With this weight loss, there is an associated improvement in
metabolic risks, such as insulin resistance, hypertension, and lipid profile, which
contribute to an overall improvement in MASLD patients. This could be achieved
through consistent lifestyle changes, such as dietary and exercise modifications.
There are also a few pharmacological agents being considered.
314 A. Sourianarayanane

Vitamin E is used in patients without diabetes and has shown improvement in


steatohepatitis. Incretins, such as GLP-1 agonists (liraglutide and semaglutide), are
beneficial for MASLD patients with and without diabetes. Studies have shown
improvement in liver aminotransferase levels, improvement of steatohepatitis, and
weight loss, and lowering of HbA1c among diabetes patients is seen with GLP-1
agonists. Obeticholic acid is another agent that modifies bile acid synthesis; it
reduces inflammation in the liver and fibrosis in MASH patients because of this.
These agents are in the final stages of clinical trials for MASH. Many newer phar-
macological agents are undergoing clinical trials, either alone or in combination;
they modify different metabolic processes involved in MASLD. These agents
include those modifying lipid synthesis, the oxidative process, cell death, and fibro-
sis formation. Bariatric surgery is another important and valid consideration in
appropriate patients with MASH [10–12].
Scenario 3 Liver biopsy shows steatohepatitis and stage 3 fibrosis.
13. How often would you expect to detect patients with fatty liver who are
already in stages of advanced fibrosis/cirrhosis?
Most patients with MASLD do not have symptoms; hence, many are diagnosed
incidentally on abdominal imaging for other reasons. A portion of this diagnosis can
occur in their late stages of the disease. It is estimated that <5% of patients with
MASLD may progress to cirrhosis; therefore, we can expect that <5% of MASLD
patients may have advanced fibrosis at the time of detection.
14. How reversible is this stage of liver disease? With what measures?
Fibrosis is less likely to reverse with lifestyle changes and with currently
approved pharmacological agents. A weight loss of >15% may be required for
improvement in fibrosis. Bariatric surgery is one option, especially for patients with
advanced fibrosis. A reduction in the fibrosis stage has been observed many years
after successful bariatric surgery. Newer pharmacological agents that modify col-
lagen synthesis and deposition are being considered. If these agents are successful,
they may also alter the course of this disease [10–12].
15. Assuming that you implement screening for esophageal varices and con-
sidering HCC surveillance, are there other measures you would imple-
ment now?
Esophageal varices screening and HCC surveillance are usually considered for
patients with cirrhosis. Liver societies do not recommend routine screening for vari-
ces or HCC in patients without cirrhosis. As this patient has advanced fibrosis, she
will benefit from periodic evaluation for fibrosis progression and the onset of cir-
rhosis with FibroScan® or transient elastography. Elastography, as measured in kilo-
pascals, correlates with portal pressure and decompensated liver disease. If there is
an increase in kilopascals and a significant reduction in platelets (<180), the above
screenings should be considered. When performing ultrasound-based elastography,
the results may be less reliable if the patient has a high BMI. Due to a similar rea-
son, ultrasound is less reliable in detecting liver lesions in subjects with high
BMI. Some consider an MRI (which is costly) or a CT scan (which could be a con-
cern in patients with renal dysfunction), alternating with ultrasound imaging for
HCC screening [2, 8].
67 Treatment of MASLD 315

16. Any medications you would start in this case?


In this patient, with the suggestions of steatohepatitis and the absence of overt
diabetes, adding 800 units of vitamin E to her medication regimen is appropriate.
Semaglutide, a GLP-1 agonist, or a similar agent can also be considered after appro-
priately educating the patient about the side effects and how to use it. This can
improve her body weight, insulin sensitivity, and liver aminotransferases. If she has
hyperlipidemia, a statin to reduce it would also be appropriate [10, 11].
17. Do you believe that patients with MASH truly progress to HCC without
developing cirrhosis first? Why is this, and do you screen any/all of your
patients with MASH for HCC?
About 10% of patients who have HCC with a diagnosis of MASLD do not have
cirrhosis. Many of the risk factors related to MASLD, such as a high BMI and dia-
betes, are also associated with malignancy, including HCC. However, no definitive
pathway has been found in MASH patients. Many of our screening protocols are
based on the costs involved in diagnosing a treatable condition and the quality of
life rendered to society as a result of it. This is based on the prevalence of the disease
and the incidence of HCC in a specific population. The current consensus guidelines
do not recommend screening for patients with MASH without cirrhosis for HCC,
unlike in patients with hemochromatosis or hepatitis B infection, where it is recom-
mended even in the absence of cirrhosis [2].
18. When would you refer this patient for a liver transplant? What would be
the chance of recurrence of MASH post-transplant?
MASH patients should be evaluated for liver transplantation in the appropriate
clinical setting, like any other patient. This would be in patients who have decom-
pensated liver disease or those with a relatively high model for end-stage liver dis-
ease (MELD). The referrals and transplantations for patients with MASLD are
usually no different from those for other chronic liver disorders. Many MASLD
patients also have other comorbidities that could be a concern during the periopera-
tive and postoperative phases. These should be evaluated to reduce morbidity or
mortality posttransplant through a multidisciplinary approach.
Following liver transplantation, the recurrence of hepatic steatosis among
patients with a pretransplant diagnosis of MASH is high. Hepatic steatosis occurs in
patients who have undergone liver transplantation for more than one reason [13].
Immunosuppressive agents required for the maintenance of the transplanted liver
are associated with many metabolic factors and increase the risks for hepatic steato-
sis posttransplantation. However, not all post-liver transplant patients with hepatic
steatosis develop MASH.
General Questions
1. Which patients with MASLD/MASH should consider having bariatric
surgery?
Any patient who could otherwise benefit from a bariatric procedure for their
health will also benefit if they have MASLD. Patients with MASH with advanced
fibrosis would benefit from significant weight loss following a bariatric procedure.
A weight loss of >15% occurs frequently following a bariatric procedure. A reduc-
tion or clearance of steatosis and inflammation, including ballooning cells, is found
316 A. Sourianarayanane

following a successful bariatric procedure. Stabilization or even a reduction of


fibrosis is seen after many years (>5 years). Patients with decompensated cirrhosis,
however, do not do well following bariatric surgery and are not generally recom-
mended to undergo this procedure [12].
2. How do patients with MASLD and fibrosis/cirrhosis compare to patients
with ALD in the same stage of disease? Assuming the alcoholic has stopped
drinking, which has the worse prognosis? Who will have a better outcome after
a liver transplant?
A direct comparison of outcomes between patients with ALD and MASLD is not
easy. Patients with MASLD may continue to have metabolic risks and may have
gradual MASLD progression over time. However, many patients with ALD may
improve or may not progress if their consumption of alcohol is discontinued entirely.
The patient outcomes in these two groups could also be affected by their age and
comorbidity, in addition to their liver disease-related outcomes. Many patients with
MASLD are older and have comorbidities due to the presence of metabolic risk
(i.e., high BMI, diabetes, and hyperlipidemia), such as cardiovascular, cerebrovas-
cular, and renal disorders. These could contribute to reduced overall outcomes fol-
lowing liver transplantation.
3. Is liver transplant restricted for any of the MASLD patients?
It is possible that MASLD patients have significant cardiovascular or other
comorbidities. As in any other patient, these risk factors have a bearing on periop-
erative and postoperative complications and may restrict those undergoing liver
transplantation. The body habitus of the patient may also cause some restrictions on
undergoing liver transplantations. Retrospective and large database studies suggest
that patients with a BMI >50 do not do as well as those with a BMI <50 following
liver transplantation. With the increasing technology and training of surgeons, many
patients with a higher BMI are undergoing liver transplantation with a good out-
come [14].
4. In which MASLD patients do you like to use vitamin E, pioglitazone, and
GLP-1 agonists?
Currently, the FDA recommends pharmacological intervention for patients with
a histological diagnosis of MASH. Vitamin E has been found to be beneficial in
MASH patients with improvement of inflammation and balloon cells on histology
among patients without diabetes. In patients with diabetes, vitamin E has not proven
to be beneficial. Some researchers recommend a combination of pioglitazone and
vitamin E for patients with diabetes and MASH. Others feel that the benefit observed
with this combination therapy is from pioglitazone alone and that the use of vitamin
E may not be required. Pioglitazone causes a reduction in hepatic steatosis. They
also have an increase in subcutaneous fat and overall weight gain. Hence, it is not
considered a preferred treatment for most patients. GLP-1 agonists can be used in
all patients with MASH, especially those with type II diabetes. They have also been
considered for the management of obesity. These patients should be educated about
potential hypoglycemic episodes when a GLP-1 agonist is used [10, 11].
67 Treatment of MASLD 317

5. Are any other drugs you like to use today in MASLD patients?
Statins have been found to benefit patients across the spectrum of MASLD. They
have been associated with a reduction in the progression of the disease. Statins also
benefit patients with hyperlipidemia, which is an important metabolic risk factor
in MASLD.
6. Are there any other drugs that are promising that you anticipate being
available in the next 5 years?
There are many drugs undergoing clinical trials in various phases that could be
used for the treatment of MASH. Obeticholic acid, an FXR agonist, has been prom-
ising. It reduces the inflammatory process in addition to fibrosis reduction in MASH
patients. A variation of this agent (Cilofexor) with fewer side effects is also being
considered. Cenicriviroc, a chemokine antagonist, reduced fibrosis in MASH
patients. Aramchol, an acetyl-co A inhibitor, and resmetirom, a thyroid hormone
receptor (ꞵ) agonist, reduced hepatic steatosis among MASLD patients. Elafibranor,
lanifibranor, and other peroxisome proliferator-activated receptor agonists reduce
inflammation in MASH. Agents that directly act on collagen and elastin cross-­
linkage (e.g., simtuzumab), reduce cell death and apoptosis (e.g., selonsertib), and
reduce fibrosis may also be available. These drugs could be used either alone or in
combination for an overall effective management of MASH in the future [10, 11].

References

1. Rinella ME, Lazarus JV, Ratziu V, et al. A multi-society Delphi consensus statement on new
fatty liver disease nomenclature. Hepatology. 2023;29(1):101133. https://doi.org/10.1097/
HEP.0000000000000520.
2. Chalasani N, Younossi Z, Lavine JE, et al. The diagnosis and management of nonalcoholic
fatty liver disease: practice guidance from the American Association for the Study of Liver
Diseases. Hepatology. 2018;67:328–57.
3. Sourianarayanane A, Pagadala MR, Kirwan JP. Management of non-alcoholic fatty liver dis-
ease. Minerva Gastroenterol Dietol. 2013;59(1):69–87.
4. Services. USDoAaUSDoHaH. Dietary Guidelines for Americans, 2020–2025. 2020; 9th
Edition.
5. Sourianarayanane A, McCullough AJ. Accuracy of steatosis and fibrosis NAFLD scores in
relation to vibration controlled transient elastography: an NHANES analysis. Clin Res Hepatol
Gastroenterol. 2022;46(7):101997.
6. Sumida Y, Yoneda M, Tokushige K, Kawanaka M, Fujii H, Yoneda M, Imajo K, Takahashi
H, Eguchi Y, Ono M, Nozaki Y, Hyogo H, Koseki M, Yoshida Y, Kawaguchi T, Kamada Y,
Okanoue T, Nakajima A, Japan Study Group Of Nafld Jsg-Nafld. FIB-4 first in the diagnostic
algorithm of metabolic-dysfunction-associated fatty liver disease in the era of the global meta-
bodemic. Life (Basel). 2021;11(2):143.
7. Sasso M, Miette V, Sandrin L, Beaugrand M. The controlled attenuation parameter (CAP):
a novel tool for the non-invasive evaluation of steatosis using Fibroscan. Clin Res Hepatol
Gastroenterol. 2012;36(1):13–20.
8. Caussy C, Chen J, Alquiraish MH, Cepin S, Nguyen P, Hernandez C, Yin M, Bettencourt
R, Cachay ER, Jayakumar S, Fortney L, Hooker J, Sy E, Valasek MA, Rizo E, Richards L,
318 A. Sourianarayanane

Brenner DA, Sirlin CB, Ehman RL, Loomba R. Association between obesity and discordance
in fibrosis stage determination by magnetic resonance vs transient elastography in patients
with nonalcoholic liver disease. Clin Gastroenterol Hepatol. 2018;16(12):1974–1982.e7.
9. Angulo P, Kleiner DE, Dam-Larsen S, Adams LA, Bjornsson ES, Charatcharoenwitthaya P,
Mills PR, Keach JC, Lafferty HD, Stahler A, Haflidadottir S, Bendtsen F. Liver fibrosis, but no
other histologic features, is associated with long-term outcomes of patients with nonalcoholic
fatty liver disease. Gastroenterology. 2015;149(2):389–97.
10. Sourianarayanane A, Challa SR. Non-alcoholic fatty liver: current management and future
trends. Gut Gastroenterol. 2020;3:001–17.
11. Oseini AM, Sanyal AJ. Therapies in non-alcoholic steatohepatitis (NASH). Liver Int.
2017;37(Suppl 1):97–103. https://doi.org/10.1111/liv.13302.
12. Sasaki A, Nitta H, Otsuka K, Umemura A, Baba S, Obuchi T, Wakabayashi G. Bariatric
surgery and non-alcoholic fatty liver disease: current and potential future treatments. Front
Endocrinol (Lausanne). 2014;5:164.
13. Sourianarayanane A, Arikapudi S, McCullough AJ, Humar A. Nonalcoholic steatohepatitis
recurrence and rate of fibrosis progression following liver transplantation. Eur J Gastroenterol
Hepatol. 2017;29(4):481–7.
14. Alvarez J, Mei X, Daily M, Shah M, Grigorian A, Berger J, Marti F, Gedaly R. Tipping the
scales: liver transplant outcomes of the super obese. J Gastrointest Surg. 2016;20(9):1628–35.
Chapter 68
Alcohol and Liver Disease in Women

Veronica Loy

A 60-year-old female is referred for elevated liver enzymes. She and her husband
enjoy drinking wine with dinner and have it most evenings. They generally each
have two glasses of wine, finishing off about 2/3 of the bottle.
Her physical exam is unremarkable. Her BMI is 20. Her AST is 90, ALT is 70, Alk
phos is 80, and bilirubin is 1.0. Hepatitis panel, autoimmune panel, ceruloplasmin,
and alpha-1 antitrypsin are all unremarkable. An US of the liver shows mild steato-
sis and no nodularity.
She is surprised because her husband recently had his annual physical exam and
his liver enzymes were normal.
What is your interpretation of these findings, and can you describe the
increased susceptibility women have to alcoholic liver disease?
It looks to me like this woman has some mild alcohol-related steatohepatitis.
Thankfully, her synthetic function is intact with that normal bilirubin. Her ultra-
sound does not show nodularity, so there is no obvious cirrhosis, but that doesn’t
necessarily mean that she doesn’t have some fibrosis, because that is difficult to
assess by ultrasound alone. She does appear to have some mild alcohol-related
steatohepatitis.
Why would she have this and her husband doesn’t? I think that in every indi-
vidual, their own risk for developing steatohepatitis and ultimately cirrhosis of the
liver is multifactorial. One of those factors is gender. There are certainly a lot of
other factors. I don’t know other health differences between her and her husband,
but gender alone is a driving factor in the risk of developing fibrosis, cirrhosis, or
just alcohol-related steatohepatitis.

V. Loy (*)
Division Gastroenterology and Hepatology, Department of Medicine, Medical College of
Wisconsin, Milwaukee, WI, USA
e-mail: [email protected]

© The Author(s), under exclusive license to Springer Nature 319


Switzerland AG 2023
W. H. Sobin et al. (eds.), Managing Complex Cases in Gastroenterology,
https://doi.org/10.1007/978-3-031-48949-5_68
320 V. Loy

There are many reasons for gender differences in liver disease. First, we know
that female patients metabolize alcohol differently than males. As a result, females
have a higher rate of steatosis and inflammation and cytokine release at a much
lower level of alcohol intake than males. Over the years, two to three glasses of wine
a night, for a decade, for a female, is enough to put someone at risk for cirrhosis.
Whereas in a male, it would be more like four to six a night. There is a very big dif-
ference in the way the body metabolizes alcohol at a lower level, which presents a
higher risk to these people. It also relates to body size and body mass. A smaller
stature person is going to have a lower threshold for damage from alcohol and gen-
erally women are smaller than men.
Most women are not aware of this distinction. Most of the women I see say that
they’re not drinking any more than their peers and they’re not drinking more than
their husband. They are astonished to end up in this situation. I really don’t think
there is enough awareness of the increased risk females have with alcohol
consumption.
We also know that in the primary care setting and the ER setting, women are far
less likely to be screened for alcohol use disorder. Even if female patients are
screened, they are far less likely to be referred for counseling or medical therapies.
We are really just not doing a good job of awareness in the medical community to
help these people prevent alcohol related liver damage.
What do you make of the AST > ALT?
Briefly, the AST, being greater than the ALT, is thought to be predictive of
alcohol-­related liver disease. The one caveat to that is once someone is cirrhotic,
regardless of the cause of cirrhosis, typically the AST is going to be higher than
the ALT.
I often see patients where their medical team is grilling them and accusing them
of drinking, because their AST is higher than their ALT. And they’re not drinking,
but they’re cirrhotic and that’s just what happens once someone is cirrhotic.
Would you simply advise the patient to stop drinking and follow the enzyme
levels over the next 3–6 months, or would you do further investigations (i.e.,
FibroScan®, biopsy, etc.).
As far as what to do next, I would probably advise her to work on minimizing or
stopping alcohol and repeat the enzymes in 3–6 months. The reason that I would not
advocate for a FibroScan® or a biopsy is that, first of all, FibroScan® is not validated
in people who are actively using alcohol. We tend to see an overestimate of the level
of fibrosis on FibroScan® in someone who’s using alcohol. It’s just not accurate in
my opinion. Similarly, with biopsy, if a biopsy is obtained on someone when there
is a lot of active inflammation, it’s very challenging for the pathologist to estimate
the amount of fibrosis. Finally, there is a really good chance in this woman that with
complete abstinence from alcohol the liver has a great chance to heal and regener-
ate. Therefore, what you see on a biopsy today may be very different from what you
see after 6 months of abstinence, during which you give the body the tools it needs
68 Alcohol and Liver Disease in Women 321

to repair itself. If she’s willing to stop, I think you’ll get a more accurate assessment
after waiting 6 months. Unfortunately, many people aren’t willing to stop. Without
alcohol cessation, I would move on to a biopsy, not a FibroScan®. If they’re not
going to stop alcohol, the FibroScan® isn’t something I offer them.
Chapter 69
Alcoholic Hepatitis

Francisco Durazo

A 52-year-old executive is fired from his job. Prior to that he would have two hard
drinks a day after work. After being fired, he goes on a drinking binge for a month.
He finally gets dragged in to see the doctor by his wife when he becomes jaundiced.
His PT is 17, INR is 2.1, bilirubin is 12, AST is 80, ALT is 50, Alk phos is 120, creati-
nine is 1.2, Hgb is 11.8, and Maddrey score is 34.
How would you manage this patient with presumed acute alcoholic hepatitis?
These days, acute alcoholic hepatitis is overdiagnosed. It seems like everyone
who is drinking and presents with jaundice is immediately diagnosed with alcoholic
hepatitis. As a result, we see many patients placed on steroids and getting infections,
where steroids weren’t needed in the first place.
So, the first thing is to make sure this really is acute alcoholic hepatitis. On physi-
cal exam, we need to see an enlarged liver with a bruit. If we examine the patient
and there is not an enlarged liver, then it is probably alcoholic cirrhosis we are deal-
ing with.
The benefit of steroids is limited. Even in the studies where acute alcoholic hepa-
titis was appropriately diagnosed and steroids given according to protocol, there
were a lot of complications, and the benefit of steroids was quite modest. Therefore,
we refrain from using steroids, unless a patient presents with florid alcoholic hepa-
titis, where the diagnosis is obvious. Steroids are approved if the Maddrey score is
>32 (I prefer to hold out for an even higher Maddrey score). In that case, we will
give prednisolone for up to 28 days.

F. Durazo (*)
Division of Gastroenterology and Hepatology, Department of Medicine,
Medical College of Wisconsin, Milwaukee, WI, USA
e-mail: [email protected]

© The Author(s), under exclusive license to Springer Nature 323


Switzerland AG 2023
W. H. Sobin et al. (eds.), Managing Complex Cases in Gastroenterology,
https://doi.org/10.1007/978-3-031-48949-5_69
324 F. Durazo

There has been a recent push for transplanting patients with acute alcoholic
hepatitis. What are your thoughts?
Those studies were done in France, where there is a national healthcare program
and patients got close follow-up and rehabilitation for their alcoholism. In those
studies, the results of transplant were excellent and alcohol recidivism low.
However, in this country, we are not good at treating the major disease, which is
alcoholism. We may be good at treating liver problems, but our approach to alcohol
rehabilitation is poor.
In centers in the USA, where transplant for acute alcoholic hepatitis has been
more successful, there have been four qualifiers that are essential: (1) this is the first
episode of acute alcoholic hepatitis, (2) the patient has no history of relapse from
sobriety, (3) there is strong family support, and (4) the patient has no history of
psychological problems. Under these conditions, there is a much higher likelihood
of success without relapse.
Chapter 70
Fatty Liver on Ultrasound with Normal
LFTs

Francisco Durazo

A 43-year-old male was having mid-abdominal pain, and an ultrasound was


obtained to rule out gallbladder disease. This showed a normal gallbladder but
some fatty steatosis. The patient drinks two glasses of wine a week. His liver enzymes
are totally normal. His BMI is 26.
In this case, does the finding of steatosis with normal liver enzymes merit
further workup?
All patients with fatty liver deserve a workup for chronic hepatitis. Some patients
with autoimmune hepatitis will present with fatty liver, some patients with Wilson’s
disease will present with fatty liver, and some patients with hepatitis C, particularly
those with genotype 3, will present with fatty liver. We also want to look for signs
of metabolic syndrome.
It would be appropriate to send off labs but not necessarily to perform liver
biopsy. In terms of liver biopsy, there is not enough manpower to do liver biopsies
on the huge number of people with fatty liver. We want to select those patients who
are most at risk of having NASH, those patients who demonstrate more components
of the metabolic syndrome. I personally use the NAFLD fibrosis score to help me
decide whether to perform a liver biopsy in these patients.

F. Durazo (*)
Division of Gastroenterology and Hepatology, Department of Medicine,
Medical College of Wisconsin, Milwaukee, WI, USA
e-mail: [email protected]

© The Author(s), under exclusive license to Springer Nature 325


Switzerland AG 2023
W. H. Sobin et al. (eds.), Managing Complex Cases in Gastroenterology,
https://doi.org/10.1007/978-3-031-48949-5_70
Chapter 71
Evaluation for Liver Damage
from Chronic ETOH Use

Francisco Durazo

A 52-year-old male is concerned about his drinking history. He has 3–4 glasses of
wine or 2–3 hard drinks a day. He is concerned that this could be causing liver dam-
age. His liver enzymes are normal as is his ultrasound.
Is there any role for FibroScan®? Has FibroScan® become a basic compo-
nent of the exam and evaluation of most patients with liver problems?
The jury is still out on how useful the FibroScan® is. Some practitioners find it
very useful while others believe that a good history, physical exam, and a basic
ultrasound will give you the same information. They point out that FibroScan® is
most accurate in detecting extremes – full-blown cirrhosis or a normal liver. These
clinicians feel that the basic workup will yield the same information. For those in-­
between cases, they believe there is too much discrepancy between the FibroScan®
results and liver biopsy, or MR elastography.
What are some factors that can confound FibroScan® results?
Factors that can blur FibroScan® results include hepatic congestion, biliary
obstruction, and obesity. Some believe that FibroScan® is very unreliable in patients
with NAFLD; it is most accurate in patients with viral hepatitis.

F. Durazo (*)
Division of Gastroenterology and Hepatology, Department of Medicine,
Medical College of Wisconsin, Milwaukee, WI, USA
e-mail: [email protected]

© The Author(s), under exclusive license to Springer Nature 327


Switzerland AG 2023
W. H. Sobin et al. (eds.), Managing Complex Cases in Gastroenterology,
https://doi.org/10.1007/978-3-031-48949-5_71
Chapter 72
Ascites in a Patient Without Obvious
Cirrhosis

Jose Franco

The patient is a 60-year-old male who presents with a complaint of abdominal dis-
tension. There is no history of heavy alcohol abuse; he has 1–2 drinks a day on
weekends only. The patient does have a strong family history of heart disease. He
has very mild dyspnea on exertion. On exam, there is moderate abdominal ascites;
his liver is not palpable. There is mild peripheral edema. Lungs are clear. There is
no S3. However, he has a BNP of 400. His albumin is 3.2, AST is 30, ALT is 45, alk
phos is 140 (Nl < 80), INR is 1.2, Hgb is 12.8, and platelet count is 140,000.
Hepatitis serology, autoimmune markers, and Fe/TIBC are all negative. An ultra-
sound of the liver does not show any liver nodularity, but there is some dilation of
the IVC and hepatic vein, and moderate ascites is present. EF is 30% on echocar-
diogram. The patient is started on diuretics for mild peripheral edema, and a para-
centesis is performed to evaluate the ascites. The SAAG is 1.2 with ascites albumin
of 2.0 and ascites total protein of 3.2. Cytology and AFB are negative. Cell count is
20 PMNs. FibroScan® shows 8 kPa.
What is your differential diagnosis for patients who present with ascites but
do not have obvious cirrhosis?
I always tell students that cirrhosis is the leading cause of ascites, but it’s cer-
tainly not the only cause. When we do a paracentesis, we send an ascites albumin
and serum albumin to determine the SAAG (serum albumin-ascites albumin gradi-
ent) as well as an ascites total protein (TP). These results help us determine the eti-
ology of the ascites.
We break the findings into four categories:
Category 1: SAAG >1.1 g/dL and ascites TP < 2.5 g/dL. Most cases of cirrhosis,
also seen in fulminant liver disease.

J. Franco (*)
Department of Medicine-Division of Gastroenterology and Hepatology, Medical College of
Wisconsin, Milwaukee, WI, USA
e-mail: [email protected]

© The Author(s), under exclusive license to Springer Nature 329


Switzerland AG 2023
W. H. Sobin et al. (eds.), Managing Complex Cases in Gastroenterology,
https://doi.org/10.1007/978-3-031-48949-5_72
330 J. Franco

Category 2: SAAG >1.1 g/dL and TP > 2.5 g/dL. Patients with CHF, constrictive
pericarditis, Budd-Chiari, and veno-occlusive disease.
Category 3: SAAG <1.1 g/dL and TP < 2.5 g/dL. Patients with nephrotic syndrome,
peritoneal carcinomatosis (where the TP is usually high but on occasion low).
Category 4: SAAG <1.1 g/dL and TP > 2.5 g/dL. Patients with peritoneal carcino-
matosis, TB, pancreatic ascites, and chylous ascites.
This patient has a SAAG of 1.2. The high SAAG indicates portal hypertension.
If we look at the differential diagnosis for high SAAG ascites fluid, we have cirrho-
sis, fulminant liver disease, CHF, constrictive pericarditis, Budd-Chiari syndrome,
and veno-occlusive disease (VOD) or sinusoidal obstruction syndrome (SOS).
Occasionally, in advanced CHF, you will see a low SAAG. In a patient like this with
mild CHF, you would expect to see a high SAAG.
This patient has an elevated ascites fluid total protein of 3.2, which is usually not
seen in cirrhosis by itself. It is seen in cases of CHF, constrictive pericarditis and
Budd-Chiari. The reason for the high total protein in these cases is that hepatic syn-
thetic function is usually preserved. In this case, the patient’s serum albumin of 3.2
and INR of 1.2 demonstrates good functional reserve. This is quite different from
the cirrhotic patient who presents with a serum albumin of 2.3 or 2.5. It is more
similar to patients with portal vein thrombosis, who have preserved synthetic func-
tion and a high total ascites protein.
This patient received some diuretics prior to paracentesis. Will this have any
effect on the SAAG or total protein?
Usually, you don’t see any change but occasionally the total protein will increase
due to volume contraction.
In cases where ascites is due to CHF, how often do they have cardiac
cirrhosis?
Most patients with ascites due to heart failure do not have cirrhosis. Most of the
time, they have congestive hepatopathy, with associated portal hypertension. This
congestion is verified by the finding of a dilated IVC and hepatic veins on ultra-
sound. Patients who have severe, long-standing CHF, however, may get end-stage
cardiac cirrhosis after many years of complications.
Is there any role for doing an EGD looking for varices in a patient like this?
Yes, to me, this is no different than evaluating a patient with portal vein throm-
bosis. These patients can get esophageal and gastric varices from non-cirrhotic por-
tal hypertension. If you ask what the most feared complication is in a patient like
this it is bleeding varices. Ascites fluid could get infected, but the highest mortality
would be bleeding varices, and you don’t want to miss these. So, you do need to
do an EGD.
Another thing we look at in deciding whether to do an EGD is the platelet count.
If the platelet count is <150,000, they are more likely to have varices. In this case, it
is 140,000.
Chapter 73
Ascites in Cirrhosis of Unclear Etiology
with Increased Mononuclear Cells

Jose Franco

A 57-year-old female with a normal BMI is found to have marked ascites. There is
no history of significant alcohol use. Ultrasound shows a mildly nodular liver. The
AST is 34, ALT is 47, Alk phos is 90, bilirubin 1.3, and albumin is 2.9. Hepatitis
panel and autoimmune markers – including ANA, ASMA, and AMA – are negative.
A paracentesis is performed, which reveals 300 wbc/hpf, and 80% are mononuclear
cells. The Aa gradient is 1.0.
There is no obvious cause for her apparently cirrhotic liver, what are some
possible etiologies for this?
You’ve ruled out viral hepatitis. PBC is unlikely with the normal alkaline phos-
phatase and negative AMA. The patient could have late-stage PSC – but there is no
history of IBD here. I always check for alpha-1 antitrypsin deficiency and Wilson’s.
You know, alpha-1 antitrypsin deficiency is the leading indication for metabolic
liver transplant, in the pediatric population. But while we always check for alpha-1
and Wilson’s, it would be very unusual for the first presentation to be at age 57.
Another consideration is hemochromatosis. A 57-year-old female is now post-
menopausal. It is possible for hemochromatosis to first manifest itself at this age in
a woman. Remember, men will present much earlier than women. Women tend to
present when they are 50, 60, and 70. When they are younger, they are auto-­
phlebotomizing through menstruation. So, it is a diagnosis worth considering.
The most common cause of cirrhosis, however, in a 57-year-old female would be
nonalcoholic fatty liver disease. Patients don’t have to be obese; they just have to
have insulin resistance and/or elevated triglycerides, those by themselves are risk

J. Franco (*)
Department of Medicine-Division of Gastroenterology and Hepatology, Medical College of
Wisconsin, Milwaukee, WI, USA
e-mail: [email protected]

© The Author(s), under exclusive license to Springer Nature 331


Switzerland AG 2023
W. H. Sobin et al. (eds.), Managing Complex Cases in Gastroenterology,
https://doi.org/10.1007/978-3-031-48949-5_73
332 J. Franco

factors. Obviously, there are no blood tests to diagnose nonalcoholic fatty liver dis-
ease; you diagnose it by ruling out other possible causes, and then, consider doing a
liver biopsy. So that’s how I would approach it.
In this case, the tests are all negative, yet I know she has cirrhosis. If my sero-
logic workup remains negative, I would consider doing a liver biopsy to see if it
helps me establish an etiology.
What would be causing the increase in mononuclear cells?
An increase in mononuclear cells on paracentesis is not something we commonly
see. You have to think about atypical things, like fungal infections or tuberculosis.
But it is also possible that they might represent cancer cells. Could the patient have
peritoneal carcinomatosis? If the fluid looked milky, we’d consider chylous ascites,
which could involve the lymphatics leading to ascites fluid with increased
lymphocytes.
Chapter 74
Low SAAG Ascites

Jose Franco

A 67-year-old male who drinks alcohol on a daily basis is being seen for moderate
ascites. He has anorexia and has lost 20 lbs. over the past month. Ultrasound shows
a nodular liver, with marked ascites and no obvious masses. His AST is 40, ALT is
25, alk phos is 120, and bilirubin is 1.4. A paracentesis is performed and reveals an
Aa gradient of 0.8. The ascites total protein is 2.3. Fluid cell count is 20 wbc.
Cytology is negative. UA is negative for protein. Because the patient’s creatinine is
1.8, you are reluctant to do a contrast-enhanced CT scan. An MR abdomen shows
no obvious cancer.
Normally, if it were just alcoholic cirrhosis, you would expect a high Aa
gradient. What might be causing a low gradient here? What further investiga-
tions would you do?
So, the patient has a nodular liver and normally you would expect to see a high
SAAG, but here it is low. Let’s review our categories again:
Category 1: SAAG >1.1 g/dL and ascites TP < 2.5 g/dL. Most cases of cirrhosis,
also seen in fulminant liver disease.
Category 2: SAAG >1.1 g/dL and TP > 2.5 g/dL. Patients with CHF, constrictive
pericarditis, Budd-Chiari, and veno-occlusive disease.
Category 3: SAAG <1.1 g/dL and TP < 2.5 g/dL. Patients with nephrotic syndrome,
peritoneal carcinomatosis (in which the TP is usually high but on occasion low).
Category 4: SAAG <1.1 g/dL and TP > 2.5 g/dL. Patients with TB, pancreatic asci-
tes, and chylous ascites.

J. Franco (*)
Department of Medicine-Division of Gastroenterology and Hepatology, Medical College of
Wisconsin, Milwaukee, WI, USA
e-mail: [email protected]

© The Author(s), under exclusive license to Springer Nature 333


Switzerland AG 2023
W. H. Sobin et al. (eds.), Managing Complex Cases in Gastroenterology,
https://doi.org/10.1007/978-3-031-48949-5_74
334 J. Franco

In this case, I am concerned that the low SAAG may represent peritoneal carci-
nomatosis. The negative cytology does not rule this out. That’s because cytology of
ascites fluid is a poor test, less than a quarter of cases will actually be positive. It’s
great for diagnosing infection, but if you’re trying to find malignant cells, it is not a
very good test.
I believe that this patient has cancer, and we just haven’t found it. In terms of the
other diagnoses for a low SAAG ascites, I don’t think he has pancreatic ascites,
because you would’ve seen something on an MRI. He doesn’t have chylous ascites,
because you obviously would have seen milky-looking fluid. There’s nothing to
suggest nephrotic syndrome or TB. So, to me, this patient has a primary with peri-
toneal metastases causing ascites, and we just haven’t found the primary. So, he
might need a laparoscopy, to look at what is going on in his peritoneal lining.
Chapter 75
Banding Esophageal Varices

Kia Saeian

A 43-year-old male with a long history of alcoholism presented to your ER a week


earlier. One of the newer gastroenterologists on staff did an EGD and found grade
3 esophageal varices with stigmata of prior hemorrhage but were not actively
bleeding. He placed a total of 10 bands on the varices. The patient was discharged
and now presents again to the ER with torrential UGI bleeding. Endoscopy reveals
several post-banding ulcers, one of which is actively bleeding.
How do you stop bleeding from these ulcers? Can it be done endoscopically
or do you need IR?
In general, we think that banding ulcers resulting in bleeding occur in less than
5% of the cases, and while most of these bleeding episodes are minor, some of them
can be significant, and fatal cases have been reported. Ulceration itself is almost
universal and typically occurs 3–7 days after the banding with healing expected
within the first 2–3 weeks. While the thought is that it is often the perforating veins
as opposed to the main varix that is bleeding in the setting of post-banding ulcer-
ation resulting in lower volume bleeding, they are often very difficult to control if
they bleed significantly.
It may seem flippant to answer in this way, but the short answer is that you pretty
much do whatever it takes to stop the bleeding. There are reports of attempting
almost anything you can think of to try to stop them from bleeding, including repeat
band ligation, cyanoacrylate glue injection, proceeding with TIPS or other portal
decompressive procedures, or even esophageal stents and Hemospray®. In my expe-
rience, I start with band ligation and then in a couple of cases have used glue

K. Saeian (*)
Division of Gastroenterology & Hepatology, Medical College of Wisconsin,
Milwaukee, WI, USA
e-mail: [email protected]

© The Author(s), under exclusive license to Springer Nature 335


Switzerland AG 2023
W. H. Sobin et al. (eds.), Managing Complex Cases in Gastroenterology,
https://doi.org/10.1007/978-3-031-48949-5_75
336 K. Saeian

injection (This is not FDA-approved in the United States and is costly). I have not
had to resort to TIPS or other modalities as a primary measure but have considered
TIPS more actively after stopping the bleeding if the TIPS is otherwise not contra-
indicated for the particular patient. Temporizing with an esophageal stent or using
balloon tamponade seem reasonable as a bridge to TIPS.
What are your tips for initial banding of varices to best initiate control of
varices and also avoid complications?
In the setting of active bleeding initially, I do start as close to the GE junction as
possible and band in a preferably spiral fashion upward until I gain control. As with
standard band ligation of esophageal varices, I limit my banding to the distal 5 cm
of the esophagus as this is the culprit region in the vast majority, and banding more
proximally is less likely to be effective but can precipitate ulceration and bleeding
that’s much more difficult to control. Without much data to support my practice, I
do routinely avoid placing a lot more bands if I have gained control of the bleeding
and I administer sucralfate slurry 1 g four times daily for at least 7 days and in some
patients for 14 days. While many follow the recommendation to bring the patient
back within the first 2–4 weeks for repeat band ligation, if there is ulceration still
present, which there often is if you bring the patient back in 2 weeks, I do not band
in the presence of any suggestion of ulceration. My practice has changed over the
years, and I typically bring the patients back closer to 4 weeks to avoid putting the
patient through an extra unnecessary endoscopy.
Chapter 76
Gastric Varices

Kia Saeian

A 54-year-old male presents with a history of hematemesis, which stops after initia-
tion of octreotide and PPI. He has a long history of alcohol abuse. On exam, there
are multiple stigmata of cirrhosis. The Hbg is 8, his platelet count is 90,000, and
INR is 2.3. EGD reveals small esophageal varices and large gastric varices in the
fundus and body not felt to be an extension of esophageal varices. It is your impres-
sion that bleeding occurred from gastric varices but has stopped for the time being.
What are the scenarios in which you most commonly encounter significant
gastric varices, either isolated or in association with esophageal varices?
When I see a patient with significant gastric varices, I always suspect they may
have thrombosis. We sometimes see isolated gastric varices in patients with chronic
pancreatitis who have developed thrombosis of their splenic vein. In cirrhotics, we
sometimes see gastric varices in the setting of thrombosis of the portal and/or mes-
enteric veins. We see occasional patients with myeloproliferative disorders who
develop thromboses in the portal system and can develop pretty large gastric varices.
I would want cross-sectional imaging in this case.
Cross-sectional imaging reveals portal vein thrombosis in a cirrhotic liver with
significant splenorenal shunting and large gastric varices.
How would you manage this situation?
From a management standpoint, the presence of the thrombus creates an extra
layer of complexity, especially if it is chronic, because this may take TIPS out of the
equation.
If you do think the bleeding is from a gastric varix, particularly if there is an ero-
sion or a red mark present, I think you have to evaluate whether you have the option

K. Saeian (*)
Division of Gastroenterology & Hepatology, Medical College of Wisconsin,
Milwaukee, WI, USA
e-mail: [email protected]

© The Author(s), under exclusive license to Springer Nature 337


Switzerland AG 2023
W. H. Sobin et al. (eds.), Managing Complex Cases in Gastroenterology,
https://doi.org/10.1007/978-3-031-48949-5_76
338 K. Saeian

of performing TIPS or BRTO. BRTO is balloon occluded retrograde transvenous


obliteration of the varices. This procedure is becoming more and more common.
Another option is to perform an EGD with cyanoacrylate glue injection of gastric
varices. Glue injection of varices is not yet FDA-approved in the United States, but
it is widely used in other countries around the world. However, we are getting much
more experience using it, off-label and it should be considered when BRTO is not
an option.
In the acute setting, of course, you do the standard things including infusing a
vasoactive substance, which in this country is usually octreotide and in other coun-
tries tends to be terlipressin. Terlipressin has recently been approved in the United
States for the treatment of hepatorenal syndrome and may soon replace octreotide
as the vasoactive agent of choice. We also start antibiotics, typically ceftriaxone.
After that, you need to make a decision about which of the more robust treat-
ments for gastric varices you want to attempt. At our institution, more and more
frequently it means going to BRTO. Our interventional radiologists have a lot of
experience with BRTO, but there are a couple of caveats you need to know about.
The first is that BRTO can actually worsen ascites, and another is that it can lead to
enlargement and possibly bleeding from esophageal varices if they are present. So,
in those patients who have esophageal varices as well as gastric varices and either
could be the source of bleeding, we prefer to go with TIPS placement. However, in
some patients with severe thrombosis, TIPS is not a feasible option, and therefore
BRTO is performed. You really want to get good cross-sectional imaging to allow
the radiologist to evaluate the anatomy to see if he is a good candidate for
BRTO. They can look at the imaging and see if there is a splenorenal shunt or some
other systemic-portal venous shunt that would allow you to do the BRTO.
How is BRTO performed?
The interventional radiologist usually inserts a catheter via a transfemoral
approach and advances the catheter into the splenic vein (or another vein) injects
contrast and gets opacification. Once they get filling of the gastric varix, they will
advance the balloon occlusion catheter and position it in the descending portion of
the veins that are draining the varix. Then, they inflate the balloon to prevent injected
material from escaping the system. They usually inject ethanolamine oleate (EO),
which acts similarly to our use of cyanoacrylate glue to help occlude the varix.
However, thanks to the balloon occlusion, their glue remains in contact with the
varix for a longer interval, and they can inject a larger volume of EO. As a result,
there is usually a better result from BRTO than endoscopic glue injection of gastric
varices. In theory, it works in a similar manner to our variceal glue injections.
Why might BRTO lead to enlargement and possible bleeding from esopha-
geal varices?
If the occluded gastric varix is in continuity with esophageal varices, you have
now occluded the drainage system of the esophageal varices. The portal hyperten-
sion then gets transmitted back into the esophageal varices, leading to enlargement
and possible bleeding. This is the result of occluding one of the escape channels for
the elevated portal pressures.
76 Gastric Varices 339

A couple of our patients who received BRTO developed such prominent enlarge-
ment of esophageal varices that the interventional radiologist asked us to scope the
patient and consider banding the esophageal varices. One of them had bleeding
from the esophageal varices before we were able to do her endoscopy.
When you have your conference with the radiology department to evaluate
the patient with portal vein thrombosis, how do you decide whether they are
TIPS candidates or not?
If the portal vein thrombosis is not that extensive, sometimes the interventional
radiologist can recanalize the vein and TIPS placement can be performed. Following
TIPS placement, anticoagulation is started to prevent thrombosis and help maintain
patency of the TIPS. However, if the thrombosis is very extensive, the standard
TIPS approach may not be an option. Another option, in some of these cases, is
DIPS (direct intrahepatic portosystemic shunt). While TIPS involves a transvenous
approach to the portal system, DIPS involves a percutaneous approach to the portal
vein and its branches. The patient has to have a large enough vein target for the
percutaneous approach. The number of cases of DIPS performed is much lower than
the number of TIPS at our institution.
Now, if neither TIPS nor DIPs are feasible, BRTO remains a promising option.
The other group of patients in whom BRTO is preferred are those patients in whom
TIPS is relatively contraindicated. This includes those with hepatic encephalopathy,
pulmonary hypertension, or CHF. In these cases, TIPS may be off the table.
It is really important that the hepatologist uses good judgment. Whenever you
decide between TIPS and BRTO, you want to consider whether there are secondary
benefits from one approach over the other. Certainly, in patients with gastric varices
who also have significant esophageal varices or ascites, the secondary benefits of
TIPS are really attractive. In hepatic encephalopathy and CHF, BRTO is preferred.
Why is banding of gastric varices usually avoided, and why may there be
some concern about banding esophageal varices when gastric varices are
present?
Banding of large isolated gastric varices is avoided because patients can develop
post-banding ulcers, and bleeding from those ulcers can be torrential. This doesn’t
apply to banding of GOV-1 varices, the proximal gastric varices that are in contigu-
ity with esophageal varices. Banding for GOV-1 varices is widely accepted. It is the
banding of GOV-2 and other gastric varices that should be avoided because that can
lead to catastrophic consequences.
The concern about banding of esophageal varices, when gastric varices are pres-
ent, is that after esophageal varices are banded, you will occasionally see enlarge-
ment of gastric varices, which might precipitate bleeding.
With the initial endoscopy, no intervention was done. While deciding which pro-
cedure to perform to definitively treat the gastric varices, the patient develops much
more active bleeding. He is intubated and repeat endoscopy initiated; however,
there is too much blood to see anything.
How do you manage the bleeding diathesis in cirrhotics with an elevated
INR, and how do you manage the torrential bleeding at the time of
endoscopy?
340 K. Saeian

It is difficult to evaluate the degree of anticoagulation in many of our cirrhotic


patients. Most of our experience demonstrates that the INR is not a good indicator
of the bleeding propensity of these patients. Unfortunately, the thromboelastogram
(TEG) is not a panacea either. Some of our literature supports checking fibrinogen
levels, targeting a fibrinogen level between 170 and 200. We generally try to replen-
ish fibrinogen using FFP, because it is cheaper than some of the other agents.
Kcentra certainly works as well.
My preference in these cases is to improve the coagulopathy as rapidly as pos-
sible so we do not delay definitive therapy. We realize that it is generally impossible
to totally correct the coagulopathy. So, in this case, where the patient is actively
bleeding, I would try to correct things using FFP because it is more readily acces-
sible in our center, and then we can proceed to the next step in management as
quickly as we can.
With the patient bleeding so heavily, I might try to temporize things by inserting
a Minnesota tube (we use these at our center rather than the Segnstaken-Blakemore
tube). My approach to placing the tube is to do it blindly, and then, after placement,
call for a C-arm and confirm the position radiographically. On occasion, I have
placed a transnasal scope alongside the tube to confirm that the balloon is in the
proper position. But generally, the use of the endoscope is unnecessary. We are usu-
ally comfortable relying on the radiographic confirmation alone. And when dealing
with a bleeding gastric varix, we just need to inflate the gastric balloon; we do not
need to inflate the esophageal balloon, which could risk esophageal pressure necro-
sis. Even in the setting of bleeding from esophageal varices, inflation of just the
gastric balloon may suffice in stopping the bleeding.
Chapter 77
Increased Ascites in a Stable Cirrhotic

Kia Saeian

A 54-year-old male with alcoholic cirrhosis, diagnosed 6 months earlier, has been
abstinent for 6 months following drinking heavily for about 30 years. He presents
because of new-onset ascites. He has gynecomastia, palmar erythema, shifting dull-
ness and a fluid wave, and palpable splenomegaly. His last ultrasound, done
6 months earlier, showed cirrhosis without any liver mass and an EGD was negative
for varices at that time.
When a patient with alcoholic cirrhosis who is abstinent starts decompen-
sating, what are your considerations?
With stable alcoholic cirrhotics who are abstinent, we expect their symptoms to
get better not worse. This patient’s turn for the worse is surprising, and I consider
the possible decompensating events. First, I question whether the patient may have
gone back to drinking, although this does not appear to be the case here. Second, I
worry about portal vein thrombosis or any kind of venous thrombosis as a precipi-
tant. And third, even though the ultrasound was okay previously, you always worry
about malignancy. Those are the three things that are highest in the differential for
me. Portal vein thrombosis is very common in our cirrhotic population so that is
probably the most common explanation we encounter.
In terms of whether he has gone back to drinking, do you use the PEth
(phosphatidylethanol) levels often?
We do, particularly in our transplant patients, but there are a couple of caveats
to using the PEth that the reader has to be aware of. Patients can be abstinent and
still have the PEth return positive, because you get a numerical result, and it can be
dropping and takes a long time to return zero. It’s more useful if you have a

K. Saeian (*)
Division of Gastroenterology & Hepatology, Medical College of Wisconsin,
Milwaukee, WI, USA
e-mail: [email protected]

© The Author(s), under exclusive license to Springer Nature 341


Switzerland AG 2023
W. H. Sobin et al. (eds.), Managing Complex Cases in Gastroenterology,
https://doi.org/10.1007/978-3-031-48949-5_77
342 K. Saeian

baseline value taken when the patient was drinking. While the test is helpful, it
does detect alcohol use for 30 days and perhaps up to 90 days. We have had cases
where patients have claimed abstinence and a positive PEth result has led the
patient to feel like they have been falsely accused of drinking. Patients need to
know that the threshold of the test is so low that a positive result may simply be an
indicator of prior alcohol use.
Another test we use is the serum ethanol glucuronide (EtG), which can detect
alcohol use for 72–96 h. The advantage of this test (at our hospital) is that the turn-
around on this test is much more rapid than the PEth. In the inpatient setting, the
Peth test may take too long to return, and the patient gets discharged before you get
the results back. So, if you are basing decision-making on these tests, the ethanol
glucuronide may be preferable if you need quick results.
An US is performed and there is a 3-cm lesion, suggestive of hepatoma that was
absent on the last US. Triple-phase CT scan confirms the likely diagnosis of HCC in
the background of cirrhosis.
Are there any clues that make you suspect a patient may have developed
hepatoma? Do you develop a sixth sense for who may develop a hepatoma?
I wish I had a sixth sense that would tell me who the patient is who will develop
a hepatoma. But there are some patients who you suspect are at higher risk. We do
not see as many hepatitis C patients with hepatomas as we used to, now that we are
eradicating hepatitis C, but those patients with cirrhosis from alcohol and hepatitis
C are at increased risk. Also, those patients with cirrhosis from hemochromatosis
and alcohol are at greater risk, as are hepatitis B patients. For patients with these risk
factors, we have a much lower threshold for being worried about HCC.
Another worrisome scenario is when a stable patient starts doing more poorly
unexpectedly, for instance, a patient who develops anorexia, or starts losing a lot of
muscle mass, or starts developing ascites, in spite of good control of their underly-
ing liver disease. In cases like those I start worrying about HCC.
In terms of detecting bruits on auscultation and those sorts of things that have
been advocated in the past for detecting hepatoma, honestly, now with the excellent
imaging at our fingertips, we usually don’t come across those secondary manifesta-
tions before we get back positive results from our radiology exams.
I would say in this particular patient, though, that once he developed ascites, I
would have gone directly to CT imaging, or if I was going to do an ultrasound make
it a Doppler ultrasound. Because I am worried that there could be a portal vein
thrombus with or without HCC, I would get one of those examinations. Now, usu-
ally a 3-cm HCC is not going to cause this degree of ascites, so I would be worried
that there could be a tumor thrombus.
Do you follow the AFP in surveillance of your cirrhotic patients?
I do, although I know the national guidelines from the AASLD have gone back
and forth on this. The AASLD had removed alpha-fetoprotein a number of years
ago from their guidelines but societies are now recommending it again. But in my
surveillance of cirrhotics for HCC, I have found it to be a helpful tool. There have
been cases where an elevated alpha-­fetoprotein would jump-start me to order a CAT
77 Increased Ascites in a Stable Cirrhotic 343

scan in a patient whose ultrasound was negative for a mass, and we’ve found some
early HCCs this way. Occasionally, it’s also useful to have a baseline alpha-fetopro-
tein because it helps you in terms of therapeutic decisions down the line, especially
when the patients get liver-directed therapy.
What if this patient was not abstinent and could not quit drinking and asks
you for help, are there meds you would prescribe to prevent alcohol relapse?
Yes, first we have been fortunate in that we have a transplant psychology team
that we can use even in patients who we are not being evaluated for transplant. In
that subset of patients, having our psychology team involved early is helpful in
decreasing recidivism.
Personally, I have started a number of patients on baclofen. The original article
in the Lancet [1] that used baclofen to decrease recidivism showed that it was prom-
ising; however, the study has not always been replicated. But if I am going to use
something, I rely on baclofen. Some other people may use other agents like
Suboxone. I am most comfortable with baclofen, and I have found that it helps
patients with their cravings. It also gives the patient the sense that he is taking action
to do something about his problem and makes him feel empowered that he has a
shot at not going back to drinking. I will start at 5 mg bid and then go up to 10 mg
and may go up to 15 mg, increasing the frequency to tid. We treat the patient for
6 weeks. More recently, there is now more and more experience with the use of both
naltrexone and acomprosate. While many use naltrexone as first line, it should be
avoided in acute alcoholic hepatitis or acute on chronic liver failure thus I tend to
use acomprosate (666 mg three times daily with normal renal function) for my
patients intolerant of baclofen. Acomprosate should be avoided in those with severe
renal failure.
Another intervention to help protect patients from relapsing is to try to engage
family and others to help the patient maintain his sobriety. It is particularly effective
when patients have kids that are involved with their care, we find they are much
more apt not to go back to alcohol.

Reference

1. Addolorato G, Leggio L, Ferrulli A, Cardone S, Vonghia L, Mirijello A, Abenavoli L, D'Angelo


C, Caputo F, Zambon A, Haber PS. Effectiveness and safety of baclofen for maintenance of
alcohol abstinence in alcohol-dependent patients with liver cirrhosis: randomized, double-­
blind controlled study. Lancet. 2007;370(9603):1915–22.
Chapter 78
Complications Related to Managing
Ascites

James Esteban

A 46-year-old male with a history of heavy alcohol use presents for evaluation and
management of ascites. On presentation, his creatinine is 1.1 mg/dL. A therapeutic
and diagnostic paracentesis reveals straw-colored fluid that is a transudate, with
low WBC and only 80 neutrophils per mm3. The serum albumin-ascitic fluid gradi-
ent is 1.5. You start furosemide 40 mg and spironolactone 100 mg once daily. One
week later, the creatinine has increased to 1.4 mg/dL. His ascites is improved,
though still present.
Would you change the dose of his diuretic? Do you monitor urine sodium in
these patients?
I might decrease the dose while making sure that the patient follows a sodium-­
restricted diet (<2000 mg of sodium per day). The alternative is to maintain the
patient on the current dose while closely monitoring the creatinine and electrolytes.
I don’t routinely check urine sodium, although this can be beneficial in determining
whether persistent ascites is due to inadequate diuresis or nonadherence to sodium
restriction [1].
Staying on the same diuretic dose, his creatinine increases to 1.7 mg/dL the fol-
lowing week.
Assuming the patient is adhering to sodium restriction, it looks like he is intoler-
ant of diuretics. I would have to decrease his diuretic dosing, and in some patients,
we may have to discontinue diuretics entirely. He would have to continue serial
large volume paracentesis to manage his symptoms.
I would consider referring him to interventional radiology for transjugular intra-
hepatic portosystemic shunt, or TIPS, if he does not have contraindications. For

J. Esteban (*)
Division of Gastroenterology and Hepatology, Department of Medicine, Medical College of
Wisconsin, Milwaukee, WI, USA
e-mail: [email protected]

© The Author(s), under exclusive license to Springer Nature 345


Switzerland AG 2023
W. H. Sobin et al. (eds.), Managing Complex Cases in Gastroenterology,
https://doi.org/10.1007/978-3-031-48949-5_78
346 J. Esteban

this, I would order an echocardiogram, to ensure cardiac function and pulmonary


arterial pressure would tolerate TIPS, and contrast-enhanced CT or MRI of the
abdomen (based on his kidney function), to check the vascular anatomy of the liver.
We have to calculate his model for end-stage liver disease or MELD score. If his
MELD score is high enough, typically above 15, we should begin a liver transplant
evaluation [2]. Many hepatologists would hesitate to insert a TIPS in a patient with
a MELD score of 18–20 and above, although there is not necessarily a specific
MELD cutoff for TIPS placement [3].
TIPS is performed, and ascites improves; however, the patient starts to develop
increased lethargy and forgetfulness. You start lactulose and rifaximin. The patient
improves, but a few weeks later he becomes markedly confused and is hospitalized
frequently. You confirm that he is having 3–4 bowel movements daily with lactulose
and takes rifaximin as prescribed.
How would you manage this?
Hepatic encephalopathy may occur in 20–30% of patients after TIPS insertion.
In many cases, medical management with lactulose and rifaximin is sufficient to
treat encephalopathy. However, in this patient, encephalopathy is significant and
becomes difficult to treat medically. I would refer him back to interventional radiol-
ogy to consider restricting, or narrowing, the TIPS stent. Unfortunately, restricting
the stent may cause the return of ascites.

References

1. Biggins SW, Angeli P, Garcia-Tsao G, Ginès P, Ling SC, Nadim MK, Wong F, Kim
WR. Diagnosis, evaluation, and management of ascites, spontaneous bacterial peritonitis and
hepatorenal syndrome: 2021 practice guidance by the American Association for the Study of
Liver Diseases. Hepatology. 2021;74(2):1014–48.
2. Brown RS, Lake JR. The survival impact of liver transplantation in the MELD era, and the
future for organ allocation and distribution. Am J Transplant. 2005;5(2):203–4.
3. Boike JR, Thornburg BG, Asrani SK, Fallon MB, Fortune BE, Izzy MJ, Verna EC, Abraldes
JG, Allegretti AS, Bajaj JS, Biggins SW. North American practice-based recommendations
for transjugular intrahepatic portosystemic shunts in portal hypertension. Clin Gastroenterol
Hepatol. 2022;20(8):1636–62.
Chapter 79
Portal Vein Thrombosis in Cirrhosis

James Esteban

A 56-year-old male was diagnosed with alcohol-associated cirrhosis 2 years ago


and stopped drinking at that time. He has been undergoing abdominal ultrasound
every 6 months for hepatocellular cancer screening. On his last ultrasound, a portal
vein thrombus was seen. The last ultrasound 6 months ago did not show PVT.
Would you anticoagulate this patient?
Portal vein thrombosis (PVT) is a known complication of cirrhosis. It appears
paradoxical that patients with cirrhosis, who often have prolonged prothrombin
time and thrombocytopenia, would develop PVT. However, cirrhosis is a state of
rebalanced hemostasis, where patients experience simultaneous changes in both
pro- and anticoagulant factors [1]. Decisions on anticoagulation for patients with
PVT, who may have varices, bleeding portal hypertensive gastropathy, or GAVE,
can be challenging.
The first thing I would do is obtain contrast-enhanced CT or MRI, to confirm the
PVT; determine the presence of collaterals (or cavernoma), which indicate chronic-
ity, determine the extent of the clot, in terms of being partially or totally occlusive
and extension into superior mesenteric vein (SMV) or splenic veins; and make sure
it is not a malignant thrombus. My decision about anticoagulation will be based on
chronicity of the clot, extent of the clot and the presence of symptoms of intestinal
ischemia, and the patient’s potential liver transplant candidacy [2, 3].
If the clot is chronic, with well-established collaterals and cavernoma, there is no
established benefit with anticoagulation. We would be continuing medical manage-
ment and surveillance of portal hypertension.

J. Esteban (*)
Division of Gastroenterology and Hepatology, Department of Medicine, Medical College of
Wisconsin, Milwaukee, WI, USA
e-mail: [email protected]

© The Author(s), under exclusive license to Springer Nature 347


Switzerland AG 2023
W. H. Sobin et al. (eds.), Managing Complex Cases in Gastroenterology,
https://doi.org/10.1007/978-3-031-48949-5_79
348 J. Esteban

If the clot is recent (<6 months) but involves only intrahepatic PV branches or
only partially occludes the main PV, I would offer either anticoagulation or surveil-
lance imaging with expectant management. It is reasonable to repeat abdominal
imaging after 2–3 months, and anticoagulate only if the clot progresses.
If the clot is recent and extensive, which is to say that it totally occludes the main
PV or extends into the SMV, particularly if the patient is a liver transplant candidate
or has symptoms of intestinal ischemia, I would anticoagulate as soon as possible. I
would go as far as discussing with interventional radiology and transplant surgery
whether we should consider portal vein recanalization and transjugular intrahepatic
portosystemic shunt (TIPS), especially if the patient has another indication for TIPS
(e.g., ascites or variceal bleeding) or if clot extension threatens portal anastomosis
during transplant.
In this case, the patient is felt to have a recent clot, which is totally occluding the
main portal vein. He does not have abdominal pain or bloody diarrhea. The patient’s
last EGD was 2 years earlier, and at that time, there were no varices present. The
decision is made to repeat EGD, and this reveals two grade 2 varices and one grade
3 varix, with a red mark.
How do you want to manage this?
Since the patient has high-risk esophageal varices and needs anticoagulation, I
would pursue band ligation.
If you band, how long will you wait before starting anticoagulation?
It is debatable if anticoagulation should be delayed until varices are eradicated. I
feel that cirrhotic patients with totally occlusive clots of the main PV and/or SMV
should be anticoagulated without delay, while patients with less extensive clots may
hold off anticoagulation until varices are banded or eradicated. Early anticoagula-
tion of PVT is important for clot recanalization and prevention of clot progression
[4, 5]. The AASLD does recommend initiating anticoagulation as soon as possible
and not delaying until varices are eradicated [3]. Available data do not suggest
higher risk of bleeding with this practice. Moreover, the data suggest that anticoagu-
lation can continue uninterrupted through band ligation without risk of bleeding [6].
I treat patients with small and low-risk varices with nonselective beta-blockers
such as carvedilol (my preference), propranolol, or nadolol instead of band ligation,
with the goal of preventing the growth and rupture of varices. I prefer carvedilol
over propranolol because studies show that it more effectively lowers portal pres-
sure [7] as a result of its additional effect on alpha-1 adrenergic receptors, which
reduce intrahepatic vascular resistance in addition to reducing portal blood flow.
I also use carvedilol in patients even after large varices are eradicated by band
ligation because studies show that carvedilol not only reduces the risk of variceal
bleeding but also hepatic decompensation events such as ascites [8].
What anticoagulant agents do you use and how long do you anticoagulate
patients?
I have used warfarin and direct oral anticoagulants as first-line agents for antico-
agulation in PVT. Warfarin probably has the most data on efficacy and safety, and it
is also cheaper and more easily reversible if the patient develops a bleeding compli-
cation, but it requires normal baseline INR – which many cirrhotics don’t
79 Portal Vein Thrombosis in Cirrhosis 349

have – and also frequent INR monitoring. Direct oral anticoagulants (DOAC) do not
require INR monitoring and are thus easier to use. Many recent studies show
DOAC’s success in PVT, but it is unclear if they are safe to use in patients with more
advanced stages of liver disease (i.e., Child’s C). I would request hematology con-
sultation in more complicated cases, such as patients with progressive clot despite
anticoagulation or underlying thrombophilia.
I anticoagulate patients for at least 6 months. I obtain cross-sectional abdominal
imaging after 2–3 months to check if the clot resolves, improves, or progresses. If
the clot completely resolves, I will discontinue anticoagulation, but continue routine
surveillance with doppler ultrasound or cross-sectional imaging every 6 months. If
the clot progresses, assuming the patient is being adherent, my options are changing
anticoagulants with the guidance of hematology or interventional radiology consul-
tation for intravascular procedures or TIPS.

References

1. Lisman T, Caldwell SH, Intagliata NM. Haemostatic alterations and management of haemosta-
sis in patients with cirrhosis. J Hepatol. 2022;76(6):1291–305.
2. De Franchis R, Bosch J, Garcia-Tsao G, Reiberger T, Ripoll C, Abraldes JG, Albillos A, Baiges
A, Bajaj J, Bañares R, Barrufet M. Baveno VII–renewing consensus in portal hypertension. J
Hepatol. 2022;76(4):959–74.
3. Northup PG, Garcia-Pagan JC, Garcia-Tsao G, Intagliata NM, Superina RA, Roberts LN,
Lisman T, Valla DC. Vascular liver disorders, portal vein thrombosis, and procedural bleed-
ing in patients with liver disease: 2020 practice guidance by the American Association for the
Study of Liver Diseases. Hepatology. 2021;73(1):366–413.
4. Loffredo L, Pastori D, Farcomeni A, Violi F. Effects of anticoagulants in patients with cir-
rhosis and portal vein thrombosis: a systematic review and meta-analysis. Gastroenterology.
2017;153(2):480–7.
5. Delgado MG, Seijo S, Yepes I, Achécar L, Catalina MV, García-Criado Á, Abraldes JG, de la
Peña J, Bañares R, Albillos A, Bosch J. Efficacy and safety of anticoagulation on patients with
cirrhosis and portal vein thrombosis. Clin Gastroenterol Hepatol. 2012;10(7):776–83.
6. Guillaume M, Christol C, Plessier A, Corbic M, Péron JM, Sommet A, Rautou PE, Consigny
Y, Vinel JP, Valla CD, Bureau C. Bleeding risk of variceal band ligation in extrahepatic por-
tal vein obstruction is not increased by oral anticoagulation. Eur J Gastroenterol Hepatol.
2018;30(5):563–8.
7. Jachs M, Hartl L, Simbrunner B, Bauer D, Paternostro R, Balcar L, Hofer B, Pfisterer N,
Schwarz M, Scheiner B, Stättermayer AF. Carvedilol achieves higher hemodynamic response
and lower rebleeding rates than propranolol in secondary prophylaxis. Clin Gastroenterol
Hepatol. 2022;21(9):2318–2326.e7.
8. Villanueva C, Albillos A, Genescà J, Garcia-Pagan JC, Calleja JL, Aracil C, Bañares R,
Morillas RM, Poca M, Peñas B, Augustin S. β blockers to prevent decompensation of cirrhosis
in patients with clinically significant portal hypertension (PREDESCI): a randomized, double-­
blind, placebo-controlled, multicentre trial. Lancet. 2019;393(10181):1597–608.
Chapter 80
PVT and Budd Chiari as a Complication
of UC

Kia Saeian

The patient is a 28-year-old female with a history of ulcerative colitis that was well
controlled on mesalamine. She presents with increased diarrhea and rectal bleeding
but also has marked abdominal distension. An US of the abdomen confirms the pres-
ence of a large amount of ascites. There is no history of liver disease, alcohol use,
etc. Her labs show AST of 120, ALT of 140, AP of 100, and bilirubin of 1.4. Hepatitis
panel and autoimmune panel are negative. A CT is obtained, which shows thrombus
in the hepatic vein, IVC, and portal vein. There is a suggestion of gastroesophageal
varices. There is hepatomegaly but no suggestion of cirrhosis. She has a combined
EGD/colon, and while on EGD there are grade 2 esophageal varices, on colon
there is evidence for moderately severe ulcerative colitis. The patient is placed on
IV steroids and beta-blockers. Anticoagulation is started (IV heparin), and the next
day the patient starts having more active rectal bleeding. Her abdominal distension
is getting worse. Heparin is put on hold.
How would you manage the presumed Budd-Chiari and portal vein
thrombosis?
First, I’m not sure I would have started her on beta-blockers in the first place,
particularly in the setting of a large amount of ascites, where the use of beta-­blockers
is controversial. And with grade 2 varices, I don’t think you have to jump in and
immediately do anything about her varices.
But, of course, we do have to address the thromboses. Here, it’s a little unusual,
it’s not only Budd-Chiari, and it’s also portal vein thrombosis and extension into the
IVC, which makes it a pretty extensive thrombus. She’s very young, so I’m not as
concerned about malignancy. But while the IBD could be a trigger for her

K. Saeian (*)
Division of Gastroenterology & Hepatology, Medical College of Wisconsin,
Milwaukee, WI, USA
e-mail: [email protected]

© The Author(s), under exclusive license to Springer Nature 351


Switzerland AG 2023
W. H. Sobin et al. (eds.), Managing Complex Cases in Gastroenterology,
https://doi.org/10.1007/978-3-031-48949-5_80
352 K. Saeian

hypercoagulable state, it seems a little out of proportion. Since it’s very extensive, I
would look for other causes of hypercoagulable state, even though IBD might
remain the sole explanation. I’d want to make sure that there is nothing else going
on with her bone marrow. These days, it is almost mandatory, when you have some-
one who presents with Budd-Chiari, to rule out polycythemia vera, or some of the
other myeloproliferative disorders that can certainly trigger this. That would be on
my radar as well.
Looking at the bigger picture, we have to decide how to manage this very com-
plex patient who is bleeding but needs treatment for thrombosis. In this setting, we
have really discovered that TIPS is the way to go. I would get interventional radiol-
ogy and transplant surgery involved. While TIPS is generally feasible in Budd-­
Chiari, the portal vein thrombus in this case presents an increased level of complexity
for performing a successful TIPS. And, if you are successful in placing a TIPS, you
want to anticoagulate afterward to prevent shunt thrombosis. So, in this case you
should notify colorectal surgery as well, because even though I’m not recommend-
ing a colectomy at the time, that’s where you may end up heading if rectal bleeding
becomes intractable.
Even though she is young, I would get an echocardiogram, to see whether she
has any pulmonary hypertension and whether she is a suitable candidate for TIPS,
and get IR involved to see if it’s feasible to place a TIPS and maintain its patency.
In most cases of Budd-Chiari, the thrombus is fairly new and the radiologist can
get a wire through it. In this case, however, the clot is more extensive, and it goes
into the IVC, and so the question is whether there are other channels that IR can get
into. In some of these cases, it might require DIPS to be successful.
If you only have hepatic vein thrombosis, is stenting the vein ever an option?
You could consider it, but in almost all cases, the thrombus in Budd-Chiari
involves multiple small venous tributaries, almost like a spider web configuration.
If you have the classic Asian type of Budd-Chiari, where there is an IVC web, then
a stent is a very good option for those patients. But in the patients we tend to see
who have involvement of all the little venous radicals, you really do need TIPS
decompression. There’s good evidence that if you intervene early with TIPS, you
can help prevent some of these patients from needing a liver transplant. But if you
don’t act quickly and aggressively, cirrhosis can develop pretty quickly.
Chapter 81
HCC Management, Solitary Lesion
in a Child’s A Cirrhosis

Francisco Durazo

A 63-year-old man with chronic HCV is sent to you with Child’s A cirrhosis and a
3-cm mass in the liver with characteristics of an HCC. The patient’s overall health
is otherwise quite good.
Can you ever consider surgical resection of a hepatoma in a patient with
cirrhosis? Is there anything that would make you want to put this patient on a
transplant track?
Our current guidelines state that if a patient with Child’s A cirrhosis has a single
resectable lesion, surgery is the treatment of choice. There is a 50% chance of recur-
rence after surgical resection, but that means 50% of patients may be cured.
If, instead, you recommend liver transplant, the patient needs to undergo trans-
plant evaluation, go on a waiting list, and will require locoregional therapy as a
temporizing bridge to transplant. If he gets transplanted, the patient will be con-
demned to lifelong immunosuppression.
This patient is a male, and it should be noted that HCC is much more common in
males; the management is the same in males and females.

F. Durazo (*)
Division of Gastroenterology and Hepatology, Department of Medicine, Medical College of
Wisconsin, Milwaukee, WI, USA
e-mail: [email protected]

© The Author(s), under exclusive license to Springer Nature 353


Switzerland AG 2023
W. H. Sobin et al. (eds.), Managing Complex Cases in Gastroenterology,
https://doi.org/10.1007/978-3-031-48949-5_81
Chapter 82
Management of Unresectable HCC
in a Cirrhotic

Francisco Durazo

A 59-year-old male with alcoholic cirrhosis has been abstinent for 5 years and
presents with a 6 cm HCC and portal vein thrombosis. The AFP is 400. You had last
seen him 3 years ago, at which time ultrasound showed no lesion in the liver. He was
subsequently lost to follow-up, until now.
What do you recommend for HCC surveillance?
For patients with cirrhosis, we get an ultrasound and AFP every 6 months. If
there is an abnormality on ultrasound, we order a CT or MRI.
Is AFP still recommended for surveillance?
Actually, the American Association for the Study of Liver Diseases (AASLD)
“fired” the AFP a while back, but the 2018 recommendations had a soft reintroduc-
tion, with the recommendation of surveillance using ultrasound, with or without
AFP, every 6 months. However, we still find the AFP to be helpful. There are occa-
sional patients who have a normal ultrasound but an AFP of 1000, so we order an
MRI. On MRI, we see the hepatoma that was missed on US. I think that most hepa-
tologists still follow the AFP.
How would you manage this patient if there was no portal vein thrombosis?
We think about possible liver transplant to cure the HCC and cirrhosis, but
according to Milan criteria the lesion is too large. Therefore, we would try to down-
size the lesion. For lesions under 4 cm in size, ablation with radiofrequency or
microwave is effective. For lesions larger than 4 cm, transarterial catheterization
and chemoembolization (TACE) or radioembolization, with Yttrium 90 (TARE), are
recommended for downsizing, with TARE causing fewer side effects than TACE,
although the two modalities have not been compared head-to-head for efficacy.

F. Durazo (*)
Division of Gastroenterology and Hepatology, Department of Medicine,
Medical College of Wisconsin, Milwaukee, WI, USA
e-mail: [email protected]

© The Author(s), under exclusive license to Springer Nature 355


Switzerland AG 2023
W. H. Sobin et al. (eds.), Managing Complex Cases in Gastroenterology,
https://doi.org/10.1007/978-3-031-48949-5_82
356 F. Durazo

Do you ever see cures with locoregional therapies?


Yes, the lesions may disappear and be absent in the liver explant. In spite of this,
transplant is still recommended, because new tumors will form in the cirrhotic liver;
you treat one tumor and a new one pops up.
This patient has portal vein thrombosis. In patients with HCC, are the por-
tal vein thrombi assumed to be malignant?
No, benign portal vein thrombosis is common in cirrhotics. Malignant portal
vein thrombi are generally easy to differentiate from benign PVT. They are usually
contiguous with the HCC, and they take up contrast on CT and MRI, so usually they
are easily discernible. The distinction between malignant and benign portal vein
thrombosis is important because malignant PVT closes the door on transplant.
How do you manage benign PVT in a transplant candidate?
Some centers will anticoagulate these patients, usually with coumadin. Other
centers will take the patient to transplant with a PVT and do thrombectomy at the
time of transplant, as long as the PVT does not extend to the SMV (in which case
anticoagulation is necessary).
This patient gets therapy with TACE and the lesion shrinks to 3 cm. AFP is 500.
Can you list the patient for transplant now?
Not yet. Once the lesion is downsized and the AFP is under 500, you then have
to wait 6 months before giving exception points to the patient. It was found that
delaying transplant leads to better outcomes by weeding out those tumors that have
the most unfavorable biology. In addition, it was found that hepatomas with an
AFP >1000 had a very unfavorable prognosis, leading to the requirement that
patients had to have an AFP under 500 to be listed.
Chapter 83
HCC in NAFLD Without Cirrhosis

Francisco Durazo

A 57-year-old, obese female is seen in the community and referred for elevated
LFTs and a mass in the liver. Her labs include ALT of 220, AST of 150, Alk phos of
170, and platelets of 180,000. US reveals diffuse steatosis and a 4-cm mass in the
liver. All viral and autoimmune serology is negative. CT scan shows a lesion that
has characteristic findings of hepatocellular carcinoma and no evidence of cirrho-
sis. There is no liver nodularity or enlarged spleen nor suggestion of varices.
Do some patients with NAFLD develop HCC without first having cirrhosis?
Over the past 10 years, there have been cases of HCC in patients with NASH
without cirrhosis, although it remains uncommon. This is called steatohepatitic
hepatocellular carcinoma, a kind of noncirrhotic HCC in patients with underlying
fatty liver. Many of these patients have a single-nucleotide polymorphism. We’ve
seen a couple of cases like this at our institution.
How should we be screening for this possibility?
In the absence of warning signals, we continue to screen patients with NAFLD
and cirrhosis for HCC every 6 months, but now I am also screening patients with
fatty liver without cirrhosis every 12 months with ultrasound and AFP. A number of
our colleagues are doing the same thing, to help us all sleep better at night.

F. Durazo (*)
Division of Gastroenterology and Hepatology, Department of Medicine, Medical College of
Wisconsin, Milwaukee, WI, USA
e-mail: [email protected]

© The Author(s), under exclusive license to Springer Nature 357


Switzerland AG 2023
W. H. Sobin et al. (eds.), Managing Complex Cases in Gastroenterology,
https://doi.org/10.1007/978-3-031-48949-5_83
Chapter 84
Managing Primary Biliary Cholangitis

Juan Trivella

The patient is a 45-year-old female who’s referred for abnormal liver enzymes. She
has an alkaline phosphatase of 240 (nl < 80), and her AST is 25, ALT is 30, and bili-
rubin is 1.0. Her GGTP is three times normal. Her anti-mitochondrial antibody is
positive at 1:80, ANA is negative, anti-smooth-muscle antibody is negative, albumin
is 3.6, platelets are 210,000, and ultrasound of the liver is normal. The patient
doesn’t drink alcohol and is on no medications.
Are you comfortable making the diagnosis of primary biliary cholangitis?
Do you need a liver biopsy?
Yes, I am comfortable making the diagnosis. This female patient has cholestasis
and a clearly positive AMA, with no risk factors for other chronic liver disorders.
No liver biopsy is needed in this case. If you are considering a superimposed diag-
nosis or are unable to stage the degree of fibrosis via noninvasive methods, then I
would perform a liver biopsy [1].
It appears that the majority of patients with PBC are asymptomatic at the
time of diagnosis. Is that your experience?
Yes, this has been my experience, and although most patients are asymptomatic
at diagnosis, almost everyone with PBC will experience symptoms within two
decades [2]. The presence or absence of symptoms at diagnosis or on follow-up
does not correlate well with the stage of the disease and has shown no difference in
survival outcomes.

J. Trivella (*)
Division of Gastroenterology and Hepatology, Department of Medicine, Medical College of
Wisconsin, Milwaukee, WI, USA
e-mail: [email protected]

© The Author(s), under exclusive license to Springer Nature 359


Switzerland AG 2023
W. H. Sobin et al. (eds.), Managing Complex Cases in Gastroenterology,
https://doi.org/10.1007/978-3-031-48949-5_84
360 J. Trivella

Primary care providers are more aware of the disease. In turn, patients with
chronic cholestasis are referred earlier to the hepatology clinic, and treatment is
started before decompensated cirrhosis has ensued. This is why clinical signs and
symptoms of portal hypertension are a rather uncommon presentation in patients
with PBC nowadays.
Now, we are most aware of PBC occurring in females. Are you seeing it in
males, are they being diagnosed later? Is the course more severe in males?
PBC affects people of all sexes, races, and ethnicities. Earlier reports showed a
median female-to-male ratio of 10:1, but these were case-finding studies and, there-
fore, subjected to numerous biases. Modern literature from epidemiologic studies—
which, although not perfect, provide a more general vision of the actual number of
cases—have shown that the female to male prevalence ratio is closer to 4–6:1 [3, 4].
PBC in males is often diagnosed at an older age once the disease is more
advanced. It has also been associated with a lower biochemical response to ursode-
oxycholic acid (UDCA), greater progression to cirrhosis, higher rates of liver-­
related death or transplantation, and higher risk for primary liver cancer [5].
It’s important that we continue to create awareness among providers that PBC
can affect people from all backgrounds. Being vigilant will lead to an earlier diag-
nosis, particularly in males and racial minorities, allowing for prompt treatment
initiation, which will in turn slow down the rate of progression to cirrhosis.
If patients are diagnosed early with PBC and started on ursodeoxycholic
acid immediately, do you find that the vast majority will do well for years,
decades even?
Yes, the introduction of ursodeoxycholic acid in the mid-1990s has helped
reshape the transplant-free survival in patients with PBC. Those treated with the
medication have a survival rate of 90% at 5 years and 66% at 15 years. In the pre-­
ursodeoxycholic acid era, these percentages were significantly lower with a 79%
transplant-free survival at 5 years and 32% at 15 years [6].
Why does ursodeoxycholic acid work so remarkably well for PBC?
Ursodeoxycholic acid is a hydrophilic bile acid that is normally present in human
bile at a low concentration (around 3% of the total bile acid pool). When adminis-
tered orally, only 30–60% is absorbed by the bowel. In the hepatocytes and biliary
ducts, ursodeoxycholic acid has anti-inflammatory properties, solubilizes the bile,
and stimulates bile flow [7].
When you see patients who are initially doing well but then take a turn for
the worse, what sort of symptoms do they usually present with?
Symptoms do not correlate well with the degree of cholestasis or fibrosis,
although, observationally, patients with more severe disease tend to have more
symptoms.
Symptoms can appear at any point in the disease course, with fatigue being the
most common one. It is experienced by about 80% of the patients, and it is very
disabling. Fatigue is also very challenging to manage, with no effective medications
currently approved for its treatment. The second most frequent symptom is pruritus.
It is more intense in the limbs, particularly in the palms and soles, and has a
84 Managing Primary Biliary Cholangitis 361

circadian rhythm. It is more intense in the evening and overnight. Pruritus is associ-
ated with sleep deprivation, worsening fatigue, depression, social isolation, and
self-­mutilation, thus leading to significant impairment in quality of life [1].
Sicca complex symptoms, including dry mouth and dry eyes, are usually under
reported by patients but are highly prevalent in PBC. Providers should proactively
ask about them when following patients with PBC and treat them accordingly [1].
Right upper quadrant abdominal pain is present in about 25–30% of patients.
This is a very nonspecific symptom, and other etiologies like gallbladder or pancre-
atic disease and abdominal/chest wall pathology would have to be excluded before
attributing the pain solely to PBC [1].
How do you like to manage pruritus in PBC?
My initial approach consists of utilizing commercially available skin moisturiz-
ers, emollients, and topical agents with camphor or menthol. The only currently
approved treatment for pruritus in PBC is cholestyramine. This is a bile acid binding
resin. It is usually started at a dose of 4 g by mouth per day and can be increased to
a maximum of 16 g per day. It is important to remember that cholestyramine can
inadvertently bind other medications—particularly ursodeoxycholic acid—when
taken at the same time, preventing their absorption. Therefore, it is recommended
that cholestyramine should be taken 3–4 h before or after other prescribed drugs. A
recent trial utilizing bezafibrate (400 mg by mouth per day) for the management of
cholestatic itch showed a 50% reduction in the severity of pruritus in PBC patients
treated with this fibrate. This medication can be considered an alternative to chole-
styramine, where it is available. Rifampin (150–300 mg PO BID) enhances the rate
of bile acid metabolism and increases the excretion of pruritogens. This medication
works well in PBC, but its use is off-label, and although very rare, it has been asso-
ciated with severe hepatitis.
Now, if you have a patient who doesn’t tolerate cholestyramine, does colesti-
pol work for pruritus?
Colestipol can be used for PBC since this is also a bile acid binding resin, but it
has not been FDA-approved for this particular indication.
She is started on UDCA at a dose of 13–15 mg/kg per day.
When you start ursodeoxycholic acid, when do you want to recheck liver
enzymes?
Liver chemistries should be rechecked every 3–6 months in patients treated with
ursodeoxycholic acid. Alkaline phosphatase has a log linear association with the
risk of liver transplant and death and can be used to monitor response to therapy.
There are multiple other scores like the UK-PBC and the GLOBE score that are
excellent at estimating outcomes and can be found online. In clinical practice, the
goal should be to normalize or near normalize the ALP after 1 year of treatment
with UDCA. This is only possible in about two thirds of the patients. Those who do
not respond may require the combination of UDCA with other drugs, like obeticho-
lic acid or fibrates, where it is available [8].
In this case, recheck of the alkaline phosphatase at 6 months returns 160
(nl < 80).
362 J. Trivella

Would you change your management?


Although the ALP is still elevated, it is much improved in comparison to her
baseline numbers. Since the patient has only been on UDCA for 6 months, I would
keep her on this medication. I would make sure that the dose is appropriate to her
weight and that she is being compliant with the medication. It is also important to
assess for side effects like stomach upset, hair thinning, and weight gain each fol-
low-up visit.
Will you go to a higher dose of ursodeoxycholic acid?
No, the dose of UDCA should range between 13 and 15 mg/kg per day, and
higher doses have not shown any benefit in terms of biochemical response or mor-
tality. I would only increase the dose of this medication if it has been miscalculated
previously and the patient is being undertreated [9].
The patient remains on ursodeoxycholic acid, and at the end of 1 year, the alka-
line phosphatase is 150.
Would you change your treatment?
This is a difficult question to answer since the patient responded well to the
medication but did not completely normalize her ALP. Data supports that this near
normalization of her ALP should decrease her rate of PBC progression, transplant-­
free survival, and overall mortality. I would have a discussion with the patient
regarding her overall prognosis and pros and cons of starting obeticholic acid (OCA)
and come up with a collaborative plan after [10].
Do patients tend to have problems tolerating ursodeoxycholic acid or toler-
ating obeticholic acid?
Patients usually tolerate ursodeoxycholic acid well. Some patients complain of
hair thinning, stomach upset (particularly bloating and diarrhea), and weight gain.
UDCA should be taken with food. If patients can’t tolerate a single dose of the
medication per day, then UDCA can be split into two doses.
OCA is a synthetic bile acid, which acts as a very potent farnesoid X receptor
agonist. It was approved by the FDA for the management of PBC patients who
respond inadequately to UDCA after 1 year of therapy. Although over 45% of
patients achieve a significant ALP improvement with the concomitant use of OCA
and UDCA, this medication is associated with pruritus in a dose-dependent fashion,
leading to the discontinuation of the drug in up to 25% of patients. This is why OCA
should be started at a dose of 5 mg per day and then slowly increase the dose up to
10 mg PO daily. OCA also decreases HDL cholesterol and increases LDL choles-
terol independently of the dose. Providers should be aware that OCA is contraindi-
cated in patients with cirrhosis with a Child-Pugh score of B or C and in those with
a Child-Pugh score of A who have previous or current clinical evidence of portal
hypertension [8].
Can you use it in a grade A cirrhosis?
Yes, you can use it in cirrhotic patients with a Child-Pugh score of A, as long as
there is no evidence of portal hypertension. In these patients, the recommended
maximum dose is 5 mg PO/daily.
84 Managing Primary Biliary Cholangitis 363

Is it known why OCA might precipitate hepatic decompensation?


I don’t think this has been fully clarified. The current black label warning is
based on post-marketing reports.
In this case, the alkaline phosphatase drops down to 100 on the combination of
OCA and ursodeoxycholic acid.
So, I assume you just continue the two medicines indefinitely?
Yes, I would continue both medications indefinitely.
Besides tracking liver enzymes, how are you following your PBC patients?
The holistic management of PBC should include four different aspects. The first
one is the disease treatment per se, which includes the utilization of UDCA and
monitoring liver chemistries for response every 3–6 months. The second important
aspect is to stage the disease in terms of fibrosis status via elastography. Elastography
should be performed every 2–3 years in those PBC patients without evidence of
advanced fibrosis at baseline and on a yearly basis in those with evidence of
advanced fibrosis at baseline or on follow-up. In general, a liver stiffness measure-
ment below 6.5 kPa or above 11 kPa measured by transient elastography accurately
discriminates between the absence or presence of advanced fibrosis respectively
with high sensitivity and specificity. Surveillance for hepatocellular carcinoma and
esophageal varices should follow the same guideline recommendations that apply
for cirrhotic patients from other etiologies. A third important aspect is to address
and manage symptoms directly derived from PBC like pruritus, fatigue, and sicca
complex. Lastly, patients with cholestasis should be screened and treated for bone
disease, dyslipidemia, and liposoluble vitamin deficiencies, particularly if jaundice
has ensued [8].

References

1. Lindor KD, Bowlus CL, Boyer J, Levy C, Mayo M. Primary biliary cholangitis: 2018 prac-
tice guidance from the American Association for the Study of Liver Diseases. Hepatology.
2019;69(1):394–419.
2. Prince MI, Chetwynd A, Craig WL, Metcalf JV, James OF. Asymptomatic primary biliary
cirrhosis: clinical features, prognosis, and symptom progression in a large population based
cohort. Gut. 2004;53(6):865–70.
3. Lu M, Zhou Y, Haller IV, Romanelli RJ, VanWormer JJ, Rodriguez CV, et al. Increasing preva-
lence of primary biliary cholangitis and reduced mortality with treatment. Clin Gastroenterol
Hepatol. 2018;16:1342–50.e1.
4. Lleo A, Jepsen P, Morenghi E, Carbone M, Moroni L, Battezzati PM, et al. Evolving trends
in female to male incidence and male mortality of primary biliary cholangitis. Sci Rep.
2016;6:25906.
5. John BV, Aitcheson G, Schwartz KB, Khakoo NS, Dahman B, Deng Y, et al. Male sex is asso-
ciated with higher rates of liver related mortality in primary biliary cholangitis and cirrhosis.
Hepatology. 2021;74:879–91.
6. Lammers WJ, van Buuren HR, Hirschfield GM, Janssen HL, Invernizzi P, Mason AL,
et al. Levels of alkaline phosphatase and bilirubin are surrogate end points of outcomes of
364 J. Trivella

patients with primary biliary cirrhosis: an international follow-up study. Gastroenterology.


2014;147(6):1338–49.e5; quiz e15.
7. Gulamhusein AF, Hirschfield GM. Primary biliary cholangitis: pathogenesis and therapeutic
opportunities. Nat Rev Gastroenterol Hepatol. 2020;17(2):93–110.
8. Trivella J, John BV, Levy C. Primary biliary cholangitis: epidemiology, prognosis, and treat-
ment. Hepatol Commun. 2023;7(6):e0179. https://doi.org/10.1097/HC9.0000000000000179.
9. Angulo P, Jorgensen RA, Lindor KD. Incomplete response to ursodeoxycholic acid in primary
biliary cirrhosis: is a double dosage worthwhile? Am J Gastroenterol. 2001;96(11):3152–7.
10. Corpechot C, Poujol-Robert A, Wendum D, Galotte M, Chretien Y, Poupon RE, et al.
Biochemical markers of liver fibrosis and lymphocytic piecemeal necrosis in UDCA-treated
patients with primary biliary cirrhosis. Liver Int. 2004;24(3):187–93.
Chapter 85
Advanced PBC

Juan Trivella

A 55-year-old female, who hasn’t seen a doctor in years, comes in with the com-
plaint of severe itching and fatigue. She drinks only on weekends, two beers a day.
She has no stigmata of chronic liver disease on exam. However, her alkaline phos-
phatase is elevated to 360 (fractionation shows the increase is from the liver), bili-
rubin is 1.8, albumin is 3.2, AST is 40, ALT is 70, Hbg is 11, platelets is 100k, and
AMA is +1:160. US shows mild nodularity of liver. LSM (liver stiffness measure-
ment) is 11 kPa on transient elastography.
Would you want a liver biopsy in this case?
This female patient with classic symptoms for PBC, a predominantly cholestatic
pattern of elevated liver chemistries, and a positive AMA does not require a biopsy
to make the diagnosis of PBC. Despite this, if the possibility of a concomitant diag-
nosis is also being considered, for example, metabolic associated steatohepatitis
(MetASH), or there are concerns for overlap syndrome with autoimmune hepatitis
on additional serologies, then a liver biopsy may be necessary. The combination of
findings on US and elastography are concerning for progression to cirrhosis, and the
thrombocytopenia could imply the development of portal hypertension. I would feel
comfortable managing the patient as such and would obtain the appropriate screen-
ing per guidelines and monitor closely her liver chemistries [1].
This patient has an EGD, and there are no varices and no GAVE.
A normal endoscopy in the above scenario would not change the diagnosis of
PBC or the presence of at least advanced fibrosis (given the results on elastography).
The only aspect that would require further investigation is the thrombocytopenia,
since initially this was believed to be secondary to portal hypertension—and

J. Trivella (*)
Division of Gastroenterology and Hepatology, Department of Medicine, Medical College of
Wisconsin, Milwaukee, WI, USA
e-mail: [email protected]

© The Author(s), under exclusive license to Springer Nature 365


Switzerland AG 2023
W. H. Sobin et al. (eds.), Managing Complex Cases in Gastroenterology,
https://doi.org/10.1007/978-3-031-48949-5_85
366 J. Trivella

although this may still be true—the normal upper endoscopy makes this possibility
somewhat less likely. An alternative explanation for the low platelets would have to
be ruled out.
Would you want to start ursodeoxycholic acid, even if there is advanced
cirrhosis?
Yes, absolutely. UDCA improves liver chemistries, delays histological progres-
sion, and delays the development of varices. Furthermore, improved survival with
the use of UDCA has been shown regardless of sex, fibrosis stage, or even in those
with an inadequate response to the medication [2].
Do you monitor the AMA levels during treatment?
No, AMA titers do not correlate with disease activity, severity, or response to
therapy and are devoid of prognostic significance. AMA should not be followed
longitudinally since its value will not change the patient’s management [3].
In this patient, ursodeoxycholic acid is started and the alkaline phosphatase
doesn’t drop.
Would you add OCA?
To answer this question accurately, additional clinical information is needed, but
in general, OCA is indicated in patients with an inadequate response to UDCA after
1 year of monotherapy with this drug. This is exclusively for non-cirrhotics or for
those with cirrhosis and a Child’s Pugh score no worse than A without previous or
current evidence of portal hypertension or decompensated disease [1].
What do you think about using fibrates instead of OCA?
You can consider using fibrates as an off-label add-on therapy for the manage-
ment of inadequate responders. The caveat is that bezafibrate is not currently com-
mercially available in the United States. Most of the data on the effectiveness of
bezafibrate in combination with UDCA comes from Europe and Japan, where this
medication has been repurposed for the treatment of PBC and cholestatic itch for
years [4]. In the United States, only fenofibrate is available in retail, and data on its
use for the management of inadequate responders is considerably less robust.
Although gastrointestinal and musculoskeletal side effects, like myalgias and
arthralgias, are more common in patients with PBC on dual UDCA-fibrates, no
significant difference in the frequency of serious side effects (including elevations
of aminotransferases >5 times the ULN) has been reported when comparing the
combined treatment with those on UDCA monotherapy, making this a relatively
safe drug for use in PBC.
In this case, the cholesterol comes back 290. LDL is 160.
Is it unusual to see lipid levels like this in PBC, and would you start a statin?
Hyperlipidemia is very common among patients with PBC. At initial presenta-
tion, around three quarters of patients with PBC have a cholesterol level >200 mg/
dL with a proportion of them also experiencing milder elevations of low-density
lipoproteins (LDL) and marked elevations in high-density lipoproteins (HDL).
Late-stage disease is associated with marked LDL elevations. Clinically, dyslipid-
emia alone does not appear to increase the risk of cardiovascular events in patients
with PBC and therefore, its management should follow the same criteria that is used
85 Advanced PBC 367

to treat lipid disorders in other populations with no history of PBC. In other words,
statins are not contraindicated in patients with PBC and should be used as needed
and according to the regular dyslipidemia guidelines used in preventive medicine
and primary care [5, 6].
So, to summarize, you are going to add OCA to ursodeoxycholic acid and
probably start a statin. Are there any other treatments you would start in the
meantime?
If this patient is confirmed to have cirrhosis and portal hypertension and her
numbers fail to normalize on combination therapy, then a conversation regarding
the potential need for liver transplantation will be necessary. This is especially
important if her synthetic function continues to deteriorate, or she clinically decom-
pensates. She will also need appropriate hepatocellular carcinoma and esophageal
varices screening according to guidelines. The patient will also require symptomatic
management for pruritus, sicca complex, and fatigue. Lastly, she should receive
appropriate screening for bone disease and liposoluble vitamin deficiencies (the lat-
ter only if jaundice is present) and her hepatitis A virus (HAV) and hepatitis B virus
(HBV) vaccination status should be addressed.

References

1. Lindor KD, Bowlus CL, Boyer J, Levy C, Mayo M. Primary biliary cholangitis: 2018 prac-
tice guidance from the American Association for the Study of Liver Diseases. Hepatology.
2019;69(1):394–419.
2. Shi J, Wu C, Lin Y, Chen YX, Zhu L, Xie WF. Long-term effects of mid-dose ursodeoxycho-
lic acid in primary biliary cirrhosis: a meta-analysis of randomized controlled trials. Am J
Gastroenterol. 2006;101(7):1529–38.
3. Levy C, Bowlus CL. Role of antinuclear antibodies in primary biliary cholangitis. Am J
Gastroenterol. 2020;115(10):1604–6.
4. Tanaka A, Hirohara J, Nakano T, Matsumoto K, Chazouilleres O, Takikawa H, et al. Association
of bezafibrate with transplant free survival in patients with primary biliary cholangitis. J
Hepatol. 2021;75:565–71.
5. Assis DN. Chronic complications of cholestasis: evaluation and management. Clin Liver Dis.
2018;22:533–44.
6. Chalifoux SL, Konyn PG, Choi G, Saab S. Extrahepatic manifestations of primary biliary chol-
angitis. Gut Liver. 2017;11:771–80.
Chapter 86
Autoimmune Hepatitis

Francisco Durazo

The patient is a 24-year-old female with a history of hypothyroidism who presents


with weakness. She is on no medications, except oral contraceptives. Her labs
reveal an ALT of 520, AST of 319, Alk phos of 80, and bilirubin of 1.0. Hepatitis
panel is negative, ANA returns +1:160, and ASMA is +1:80. Gamma globulin is two
times normal.
How do you manage this patient?
All things point to this being autoimmune hepatitis, but we want to send the rest
of our chronic hepatitis workup for completeness. We need to do a liver biopsy to
confirm the diagnosis.
Liver biopsy is consistent with AIH.
What treatment do you recommend for AIH?
We like to start with prednisone monotherapy. Once we see a definite response to
steroids, with the liver enzymes improving, we add on azathioprine. We don’t start
azathioprine initially because we want to ensure that the hepatitis is steroid respon-
sive. If it is not steroid responsive, we may be dealing with a different diagnosis. We
start azathioprine at 50 mg. Once azathioprine is started, the goal is to taper the
steroids and hopefully maintain the patient on azathioprine monotherapy. Unlike
IBD, where doses of azathioprine may be 150 or 200 mg or more, in AIH hepatolo-
gists generally don’t go above 50 mg. And, unlike in IBD, many hepatologists do
not bother checking TPMT status before starting azathioprine.
What about the patient who has autoimmune hepatitis and cirrhosis?
We tend to avoid steroids in the patients with AIH and cirrhosis. The steroids are
more likely to promote sodium retention, ascites, and decompensation. So, in

F. Durazo (*)
Division of Gastroenterology and Hepatology, Department of Medicine, Medical College of
Wisconsin, Milwaukee, WI, USA
e-mail: [email protected]

© The Author(s), under exclusive license to Springer Nature 369


Switzerland AG 2023
W. H. Sobin et al. (eds.), Managing Complex Cases in Gastroenterology,
https://doi.org/10.1007/978-3-031-48949-5_86
370 F. Durazo

cirrhotics, we generally use azathioprine monotherapy, or CellCept (mycopheno-


late), or just maintenance therapy. In younger patients, AIH tends to present as a
more acute hepatitis with very high transaminases and jaundice. In older patients,
the disease usually presents as a more chronic hepatitis and a more subacute course,
and these patients are more likely to have well-established fibrosis or cirrhosis and
portal hypertension.
Chapter 87
Autoimmune Hepatitis

James Esteban

The patient is a 28-year-old female with a history of hypothyroidism due to


Hashimoto’s disease, who presents with weakness and anorexia for 4 weeks. She
has one or two cocktails per weekend. Her only medication is levothyroxine. Her
physical exam is fairly unremarkable with a BMI of 20. She has ALT of 420 U/L,
AST of 300 U/L, Alk phos of 70 U/L, and a total bilirubin of 1.2 mg/dL. Her viral
hepatitis panel is negative. She has elevated IgG at 2200 mg/dL, elevated titers of
ANA 1:640, and anti-smooth muscle antibody of 1:160. Anti-mitochondrial anti-
body is negative.
This case seems fairly clear-cut, in terms of the strongly positive autoim-
mune markers. What is your approach to the patient with suspected autoim-
mune hepatitis (AIH)?
Even though the patient has classic liver biochemistries (hepatocellular) and
autoantibodies, I will still obtain a liver biopsy. The liver biopsy is necessary to
establish the diagnosis, evaluate concurrent liver disease (for example, NASH or
alcohol), and define a baseline for future comparisons if the patient does not respond
to corticosteroids and first-line immunomodulator therapy.
This patient’s liver biopsy shows interface hepatitis with a dense infiltrate of
plasma cells. She has minimal hepatic steatosis and hepatic fibrosis (stage 1). You
start the patient on prednisone 60 mg a day, and the patient’s liver enzymes start
improving immediately, but the patient also complains of mood swings and insomnia.

J. Esteban (*)
Division of Gastroenterology and Hepatology, Department of Medicine, Medical College of
Wisconsin, Milwaukee, WI, USA
e-mail: [email protected]

© The Author(s), under exclusive license to Springer Nature 371


Switzerland AG 2023
W. H. Sobin et al. (eds.), Managing Complex Cases in Gastroenterology,
https://doi.org/10.1007/978-3-031-48949-5_87
372 J. Esteban

How would you manage this adverse side effect?


I would see how she does on a slightly lower dose of prednisone, perhaps 40 mg
a day. I will do labs weekly and try to taper her prednisone dose further as soon as
her transaminases improve. I start azathioprine once I have prednisone down to
20–30 mg a day, and this may help speed up the prednisone taper.
Another option is budesonide. A 9 mg dose of budesonide is 30–40 mg of pred-
nisone, but due to 90% first-pass metabolism in the liver, budesonide is considered
a “topical” steroid and has much fewer steroid-related side effects than prednisone.
Budesonide is at least as effective (possibly more effective based on trials) than
prednisone in inducing and maintaining biochemical remission in non-cirrhotic
patients with AIH [1].
The patient is started on budesonide 9 mg a day, and the steroid side effects
cease. Her liver enzymes continue improving.
Would you add azathioprine at this point? If so, what dose do you give? Do
you bother to check the TPMT status?
Once I verify a positive response to corticosteroids, I add on azathioprine (or
6-mercaptopurine, 6-MP). Since azathioprine can cause drug-induced liver injury
(either related to 6-MMP metabolites or as an idiosyncratic reaction), I prefer to
start azathioprine only when transaminases are normal to prevent diagnostic
confusion.
I start azathioprine at 50 mg a day. I send off a thiopurine methyltransferase
(TPMT) activity to identify that rare patient who will have zero or near-zero TPMT
activity who will suffer from severe myelosuppression with azathioprine. Because
this test takes time to result, I send it out as soon as AIH is diagnosed. The AASLD
recommends sending TPMT activity [2].
The intent is for azathioprine (or 6-MP) to be the long-term, steroid-sparing
maintenance regimen for AIH. For this reason, I continue to taper prednisone
down to a dose of prednisone 5 mg. I try to discontinue prednisone altogether to
keep patients on azathioprine (or 6-MP) monotherapy for maintenance, and I have
reasonable success. Azathioprine at a dose of 50 mg is often sufficient, but I have
gone up to 75–150 mg of azathioprine if patients experience small flare-ups in
their transaminases. In some cases, the only way to keep patients in remission is
through combination therapy of azathioprine and low-dose prednisone,
5–10 mg daily.
The patient’s TPMT phenotype returns normal, and you start azathioprine 50 mg
a day. However, 2 weeks later the patient develops severe pain, goes to the ER, and
is found to have acute pancreatitis. Azathioprine is stopped.
Do you have any experience using mycophenolate or tacrolimus in this
situation?
Mycophenolate mofetil (MMF) and mycophenolic acid (MPA) are my second-­
line agents for patients who do not tolerate or do not respond to azathioprine.
Mycophenolate is particularly effective if the reason for withdrawing azathioprine
was intolerance, rather than treatment nonresponse. However, mycophenolate is a
category X drug in pregnancy, thus at our patient’s age, she needs to use at least two
forms of effective contraception.
87 Autoimmune Hepatitis 373

Calcineurin inhibitors (CNI), like tacrolimus and cyclosporine, and biologics are
also second-line agents for patients who are refractory to or intolerant of azathio-
prine or steroids. I almost never have had to use these agents in my non-transplant
AIH patients, but there are studies showing CNIs as effective agents in normalizing
transaminases.
Working in the community, we do not have much experience with mycophe-
nolate. What is the dose you use, and what are the side effects?
You can use a bottom-up approach, starting at 500 mg BID of MMF (or 360 mg
BID of MPA), and then gradually make your way up to 1000 mg BID of MMF (or
720 mg BID of MPA) based on the patient’s response. You can also use a top-down
approach, especially if the patient has higher transaminase levels, where you start at
1000 mg BID of MMF until you’re able to induce and maintain biochemical remis-
sion, and then gradually make your way down to the minimum effective dose. The
most common side effects of MMF are GI upset (e.g., nausea, vomiting, abdominal
pain, diarrhea) and cytopenias.
The patient says she hopes to get pregnant in the next year or two. What
options do you have?
If she hopes to get pregnant, my options are to either keep her on steroid mono-
therapy, since she responded to that, or use CNIs such as tacrolimus or cyclosporine
instead of mycophenolate. Both are pregnancy category C. Azathioprine is category
D but may be continued during pregnancy to prevent a flare-up.
For tacrolimus, a good starting dose is 0.1 mg/kg/day divided into two doses,
which roughly translates to 2–3 mg BID [3]. For cyclosporine, a good starting dose
is 2–3 mg/kg/day, which roughly translates to 100–150 mg BID [4]. Both tacroli-
mus and cyclosporine require drug-level monitoring to ensure efficacy and prevent
toxicities. Drug-level monitoring requires blood testing at the same time of day,
typically in the morning before taking the first dose of the day. Patients would likely
need higher drug levels to induce remission (tacrolimus 6–8 ng/dL, cyclosporine
100–150 ng/dL), which could be lowered after a few months to a year of remission
for maintenance (tacrolimus 4–6 ng/dL, cyclosporine 100–150 ng/dL).
The main side effects of CNIs are nephrotoxicity, neurotoxicity, and metabolic
dysfunction, such as diabetes, hyperlipidemia, hypertension, and weight gain.
There are patients with AIH who have cirrhosis on their initial liver biopsy,
occurring in about 30% of cases. Is this fairly uncommon in young patients
presenting with AIH?
It is. Cirrhosis as the presenting phenotype of AIH is usually seen in older
patients and African-Americans.
Is it true that if you have full-blown cirrhosis, you may already have burned
out AIH and you may choose not to treat it?
Some patients with AIH and cirrhosis will have normal to mildly elevated trans-
aminases and minimal inflammation on biopsy and would be labeled as “burned
out” AIH. I think it would be reasonable to withhold immunosuppressive treatment
for these patients because any potential benefit is not worth treatment-related side
effects. These patients should continue to have periodic assessment of liver
chemistries.
374 J. Esteban

Some patients will have normal transaminases but will continue to have inflam-
matory activity on biopsy. I would treat these patients with maintenance immuno-
suppression to prevent progression of fibrosis (and thus portal hypertension and
hepatic decompensation) and a severe, acute flare-up of AIH. We avoid azathioprine
in patients with decompensated cirrhosis.

References

1. Manns M, Woynarowski M, Kreisel W, Lurie Y, Rust C, Zuckerman E, et al. Budesonide


induces remission more effectively than prednisone in a controlled trial of patients with auto-
immune hepatitis. Gastroenterology. 2010;139(4):1198–206.
2. Mack CL, Adams D, Assis DN, Kerkar N, Manns MP, Mayo MJ, et al. Diagnosis and manage-
ment of autoimmune hepatitis in adults and children: 2019 practice guidance and guidelines from
the American Association for the Study of Liver Diseases. Hepatology. 2020;72(2):671–722.
3. Hanouneh M, Ritchie MM, Ascha M, Ascha MS, Chedid A, Sanguankeo A, et al. A review
of the utility of tacrolimus in the management of adults with autoimmune hepatitis. Scand J
Gastroenterol. 2019;54(1):76–80.
4. Fernandes NF, Redeker AG, Vierling JM, Villamil FG, Fong TL. Cyclosporine therapy in
patients with steroid resistant autoimmune hepatitis. Am J Gastroenterol. 1999;94(1):241–8.
Chapter 88
Primary Sclerosing Cholangitis

Juan Trivella

A 38-year-old male is referred because of elevated liver enzymes. He has one or two
drinks a week, does not use any drugs, and is on no medications. He has mild itch-
ing, no belly pain, and no jaundice. He has an alkaline phosphatase of 240, GGT of
200, ALT of 60, AST of 45, bilirubin of 0.9, INR of 1.1, WBC of 4.3, Hgb of 11.8,
platelets of 180,000, and albumin of 3.5. Ultrasound of the liver shows normal-­
appearing liver, normal gallbladder, and CBD of 8 mm in diameter. Hepatitis panel
is negative, ANA is (+)1:16, ASMA is (−), and AMA is (−). An MRCP is performed
and this demonstrates ductal narrowing both intra- and extrahepatic.
What is your diagnosis and what other tests would you want?
The presence of cholestasis and intra- and extrahepatic ductal narrowing on
MRCP in a male patient who has no other previous history of biliary or liver pathol-
ogy suggests the diagnosis of PSC, but careful exclusion of secondary causes of
sclerosing cholangitis is required—especially in the absence of IBD. The negative
AMA and the presence of intra- and extrahepatic ductal beading makes PBC
unlikely. It is not uncommon to have a slight elevation of transaminases in PSC, like
in the case presented above. The diagnosis of overlap AIH-PSC should be consid-
ered only when the transaminases approach five times the upper limit of normal or
when these are the predominant pattern of elevation. It would also be important to
order an IgG4 to exclude IgG4 sclerosing cholangitis as a potential diagnosis. A
baseline CA19-9 and a colonoscopy to evaluate for inflammatory bowel disease
should also be obtained [1].

J. Trivella (*)
Division of Gastroenterology and Hepatology, Department of Medicine, Medical College of
Wisconsin, Milwaukee, WI, USA
e-mail: [email protected]

© The Author(s), under exclusive license to Springer Nature 375


Switzerland AG 2023
W. H. Sobin et al. (eds.), Managing Complex Cases in Gastroenterology,
https://doi.org/10.1007/978-3-031-48949-5_88
376 J. Trivella

What is the current role of colonoscopy in PSC, both IgG4 positive and
negative?
In patients with PSC without a known history of IBD, a diagnostic colonoscopy
with histologic sampling should be performed at the diagnosis of PSC and then
every 5 years if IBD is not initially detected, or earlier if the patient develops gas-
trointestinal symptoms to suggest IBD. Colon cancer surveillance should begin at
age 15 years in patients with PSC and IBD and should be repeated yearly, given
their higher risk for developing colorectal cancer.
Is that true for IgG4 sclerosing cholangitis?
No. Although there is paucity of data regarding the course of IgG4 sclerosing
cholangitis, given its rarity, no strong association with IBD and/or colorectal cancer
has been established. Therefore, surveillance colonoscopy is not currently recom-
mended for these patients.
What is the role of ursodeoxycholic acid in PSC?
The efficacy of UDCA in PSC has not been consistent, despite this being the
most studied drug for this particular disease. Low-dose UDCA (13–15 mg/kg/day)
has shown an improvement in ALP (in those with baseline elevated ALP) but has
shown no benefits on liver histology or transplant-free survival. High-dose UDCA
(28–30 mg/kg/day) should be avoided in patients with PSC, given the high risk for
serious adverse events.
Intermediate-dose UDCA (17–23 mg/kg/day) has shown both ALP and histo-
logical improvement in a proportion of patients with PSC, but research has failed to
demonstrate a statistically significant reduction in the need for liver transplantation,
cholangiocarcinoma, or overall mortality. This is with the caveat that most of these
reports have been underpowered. Medical societies like the AASLD currently sup-
port trials of intermediate-dose UDCA in PSC patients with baseline elevated
ALP. This medication can be continued lifelong if the patient experiences improve-
ment of symptoms and/or a meaningful alkaline phosphatase reduction after
12 months of treatment [1].
How do you do surveillance studies in PSC?
Cholangiocarcinoma and gallbladder carcinoma surveillance should be per-
formed annually and include MRI/MRCP and a CA19-9. Surveillance is currently
not recommended for patients with PSC who are younger than 18 years of age or in
those with exclusively small duct PSC.
In this case, the patient remains stable for a couple of years, with a stable MRCP
and CA19-9, and then, without explanation, his alkaline phosphatase goes up to
350, and his bilirubin goes up to 3. Another MRCP is performed, and there’s a
dominant stricture in the common bile duct.
How would you manage this?
This patient was found to have a relevant stricture on MRI/MRCP and should
receive an ERCP. ERCP is indicated for the evaluation of relevant strictures as well
as for the evaluation of new onset or worsening of pruritus, unexplained weight loss,
worsening serum liver chemistry abnormalities, rising CA19-9, recurrent bacterial
cholangitis, or progressive ductal dilatation on MRI/MRCP surveillance.
88 Primary Sclerosing Cholangitis 377

On ERCP brushings of the stricture are performed. Cytology returns suspicious


and FISH cytology is pending.
The first step should be to wait for the FISH analysis to return. If negative, then
the patient should receive a repeat ERCP in 3 months. If positive, this case should
be discussed with a multidisciplinary team that includes pancreaticobiliary and
transplant surgery. Depending on the location of the stricture (intrahepatic vs. hilar
vs. distal CBD), this patient may qualify for resection or liver transplantation.
Can you do a liver transplant in a PSC patient who has a
cholangiocarcinoma?
The management of cholangiocarcinoma in patients with PSC depends on sev-
eral factors. The size and the location of the primary tumor in the biliary tree is one
of them. Well-selected patients with PSC and perihilar early-stage cholangiocarci-
noma can be considered for liver transplantation and can even qualify for exception
points after a rigorous process that involves close monitoring for metastatic disease,
neoadjuvant chemoradiation, and pretransplant staging laparoscopy. This is com-
monly referred to as the “Mayo protocol” and has shown an overall survival of
around 65% and a recurrence-free survival close to 80% at 5 years [2].
Early studies evaluating liver transplantation in patients with intrahepatic chol-
angiocarcinoma showed a posttransplant recurrence in up to 25% and an overall
survival of about 40% at 5 years, making this approach undesirable for quite some
time. Most recently, retrospective data looking at liver explants of patients with PSC
and an incidentally found, small (<3 cm), solitary intrahepatic cholangiocarcinoma
showed an overall survival of around 61% and a recurrence-free survival of 75% at
5 years. This has prompted different centers around the country to develop prospec-
tive research protocols to reevaluate LT as a possibility for those with a single small
liver lesion. Results from these trials are eagerly awaited.
Distal cholangiocarcinomas are best treated with surgical resection when possi-
ble, removing the bile duct, gallbladder, head of the pancreas, and the first portion
of the duodenum (pancreaticoduodenectomy). There is currently no role for liver
transplantation in patients with distal cholangiocarcinoma.

References

1. Bowlus CL, Arrivé L, Bergquist A, Deneau M, Forman L, Ilyas SI, Lunsford KE, Martinez
M, Sapisochin G, Shroff R, Tabibian JH, Assis DN. AASLD practice guidance on primary
sclerosing cholangitis and cholangiocarcinoma. Hepatology. 2023;77(2):659–702. https://doi.
org/10.1002/hep.32771.
2. Rosen CB, Heimbach JK, Gores GJ. Surgery for cholangiocarcinoma: the role of liver trans-
plantation. HPB (Oxford). 2008;10(3):186–9. https://doi.org/10.1080/13651820801992542.
Chapter 89
PBC-AIH Overlap

Juan Trivella

A 35-year-old female is referred for lethargy and elevated LFTs. Her alkaline phos-
phatase is 320, AST is 289, ALT is 391, and bilirubin is 1.2. He is in no ETOH or
meds. AMA +1:320, ANA +1:160, ASMA +1:80, IgG 2200.
What is your diagnosis, and do you want a liver biopsy?
The information provided suggests a diagnosis of PBC-AIH overlap syndrome.
Although there is no standardized definition, this clinical entity is commonly char-
acterized by clinical, histologic, and serologic features of both autoimmune hepati-
tis and PBC, ensuing either simultaneously or sequentially.
According to the Paris criteria, which has a sensitivity of 92% and specificity of
97% for the diagnosis of overlap syndrome, two out of three key features for the
diagnosis of PBC and AIH, respectively, should be met to establish the diagnosis.
One of these is the presence of interface hepatitis and/or the presence of florid bile
duct lesions on histology. A liver biopsy in the case presented is not only indicated
to establish the diagnosis of overlap syndrome but also to exclude other potential
explanations for her elevated liver chemistries and to stage the fibrosis status of her
liver [1].
Liver biopsy is performed and shows granulomas with bile duct inflammation
and also mild interface hepatitis with lymphocytes and plasma cells.
Would you start the patient on ursodeoxycholic acid alone, steroids alone, or
a combination?
The therapeutic approach for overlap syndrome has not been completely defined
because its low prevalence prevents the performance of large randomized clinical
trials. Most agree that the treatment should be directed toward the predominant

J. Trivella (*)
Division of Gastroenterology and Hepatology, Department of Medicine, Medical College of
Wisconsin, Milwaukee, WI, USA
e-mail: [email protected]

© The Author(s), under exclusive license to Springer Nature 379


Switzerland AG 2023
W. H. Sobin et al. (eds.), Managing Complex Cases in Gastroenterology,
https://doi.org/10.1007/978-3-031-48949-5_89
380 J. Trivella

component, particularly when patients do not completely meet Paris criteria. The
rationale for this approach comes from the premise that rather than being two sepa-
rate disorders occurring in parallel, overlap syndrome appears to be the result of a
single disease with atypical features. In these circumstances, steroids or UDCA
induce biochemical response when selected toward the predominant phenotype. In
those who meet Paris criteria, combination therapy with immunosuppression and
UDCA has proven to be superior in achieving biochemical response than either of
them alone. Since this patient meets all Paris criteria, then combination therapy
would be more appropriate [2, 3].
What steroid would you use?
Both prednisone and prednisolone can be used for the management of overlap
syndrome in combination with UDCA. Steroids should be tapered slowly to about
10 mg daily after 4 weeks of treatment to decrease the risk of adverse events.
Would you add azathioprine as you attempt to taper steroids?
Yes, I would attempt this. Given the low prevalence of PBC-AIH overlap syn-
drome, guidelines for its long-term treatment lack robust evidence. Despite this,
azathioprine has successfully been used as the long-term immunosuppressive agent
of choice for those with AIH. It would be reasonable to extrapolate this information
to patients with overlap syndrome, especially when considering the adverse events
that are commonly associated with long-term steroid use.

References

1. Lindor KD, Bowlus CL, Boyer J, Levy C, Mayo M. Primary biliary cholangitis: 2018 prac-
tice guidance from the American Association for the Study of Liver Diseases. Hepatology.
2019;69(1):394–419.
2. Chazouilleres O, Wendum D, Serfaty L, Montembault S, Rosmorduc O, Poupon R. Primary
biliary cirrhosis–autoimmune hepatitis overlap syndrome: clinical features and response to
therapy. Hepatology. 1998;28:296–301.
3. Kuiper EM, Zondervan PE, van Buuren HR. Paris criteria are effective in diagnosis of primary
biliary cirrhosis and autoimmune hepatitis overlap syndrome. Clin Gastroenterol Hepatol.
2010;8:530–4.
Chapter 90
Immune-Tolerant Chronic HBV

James Esteban

A 24-year-old male, born in the United States and whose parents emigrated from
Taiwan, is evaluated in the clinic for a hepatitis B infection. This was detected when
he went to donate blood at a blood drive. He has no history of intravenous drug use,
tattoos, or blood transfusions. He is in a monogamous sexual relationship with a
female spouse. He is asymptomatic and has normal transaminases. He is hepatitis
B surface antigen (HBsAg) positive with an HBV DNA of 10 million IU/mL. His
total hepatitis B core antibody (anti-HBc total) is positive, and the anti-HBc IgM is
negative. He is hepatitis B envelope antigen positive (HBeAg).
How would you describe the phase of the patient’s hepatitis B infection?
We first have to establish the chronicity of hepatitis B infection. Chronic hepati-
tis B is the presence of hepatitis B surface antigen and HBV viremia for at least
6 months. The absence of significant hepatocellular liver injury and anti-HBc IgM
strongly suggests chronic hepatitis B. I would make sure to repeat HBV serologies,
HBV DNA, and liver enzymes in the next 3–6 months to verify chronic hepatitis B.
After we have established chronic hepatitis B, we need to determine the phase of
HBV infection the patient is experiencing. In the immune-tolerant phase, there is
normal ALT, minimal to no hepatic inflammation, and high HBV DNA viral load.
Immune-tolerant hepatitis B is commonly seen in perinatally or vertically acquired
infection, which is potentially the route of transmission for our patient. These
patients are typically HBeAg-positive.

J. Esteban (*)
Division of Gastroenterology and Hepatology, Department of Medicine, Medical College of
Wisconsin, Milwaukee, WI, USA
e-mail: [email protected]

© The Author(s), under exclusive license to Springer Nature 381


Switzerland AG 2023
W. H. Sobin et al. (eds.), Managing Complex Cases in Gastroenterology,
https://doi.org/10.1007/978-3-031-48949-5_90
382 J. Esteban

It is my impression that immune-tolerant chronic hepatitis B refers to the


fact that the immune system does not recognize the hepatitis B virus as a for-
eign invader. This is a unique situation where the virus keeps replicating,
undisturbed by the immune system, and you can have hundreds of thousands,
or even millions of viral particles present with totally normal liver enzymes. If
you were to do a liver biopsy, there would be no signs of liver damage. Is that
correct?
This is the expected finding. The immune-tolerant phase of chronic HBV is
thought to be the result of reduced responsiveness of immunologic T cells to the
virus, which leads to unrestricted hepatitis B viral replication and, thus, high HBV
DNA viral load. Liver injury in chronic hepatitis B is the result of immune-mediated
(e.g., cytotoxic T cells, NK cells, and neutrophils) cell and tissue injury. Therefore,
a hypoactive immune response also means that there is little to no liver injury, which
explains the typically normal transaminases in immune-tolerant chronic hepatitis B.
Would you start antiviral therapy?
AASLD [1] and EASL [2] do not recommend starting antiviral treatment in
immune-­tolerant chronic hepatitis B infection, unless the patient has any of the
following:
1. Cirrhosis
2. Extrahepatic manifestations (usually immune-mediated) of HBV infection such
as vasculitis, purpura, polyarteritis nodosa, arthralgias, neuropathy, and
glomerulonephritis
3. Receiving immunosuppressive therapy
4. Pregnant women with HBV DNA >200,000 IU/mL in the third trimester to help
prevent mother-to-child transmission of HBV (tenofovir disoproxil fumarate is
the preferred antiviral medicine in pregnancy)
We are not starting antiviral therapy. How would you monitor this patient
after this initial encounter?
In these patients, liver chemistries and HBV DNA should be monitored every
3–6 months to determine if they are transitioning to the immune-active or chronic
inactive phases of infection. Increasing transaminases warrants more frequent mon-
itoring. Patients should meet criteria for immune-active chronic hepatitis B (ALT
>2 × ULN and HBV DNA >20,000 IU/mL if HBeAg-positive and >2000 if HBeAg-­
negative) for at least 3–6 months before committing to antiviral therapy, since it is
possible that increased immunologic activity may lead to viral clearance and entry
into the chronic inactive phase of infection.
Some patients will have elevated transaminases that are less than 2× the upper
limit of normal. In addition to further testing to ensure that they do not have other
causes of liver disease, such as metabolic dysfunction or alcohol-associated fatty
liver diseases, liver biopsy and/or elastography should be considered for these
patients because significant inflammation or fibrosis will warrant antiviral therapy.
What other issues need to be addressed for this patient?
Certain subpopulations of patients with chronic hepatitis B should undergo
regular screening for hepatocellular carcinoma (HCC) with abdominal imaging
90 Immune-Tolerant Chronic HBV 383

(e.g., ultrasound, CT or MRI) and a-fetoprotein every 6 months. Subpopulations


who should undergo regular HCC screening are:
1. All patients with cirrhosis
2. Asian and Black men above 40 years old, irrespective of cirrhosis
3. Asian women above 50 years old, irrespective of cirrhosis
4. First-degree relatives with HCC, irrespective of cirrhosis
Patients with immune-tolerant chronic hepatitis B are highly viremic and are
therefore very contagious. Sexual partners and household members of the patient
should be screened for chronic hepatitis B with HBsAg and anti-HBs testing.
Uninfected and nonimmune sexual partners and household members should com-
plete the hepatitis B vaccine series, followed by repeat anti-HBs testing to ensure an
adequate response to the vaccine.
The patient should also be counseled the following:
1. Use barrier contraception if their sexual partner is not vaccinated or naturally
immunized
2. Not to share toothbrushes and razors
3. Not to share injection equipment, including glucose testing equipment for
diabetics
4. Cover all open cuts and scratches
5. Clean blood spills with bleach solution
6. Not to donate blood, organs, or sperm
Is there a certain age when you would start antivirals in a patient with
immune-tolerant phase chronic hepatitis B?
AASLD [1] recommends liver biopsy in patients with immune-tolerant chronic
hepatitis B aged 40 years and above, especially those infected at a young age and
with very elevated HBV viral load (HBV DNA >1 million IU/mL) to assess severity
of inflammation and fibrosis. Immune-tolerant hepatitis B patients with at least
moderate inflammation and/or stage 2 fibrosis should receive antiviral therapy.
These patients could alternatively undergo noninvasive testing of hepatic fibrosis
through elastography first. Patients with significant fibrosis (i.e., F2 or greater) on
elastography should warrant antiviral therapy.
EASL [2] is more aggressive and recommends antiviral therapy in patients with
immune-tolerant hepatitis B age 30 years and above, regardless of the severity of
liver histologic lesions.

References

1. Terrault N, Lok ASF, McMahon BJ, Chang KM, Hwang JP, Jonas MM, et al. Update on pre-
vention, diagnosis, and treatment of chronic hepatitis B: AASLD 2018 hepatitis B guidance.
Hepatology. 2018;67(4):1560–99.
2. European Association for the Study of the Liver. EASL 2017 clinical practice guidelines on the
management of hepatitis B virus infection. J Hepatol. 2017;67(2):370–98.
Chapter 91
Immune-Active Chronic HBV

James Esteban

A 30-year-old Asian male has had a diagnosis of chronic hepatitis B since his teen
years. His mother also has chronic hepatitis B. He previously had normal transami-
nases, HBV viral load ranging from 10 to 20 million IU/mL, and positive HBeAg.
On routine testing 3 months ago, he was found to have elevated transaminases (ALT
of 140, AST of 80). His HBV DNA remains elevated, but at a lower level than previ-
ously, currently at around 100,000 IU/mL. On repeat testing today, he continues to
have elevated transaminases (ALT of 120, AST of 80) and elevated HBV DNA
(90,000 IU/mL). He remains HBeAg-positive.
The fact that his liver chemistries increased to more than twice the upper
limit normal while maintaining significant hepatitis B viremia indicates that
he has evolved into immune-active chronic hepatitis B. Can you explain the
significance of the positive HBeAg and the usual transition to nega-
tive HBeAg?
The hepatitis B envelope antigen (HBeAg) is a hepatitis B viral protein that
indicates active viral replication. Thus, patients who are HBeAg-positive typically
have very high titers of HBV DNA in serum. HBeAg-positive infection, particu-
larly in younger persons, will have normal or near-normal transaminases, because
HBV does not have a direct cytopathic effect and does not cause liver inflammation
on its own. This is known as the immune-tolerant phase of chronic hepatitis B
infection, where the body’s T-cell-mediated response to the virus is thought to be
blunted.

J. Esteban (*)
Division of Gastroenterology and Hepatology, Department of Medicine, Medical College of
Wisconsin, Milwaukee, WI, USA
e-mail: [email protected]

© The Author(s), under exclusive license to Springer Nature 385


Switzerland AG 2023
W. H. Sobin et al. (eds.), Managing Complex Cases in Gastroenterology,
https://doi.org/10.1007/978-3-031-48949-5_91
386 J. Esteban

The body may attempt to clear the virus at some point through an immune-­
mediated and inflammatory response, which leads to hepatic injury and inflamma-
tion, causing a rise in transaminases and eventually hepatic fibrosis if left unabated.
HBV DNA titers remain elevated, but typically not at the same magnitude as in the
immune-tolerant phase. This is known as the immune-active phase of chronic hepa-
titis B infection.
The immune-active phase of chronic hepatitis infection is defined by the
AASLD as [1]:
1. HBsAg-positive for at least 6 months
2. ALT or AST > 2× upper limit of normal, defined as 35 U/L in males and 25 U/L
in females
3. HBV DNA >20,000 IU/mL in HBeAg-positive hepatitis B and >2000 IU/mL in
HBeAg-negative hepatitis B
The immune-active phase can lead to a couple of possible outcomes. The patient
can continue to have viral replication (HBeAg-positive, high HBV DNA) with con-
tinuing liver injury and inflammation, and this warrants antiviral therapy. On the
other hand, the immune system may succeed in suppressing the infection, which
can lead to loss of HBeAg and low HBV DNA viral loads (defined as <2000 IU/
mL), indicating suppression of viral replication and improving liver chemistries.
This is known as the inactive phase of chronic hepatitis B infection. Spontaneous
HBeAg seroconversion, which is a way of saying that HBeAg is lost and anti-HBe
antibodies have appeared, occurs in 15% of cases per year [2]. Some patients may
lose hepatitis B surface antigen (HBsAg) altogether, representing a functional cure
of the infection, but this occurs much more rarely at a rate of 1–2% per year [3].
A few patients will have lost HBeAg but will continue to have active viral repli-
cation (i.e., high HBV DNA) and hepatic inflammation (i.e., ALT >2× ULN). These
patients have HBeAg-negative, immune-active chronic hepatitis B, and typically
mutations in the precore or basal core promoter region of the HBV genome.
You decide that the patient has immune-active chronic hepatitis B and will
need antiviral therapy. What are the options and what would you recommend
for the patient?
AASLD [1] and EASL [4] recommend antiviral therapy in patients with immune-­
active hepatitis B (Fig. 91.1). (Note: “Chronic hepatitis B” is the EASL nomenclature
for immune-active hepatitis B, while “chronic HBV infection” is the EASL nomen-
clature for immune-tolerant hepatitis B) Antiviral therapy is also recommended in all
patients with cirrhosis and in patients who have significant inflammation or fibrosis
on liver biopsy or elastography, regardless of ALT and HBV DNA titers.
Patients who have elevated transaminases, but do not quite meet criteria for
immune-active hepatitis B, should undergo further evaluation for other liver dis-
eases. They should also be considered for liver biopsy or elastography. The presence
of significant hepatic inflammation and fibrosis stage 2 or greater warrants antiviral
treatment. Antiviral therapy is not recommended in most patients with immune-­
tolerant hepatitis B, and this is discussed in a separate chapter.
91 Immune-Active Chronic HBV 387

Fig. 91.1 Treatment algorithm for chronic hepatitis B according to HBeAg status. (a) HBeAg-­
positive. (b) HBeAg-negative

The two treatment options for chronic hepatitis B include pegylated interferon
(PEG-IFNa) and nucleos(t)ide analogs (NA). Earlier generation NAs (lamivudine,
telbivudine, adefovir) have a low genetic barrier to resistance. Newer NAs (enteca-
vir, tenofovir disoproxil fumarate [TDF], tenofovir alafenamide [TAF]) have high
genetic barriers to resistance and are the recommended first-line antiviral therapy
for chronic hepatitis B. In all honesty, we can choose either entecavir, TDF, or TAF
for our patient.
PEG-IFNa has the advantages of finite treatment duration (48 weeks) and slightly
higher rates of HBsAg loss, but significant side effects limit its wider use in clinical
practice, relative to entecavir and tenofovir. PEG-IFNa is contraindicated in decom-
pensated cirrhosis and acute liver failure.
388 J. Esteban

The patient has normal kidney function. He underwent vibration-controlled tran-


sient elastography (VCTE or FibroScan®), which showed liver stiffness of 5 kPa
(F0). You recommend starting the patient on tenofovir disoproxil fumarate (TDF)
400 mg once daily.
The patient asks about the effectiveness of TDF and possible long-term side
effects.
The effectiveness of TDF and other NAs in clinical trials are summarized
below [4]:

Entecavir TDF TAF


HBeAg-positive
 Anti-HBe seroconversion 21% 21% 10%
 HBV DNA suppression 67% 76% 64%
 ALT normalization 68% 68% 72%
 HBsAg loss 2% 3% 1%
HBeAg-negative
 Anti-HBe seroconversion N/A N/A N/A
 HBV DNA suppression 90% 93% 94%
 ALT normalization 78% 76% 83%
 HBsAg loss 0% 0% 0%

Long-term treatment with NA, specifically due to result of suppression of viral


replication, halts the progression of liver disease, improves hepatic fibrosis (poten-
tially even regression of cirrhosis) and hepatic recompensation in cases of decom-
pensated cirrhosis, reduces the need for liver transplantation, and reduces the risk of
HCC [5].
TDF has been associated with renal dysfunction, including Fanconi syndrome,
and bone mineral density loss. Entecavir or TAF are preferred in older patients (age
>60 years), patients with established osteopenia or osteoporosis or at risk for these
conditions (e.g., long-term use of corticosteroids), and patients with renal dysfunc-
tion (e.g., estimated GFR <60 mL/min/1.73 m2, established albuminuria).
After 9 months on TDF, he becomes HBeAg-negative and develops anti-HBe. His
transaminases normalize and HBV DNA becomes undetectable.
Would you consider stopping TDF in this patient now or in the near future?
I would have him continue TDF for an additional 12 months as consolidation
therapy. If transaminases remain normal and HBV DNA remain undetectable at the
end of consolidation therapy—assuming repeat VCTE or FibroScan® continues to
show absence of significant fibrosis, he does not have extrahepatic manifestations,
and he is not receiving immunosuppressive therapy of any kind—then, yes, I will
consider discontinuing TDF. I would obtain transaminases and HBV DNA every
3 months for at least a year to make sure that he does not experience a relapse and
hepatitis flare-up.
The majority of initially HBeAg-positive patients who seroconvert remain in
remission [6]. Treatment discontinuation is discussed in more detail in another
chapter.
91 Immune-Active Chronic HBV 389

References

1. Terrault N, Lok ASF, McMahon BJ, Chang KM, Hwang JP, Jonas MM, et al. Update on pre-
vention, diagnosis, and treatment of chronic hepatitis B: AASLD 2018 hepatitis B guidance.
Hepatology. 2018;67(4):1560–99.
2. Dusheiko G, Agarwal KM, Maini MK. New approaches to chronic hepatitis B. N Engl J Med.
2023;388(1):55–69.
3. Yeo YH, Ho HJ, Yang HI, Tseng TC, Hosaka T, Trinh HY, et al. Factors associated with rates
of HBsAg Seroclearance in adults with chronic HBV infection: a systematic review and meta-­
analysis. Gastroenterology. 2019;156(3):635–646.e9.
4. European Association for the Study of the Liver. EASL 2017 clinical practice guidelines on the
management of hepatitis B virus infection. J Hepatol. 2017;67(2):370–98.
5. Su TH, Hu TH, Chen CY, Huang YH, Chuang WL, Lin CC, et al. Four-year entecavir therapy
reduces hepatocellular carcinoma, cirrhotic events and mortality in chronic hepatitis B patients.
Liver Int. 2016;36(12):1755–64.
6. Papatheodoridis G, Vlachogiannakos I, Cholongitas E, Wursthorn K, Thomadakis C, Touloumi
G, et al. Discontinuation of oral antivirals in chronic hepatitis B: a systematic review.
Hepatology. 2016;63(5):1481–92.
Chapter 92
HBV Treatment Discontinuation

James Esteban

A 42-year-old former IV heroin user with immune-active, HBeAg-positive, chronic


hepatitis B was treated with tenofovir disoproxil fumarate (TDF). After 6 months of
treatment, he had normal liver chemistries and undetectable HBV DNA. He was still
HBeAg-positive. Shortly after, he ran out of TDF and went about 6 months without
medication before returning to the office with malaise, jaundice, and right upper
quadrant tenderness. His current labs are ALT of 350, AST of 240, total bilirubin of
4 mg/dL, and HBV DNA of 25 million IU/mL. He continues to be HBeAg-positive.
Are patients with chronic hepatitis B able to come off antiviral therapy
without risk of relapse?
It is clear that our patient relapsed after treatment interruption of tenofovir.
Patients who are HBeAg-positive can potentially stop nucleos(t)ide analog (NA)
therapy after a finite duration of treatment. HBeAg-positive chronic hepatitis B
patients being considered for treatment discontinuation should, at minimum, have
achieved HBeAg seroconversion (i.e., HBeAg loss with appearance of anti-HBe),
normal liver chemistries, and undetectable HBV DNA. They should then continue
NA therapy for an additional 12 months (consolidation therapy) before NA
discontinuation.
Liver chemistries, HBV DNA, and signs and symptoms of hepatic decompensa-
tion should be followed very closely (i.e., every 3 months) for the first year after
treatment discontinuation. Available data indicate that, at 12 months after treatment
discontinuation, initially HBeAg-positive patients will remain in virologic remis-
sion, biochemical remission, and HBeAg-negative in 60%, 75%, and 90% of cases,
respectively [1].

J. Esteban (*)
Division of Gastroenterology and Hepatology, Department of Medicine, Medical College of
Wisconsin, Milwaukee, WI, USA
e-mail: [email protected]

© The Author(s), under exclusive license to Springer Nature 391


Switzerland AG 2023
W. H. Sobin et al. (eds.), Managing Complex Cases in Gastroenterology,
https://doi.org/10.1007/978-3-031-48949-5_92
392 J. Esteban

The AASLD recommends that patients who are HBeAg-negative should remain
on NA therapy indefinitely or until HBsAg is lost, based on data indicating high
rates of virologic (60%) and biochemical (45%) relapse [1, 2]. However, there is
evidence that NA therapy can be safely discontinued in select HBeAg-negative
patients with reasonably low risk of virologic relapse and hepatitis flare, especially
if patients have undetectable HBV DNA for at least 2–3 years while on treatment [1,
3]. As with HBeAg-positive patients, these patients should undergo frequent moni-
toring after stopping NA therapy. Interestingly, HBeAg-negative patients who stop
NA therapy may have a greater chance for HBsAg loss (up to 30%) if quantitative
HBsAg titers are <1000 IU/mL at treatment discontinuation [4, 5]. Unfortunately,
quantitative HBsAg testing is not widely available in the United States.
Patients with cirrhosis, irrespective of HBeAg status, should remain on antiviral
treatment indefinitely due to increased risk of fatal acute-on-chronic liver failure.
How would you manage this situation? Do you restart the same antiviral or
do you need to start something different because of concerns about resistance?
Since he continues to have immune-active, HBeAg-positive, chronic hepatitis B,
I will retreat and resume tenofovir. Entecavir and tenofovir are NAs with high
genetic barriers to resistance, and reported antiviral resistance to these agents is
uncommon (1% or less) [6]. The likelihood of resistance is proportionate to the
duration of treatment, and our patient has only been on treatment for a few months.
Antiviral-resistant viruses are in general also “less fit” for replication than their
wild-type counterparts.
If he develops virological failure to tenofovir, defined as less than tenfold reduc-
tion in HBV DNA at 3 months of treatment, or virological breakthrough, defined as
tenfold or more increase from the lowest HBV DNA level, then I would first ascer-
tain that he is adhering to treatment. I will obtain HBV genotype and antiviral resis-
tance testing only after I am able to determine that he is adherent.
Switching to entecavir is the next step if he is tenofovir-resistant. Conversely,
switching to tenofovir is the next step if a patient is entecavir-resistant. Entecavir
should not be used if a patient is resistant to lamivudine or telbivudine. Combination
therapy with entecavir and tenofovir is typically not needed, since the individual
agents are highly effective on their own, but should be considered in cases of docu-
mented multidrug resistance or persistent plateauing viremia after 12 months or
more of treatment.
If our patient was HBeAg-negative at the start of NA treatment, how would
you manage the situation?
Experts recommend delaying retreatment of relapse in HBeAg-negative patients
for as long as possible, because this potentially increases the likelihood of spontane-
ously losing HBsAg and achieving functional cure [4, 5, 7]. These patients should
be followed up very closely. Some indications for retreatment include severe (e.g.,
ALT >10× ULN for more than 1 week) or persistent (e.g., ALT >2× ULN for more
than 3 months) hepatitis flare or jaundice [5, 7]. My choice of NA therapy and deci-
sion to pursue antiviral resistance testing would be similar to HBeAg-positive
patients.
92 HBV Treatment Discontinuation 393

References

1. Papatheodoridis G, Vlachogiannakos I, Cholongitas E, Wursthorn K, Thomadakis C, Touloumi


G, et al. Discontinuation of oral antivirals in chronic hepatitis B: a systematic review.
Hepatology. 2016;63(5):1481–92.
2. Terrault N, Lok ASF, McMahon BJ, Chang KM, Hwang JP, Jonas MM, et al. Update on pre-
vention, diagnosis, and treatment of chronic hepatitis B: AASLD 2018 hepatitis B guidance.
Hepatology. 2018;67(4):1560–99.
3. European Association for the Study of the Liver. EASL 2017 clinical practice guidelines on the
management of hepatitis B virus infection. J Hepatol. 2017;67(2):370–98.
4. van Bömmel F, Berg T. Risks and benefits of discontinuation of nucleos(t)ide analogue treat-
ment: a treatment concept for patients with HBeAg-negative chronic hepatitis B. Hepatol
Commun. 2021;5(1):1632–48.
5. van Bömmel F, Stein K, Heyne R, Petersen J, Buggisch P, Berg C, et al. A multicenter
randomized-­controlled trial of nucleos(t)ide analogue cessation in HBeAg-negative chronic
hepatitis B. J Hepatol. 2023;78(5):926–36.
6. Tenney DJ, Rose RE, Baldick CJ, Pokornowski KA, Eggers BJ, Fang J, et al. Long-term
monitoring shows hepatitis B virus resistance to entecavir in nucleoside-naïve patients is rare
through 5 years of therapy. Hepatology. 2009;49(5):1503–14.
7. Hadziyannis SJ, Sevastianos V, Rapti I, Vassilopoulos D, Hadziyannis E. Sustained responses
and loss of HBsAg in HBeAg-negative patients with chronic hepatitis B who stop long-term
treatment with Adefovir. Gastroenterology. 2012;143(3):629–36.
Chapter 93
Differentiating Acute HBV from an Acute
Exacerbation of Chronic HBV

James Esteban

A patient presents with fever and elevated transaminases—ALT of 350, AST of 260,
and total bilirubin of 5 mg/dL. He has no past history of hepatitis but did use intra-
venous heroin several times, about 10 years ago. One month before the presenta-
tion, he got a tattoo in a parlor. Labs show a positive hepatitis B surface antigen
(HBsAg), positive hepatitis B core antibody (anti-HBc) IgM and total, and elevated
HBV DNA of greater than 20 million IU/mL.
Are you able to say if this is a case of acute hepatitis B infection or an exac-
erbation of chronic hepatitis B infection?
Chronic hepatitis B is defined as the persistence of HBsAg or positive HBV
DNA for at least 6 months [1, 2]. Without prior HBV serologic testing, we have not
established HBV chronicity for this patient. I would make sure to look for signs of
liver disease chronicity and portal hypertension on the physical exam and abdomi-
nal ultrasound.
The presence of anti-HBc IgM is a strong indicator of acute hepatitis B infection,
but some patients with chronic hepatitis B who are experiencing an acute flare-up or
exacerbation may also be anti-HBc IgM-positive.
Would you start antiviral therapy?
First, I would examine him for hepatic encephalopathy, obtain hepatitis B enve-
lope antigen (HBeAg), and antibody (anti-HBe), check his international normalized
ratio (INR), and obtain abdominal ultrasound to determine if he has cirrhosis and
portal hypertension.

J. Esteban (*)
Division of Gastroenterology and Hepatology, Department of Medicine, Medical College of
Wisconsin, Milwaukee, WI, USA
e-mail: [email protected]

© The Author(s), under exclusive license to Springer Nature 395


Switzerland AG 2023
W. H. Sobin et al. (eds.), Managing Complex Cases in Gastroenterology,
https://doi.org/10.1007/978-3-031-48949-5_93
396 J. Esteban

If he has acute liver failure, defined as severe liver injury with hepatic synthetic
dysfunction (coagulopathy, INR > 1.5), and hepatic encephalopathy within 26 weeks
of disease onset, in a person without a history of cirrhosis, I will start antiviral
therapy in addition to getting him evaluated for liver transplantation.
If he does not have acute liver failure, I would follow him up closely and obtain
liver chemistries and INR frequently (every 1–2 weeks for 1–2 months). If he has
persistent hyperbilirubinemia (total bilirubin > 3 mg/dL) or coagulopathy
(INR > 1.5), which indicate severe disease, for greater than 4 weeks, then I will start
antiviral therapy because he has a protracted course of severe acute HBV infection.
The preferred antiviral therapies are the nucleos(t)ide analogs entecavir, tenofo-
vir disoproxil fumarate, and tenofovir alafenamide. Interferon is contraindicated in
acute hepatitis B infection due to risk of exacerbating liver inflammation.
If he has neither of these, then close monitoring and expectant management are
appropriate. More than 95% of immunocompetent patients with acute hepatitis B
infection will recover spontaneously and lose HBsAg over time. In a large clinical
trial of patients with severe acute hepatitis B infection, lamivudine was no better
than placebo in terms of biochemical improvement and rates of HBsAg loss (both
~93% at 12 months) [3].
He is HBeAg-positive and anti-HBe-negative. His INR is 1.3. On exam, he was
alert and oriented. He did not have spider angiomas, abdominal distention, fluid
wave, or asterixis. Abdominal ultrasound shows a normal-appearing liver, without
splenomegaly or ascites. You decided that he does not require immediate initiation
of antiviral therapy. You monitor his liver chemistries and INR weekly. In week 4, his
liver chemistries showed the following: ALT of 120, AST of 100, total bilirubin of
1.8 mg/dL, and INR of 1.2.
How would you follow him up?
I would continue following his liver chemistries, and I will, in addition, be fol-
lowing his HBsAg, anti-HBs, HBeAg, anti-HBe, and HBV DNA periodically
because we need to know if he will “clear” the HBV infection or if he will go on to
develop chronic hepatitis B infection. I will obtain hepatitis B serologies at 3 and/or
6 months. If he remains HBsAg positive at 6 months or more, then we have estab-
lished that he has chronic hepatitis B infection. Management of chronic hepatitis B
is discussed in other chapters in the book.

References

1. Terrault N, Lok ASF, McMahon BJ, Chang KM, Hwang JP, Jonas MM, et al. Update on pre-
vention, diagnosis, and treatment of chronic hepatitis B: AASLD 2018 hepatitis B guidance.
Hepatology. 2018;67(4):1560–99.
2. European Association for the Study of the Liver. EASL 2017 clinical practice guidelines on the
management of hepatitis B virus infection. J Hepatol. 2017;67(2):370–98.
3. Kumar M, Satapathy S, Monga R, Das K, Hissar S, Pande C, et al. A randomized controlled
trial of lamivudine to treat acute hepatitis B. Hepatology. 2007;45(1):97–101.
Chapter 94
Acute HBV and Risk of HBV Reactivation

James Esteban

A 25-year-old male gets acute hepatitis B from a sexual encounter. He completely


clears the infection spontaneously, without treatment.
Is there a need for extended follow-up of any kind?
Clearance of hepatitis B surface antigen (HBsAg) and formation of hepatitis B
surface antibody (HBsAb) defines functional cure. Loss of HBsAg is associated
with durable suppression of viral replication. Thus, he will not require extended
follow-up [1].
What if, 25 years from now, he needs to receive rituximab-based regimen for
lymphoma, will he need special precautions?
After entry into the hepatocyte, hepatitis B virus (HBV) translocates into the
nucleus, where the double-stranded HBV DNA (dsDNA) is repaired into an
extremely stable covalently closed circular DNA (cccDNA). cccDNA persists in
hepatocytes long after loss of HBsAg. HBV dsDNA may also integrate with the
human genome as HBV integrated or iDNA. The persistence of cccDNA and iDNA
in hepatocytes, despite complete suppression of HBV replication and even loss of
HBsAg, is the reason why true virologic cure is not achievable in hepatitis B infec-
tion with current medical therapies.
Immunosuppressive therapy may release immunologic control over HBV, lead-
ing to resumption of transcription of HBV cccDNA and/or iDNA. This leads to
reappearance of HBsAg and HBV DNA in serum and hepatitis flares in some cases.
This is called HBV reactivation. Risk of HBV reactivation varies by intensity of
immunosuppression. B-cell depleting agents such as rituximab, or prolonged use of

J. Esteban (*)
Division of Gastroenterology and Hepatology, Department of Medicine, Medical College of
Wisconsin, Milwaukee, WI, USA
e-mail: [email protected]

© The Author(s), under exclusive license to Springer Nature 397


Switzerland AG 2023
W. H. Sobin et al. (eds.), Managing Complex Cases in Gastroenterology,
https://doi.org/10.1007/978-3-031-48949-5_94
398 J. Esteban

Table 94.1 Risk of HBV reactivation according to immunosuppressive agent and HBsAg status
HBsAg+/ HBsAg−/
HBcAb+ HBcAb+
B-cell depleting therapy (e.g., rituximab, belimumab, High High
alemtuzumab, etc.)
High-dose steroids (pred >20 mg/day × >4 weeks) High Moderate
Med-dose steroids (pred 10–20 mg/day × >4 weeks) Moderate Low
Low-dose steroids (pred <10 mg/day), intra-articular steroids Low Low
Anthracycline-based systemic chemotherapy High Moderate
Non-anthracycline systemic chemotherapy Moderate Moderate
Immune checkpoint inhibitors (e.g., pembrolizumab, High Moderate
nivolumab, ipilimumab)
Anti-TNF agents (e.g., infliximab, adalimumab, High Moderate
certolizumab, golimumab)
Cytokine therapies (e.g., ustekinumab, abatacept, Moderate Moderate
natalizumab, vedolizumab)
TK inhibitors (e.g., imatinib, nilotinib) Moderate Moderate
Proteosome inhibitors (e.g., bortezomib) Moderate Moderate
Histone deacetylase inhibitors (e.g., romidepsin) Moderate Moderate
Calcineurin inhibitors (e.g., tacrolimus, cyclosporine) Moderate Moderate
Antimetabolites (e.g., 6-MP, azathioprine, methotrexate, Low Low
mycophenolate)
High risk, >10% risk of HBV reactivation; Moderate risk, 1–10% risk of HBV reactivation; Low
risk, <1% risk of HBV reactivation

high-dose corticosteroids, are associated with the highest risk of HBV reactivation
(>10%). Patients with a history of hepatitis B infection, i.e., HBsAg and hepatitis B
core antibody (HBcAb) positive, who will be receiving high-risk immunosuppres-
sive agents (Table 94.1), should be provided with antiviral prophylaxis with
nucleos(t)ide analogs such an entecavir, tenofovir disoproxil fumarate, or tenofovir
alafenamide to prevent HBV reactivation [2, 3].
Some immunosuppressive agents, such as tumor necrosis factor inhibitors or
cytokine inhibitors, have a lower, but not negligible, risk of HBV reactivation. Some
patients who will be receiving these agents may be offered on-demand therapy
(Fig. 94.1), where we periodically check (typically every 3 months) HBV DNA,
transaminases, and HBsAg for HBV reactivation or a hepatitis flare (i.e., ALT >3×
upper limit of normal). Patients would then be promptly treated with nucleos(t)ide
analogs upon diagnosis of HBV reactivation.
In patients who are HBsAg positive, reactivation may be defined as:
• 100× increase in HBV DNA
• HBV DNA 1000 IU/mL if previously undetectable
• HBV DNA 10,000 IU/mL if previous level unknown
In patients who are HBsAg negative and HBcAb positive, reactivation may be
defined as:
• Any detectable HBV DNA
• Reverse seroconversion to HBsAg positive
94 Acute HBV and Risk of HBV Reactivation 399

Fig. 94.1 Algorithm for prevention and management of HBV reactivation in patients receiving
immunosuppressive therapy

References

1. Terrault N, Lok ASF, McMahon BJ, Chang KM, Hwang JP, Jonas MM, et al. Update on pre-
vention, diagnosis, and treatment of chronic hepatitis B: AASLD 2018 hepatitis B guidance.
Hepatology. 2018;67(4):1560–99.
2. Reddy KR, Beavers KL, Hammond SP, Lim JK, Falck-Ytter Y, AGA Institute. American
Gastroenterological Association Institute guideline on the prevention and treatment of
hepatitis B virus reactivation during immunosuppressive drug therapy. Gastroenterology.
2015;1480:215–9.
3. Loomba R, Liang TJ. Hepatitis B reactivation associated with immune suppressive and bio-
logical modifier therapies: current concepts, management strategies, and future directions.
Gastroenterology. 2017;152(6):1297–309.
Chapter 95
Complicated HCV

Kia Saeian

Patient is a 35-year-old female with a history of IV drug abuse from age 18–25 who
comes in with jaundice and weakness. She is overweight with a BMI of 33 and has
no history of significant alcohol use. On exam, she has no stigmata of chronic liver
disease. Her liver enzymes are as follows: ALT of 240, AST of 150, bilirubin of 3.2,
and Alk phos of 120. Her US shows marked steatosis of the liver. Hepatitis panel is
positive for HCV ab, and her HCV PCR is (+) 1.2 million. ANA returns positive
at 1:640.
What is your impression, and how would you manage this patient?
One of the first things I would like to do is check the gamma globulin level to
corroborate the positive ANA and substantiate the impression of AIH. I would con-
sider doing a liver biopsy. There would be two reasons to consider a liver biopsy.
First, would be to confirm a diagnosis of AIH, assuming her IgG level comes back
elevated. Personally, I like to have a biopsy confirmation of AIH before initiating
therapy, although not everyone agrees.
The second would be to evaluate for fibrosis from HCV. Since her HCV exposure
was probably over 10 years ago, there is a good chance she could have fibrosis from
the chronic HCV. Most of the time with HCV we don’t need a liver biopsy, we sim-
ply treat the HCV, but in this case, we might.
An interesting question in this case is whether the patient has two distinct disor-
ders—both HCV and AIH (as well as potentially NAFLD). This question comes up
because some patients with HCV may have a positive ANA and features of autoim-
mune disease without having AIH. The distinction used to be much more of a quan-
dary when we used interferon to treat HCV, because interferon would dramatically
exacerbate AIH.

K. Saeian (*)
Division Gastroenterology and Hepatology, Department of Medicine, Medical College of
Wisconsin, Milwaukee, WI, USA
e-mail: [email protected]

© The Author(s), under exclusive license to Springer Nature 401


Switzerland AG 2023
W. H. Sobin et al. (eds.), Managing Complex Cases in Gastroenterology,
https://doi.org/10.1007/978-3-031-48949-5_95
402 K. Saeian

Now in this case, even if the patient has AIH, I don’t think it is urgent that we
start corticosteroids immediately, seeing how there is only a moderate level of trans-
aminase elevation. You can argue to hold off on biopsy, treat the HCV first, and see
what happens with the autoimmune findings.
The other interesting question about this case is the interplay between HCV and
NAFLD. I would check the HCV genotype, and if it is genotype 3, it might also
explain the degree of fat in the liver. Her BMI alone is probably adequate to explain
it, but genotype 3 has been associated with more significant steatosis.
You talk about fibrosis, if you weren’t going directly to biopsy, would you do
a FibroScan®?
I do use the FibroScan®, although I do not use it as liberally as some. But the fact
is that it’s available and it’s pretty easy to do. Now I perform a significant proportion
of the liver biopsies done in our institution, and I have a healthy appreciation for the
potential complications associated with liver biopsy. Therefore, I don’t rush into
doing liver biopsies as quickly as some others do. So, if the only reason for doing a
liver biopsy is to establish the diagnosis of fibrosis, I would try FibroScan® first.
However, there are confounding issues, like obesity, that can make FibroScan®
less reliable. In the aforementioned patient, because she has a BMI of 33, FibroScan®
would be of limited benefit. But if I have a case of chronic HCV (uncomplicated by
autoimmune features), where the patient is not obese and doesn’t have other con-
founding factors, I have a very low threshold to spend the few minutes doing the
FibroScan®. Then, if I don’t see any indication of fibrosis, I can treat their hepatitis
C and not need to arrange for follow-up, because I’m comfortable that fibrosis is
absent, and surveillance for HCC unnecessary.
What about the utility of FibroScan® in congestive heart failure? What if
you have a patient with congestive heart failure and elevated liver enzymes,
will you get a FibroScan® in that setting?
I would absolutely not use it in that setting because you’re likely to get a false
reading. If you look at the concept of elastography, it’s based on elasticity of the
tissue, and if you have elevated back pressures from CHF, pulmonary hypertension,
or hepatic vein obstruction, you lose that elasticity. Whether or not fibrosis is pres-
ent, the FibroScan® will imply advanced fibrosis.
Chapter 96
Acute Liver Failure

Kia Saeian

A patient presents with acute liver failure. A 27-year-old female college student,
who had complained to her roommate that she felt weak and nauseated for several
days, is now found lying in bed in her dorm room, extremely lethargic. The room-
mate calls her parents who urge her to call an ambulance. She is sent to the ER. On
the physical exam, she is responsive but keeps drifting off to sleep. Her BP is 90/70,
pulse is 90, and temperature is 99. She has no signs of portal hypertension on exam.
Liver and spleen are not palpable.
WBC, 12,000; Hbg 11.5; platelets, 180,000; AST, 1700; ALT, 2200; ALK phos,
140; Bili, 8; INR, 2.5
How would you address this problem?
This pattern of enzymes is suggestive of zone three necrosis, and a common
cause of this is acetaminophen overdose, even though there isn’t a history of her
taking that. The other things you would think about in a young woman like this (not
necessarily involving zone 3 necrosis) are acute presentations of autoimmune hepa-
titis, other drug induced injuries, or an unknown viral infection. Those are the most
likely explanations for her presentation.
The fact that she is already presenting with synthetic dysfunction and encepha-
lopathy qualifies her as having acute liver failure, so she absolutely must be at a
transplant center. Even if you think she has autoimmune hepatitis and you’re going
to treat her, you absolutely need to be prepared for the possibility of transplantation,
and that needs to be done pronto because these patients can deteriorate very rapidly.

K. Saeian (*)
Division of Gastroenterology and Hepatology, Department of Medicine, Medical College of
Wisconsin, Milwaukee, WI, USA
e-mail: [email protected]

© The Author(s), under exclusive license to Springer Nature 403


Switzerland AG 2023
W. H. Sobin et al. (eds.), Managing Complex Cases in Gastroenterology,
https://doi.org/10.1007/978-3-031-48949-5_96
404 K. Saeian

Now, we’ve had some patients present like this who turned out to have autoim-
mune hepatitis who we’ve been able to treat and have them recover without trans-
plant. But once they’ve developed encephalopathy, it’s more unlikely that they will
turn around and improve quickly enough to avoid listing them.
We have a specific liver failure protocol in which we screen for the routine viral
hepatidites—HAV, HBV, HCV. We also include lab tests looking for alternative
viruses like CMV, EBV, and hepatitis E. In addition, we would certainly do toxicol-
ogy screens and check the autoimmune markers.
We also monitor the AFP because it can be a marker of the ability of the liver to
regenerate. Factor V can also be a marker so we monitor that as well. If the patient
becomes acidotic, that is a poor prognostic factor and we get even more concerned.
So that’s our lab work.
In terms of the imaging studies, we get a CT scan of the head to make sure there
is no midline shift or cerebral edema, which would necessitate more aggressive
measures, including hypernatremia, hypothermia, and occasional osmotic diuresis
with mannitol, although we don’t use that or hyperventilation as frequently as we
used to.
We would consider initiating n-acetyl-cysteine (NAC) even in the absence of
known acetaminophen overdose. The evidence is less robust in ALF not related to
acetaminophen, but there is some decent data that it can help in that setting.
Directing our attention to the alkaline phosphatase, it is not elevated in propor-
tion to the transaminases in this patient. In cases of viral hepatitis and autoimmune
hepatitis, usually the alkaline phosphatase elevation is proportional to the transami-
nase elevation. But it is possible that the alk phos is on a slower upward course, it’s
still early on, and it could take longer for that alk phos to get transcribed.
The elevated bilirubin/alkaline phosphatase ratio raises the possibility of Wilson’s
disease. However, if this was Wilson’s, I would expect her Hbg to be lower, and the
alkaline phosphatase might be even lower than it is in this case. Regardless, if this
was acute Wilson’s, you’re probably headed for transplant.
However, in cases of zone 3 necrosis of the liver, the transaminases tend to be
proportionally higher than the alkaline phosphatase. Other things that could cause
zone 3 injury include hypotension and heat stroke. Heat stroke can sometimes fool
you, although in this case, the AST being lower than the ALT makes that less likely.
Would you start steroids in this patient with acute liver failure?
We are reluctant to start steroids. Our transplant surgeons prefer to avoid steroids
in a patient headed for transplant. We would certainly wait for our autoimmune
markers to come back. But even if the ANA came back elevated, I would want to see
an elevated IgG as well, because we do see elevated ANA in a number of cases that
are not autoimmune liver disease. And even with an elevated ANA and IgG, I would
prefer to have a liver biopsy before starting steroids for AIH.
Now if this patient was somewhat more stable, not encephalopathic, and time
was more on our side, I would consider doing a liver biopsy. We have done tran-
sjugular liver biopsies in some patients with an INR in this range. If a biopsy con-
firmed the diagnosis of AIH, I would be much more inclined to start steroids.
Chapter 97
Delayed Presentation of Tylenol Overdose

Kia Saeian

A patient with a history of taking a large dose of Tylenol 5 days earlier is brought
into the hospital now. She is lethargic on presentation, ALT is 3000, AST is 2400,
INR is 2.5, bilirubin is 2.2, and Alk phos is 140.
Is there any role for NAC at this late point?
I think almost everyone would give it, even 5 days after the overdose. The ques-
tion of whether it will benefit the patient is a valid one, because we don’t have many
studies on initiating NAC in this subset of patients who are presenting this late in the
clinical course. One thing about this case is that it’s unusual for the numbers to be
this high on day five. The numbers tend to peak on day three, so it is possible that
the transaminases might have been higher, say even 10,000, and are now trending
down. The patient may be recovering from the injury, but the fact that the numbers
decrease is not necessarily a good sign. It could be a sign of fulminant hepatic
necrosis.
The traditional teaching now is that you give NAC until the transaminases drop
below 1000. I would treat her as someone we might possibly be able to salvage, so
in this patient, even though it may be too late, we would start NAC and continue
until the numbers dropped to at least below 1000. The one reassuring thing in this
case is that the bilirubin is not extremely high. Treatment with NAC is generally
very safe, but some patients do develop nausea and vomiting, particularly with the
oral formulation.

K. Saeian (*)
Division of Gastroenterology and Hepatology, Department of Medicine, Medical College of
Wisconsin, Milwaukee, WI, USA
e-mail: [email protected]

© The Author(s), under exclusive license to Springer Nature 405


Switzerland AG 2023
W. H. Sobin et al. (eds.), Managing Complex Cases in Gastroenterology,
https://doi.org/10.1007/978-3-031-48949-5_97
406 K. Saeian

There are some centers that use fomepizole to treat severe episodes of acetamin-
ophen toxicity. Fomepizole is a competitive inhibitor of alcohol dehydrogenase,
which inhibits formation of toxic acetaminophen (APAP) metabolites; I have never
used it, but based on its mechanism of action, it would make sense to use it in those
who have concomitant acetaminophen and alcohol toxicity.
Chapter 98
Candidates for Liver Transplant?

Veronica Loy

A GI fellow who left your training program to join a group GI practice several
weeks ago asks to set up a meeting. He took over the practice of a retiring gastroen-
terologist and wants to discuss management of several patients. There were four
patients with ESLD who he thinks may warrant liver transplant evaluation. The
practice is located in a rural area, one and a half hours away from your academic
transplant center.
Do you think any of these patients are liver transplant candidates?
1. A 58-year-old male with alcoholic cirrhosis who has been abstinent 3 years is
living in a shelter. He has been treated for encephalopathy, recurring ascites,
and varices. He takes his meds as prescribed and manages to have a friend drive
him to the hospital for all his office visits and procedures. He is on public assis-
tance. The former doctor did not refer him for transplant evaluation because of
his lack of social and financial support.
This is a difficult case. I think that the first concern is both financial and social
support, and we need to consider why those are so important in the posttrans-
plant period. As we know, if the patient does not have access to the immunosup-
pressive medications for life, then the organ will be rejected. If that’s the case,
you’ve done the patient a disservice as well as the organ donor and another
potential recipient did not receive the organ. It is critical that we have a multidis-
ciplinary approach to assessing both financial support and social support. Our
program has our own financial liaison as well as a social worker, who hone in,
and try every avenue, to make sure that they can assure coverage for these
medicines.

V. Loy (*)
Division of Gastroenterology and Hepatology, Department of Medicine, Medical College of
Wisconsin, Milwaukee, WI, USA
e-mail: [email protected]

© The Author(s), under exclusive license to Springer Nature 407


Switzerland AG 2023
W. H. Sobin et al. (eds.), Managing Complex Cases in Gastroenterology,
https://doi.org/10.1007/978-3-031-48949-5_98
408 V. Loy

In this patient’s case, he has been taking his prescribed meds, he must have
public assistance, and Medicaid usually will cover these meds. Coverage and
compliance don’t seem to be a problem for him, so that bodes well.
However, the second thing we have to worry about is the infection risk. And
unfortunately, staying in shelters, most of the time, is something that the trans-
plant committee is really going to struggle with as far as reliable housing and
infection risk. It’s even a struggle for patients living in a nursing home because
of an increased risk of posttransplant infection. Typically, we prefer people that
have individualized housing. I think every transplant center is probably a little
different on how they would view this case. I’m pretty confident that in our cen-
ter living in a shelter would not be considered reliable housing.
What makes things more complicated is that there is a recent law about dis-
crimination and access to an organ transplant. Transplant programs must be
careful with the reasons for declining to list someone for transplant. The psycho-
social issues, especially for people with mental health or developmental delays,
are really challenging. There have been lawsuits when some of these patients
were declined. In this patient’s case, infection risk of rejection is a realistic rea-
son for declining to list the patient.
2. A 57-year-old female has NASH cirrhosis. She has had treatment for esophageal
varices and recurring ascites. Her BMI is 35. The previous doctor pushed her to
diet and lose weight. He insisted that she lose at least 20 lbs. She was not having
much success, losing only about 5 lbs. The doctor did not refer her for liver
transplant, thinking that she was not a candidate because of her obesity.
This is a very timely topic. What we know about organ transplant is that the
extremes of weight don’t do as well. The super obese or the very sarcopenic
patients don’t do well. In this case, the patient has a BMI of 35; that’s not a big
problem, and that patient certainly should be considered as a transplant candi-
date. Once you’re heading towards a BMI of 40, 45, or 50, that becomes more
challenging and controversial. It depends on where the person holds their weight.
If it’s all abdominal fat, that is more of a problem, especially if it’s a dual organ
transplant, like a liver-kidney, because the abdominal weight may cause prob-
lems with the kidney. But if they’re carrying all their weight in their legs and
their glutes, then that’s not really prohibitive to transplant. There are some really
good studies looking at outcomes in obese patients after transplant and they do
quite well as long as they’re not in the far extremes. In fact, obese patients tend
to do better than sarcopenic patients who have no muscle mass, very low BMIs,
and poor nutrition. The outcomes relate mostly to nutrition and functional status.
If you have a morbidly obese patient that has good nutrition and a good func-
tional status, that’s helpful, but many centers are doing bariatric surgery along
with the transplant if their weight is extreme. Some do it in the pre-op period,
some at the time of transplant, and others after they’ve recovered from the trans-
plant. There’s a little bit of controversy over which is the best way to do it. But a
lot of people are looking into bariatric surgery as it relates to liver transplant,
particularly in NASH patients.
98 Candidates for Liver Transplant? 409

The Mayo Clinic has done a really nice study where they compared patients
who, like this patient, were told they had to lose weight prior to transplant. And
those patients did lose weight and were transplanted, and they were compared
with a group of patients who had bariatric surgery and then were transplanted. It
turned out that those patients who had bariatric surgery and then were trans-
planted actually did much better than those that lost weight on their own and
then got their transplant. That group tended to gain back a lot of their weight
after transplant. And progression to NASH cirrhosis posttransplant happens very
quickly in those who gain their weight back. So, I think, as we move forward in
the next decade, it probably will become the standard of care that these patients
with NASH cirrhosis have some sort of bariatric procedure along with their
transplant.
3. A 49-year-old male alcoholic presented with a GI bleed and required an ICU
admission with banding of varices. He was discharged from the hospital, but
over the next 3 months, he has had repeat admissions with banding and then
bleeding from post-banding ulcers, ascites, and SBP. He has also had encepha-
lopathy controlled with lactulose. He has been abstinent since the first admis-
sion. The previous doctor would not refer him for transplant because he has only
been abstinent for 3 months.
I am really passionate about this one. Using duration of sobriety or abstinence
alone has been shown time and time again to not predict recidivism posttrans-
plant. And so, in this patient, I would absolutely encourage referring to a trans-
plant center. And at that center, there’ll be a multidisciplinary approach with a
psychology evaluation and a social work evaluation and really assess this
patient’s commitment to lifelong sobriety and their social support. Many centers
are integrating alcohol and drug addiction counseling along with their clinic
visits, and that’s been shown to improve the chances of patients having lifelong
sobriety.
It’s very important not just to use time alone as a reason not to refer someone
for transplant. It doesn’t predict posttransplant alcohol use. Even those who have
6 months or a year of sobriety have the same recidivism rate as those who have
no sobriety. So, time should really not be a factor.
4. A 72-year-old female with NASH cirrhosis has ESLD. She has encephalopathy
requiring lactulose and rifaximin. She has had a prior TIPS procedure for vari-
ceal bleeds and gastric varices. Because the encephalopathy is hard to control,
the TIPS needed to be narrowed, and now the patient has chronic GI bleeding
from GAVE. The previous doctor would not refer her for liver transplant evalua-
tion because he thought she was too old.
Once again, similar to our last discussion, this is a misconception that many
people have; it’s really an outdated thought. There is actually no age cutoff for
transplant. Most transplant centers look at the physiological age, so the health of
the patient, their functional status, their cardiac comorbidities, and pulmonary
comorbidities are far more important than the age alone. So, this patient, depend-
ing on her other medical comorbidities could certainly be considered for transplant.
Chapter 99
Frustrations with Waiting for a Transplant

Veronica Loy

A 57-year-old male with alcoholic cirrhosis is referred for liver transplant evalua-
tion. He was diagnosed 2 years earlier and has been abstinent since. He was found
on index endoscopy to have esophageal varices, which were banded and remain
obliterated on a recent endoscopy. He has been plagued with recurring ascites.
Initially diuretics were used but when doses needed to be increased, he developed
renal insufficiency. He now comes in for weekly paracentesis during which 5–7
liters of fluid is usually removed. He has had one episode of SBP and remains on
prophylactic ciprofloxacin. He has had several episodes of hepatic encephalopathy:
the first when his creatinine was elevated and the second when he had SBP. He was
placed on lactulose, which he takes regularly, except when he is in situations where
he needs to avoid having diarrhea. When he doesn’t take his lactulose, he gets very
sleepy. His INR is 1.9, albumin is 3.4, bilirubin is 1.3, AST is 30, ALT is 50, Alk phos
is 90, creatinine is 1.2, Na is 136, and MELD is 16. His local gastroenterologist tells
him that with his MELD score, he probably won’t qualify for a liver transplant for
a while.
1. How do you deal with patients who are suffering from complications of cir-
rhosis but can’t qualify for a transplant for protracted periods because
their MELD is not high enough?
This is an ongoing, common problem. But I would advocate for evaluation
and, if possible, listing for transplant. We understand that with the MELD score
of 16, they’re not likely to get an organ quickly. However, the medical condition
can deteriorate very quickly for these patients. We know that people with diuretic

V. Loy (*)
Division of Gastroenterology and Hepatology, Department of Medicine, Medical College of
Wisconsin, Milwaukee, WI, USA
e-mail: [email protected]

© The Author(s), under exclusive license to Springer Nature 411


Switzerland AG 2023
W. H. Sobin et al. (eds.), Managing Complex Cases in Gastroenterology,
https://doi.org/10.1007/978-3-031-48949-5_99
412 V. Loy

resistant or diuretic refractory ascites have a 50% 2-year mortality, which is very
high. We tend to see them develop SBP and liver decompensation, or they have
a bleed after a paracentesis and their liver decompensates, so these complica-
tions can happen very quickly. Therefore, the benefit of listing them early. I feel
that even though the MELD is on the low side, at 16, I advocate listing if it’s over
15. But sometimes, we see these patients with a MELD score of eight, and I still
think with diuretic resistant or refractory ascites or patients with varices that we
should try to get them on the transplant list because things change quickly and it
allows the patient and their family time to be educated and learn and understand.
And we can work on other things. Often, during the transplant evaluation, we
uncover other risks that we need to mitigate, like maybe they need to stop smok-
ing. And we can work on these things for the best results with transplant. Also,
if this patient did not have a history of encephalopathy, I would look into doing
a TIPS at this time, but because of the encephalopathy, I won’t.
Additionally, there are other avenues to help get a patient a liver transplant
sooner, such as living donor liver transplantation. A patient needs to be assessed
early to see if they might qualify for this.
In summary, I would try to get the patient listed, and then, I would continue
to do what this gastroenterologist is doing with the repeated paracentesis, low
sodium diet, and antibiotic prophylaxis.
2. What do you think about a system where the sicker, higher risk patients are
the ones who are having this aggressive operation? Wouldn’t you rather
your patient receives the transplant when they are less sick?
The system is not always logical but it’s meant to be fair. The people who
need the transplant the most have access to the transplant. However, we do know
that, in terms of posttransplant outcomes, there is a tipping point at which
patients have worse survival after transplant because they were so sick going into
the transplant. The ideal time to get a transplant is probably at a MELD of 24–30
instead of a MELD of 40. These patients are not as sarcopenic; they can recover
quicker. They don’t have as many complications, they’re not as
immunocompromised.
I agree it would be phenomenal to offer a transplant when a patient is less
sick, and I think we’re making some progress toward transplanting patients at a
lower MELD. Ideally, anyone who needs a transplant would have access to it,
but the real issue is an organ donor shortage. The demand far outstrips the
resources.
One way we’re making progress is by using other types of liver donations
besides just brain-dead donors. In some of these patients with the lower MELD
scores, there might be the option of using a living donor transplant, if they qual-
ify. However, patients with a lot of portal hypertension, like this one, are gener-
ally not ideal candidates for a living donor transplant, they often need a whole
donor graft.
Another potential source of livers is to use not only brain-dead donors but
also donations after cardiac death. Another group is the hepatitis C-positive
donors. We’re using them a lot more now because we can eradicate hepatitis C
99 Frustrations with Waiting for a Transplant 413

easily after the transplant. Another new thing that’s happened in the last couple
of years is the machine perfusion-donor livers. Here, they take organs that
­previously would have been discarded because of their questionable quality.
This may be the case if a liver had a lot of steatosis or the donor was elderly.
There is now available hypothermic and normothermic machine perfusion for
these donor organs, which can optimize the graft allowing for transplant in some
cases where previously they would’ve gone unused. The transplant surgeon will
take the donor liver and hook it up to this perfusion device, and they’re able to
monitor the liver and see if this liver will work in the recipient. Another tech-
nique is to do organ procurement the traditional way, then put the liver in the
cooler and get on the airplane, and then when they get back to their own hospital,
hook it up to a pump. This is something they’ve been doing with kidneys for
some time now.
3. What percentage of patients develop HCC or develop medical complica-
tions that make them inoperable while waiting to be transplanted?
This is very common because of problems waiting for access to a donor.
There is only so long that you can keep someone’s cancer within Milan criteria.
There’s only so long that you can keep someone with end-organ failure alive
before they get an infection that prohibits transplant.
Many patients will develop hemodynamic instability with which there’s no
way they could survive a transplant. What is the waiting list mortality rate? It’s
variable across the country, depending on access to grafts. But nationwide, I
would guess the wait list mortality rate is about 20%.
With HCC in particular, there have been some changes in organ allocation.
So, some patients who are listed with MELD exception points, depending on
where they live, may have better access to organs, while others have less access
to organs. We’ve seen that some of our patients with tumor exception points are
on our wait list for up to 200, even 300 days. But keeping cancer in check for a
year is quite challenging.
Chapter 100
Complications Posttransplant

Veronica Loy

A 57-year-old female receives a liver transplant for PBC. She is started on predni-
sone, tacrolimus, and mycophenolate. Within the first 3 months, she develops mul-
tiple medical problems. She has developed hypertension, which was not present
previously, she progressed from having prediabetes to diabetes, and her creatinine
has risen from 1.4 to 1.8.
Can you discuss the contribution of her antirejection meds to these different
medical problems and how you might manage the antirejection meds per. se.
When transplants were initiated in the late 1960s and early 1970s, the main cause
of death was acute cellular rejection. Things have come a really long way, thanks to
our superior antirejection meds. We have available calcineurin inhibitors, mycophe-
nolate, and azathioprine and prednisone. We are at a point where patients are living
a long time. The expectation is that survival at 10 years should be 70% or more.
Transplant rejection still remains a concern; up to 30% of patients may develop
rejection, but we can manage that much better now.
Now the major problems we are dealing with are the side effects from long-term
use of these antirejection medications. One concern is new cancers, and we encoun-
ter a lot of these, particularly skin cancers. Depending on the etiology of the patient’s
disease, we have to be very careful looking for other types of new malignancies. Of
course, infections are a big concern.
This case points out two other very challenging side effects of antirejection
meds, the metabolic syndrome and hypertension. Cardiovascular events, heart
attack and stroke, are all happening at a far greater rate in transplant recipients com-
pared with their nontransplant recipient counterparts.

V. Loy (*)
Division of Gastroenterology and Hepatology, Department of Medicine, Medical College of
Wisconsin, Milwaukee, WI, USA
e-mail: [email protected]

© The Author(s), under exclusive license to Springer Nature 415


Switzerland AG 2023
W. H. Sobin et al. (eds.), Managing Complex Cases in Gastroenterology,
https://doi.org/10.1007/978-3-031-48949-5_100
416 V. Loy

Additionally, data shows that at 5–10 years posttransplant, the rate at which peo-
ple have kidney disease, CKD3 or CKD4, approaches almost 70%. Historically, it’s
a very high percentage of people who receive the transplant of a heart or a liver who
ultimately end up needing to talk about going on dialysis or having a kidney
transplant.
The long-term effects of immunosuppression are a big problem. How can we
manage this? I think looking at strategies for minimizing immunosuppression is
very important. The level of immunosuppression that the patient is maintained on in
the immediate posttransplant period will predict how they do 10 years later. So, the
goal is to work on minimizing medication without putting the patient at risk of
rejection. There’s a lot of research in the area. The strategy used in this patient’s
case of using three drugs rather than using a high-dose calcineurin inhibitor alone is
an important step. With the calcineurin inhibitors, we’re worrying most about renal
insufficiency, but they also contribute to hypertension, hyperlipidemia, and diabe-
tes. By using mycophenolate and prednisone, we may be reducing the risk of kidney
damage, but there is a trade-off with some of these other problems and metabolic
conditions.
There’s also another class of agents, the mTOR inhibitors, which are sometimes
used with a strategy of minimizing the dose of calcineurin inhibitor. The hope is that
this will prevent renal insufficiency and may possibly help reduce the recurrence of
cancers. In this patient, I might have titrated some of the dosages of medicines she
was on to try to avoid some of these complications.
Now there is ongoing research about immunosuppression withdrawal. There are
ongoing trials to determine which patients could actually come off immunosuppres-
sion completely. I don’t think it’s yet at the point where I would feel comfortable
doing that, although some hepatologists have moved in this direction. I would work,
instead, on managing the comorbidities, trying to help the patient have better con-
trol of the hypertension, the diabetes, and the lipids. And ultimately, if renal func-
tion continues to decline, getting a nephrologist involved early on.
After 3 months, the patient comes in complaining of some increased fatigue and
weakness, and liver enzymes show a significant increase from her posttransplant
baseline. Her AST increased from 30 to 90, the ALT from 40 to 120, the Alk phos
from 100 to 160, and the bilirubin from 1.3–2.3. She is not taking any OTC meds—
nothing but what has been prescribed for her.
What are some possible explanations for this and how would you manage it?
You would certainly worry about rejection here, especially if you previously
decided to reduce immunosuppression. The other common problem I always want
to rule out in a patient like this is infection. CMV is very common; even far post-
transplant and any infection is going to cause a rise in the liver enzymes and perhaps
a cholestatic liver injury, which this patient has. Any infection, including c diff, is a
possibility, so I would screen for infection.
I also usually get some imaging to make sure that there’s no problem with the
bile duct. You know that’s very common posttransplant as well.
100 Complications Posttransplant 417

The other thing I would worry about in this patient is the possibility of PBC
recurrence posttransplant or even autoimmune hepatitis. The rate of recurrence of
PBC posttransplant is high.
In addition to rejection, I would work the patient up for infection, recurrent dis-
ease, any bile duct injury, or steatosis of the liver. For this patient, you’re probably
going to do a liver biopsy at some point. I wouldn’t send someone for a biopsy based
on one set of labs, but if the problem were to persist for a couple of weeks or even a
month or if the labs were quickly deteriorating, I would get this patient in for
a biopsy.
A liver biopsy is done that reveals lymphocytic infiltration within portal tracts,
mild inflammation of bile ducts, and lymphocytic infiltration of portal and hepatic
venules. A Doppler US shows normal blood flow without thrombosis but suggests
extrahepatic biliary dilation. An MRCP is done that shows a mild extrahepatic
stricture of questionable significance.
This looks like rejection, but the management of rejection depends on the sever-
ity of the rejection and which drugs and at what dosages the patient was on prior to
developing the rejection. The pathologist will generally give you the level of rejec-
tion based on the Banff criteria. They’re going to tell you, is this mild, moderate, or
severe rejection?
If it is mild, sometimes just increasing immunosuppression is enough. If a patient
was on cyclosporine only, then switching them to tacrolimus or adding mycopheno-
late would be my strategy for managing mild rejection. For moderate and severe
rejection, it’s going to take more than that. Usually, you’re going to need to use an
IV dosage of solumedrol, and if it’s refractory to that, sometimes even more aggres-
sive IV therapies for acute cellular rejection are required. And then, once they are
improved, you need to increase their outpatient regimen as well.
This patient’s case is tough because presumably she’s still taking prednisone,
tacrolimus, and mycophenolate. We’re not given the dosage, but you may have to
increase the prednisone and increase the trough level of the tacrolimus.
When you see patients coming from a different transplant center, what ini-
tial therapy might you be less pleased with?
I think the etiology of disease is very important to consider. For autoimmune
hepatitis, PSC, and PBC, a lot of those patients are tapered completely off pred-
nisone. But realize that they have an immune-mediated indication for transplant,
and most of these do recur posttransplant. So, I think that these patients should be
maintained on at least a low dose of prednisone. Oftentimes, I’ll see that
they’re not.
Now another drug that has fallen out of favor is cyclosporine, but depending on
where and how long-ago patients were transplanted, we may see them on cyclospo-
rine. It’s not as strong an immunosuppressive agent, and it causes more renal insuf-
ficiency and comorbid problems than tacrolimus. If patients are on cyclosporine, I’ll
often switch it. Some patients get started on cyclosporine because they have mental
status alteration posttransplant while taking tacrolimus and may be labeled with
418 V. Loy

PRES (posterior reversible encephalopathy syndrome), which is occasionally a side


effect of tacrolimus. But in many cases, the mental changes are unrelated and self-­
limited and not a reason to give up on tacrolimus.
I think that having patients on tacrolimus instead of cyclosporine is important,
whenever possible. Some patients are on mTOR inhibitors, which are not as strong
immune suppressing drugs. If a patient is on an mTOR inhibitor alone, like siroli-
mus or everolimus, they are far more likely to have rejection than a patient who is
on an mTOR inhibitor, along with a calcineurin inhibitor. While it’s pretty unusual
to be on monotherapy with an mTOR inhibitor, I do see it from time to time. Those
are some of the drug choices that are occasionally problematic.
Index

A AMA, 366
Abdominal aortic aneurysm (AAA), 96 American Association for the Study of Liver
Ablation, 50 Diseases (AASLD), 342
Abnormal liver enzymes, 281 American Society of Colon and Rectal
Achalasia Surgeons (ASCRS), 131
absent contractility, 19 Anal incontinence, 122
diagnosis of, 21 Anecdotal evidence, 112
EGJOO, 23 Anorectal manometry, 74
GERD, 18 Antibiotics, 129
manometry, 21 Anticoagulation, 347–349
pharmacologic therapy, 20 Anti-TNFs, 212
pseudoachalasia, 23 Arthralgias, 366
timed barium esophagram, 19 Ascites, 331, 332
TTS dilation, 22 alcoholic cirrhosis, 341
Acute bowel ischemia, 92 alcohol relapse, 343
Acute diarrhea, 74–75 complications, 345, 346
Acute liver failure, 396 hepatoma, 342
necrosis, 403, 404 PEth levels, 341, 342
patient history, 403 surveillance, 342
steroids, 404 without obvious
Acute mesenteric insufficiency (AMI), 101 cirrhosis, 329, 330
Acute severe ulcerative colitis (ASUC), 202, Autoimmune hepatitis (AIH)
205, 206 azathioprine, 380
Adalimumab, 187 burned out, 373, 374
AFP, 342, 355 and cirrhosis, 369
Alcoholic cirrhosis, 333, 334 diagnosis, 379
Alcoholic hepatitis, 323, 324 liver biopsy, 371, 373
Alcoholic liver disease, 319–321 mycophenolate, 372
Alcohol induced liver disease, 282 patient history, 369, 371
Alcohol relapse, 343 pregnancy, 373
Alkaline phosphatase, 404 side effect, 372, 373
ALP, 376 steroids, 380
Alpha-1-antitrypsin deficiency, 282, 295, treatment, 369
299, 331 UDCA, 379

© The Editor(s) (if applicable) and The Author(s), under exclusive license to 419
Springer Nature Switzerland AG 2023
W. H. Sobin et al. (eds.), Managing Complex Cases in Gastroenterology,
https://doi.org/10.1007/978-3-031-48949-5
420 Index

Autoimmune metaplastic atrophic Chronic pancreatitis


gastritis (AMAG) asymptomatic, 149
diagnosis, 65 differential diagnosis, 148
evaluation, 65 medications, 149
gastric cancer, 66 pain, 146
Azathioprine, 187, 369, 372, 373, 380 secretin-stimulated MRI, 146
weight loss, 147
Cirrhosis, 281, 329–331, 353, 369
B CMV, 416
Baclofen, 343 Cold EMR, 112
Balloon ablation, 51 Cold snare polypectomy, 109, 110, 115
Balloon occluded retrograde transvenous Colestipol, 361
obliteration (BRTO), 338, 339 Colon cancer, 88
Barrett’s esophagus (BE)., 51 Colon ischemia (CI), 105
Behavioral precautions, 126 Colonoscopy, 197
Biofeedback therapy, 124 Colon purge, 126
Bloating, 253 Computed tomography (CT) interpretation, 58
baseline hydrogen, 276 Constipation/irritable bowel syndrome-­
brain fog, 273 constipation predominant (IBS-C),
constipation, 273 73–74, 123
delayed small bowel motility, 275 chloride channel receptor, 262
diabetes, 275 colonoscopy, 261
distension fiber, 261, 262
FODMAP, 271, 272 guanylate cyclase receptor, 262
Linzess, 272 laxatives, 264
viscerosomatic reflex, 271 linaclotide, 262
metamucil, 274 Miralax, 262
rifaximin, 276 misoprostol, 263
SNRIs, 272 outlet dysfunction, 264
visceral hypersensitivity, 274, 275 prucalopride, 262
Blood pressure, 11 rectal disimpaction, 263
Bone disease, 367 Sitzmarks study, 265
Boston Scientific and STERIS, 111 strategies, 265
Budd-Chiari syndrome, 330, 351, 352 Corkscrew esophagus, 30
Budesonide, 372 Crohn's disease, 183, 236
bloating, 239, 240
budesonide, 184
C cholestyramine, 219, 220
Calcineurin inhibitors (CNI), 373, 416, 418 immunomodulator, 186
Cannabinoid hyperemesis syndrome infliximab, 223, 224
(CHS), 269 medical option, 185
CAPOX, 152 patency capsule, 183
Cardiac cirrhosis, 330 perianal fistulae, 186
C-arm, 340 smoking patients, 184
Cecal polyps, 116 vedolizumab levels, 185
Celiac disease, 299 Cyclosporine, 373, 417
Ceruloplasmin, 291–293 CYP2C19 polymorphism, 34
Checkpoint inhibitors, 157
Cholangiocarcinoma, 376, 377
Cholangitis lenta, 302 D
Cholestyramine, 361 Diarrhea, 241, 242
Chromoendoscopy, 200 Direct intrahepatic portosystemic shunt
Chronic abdominal pain, 87 (DIPS), 339
Chronic mesenteric ischemia, 104 Direct oral anticoagulants (DOAC), 348, 349
Index 421

Disorder of gut-brain interaction (DGBI), 90 mirtazapine, 268, 269


Distal cholangiocarcinomas, 377 Functional dyspepsia-epigastric pain
Diuretic dose, 345, 346 syndrome type, 269
Diverticulitis, 78–79, 130 Functional heartburn, 36
Dupilumab, 11
Dyslipidemia, 366
Dysphagia, 12 G
Dysplasia, 200 Gallbladder, 89, 376
Dyssynergic defecation, 126 Gastric adenocarcinoma, 66
Gastric carcinoids, 70
Gastric mapping, 67
E Gastric perforations, 63
EASL, 383 Gastric varices, 337
Elective surgery, 131 banding, 339
Endometriosis, 92 bleeding diathesis, 340
Endoscopic eradication therapy (EET), 48 BRTO, 338
Endoscopic mucosal resection esophageal varices, 337
(EMR), 5, 113 management, 337, 338
Endoscopic retrograde patient history, 337
cholangiopancreatography TIPS, 339
(ERCP), 92, 376 Gastroenterologists, 116
Endoscopy Gastroesophageal reflux disease (GERD),
pH testing, 32 3, 31, 41
PPIs, 32 Gastrointestinal (GI), 6
Entecavir, 388, 392 adenocarcinoma, 156
Eosinophilic esophagitis (EOE), 7, 8 autoimmune hepatitis, 176–177
Esophageal manometry, 43 checkpoint inhibitor, 154
Esophageal varices, 283, 335–338 chronic constipation, 171–172
Ethanolamine oleate (EO), 338 EGFR inhibitors, 153
Ethanol glucuronide (EtG), 342 eosinophilic esophagitis (EOE), 167
Extraintestinal manifestations (EIMs), 212 5FU and leucovorin, 152
Extramural lesions, 69 gastroenterologists, 177
genetic testing, 157
Helicobacter pylori, 168–171
F hepatitis B, 174
Fatigue, 360 hepatitis C, 173–174
FD-epigastric pain syndrome (EPS), 253 inflammatory bowel
Fecal incontinence, 78 disease, 175–176
Fermentable oligosaccharides, disaccharides, KRAS mutations, 153
monosaccharides and polyols leucovorin, 153
(FODMAP), 271, 272 metastatic disease, 154
FibroScan®, 327, 388, 402 pancreatic insufficiency, 174–175
Fibrostenotic disease, 12 PARP inhibitors, 158
Flexible sigmoidoscopy, 208 systemic chemotherapy, 155
Fluticasone, 10 tyrosine kinase inhibitor therapy, 155
FOLFOX, 152 Gastroparesis, 76–77
Fomepizole, 406 GERD Consensus Conference, 42
Functional diarrhea, 219, 220 GERD-related stricture, 13
Functional dyspepsia (FD), 91, 253 Gilbert’s, 297, 298
abdominal burning pain, 269 G-J tube, 57
buspirone, 268 Golytely, 125
cannabinoids, 269 GOV-1 varices, 339
delayed gastric emptying, 268 G tube dysfunction, 54
diagnosis, 268 Gut-brain interaction, 253
422 Index

H Inflammatory bowel disease (IBD), 181, 376


Hashimoto’s disease, 371 adalimumab, 245, 246
Heartburn, 37 anti-TNF, 246, 247
Hematemesis, 337 azathioprine, 246, 247
Hemochromatosis, 289, 290, 331 infliximab, 245–247
Hepatic encephalopathy, 346, 395 ozanimod, risankizumab and upadacitinib,
Hepatic steatosis, 303–305, 325 247, 248
Hepatitis A virus (HAV), 367 recurrence rate, 245, 246
Hepatitis B envelope antigen positive self-injection, 231, 232
(HBeAg), 381, 385, 386 tacrolimus, 232
Hepatitis B surface antibody (HBsAb), 397 vedolizumab, 248
Hepatitis B vaccine (HBV), 284 Infusion, 221
acute exacerbation, 395, 396 Interferon, 396
follow-up, 397 Intestinal ischemia, 347
immune-active chronic HBV Intrahepatic cholangiocarcinoma, 377
HBeAg, 385–388 Iron overload, 290
long-term treatment, 388 Irritable-bowel syndrome-diarrhea
patient history, 385 predominant (IBS-D)
TDF, 387, 388 abdominal cramps
immune-tolerant chronic HBV dicyclomine, 258
anti-viral therapy, 382 eluxadoline, 258, 259
chronic hepatitis B, 383 lidocaine, 258
issues, 382, 383 quetiapine, 259
liver biopsy, 382 SNRI, 259
patient history, 381 alosetron, 257
phase of, 381 celiac serology, 255, 256
rituximab, 397–399 colonoscopy, 255
treatment, 391, 392 depression, 256, 257
Hepatitis B virus (HBV), 367 dicyclomine, 256
Hepatitis C, 281 eluxadoline, 256
Hepatitis C vaccine (HCV) IBgard, 256
fibrosis, 402 tricyclics, 257
impression, 401, 402
patient history, 401
Hepatocellular carcinoma (HCC), 283, 353, J
355, 357, 367, 382 JAK inhibitors, 210
Hepatoma, 342 JAMA surgery, 131
Hepatosplenic T cell lymphoma (HSTCL), Jaundice, 370
225, 226
High-density lipoproteins (HDL), 366
High-grade dysplasia (HGD), 48 L
Holistic management, 363 Lamivudine, 396
Hyperbilirubinemia, 282, 301, 302 Levothyroxine, 371
Hyperlipidemia, 366 Lift polyps, 114
LINX anti-reflux system, 43
Liposoluble vitamin deficiencies, 367
I Liver biopsy, 365, 366, 371
IgG4, 376 Liver disease, 284, 319–321, 327
Ileal ulcer Liver enzyme abnormality, 299, 300
budesonide, 236 drug-induced liver injury, 286
calprotectin, 235 FibroScan®, 286
Immunosuppression, 237, 238, 416 history, 285
Ineffective motility (IEM), 44 intravenous drugs, 286
Index 423

liver biopsy, 287 classic type 1 achalasia, 29


paroxysmal nocturnal dyspnea, 285 dysphagia, 27
physical exam, 285 esophageal motility disorder, 28
thyroid disease, 286 Jackhammer esophagus, 26
Liver transplantation, 284 nutcracker, 26
candidates for, 407–409 pan-pressurization, 29
complications, 415–418 sedation, 26
frustrations, 411–413 Myalgias, 366
Locoregional therapies, 356 Mycophenolate, 416
Low-density lipoproteins (LDL), 366 Mycophenolate mofetil (MMF), 372, 373
Low SAAG ascites, 333, 334 Mycophenolic acid (MPA), 372
Myocardial infarction
ASCA/ANCA diagnosis, 193
M c diff NAAT, 195
Magnetic resonance cholangiopancreatography Crohn's disease, 194
(MRCP), 92, 376 JAK-inhibitors, 195
Mayo protocol, 377 ustekinumab, 194
Median arcuate ligament syndrome
(MALS), 89, 107
Medically refractory reflux, 45 N
Medical regimens, 38 Narcotic pain medications, 91
6-Mercaptopurine (6-MP), 372 Necrosis, 403, 404
Mesenteric vein thrombosis (MVT), 103 Neuroendocrine tumors, 70
Metabolic associated steatohepatitis Non-alcoholic fatty liver disease (NAFLD),
(MetASH), 365 303, 305, 357
Metabolic dysfunction associated steatotic biomarkers, 304
liver disease (MASLD), 303–305 FibroScan®, 304, 305
active research and clinical trials, 313 liver biopsy, 305
AST and ALT, 310 MASLD, 304, 305
bariatric surgery, 315, 316 NASH, 304
FibroScan®, 310, 311 overview, 303
GLP-1 agonist, 316 treatment, 307, 308
histological parameters, 311, 312 Non-alcoholic steatohepatitis
inflammatory process, 313 (NASH), 304
liver biopsy, 311 Non-dysplastic Barrett’s esophagus
liver transplantation, 316 (NDBE), 47
management, 317 Nonocclusive mesenteric ischemia
pioglitazone, 316 (NOMI), 105
stage 3 fibrosis, 314, 315 Non-steroidal anti-inflammatory drugs
steatohepatitis, 313 (NSAIDs), 133
steatosis, 312
vitamin E, 312, 314, 316
weight loss and abstention, 309, 310 O
Microscopic colitis (MC) Obeticholic acid (OCA), 362, 366
budesonide, 243, 244 Omeprazole-bicarbonate, 34
vedolizumab, 244 Oral contraceptive pills (OCPs), 93
Minnesota tube, 340 Oropharyngeal dysphagia, 15–16
Model for end-stage liver disease (MELD)
score, 346
Mononuclear cells, 332 P
Motility disorders Pancreatic cancer
Botox injections, 27 checkpoint inhibitors, 158
chest pain and dysphagia, 25 gastroenterologist, 158
424 Index

Pancreatic cysts Primary sclerosing cholangitis (PSC)


autoimmune pancreatitis, 143 cholangiocarcinoma, 377
genetic etiology, 142 diagnosis, 375
IPMNs, 141 IgG4, 376
MRCP, 143 management, 376, 377
recurrent idiopathic pancreatitis, 142 patient history, 375
serum CA 19-9, 140 surveillance, 376
Pantoprazole, 11 UDCA, 376
Paris classification, 117 Pyoderma gangrenosum (PG), 227
Pedunculated polyp, 118–119
PEG, 4, 57
bumper, 61 R
DOACs/Plavix, 62 Rectal exam, 124
gastroenterologists, 53 Rectal incontinence, 121
gauze, 61 Reflux hypersensitivity, 36
infections, 54 Refractory benign esophageal stricture
placement, 59 (RBES), 159–162
PEG-J tube, 63 chemoradiation, 161
Phosphatidylethanol (PEth), 341, 342 esophageal stents, 159–162
Photodynamic therapy, 50 etiology, 159
Pneumocystis jirovecii, 237, 238 extrinsic compression, 162
Polypectomy, 111–113 pancreatic cancer, 163–165
Portal hypertension, 370 self-dilation, 160
Portal vein thrombosis (PVT), 283, 347, 351, self-expanding stent, 160
352, 355, 356 stent role, 161
Post-cholecystectomy diarrhea, 75 Refractory GERD, 37–39
Post-op dysphagia, 44 Resistant/refractory proctitis, 208
Post-polypectomy bleeding, 119–120 Rheumatoid arthritis (RA), 95
Postprandial distress syndrome (PDS), 253 Rituximab, 397–399
PPI standpoint, 11
Prednisolone, 380
Prednisone, 94, 369, 380, 416 S
Pregnancy, 373 Sacral nerve stimulation, 123
Primary biliary cholangitis (PBC) Segmental colitis associated with
AMA levels, 366 diverticulosis (SCAD), 5, 134
azathioprine, 380 Senna, 74
colestipol, 361 Serotonin and norepinephrine reuptake
diagnosis, 359, 360, 379 inhibitor (SNRI), 259, 272
grade A cirrhosis, 362 Serum albumin-ascites albumin gradient
hepatic decompensation, 363 (SAAG), 283, 329, 330
lipid levels, 366 Sicca complex symptoms, 361
liver biopsy, 365, 366 Sigmoid colectomy, 136
liver enzymes, 363 Sinusoidal obstruction syndrome (SOS), 330
obeticholic acid, 362 Small bowel carcinoids, 71
OCA, 366 Small bowel obstruction, 197
patient history, 359, 365 Small intestinal bacterial overgrowth (SIBO), 4
pruritus, 361 breath tests, 85
steroids, 380 predispose, 82
symptoms, 360, 361 Small nodule, 50
treatment, 362 Steatohepatitic hepatocellular carcinoma, 357
ursodeoxycholic acid, 360–362, 366, Steroids, 369
367, 379 Superior mesenteric vein (SMV), 347
Index 425

T infliximab, 221
Tacrolimus, 372, 373, 418 JAK inhibitors, 201
Tenofovir, 392 mesalamine, 189, 215, 216
Terlipressin, 338 ozanimod, 190
Thiopurine methyltransferase (TPMT), 372 pyoderma gangrenosum, 227
Thrombocytopenia, 347 tofacitinib, 229
Thromboelastogram (TEG), 340 upadacitinib, 229
Thrombosis, 337 ustekinumab, 229, 230
Total protein (TP), 329 vedolizumab, 215, 216
Transferrin saturation (TS), 289 Upadacitinib, 203
Transjugular intrahepatic portosystemic shunt Ursodeoxycholic acid (UDCA), 360–362, 366
(TIPS), 339 PBC-AIH overlap syndrome, 379
Transoral incisionless fundoplication in PSC, 376
(TIF), 4, 46 Ustekinumab (UST), 190
Trastuzumab, 156
Tuberculosis, 332
Tylenol overdose, 403, 405 V
Vascular disorders, 101, 103
Vedolizumab (VDZ), 190
U Viokase (pancreatic enzyme), 56
Ulcerative colitis (UC), 335, 336, 351
colon stricture, 233, 234
immune modulators W
azathioprine, 225, 226 Warfarin, 348
methotrexate, 226 Wilson’s disease, 282, 291–293, 299
monotherapy, 225 Wound care, 55

You might also like